You are on page 1of 413

JEE Main 27-01-2024(Morning)

Questions

1. The acceleration due to gravity on the surface of earth is g. If the diameter of earth reduces to half of its
original value and mass remains constant, then acceleration due to gravity on the surface of earth would be:
g
(a) 2

(b) 2g
g
(c) 4

(d) 4g

2. A wire of length 10 cm and radius √– 7 × 10


−4
m is connected across the right gap of a meter bridge. When a

resistance of 4.5Ω is connected on the left gap by using a resistance box, the balance length is found to be at
60cm from the left end. If the resistivity of the wire is R × 10 Ωm, then value of R is:
−7

(a) 35
(b) 63
(c) 66
(d) 70

3. The average kinetic energy of a monoatomic molecule is 0.414 eV at temperature: (Use


−23
KB = 1.38 × 10 J /mol − K

(a) 1500 K
(b) 3200 K
(c) 1600 K
(d) 3000 K

4. A body of mass 1000kg is moving horizontally with a velocity 6 m/s. If 200 kg extra mass is added, the final
velocity (in m/s) is:
(a) 3
(b) 5
(c) 6
(d) 2

5. A wire of resistance R and length L is cut into 5 equal parts. If these parts are joined parallelly, then result
resistance will be:
(a) 1
R
25

(b) 25R
(c) R 1

(d) 5R

6. 0.08kg air is heated at constant volume through 5 C . The specific heat of air at constant volume is 0.17

kcal/kgoC and J = 4.18 joule/cal. The change in its internal energy is approximately.
(a) 284 J
(b) 298 J
(c) 142 J
(d) 318 J
7. A proton moving with a constant velocity passes through a region of space without any change in its velocity.
If E ⃗ and B⃗ represent the electric and magnetic fields respectively, then the region of space may have:
(a) E = 0, B = 0
(b) E = 0, B ≠ 0
(c) E ≠ 0, B = 0
(d) E ≠ 0, B ≠ 0
Choose the most appropriate answer from the options given below:
(a) A, C and D only
(b) A, B and C only
(c) A, B and D only
(d) B, C and D only

8. A rectangular loop of length 2.5 m and width 2m is placed at 60 to a magnetic field of 4T. The loop is

removed from the field in 10s. The average emf induced in the loop during this time is:
(a) -2V
(b) +1V
(c) -1V
(d) +2V

9. Given below are two statements:


Statement (I): Planck’s constant and angular momentum have same dimensions
Statement (II): Linear momentum and moment of force have same dimensions.
In the light of the above statements, choose the correct answer from the options given below:
(a) Statement I is true but Statement II is false
(b) Statement I is false but Statement II is true
(c) Both Statement I and Statement II are true
(d) Both Statement I and Statement II are false

10. An electric charge 10 μC is placed at origin (0, 0)m of X – Y co-ordinate system. Two points P and Q are
−6

– – –
situated at (√3, √3) m and (√6, 0)m respectively. The potential difference between the points P and Q
will be:
(a) 3V

(b) √6V

(c) √3V
(d) 0V

11. Identify the physical quantity that cannot be measured using spherometer:
(a) Radius of curvature of convex surface
(b) Radius of curvature of concave surface
(c) Specific rotation of liquids
(d) Thickness of thin plates
12. Which of the following circuits is reverse – Biased?

(a)

(b)

(c)

(d)

13. A plane electromagnetic wave propagating in x-direction is described by


) sin[1.5 × 10 t − 0.05x]; The intensity of the wave is:
−1 7
E = (200V m
y

(Use ∈ = 8.85 × 10 C N m )
0
−12 2 −1 −2

(a) 106.2W m −2

(b) 35.4W m −2

(c) 53.1W m −2

(d) 26.6W m −2

14. A convex lens of focal length 40 cm forms an image of an extended source of light on a photoelectric cell. A
current I is produced. The lens is replaced by another convex lens having the same diameter but focal length
20cm. The photoelectric current now is:
(a) 2I
(b)
I

(c) I
(d) 4I

15. Position of an ant (S in meters) moving in Y-Z plane is given by S 2^ ^


= 2t j + 5k (where t is in second). The
magnitude and direction of velocity of the ant at t = 1s will be:
(a) 16 m/s in y-direction
(b) 4 m/s in y-direction
(c) 4 m/s in x-direction
(d) 9 m/s in z-direction
16. Two bodies of mass 4g and 25 g are moving with equal kinetic energies. The ratio of magnitude of their
linear momentum is:
(a) 4 : 5
(b) 5 : 4
(c) 3 : 5
(d) 2 : 5

17. Given below are two statements:


Statement (I): Viscosity of gases is greater than that of liquids
Statement (II): Surface tension of a liquid decreases due to the presence of insoluble impurities.
In the light of the above statements, choose the most appropriate answer from the options given below:
(a) Statement I is correct but Statement II is incorrect
(b) But Statement I and Statement II is incorrect
(c) Both Statement I and Statement II are correct
(d) Statement I is incorrect but Statement II is correct

18. A train is moving with a speed of 12 m/s on rails which are 1.5 m apart. To negotiate a curve of radius 400
m, the height by which the outer rail should be raised with respect to the inner rail is (Given, g = 10m/s ): 2

(a) 6.0 cm
(b) 5.4 cm
(c) 4.8 cm
(d) 4.2 cm

19. If the refractive index of the material of a prism is cot( A


), where A is the angel of prism then the angle of
2

minimum derivation will be:


π
(a) − 2A
2
π
(b) − A
2

(c) π − A
(d) π − 2A

20. The radius of third stationary orbit of electron for Bohr’s atom is R. the radius of fourth stationary orbit will
be:
(a) R 4

(b) 16
R
9

(c) R 3

(d) 9

16
R

21. Two coils have mutual inductance 0.002H. The current changes in the first coil according to the relation
i = i sin ωt, where i = 5A and ω = 50πrad/s. The maximum value of emf in the second coil is V .
π
0 0
α

The value of α is ______


22. Two immiscible liquids of refractive indices and respectively are put in a beaker as shown in the
8 3

5 2

figure. The height of each column is 6 cm. A coil is placed at the bottom of the beaker. For near normal
vision, the apparent depth of the coil is cm. The value of α is ____.
α

23. A thin metallic wire having cross sectional area of 10 m is used to make a ring of radius 30 cm. A
−4 2

positive charge of 2πC is uniformly distributed over the ring, while another positive charge of 30pC is kept
at the centre of the ring. The tension in the ring is _____ N; provided that the ring does not get deformed
(neglect the influence of gravity).
(given, 1

4πε0
9
= 9 × 10 S I units

24. A particle executed simple harmonic motion with an amplitude of 4cm. At the mean position, velocity of the
particle is 10 m/s. The distance of the particle from the mean position when its speed becomes 5cm/s is


√ α cm, where α =____.

25. If average depth of an ocean is 4000m and the bulk modulus of water is 2 × 10 N m , the fractional
9 −2

compression of water at the bottom of ocean is α × 10 . The value of α is _____. (Given


ΔV

V
−2

g = 10ms
−2
, ρ = 1000kg m
−3
)

26. In a nuclear fission process, a high mass nuclide (A = 236) with binding energy 7.6 MeV/ Nucleon
dissociated into middle mass nuclides (A = 118), having binding energy of 8.6 MeV/Nucleon. The energy
released in the process would be _____ MeV.

27. A particle starts from origin at t = 0 with a velocity 5^i m/s and moves in x-y plane under action of a force
which produces a constant acceleration of (3^i + 2^j) m/s . If the x-coordinate of the particle at that
2

instant is 84m, then the speed of the particle at this time is √−


α m/s . The value of α is ____.

28. Two long, straight wires carry equal currents in opposite directions as shown in figure. The separation
between the wires is 5.0 cm. The magnitude of the magnetic field at a point P midway between the wires is
______ μT . (Given: μ = 4π × 10 T mA )
0
−7 −1
29. The charge accumulated on the capacitor connected in the following circuit is _____ μC . (Given
C = 150μF )

30. Four particle each of mass 1 kg are placed at four corners of a square of side 2m. Moment of inertia of
system about an axis perpendicular to its plane and passing through one of its vertex is ______ kgm .
2

31. Given below are two statements : one is labelled as Assertion (A) and the other is labelled as Reason (R).
Assertion (A): Melting point of Boron (2453 K) is unusually high in group 13 elements.
Reason (R): Solid Boron has very strong crystalline lattice.
In the light of the above statements, choose the most appropriate answer from the options given below:
(a) (A) is false but (R) is true
(b) Both (A) and (R) are correct but (R) is not the correct explanation of (A)
(c) (A) is true but (R) is false
(d) Both (A) and (R) are correct and (R) is the correct explanation of (A)

32. Two nucleotides are joined together by a linkage known as:


(a) Peptide linkage
(b) Disulphide linkage
(c) Glycosidic linkage
(d) Phosphodiester linkage
33. Highest enol content will be shown by:

(a)

(b)

(c)

(d)

34. The electronic configuration for Neodymium is:


[Atomic Number for Neodymium 60]
(a) [Xe] 4f4 6s2
(b) [Xe] 4f1 5d1 6s2
(c) [Xe] 4f6 6s2
(d) [Xe] 5f7 7s2

35. A solution of two miscible liquids showing negative deviation from Raoult's law will have:
(a) decreased vapour pressure, decreased boiling point
(b) decreased vapour pressure, increased boiling point
(c) increased vapour pressure, decreased boiling point
(d) increased vapour pressure; increased boiling point
36. Yellow compound of lead chromate gets dissolved on treatment with hot NaOH solution. The product of
lead formed is a :
(a) Neutral complex with coordination number four
(b) Tetraanionic complex with coordination number six
(c) Dianionic complex with coordination number six
(d) Dianionic complex with coordination number four

37. Which of the following has highly acidic hydrogen?

(a)

(b)

(c)

(d)

38. Which of the following electronic configuration would be associated with the highest magnetic moment?
(a) [Ar] 3d6
(b) [Ar] 3d7
(c) [Ar] 3d8
(d) [Ar] 3d3
39. Which of the following is strongest Bronsted base?

(a)

(b)

(c)

(d)

40. Choose the polar molecule from the following:


(a) CH2 = CH2
(b) CCl4
(c) CHCl3
(d) CO2

41. Given below are two statemen is :


Statement (I): p-nitrophenol is more acidic than m-nitrophenol and o-nitrophenol.
Statement (II): Ethanol will give immediate turbidity with Lucas reagent.
1n the light of the above statements, choose the correct answer from the options given below:
(a) Statement I is true but Statement II is false
(b) Statement I is false but Statement II is true
(c) Both Statement I and Statement II are false
(d) Both Statement I and Statement II are true
42. IUPAC name of the following compound (P) is:

(a) 1-Ethyl-5,5-dimethylcyclohexane
(b) 1-Ethyl-3,3-dimethylcyclohexane
(c) 3-Ethyl-1,1-dimethylcyclohexane
(d) 1,1-Dimethy 1-3-ethy-etyclohexane

43. Given below are two statements:


Statement (I): The 4f and 5f - series of elements are placed separately in the Periodic table to preserve the
principle of classification.
Statement (II): s-block elements can be found in pure form in nature.
In the light of the above statements, choose the most appropriate answer &om the options given below:
(a) Statement I is false but Statement II is true
(b) Both Statement I and Statement II are false
(c) Statement I is true but Statement II is false
(d) Both Statement I and Statement II are true

44. The correct statement regarding nucleophilic substitution reaction in a chiral alkyl halide is:
(a) Retention occurs in SN1 reaction and inversion occurs in SN2 reaction.
(b) Racemisation occurs in both SN1 and SN2 reactions.
(c) Racemisation occurs in SN1 reaction and inversion occurs in SN2 reaction.
(d) Racemisation occurs in SN1 reaction and retention occurs in SN2 reaction.

45. Consider the following complex ions


P = [FeF6]3–
Q = [V(H2O)6]2+
R = [Fe(H2O)6]2+
The correct order of the complex ions, according to their spin only magnetic moment values (in B.M) is:
(a) R < Q < P
(b) Q < R < P
(c) R < P < Q
(d) Q < P < R

46. Element not showing variable oxidation state is :


(a) Iodine
(b) Chlorine
(c) Bromine
(d) Fluorine
47. The ascending order of acidity of –OH group in the following compounds is:
(A) Bu – OH
(B)

(C)

(D)

(E)

Choose the correct answer from the options given below:


(a) (C) < (A) < (D) < (B) < (E)
(b) (A) < (C) < (D) < (B) < (E)
(c) (C) < (D) < (B) < (A) < (E)
(d) (A) < (D) < (C) < (B) < (E)

48.

Cyclohexene is ________ type of an organic compound.


(a) Acyclic
(b) Alicyclic
(c) Benzenoid non-aromatic
(d) Benzenoid aromatic
49. Given below are two statements:
Statement (I): Aqueous solution of ammonium carbonate is basic.
Statement (II): Acidic/basic nature of salt solution of a salt of weak acid and weak base depends on Ka and
Kb value of acid and the base forming it.
In the light of the above statements, choose the most appropriate answer from the options given below:
(a) Both Statement I and Statement II are incorrect
(b) Statement I is correct but Statement II is incorrect
(c) Both Statement I and Statement II are correct
(d) Statement I is incorrect but Statement· II ls correct

50. NaCl reacts with conc. H2SO4 and K2Cr2O7 to give reddish fumes (B), which react with NaOH to give
yellow solution (C). (B) and (C) respectively are :
(a) CrO2Cl2, KHSO4
(b) CrO2Cl2, Ns2Cr2O7
(c) CrO2Cl2, Na2CrO4
(d) Na2CrO4, CrO2Cl2

51. The number of electrons present in all the completely filled subshells having n = 4 and s = + 1
is ______
2

(Where n = principal quantum number and


s = spin quantum number)

52. Sum of bond order of CO and NO+ is _______

53. The mass of silver (Molar mass of Ag : 108 gmol–1) displaced by a quantity of electricity which displaces
5600 mL of O2 at S.T.P. will be ________ g.
54. Among the given organic compounds, the total number of aromatic compounds is _____
(A)

(B)

(C)

(D)

55. From the given list, the number of compounds with +4 oxidation state of Sulphur is ______
SO3, H2SO3, SOCl2, SF4, BaSO4, H2S2O7

56. If three moles of an ideal gas at 300 K expand isothermally from 30 dm3 to 45 dm3 against a constant
opposing pressure of 80 kPa, then the amount of heat transferred is ______ J.

57. Consider the following data for the given reaction


2 HI(g) → H2(g) + I2(g)

1 2 3
HI (mol L–1) 0.005 0.01 0.02
Rate (mol L–1 s–1) 7.5 × 10–4 3.0 × 10–3 1.2 × 10–2

The order of the reaction is_______

58. Among the following, total number of meta directing functional groups is ________
(Integer based)
–OCH3, –NO2, –CN, –CH3, –NHCOCH3, –COR, –OH, –COOH, –Cl

59. 3-Methylhex-2-ene on reaction with HBr in presence of peroxide forms an addition product (A).
The number of possible stereoisomers for 'A' is ______
60. Mass of methane required to produce 22 g of CO2 after complete combustion is g _______.
(Given Molar mass in g mol–1 C = 12.0, H = 1.0, O = 16.0)

61. The portion of the line 4x + 5y = 20 in the first quadrant is trisected by the lines L and L passing 1 2

through the origin. The tangent of an angle between the lines L and L is: 1 2

(a) 8

(b) 30

41

(c) 2

(d)
25

41

62. If the shortest distance of the parabola y = 4x from the centre of the circle
2

x + y − 4x − 16y + 64 = 0 is d , then d is equal to:


2 2 2

(a) 20
(b) 16
(c) 24
(d) 36

63. 10

Let a 1 , a 2 , . . . . a 10 be 10 observations such that ∑ a k = 50 and ∑ a k ⋅ a j = 1100 . Then the standard


k=1 ∀ k<j

deviation of a 1, a 2 , . . . . , a 10 is equal to:


(a) 10
(b) 5
−−−
(c) √115

(d) √5

64. If the shortest distance between the lines x−4


=
y+1
=
z
and x−λ
=
y+1
=
z−2
is 6
, then the sum of
1 2 −3 2 4 −5 √5

all possible values of λ is:


(a) 5
(b) 10
(c) 8
(d) 7

65. The length of the chord of the ellipse x


2

+
y
2

= 1, whose mid point is (1, 2


) , is equal to:
25 16 5

(a) √ 2009

(b)
√ 1741

(c)
√ 1541

(d)
√ 1691

66. Let S = {1, 2, 3, . . . . , 10} . Suppose M is the set of all the subsets of S , then the relation
R = {(A, B) : A ∩ B ≠ ϕ; A, B ∈ M } is:

(a) symmetric only


(b) symmetric and reflexive only
(c) reflexive only
(d) symmetric and transitive only
67. Let ^ ^ ^
a⃗ = i + 2 j + k,
⃗ ^ ^ ^
b = 3 ( i − j + k) . Let c ⃗ be the vector such that a⃗ × c ⃗ = b

and a⃗ ⋅ c ⃗ = 3. Then
⃗ ⃗
a ⃗ ⋅ ((c ⃗ × b) − b − c )
⃗ is equal to
(a) 24
(b) 36
(c) 32
(d) 20

n
68. If A denotes the sum of all the coefficients in the expression of (1 − 3x + 10x 2
) and B denotes the sum
n
of all the coefficients in the expression of (1 + x ) , then:
2

(a) 3A = B
(b) A = 3B
(c) B = A 3

(d) A = B 3

69. Four distinct points (2k, 3k) , (1, 0) , (0, 1) and (0, 0) lie on a circle for k equal to:
(a) 2

13

(b) 5

13

(c) 3

13

(d) 1

13

70. If S = {z ∈ C : |z − i| = |z + i| = |z − 1|} , then, n (S ) is:


(a) 3
(b) 0
(c) 2
(d) 1

71. cos x − sin x 0


⎡ ⎤
Consider the matrix f (x) = ⎢ sin x cos x 0⎥
⎣ ⎦
0 0 1

Given below are two statements:


Statement I: f (−x) is the inverse of the matrix f (x).
Statement II: f (x) f (y) = f (x + y).
In the light of the above statements, choose the correct answer from the options given below
(a) Statement I is true but Statement II is false
(b) Statement I is false but Statement II is true
(c) Both Statement I and Statement II are false
(d) Both Statement I and Statement II are true

72. The distance, of the point (7, −2, 11) from the line x−6
=
y−4
=
z−8
along the line x−5
=
y−1
=
z−5

1 0 3 2 −3 6

is:
(a) 14
(b) 18
(c) 12
(d) 21
73. √ 1+√ 1+x 4−√ 2
2

If a = lim
x
4
and b = lim
sin x

√ 2 −√ 1+cos x
, then the value of ab is:
3

x→0 x→0

(a) 25
(b) 30
(c) 32
(d) 36

74. 1
– –
If ∫ 1

√ 3+x+√ 1+x
dx = a + b√ 2 + c√ 3 , where a, b, c are rational numbers, then 2a + 3b − 4c is equal to:
0

(a) 4
(b) 8
(c) 10
(d) 7

75. Let x = x (t) and y = y (t) be solutions of the differential equations dx


+ ax = 0 and
dy
+ by = 0
dt dt

respectively, a, b ∈ R. Given that x (0) = 2; y (0) = 1 and 3y (1) = 2x (1), then value of t, for which

x (t) − y (t) , is:

(a) log 3 4

(b) log 4 3

(c) log 2 4

(d) log 2

3
2

76. Consider the function,


2
a(7x−12−x )


⎪ , x < 3


2
b|x −7x+12|

sin(x−3)
f (x) = ⎨
, , x > 3


x−[x]



b , x = 3

where [x] denotes the greatest integer less than or equals to x. If S denotes the set of all ordered pairs (a, b)
such that f (x) is continuous at x = 3, then the number of elements in S is:
(a) Infinitely many
(b) 4
(c) 2
(d) 1

77. The number of common terms in the progressions 4, 9, 14, 19, . . . . up to 25th term and 3, 6, 9, 12, . . . , up
to 37th term is:
(a) 9
(b) 8
(c) 5
(d) 7

78. The function f : N − {1} → N; defined by f (n) = the highest prime factor of n , is:
(a) one-one only
(b) neither one-one nor onto
(c) both one-one and onto
(d) onto only
79. If (a, b) be the orthocentre of the triangle whose vertices are (1, 2) , (2, 3) and (3, 1) and
b b
I1
2
I1 = ∫ x sin(4x − x )dx, I2 = ∫ sin(4x − x )dx,
2
then 36 I2
is equal to:
a a

(a) 88
(b) 72
(c) 80
(d) 66

80. n−1
C r = (k
2
− 8)
n
C r+1 if and only if:

(a) 2√ 2 < k ⩽ 3
– –
(b) 2√ 2 < k < 2√ 3
– –
(c) 2√ 3 < k < 3√ 3
– –
(d) 2√ 3 < k ⩽ 3√ 2

81. If the solution of the differential equation (2x + 3y − 2) dx + (4x + 6y − 7) dy = 0, y (0) = 3 , is


αx + βy + 3log |2x − 3y − γ| = 6, then α + 2β + 3γ is equal to _____.
e

82. Let f (x) = x


3
+ x f
2 ′
(1) + xf
′′
(2) + f
′′′
(3) , x ∈ R . Then f ′
(10) is equal to _____.

83. Let the set of all a ∈ R such that the equation cos 2x + a sin x = 2a − 7 has a solution be [p, q] and
+ tan 81 , then pqr is equal to _____.
∘ ∘ 1 ∘
r = tan 9 − tan 27 − ∘
cot 63

84. If 8 = 3 +
1
(3 + p) +
1
2
(3 + 2p) +
1
3
(3 + 3p) +. . . ∞ , then the value of p is _____.
4 4 4

85. A fair die is tossed repeatedly until a six is obtained. Let X denote the number of tosses required and let
a = P (X = 3) , b = P (X ⩾ 3) and c = P (X ⩾ 6|X > 3) . Then is equal to ____. b+c

86. 2 0 1
⎡ ⎤
Let A = ⎢1 1 0 ⎥ , B = [B 1 , B 2 , B 3 ] , where B 1, B2 , B3 are column matrics, and
⎣ ⎦
1 0 1

1 2 3
⎡ ⎤ ⎡ ⎤ ⎡ ⎤
AB 1 = ⎢ 0 ⎥ , AB 2 = ⎢ 3 ⎥ , AB 3 = ⎢ 2 ⎥ .
⎣ ⎦ ⎣ ⎦ ⎣ ⎦
0 0 1

If α = [B] and β is the sum of all the diagonal elements of B , then α 3


+ β
3
is equal to _____.

87. Let for a differentiable function f : (0, ∞) → R, f (x) − f (y) ⩾ loge (


x
) + x − y, ∀x, y ∈ (0, ∞) .
y

20

Then ∑ f

(
n
1

2
) is equal to ____.
n=1

88. Let the area of the region {(x, y) : x − 2y + 4 ⩾ 0, x + 2y ⩾ 2


0, x + 4y
2
⩽ 8, y ⩾ 0 } be m

n
, where
m and n are coprime numbers, Then m + n is equal to _____.

89. If α satisfies the equation x + x + 1 2


= 0 and (1 + α)
7
= A + Bα + C α , A, B, C ⩾ 0
2
, then
5 (3A − 2B − C ) is equal to _____.
90. The least positive integral value of α, for which the angle between the vectors α^i − 2^j + 2k
^
and
α i + 2α j − 2k is acute, is _____.
^ ^ ^
Answer Key

1. D 2. C 3. B 4. B 5. A 6. A
7. C 8. B 9. A 10. D 11. C 12. A
13. C 14. C 15. B 16. D 17. D 18. B
19. D 20. B 21. 2 22. 31 23. 3 24. 12
25. 2 26. 236 27. 673 28. 160 29. 400 30. 16
31. D 32. D 33. C 34. A 35. B 36. D
37. A 38. A 39. D 40. C 41. A 42. C
43. C 44. C 45. B 46. D 47. B 48. B
49. A 50. C 51. 16 52. 6 53. 108 54. 3
55. 3 56. 1200 57. 2 58. 4 59. 4 60. 8
61. B 62. A 63. D 64. C 65. D 66. A
67. A 68. D 69. B 70. D 71. D 72. A
73. C 74. B 75. C 76. D 77. D 78. B
79. B 80. A 81. 29.00 82. 202.00 83. 48.00 84. 9.00
85. 12.00 86. 28.00 87. 2890.00 88. 119.00 89. 5.00 90. 5.00
Solutions

1. (D)
GM 1
g = 2
⇒ g ∝ 2
R R
2
g R
2 1
= 2
g
1 R
2

R1
g 2 = 4 g 1 (R 2 = )
2

2. (C)
For null point,
4.5 R
=
60 40
ρℓ ρℓ
Also, R =
A
=
2
πr
0.1
4.5 × 40 = ρ × × 60
−8
π×7×10
−7
ρ = 66 × 10 Ω × m

3. (B)
For monoatomic molecule degree of freedom = 3.
3
∴ Kavg = KB T
2
−19
0.414×1.6×10 ×2
T = −23
3×1.38×10

= 3200K

4. (B)
Momentum will remain conserve
1000 × 6 = 1200 × v

v = 5m/s

5. (A)
Resistance of each part = R

Total resistance = 1

5
×
R

5
=
R

25

6. (A)
Q = ΔU as work done is zero [constant volume]
ΔU = msΔT

= 0.08 × (170 × 4.18) × 5

≃ 284J

7. (C)
Net force on particle must be zero i.e. qE ⃗ + qV ⃗ × B⃗ = 0

Possible cases are


(i) E &⃗ ⃗
B = 0

(ii) V ⃗ × B⃗ = 0, E ⃗ = 0
(iii) qE ⃗ = −qV ⃗ × B⃗
⃗ ⃗
E ≠ 0&B ≠ 0
8. (B)
Change in flux Δϕ
Average emf = Time
= −
Δt

0−(4×(2.5×2) cos 60 )
= = +1V
10

9. (A)
2 −1
[h] = M L T
2 −1
[L] = M L T
−1
[P ] = M LT
2 −2
[τ ] = M L T

(Here h is Planck’s constant, L is angular momentum, P is linear momentum and τ is moment of force)

10. (D)
Potential difference=
KQ KQ

r1 r2
−−−−−−−−−−−−
– 2 – 2
r1 = √(√ 3) + (√ 3)
−−−−−−−−
– 2
r2 = √(√ 6) + 0

As r = r = √6m
1 2

So potential difference = 0

11. (C)
Spherometer can be used to measure curvature of surface.

12. (A)
P end should be at higher potential for forward biasing.

13. (C)
1 2
I = ε0 E × c
2 0
1 −12 4 8
I = × 8.85 × 10 × 4 × 10 × 3 × 10
2
2
I = 53.1W /m

14. (C)
As amount of energy incident on cell is same so current will remain same.

15. (B)
ds ⃗ ^
v⃗ = = 4t j
dt

At t = 1 sec v ⃗ = ^
4j

16. (D)
2 2
P P
1 2
=
2m1 2m2

P1


m

1 2
= √ =
P2 m2 5

17. (D)
Gases have less viscosity.
Due to insoluble impurities like detergent surface tension decreases
18. (B)
2
v 12×12
tan θ = =
Rg 10×400

h
tan θ =
1.5
h 144
⇒ =
1.5 4000

h = 5.4cm

19. (D)
A+δ
min
sin( )
A 2

cot =
2 A
sin
2

A A+δmin
⇒ cos = sin( )
2 2

A+δmin π A
= −
2 2 2

δmin = π − 2A

20. (B)
2
n
r ∝
Z
2
r4 4
= 2
r3
3
16
r4 = R
9

21. (2)
ϕ = M i = M i 0 sin ωt
di
EM F = −M = −0.002 (i 0 ω cos ωt)
dt

EM F max = i 0 ω (0.002) = (5) (50π) (0.002)


π
EM F max = V
2

22. (31)
h1 h2 6 6 15 31
happ = + = 3
+ 8
= 4 + = cm
μ μ 4 4
1 2
2 5
23. (3)

kq
dθ 0
2T sin = 2
. λRdθ
2 R

Q
[λ = ]
2πR

Kq Q
0
T = 2
(R )×2π

9 −12
(9×10 )(2π×30×10 )
=
2
(0.30) ×2π
−3
9×10 ×30
= = 3N
−2
9×10

24. (12)
V at mean position = Aω ⇒ 10 = Aω
5
ω =
2
−−−−−−−
v = ω√A2 − x2
−−−−−−
5 2
5 = √4 − x2 ⇒ x2 = 16 − 4
2
− −
x = √ 12 cm

25. (2)
ΔP
B = −
ΔV
( )
V

ρgh
ΔV 1000×10×4000
−( ) = = 9
V B 2×10

= 2 × 10
−2
[-ve sign represent compression]

26. (236)
Q = BEproduct − BERectant

= 2 (118) (8.6) − 236 (7.6)

= 236 × 1 = 236M eV

27. (673)
ux = 5m/s
2
a x = 3m/s

x = 84m
2 2
vx − ux = 2ax
2
vx − 25 = 2 (3) (84)

V x = 23m/s

vx − ux = a x t
23−5
t = = 6s
3

vy = 0 + a y t = 0 + 2 × (6) = 12m/s
2 2 2 2 2
v = vx + vy = 23 + 12 = 673
−−−
v = √ 673 m/s
28. (160)
μ i −7
0 4π×10 ×10
B = ( ) × 2 =
2πa 5 −2
π×( ×10 )
2

−5
= 16 × 10 = 160μT

29. (400)

10
VA + (1) − 6 (1) = V B
3
10 8
VA − VB = 6 − = volt
3 3

Q = C (V A − V B )
8
= 150 × = 400μC
3

30. (16)

2 2 – 2
I = ma + ma + m(√ 2a)
2
= 4ma
2
= 4 × 1 × (2) = 16

31. (D)
Solid Boron has very strong crystalline lattice so its melting point unusually high in group 13 elements
32. (D)

33. (C)

34. (A)
Electronic configuration of Nd(Z = 60) is;
[Xe] 4f4 6s2

35. (B)
Solution with negative deviation has
P T < PA 0 X A + PB 0 X B
P A < PA 0 X A
P B < PB 0 X B
If vapour pressure decreases so boiling point increases.

36. (D)
PbCrO4 + NaOH (hot excess) → [Pb(OH)4]–2 + Na2CrO4
Dianionic complex with coordination number four
37. (A)

38. (A)

3d7 3d8 3d3 3d6


No. of. unpaired e– 3 2 3 4
− − – − − − −
Spin only Magnetic moment √ 15 BM √8 BM √ 15 BM √ 24 BM

39. (D)

40. (C)

μ≠0
CHCl3 is polar molecule and rest all molecules are non-polar
41. (A)
Acidic strength

Ethanol give lucas test after long time


Statement (I) → correct
Statement (II) → incorrect

42. (C)
3-Ethyl-1,1-dimethylcyclohexane

43. (C)
s-block elements are highly reactive and found in combined state.

44. (C)
SN1 – Racemisation
SN2 – Inversion

45. (B)
[FeF6]3– : Fe+3 : [Ar] 3d5
F : Weak field Ligand

No. of unpaired electron's = 5


−−− −−− −
μ = √5 (5 + 2)
−−
μ = √35 BM
[V(H2O)6]2+ : V+2 : 3d3

No. of unpaired electron's = 3


−−− −−− −
μ = √3 (3 + 2)
−−
μ = √15 BM
[Fe(H2O)6]2+ : Fe+2 : 3d6
H2O : Weak field Ligand

No. of unpaired electron's = 4


−−− −−− −
μ = √4 (4 + 2)
−−
μ = √24 BM
46. (D)
Fluorine does not show variable oxidation state.

47. (B)

48. (B)

is Alicyclic

49. (A)
Aqueous solution of (NH4)2CO3 is Basic pH of salt of weak acid and weak base depends on Ka and Kb value of
acid and the base forming it

50. (C)
NaCl + conc. H2SO4 + K2Cr2O7 → CrO2 Cl 2 + KHSO4 + NaHSO4 +H2O
(B)

Reddish brown

CrO2Cl2 + NaOH → Na2 CrO4 + NaCl +H2O


(C)

Yellow colour

51. (16)
n = 4 can have,

4s 4p 4d 4f
Total electrons 2 6 10 14
Total electrons with S = + 1

2
1 3 5 7

So, ans. 16

52. (6)
+

CO ⇒ C
¯≡
O : BO = 3

NO+ ⇒ N ≡ O+ : BO = 3
53. (108)
Eq. of Ag = Eq. of O2
Let x gm silver displaced,
as per old STP data, molar volume = 22.4 lit
x×1 5.6
= × 4
108 22.7

x= 108 gm.
Ans. 108

54. (3)
B, C and D are aromatic

55. (3)

Compounds SO3 H2SO3 SOCl2 SF4 BaSO4 H2S2O7


O.S. of Sulphur: +6 +4 +4 +4 +6 +6

56. (1200)
Using, first law of thermodynamics,
∆U = Q + W,
∆U = 0 : Process is isothermal
Q=–W
W = – Pext ∆V : Irreversible
= –80 × 103 (45 – 30) × 10–3
= –1200 J

57. (2)
Let, R = k[HI]n
using any two of given data,
−3 n
3×10 0.01
= ( )
−4
7.5×10 0.005

n=2

58. (4)
–NO2, –C ≡ N, –COR, –COOH are meta directing.

59. (4)

Number of stereoisomers = 4
60. (8)
CH4 + 2O2 → CO2 + 2H2O
Moles of CO2 = = 0.5 22

44

So, required moles of CH4 = 0.5


Mass = 0.5 × 16 = 8gm

61. (B)
Co-ordinates of A = (
5

8
,
3

8
)

Co-ordinates of B = (
10

3
,
4

3
)

Slope of OA = m1 =
8

Slope of OB = m2 =
2

m1 −m2
tan θ = ∣ ∣
∣ 1+m1 m2 ∣
6

5 30
tan θ = 16
=
41
1+
25

30
tan θ =
41

62. (A)
Equation of normal to parabola
3
y = mx − 2m − m

this normal passing through center of circle (2, 8)


3
8 = 2m − 2m − m

m = −2

So point P on parabola ⇒ 2
(am , −2am) = (4, 4)

And C = (2, 8)
− −−−− − −
P C = √ 4 + 16 = √ 20
2
d = 20
63. (D)
10

∑ a k = 50
k=1

a 1 + a 2 +. . . +a 10 = 50 . . . . (i)

∑ a k a j = 1100 . . . (ii)
∀ k<j

If a 1 + a 2 +. . . . +a 10 = 50
2
(a 1 + a 2 +. . . +a 10 ) = 2500
10
2
⇒ ∑ ai + 2 ∑ a k a j = 2500
i=1 k<j

10
2
⇒ ∑ ai = 2500 − 2 (1100)
i=1

10

∑ ai
2
= 300, standard deviation ‘σ’
i=1
−−−−−−−−−−−−−
2 −−−−−−−−−−
∑ ai
2
∑ ai 2
= √
10
− (
10
) = √
300
− (
10
50

10
) .
− −−−−− –
= √ 30 − 25 = √ 5

64. (C)
x−4 y+1 z
= =
1 2 −3

x−λ y+1 z−2


= =
2 4 −5

The shortest distance between the lines


⃗ ⃗ ⃗
∣ ⃗
(a− b)⋅(d 1 ×d 2 ) ∣
= ∣ ∣
∣ ⃗ ⃗ ∣
d 1 ×d 2
∣ ∣ ∣ ∣

∣ ∣ λ−4 0 2 ∣ ∣
∣ ∣
∣ ∣ 1 2 −3 ∣

∣ ∣
∣ ∣
∣ 2 4 −5 ∣
= ∣ ∣
∣ ^ ^ ^ ∣
i j k
∣ ∣ ∣ ∣
∣ 1 −3 ∣
∣ 2 ∣
∣ ∣
∣ ∣ 2 4 −5 ∣ ∣

∣ (λ−4)(−10+12)−0+2(4−4)

= ∣ ∣
∣ ^ ^ ^∣
2 i −1 j +0k
∣ ∣ ∣ ∣
2(λ−4)
6 ∣ ∣
= =
√5 ∣ √5 ∣

= 3 = |λ − 4|

λ − 4 = ±3

λ = 7, 1

Sum of all possible values of λ is = 8


65. (D)
Equation of chord with given middle point.
T = S1
x y 1 1
+ = +
25 40 25 100
8x+5y 8+2
=
200 200
10−8x
y = . . . . (i)
5
2
2 (10−8x )
x

25
+
400
= 1 (put in original equation)
2 2
16x +100+64x −160x
= 1
400
2
4x − 8x − 15 = 0
8±√ 304
x =
8
8+√ 304 8−√ 304
x1 = ; x2 =
8 8

Similarly, y
10−18±√ 304 2±√ 304
= =
5 5
2−√ 304 2+√ 304
y1 = ; y2 =
5 5
−−−−−−−−−−−−−−−−−−
Distance =
2 2
√(x1 − x2 ) + (y1 − y2 )
−−−−−−−−−−−
4×304 4×304 √ 1691
= √ + =
64 25 5

66. (A)
Let S = {1, 2, 3, . . . , 10}

R = {(A, B) : A ∩ B = ϕ; A, B ∈ M }

For Reflexive,
M is subset of ‘S’
So ϕ ∈ M
for ϕ ∩ ϕ = ϕ
⇒ but relation is A ⊂ B = ϕ

So it is not reflexive.
For symmetric,
ARB A ∩ B = ϕ,

⇒ BRA ⇒ B ∩ A ≠ ϕ,

So it is symmetric.
For transitive,
If A = {(1, 2) , (2, 3)}
B = {(2, 3) , (3, 4)}

C = {(3, 4) , (5, 6)}

ARB & BRC but A does not relate to C


So it not transitive
67. (A)
⃗ ⃗
a ⃗ ⋅ [(c ⃗ × b) − b − c ]

⃗ ⃗
a ⃗ ⋅ (c ⃗ × b) − a ⃗ ⋅ b − a ⃗ ⋅ c ⃗ . . . . (i)

Given a⃗ × c ⃗ = b

⃗ ⋅ b ⃗ = b.⃗ b ⃗ = ∣ b∣⃗
⇒ (a⃗ × c ) = 27
∣ ∣


⇒ a ⃗ ⋅ (c ⃗ × b) = [ a⃗ ⃗ ⃗ ⋅ b ⃗ = 27
c⃗ b ] = (a ⃗ × c ) . . . (ii)

Now a⃗ ⋅ b ⃗ = 3 − 6 + 3 = 0 . . . . (iii)

a⃗ ⋅ c ⃗ = 3 . . . . (iv) (given)
By (i), (ii), (iii) & (iv)
27 − 0 − 3 = 24

68. (D)
Sum of coefficients in the expansion of
n
2
(1 − 3x + 10x ) = A

then A = (1 − 3 + 10) = 8 (put x = n n


1)

and sum of coefficients in the expansion of


n
2
(1 + x ) = B

then B
n n
= (1 + 1) = 2
3
A = B

69. (B)
(2k, 3k) will lie on circle whose diameter is AB.

(x − 1) (x) + (y − 1) (y) = 0
2 2
x + y − x − y = 0 . . . (i)

Satisfy (2k, 3k) in (i)


2 2
(2k) + (3k) − 2k − 3k = 0
2
13k − 5k = 0
5
k = 0, k =
13

Hence k =
5

13

70. (D)
|z − i| = |z + i| = |z − 1|

ABC is a triangle. Hence its circum-centre will be the only point whose distance from A, B, C will be same.
So n (S ) = 1
71. (D)
cos x − sin x 0
⎡ ⎤
f (x) = ⎢ sin x cos x 0⎥
⎣ ⎦
0 0 1

cos x sin x 0
⎡ ⎤
f (−x) = ⎢ − sin x cos x 0⎥
⎣ ⎦
0 0 1

1 0 0
⎡ ⎤
f (x) ⋅ f (−x) = ⎢ 0 1 0 ⎥ = I
⎣ ⎦
0 0 1

Hence statement- I is correct


Now, checking statement II
⎡ cos y − sin y 0⎤

f (y) = ⎢ sin y cos y 0⎥


⎣ ⎦
0 0 1

cos(x + y) − sin(x + y) 0
⎡ ⎤

f (x) ⋅ f (y) = ⎢ sin(x + y) cos(x + y) 0⎥


⎣ ⎦
0 0 1

⇒ f (x) ⋅ f (y) = f (x + y)

Hence statement-II is also correct.

72. (A)
B = (2λ + 7, −3λ − 2, 6λ + 11)

y−4
Point B lies on
x−6 z−8
= =
1 0 3
2λ+7−6 −3λ−2−4 6λ+11−8
= =
1 0 3

−3λ − 6 = 0

λ = −2

B ⇒ (3, 4, −1)
−−−−−−−−−−−−−−−−−−−−−−−−
2 2 2
AB = √(7 − 3) + (4 + 2) + (11 + 1)

− −−−−−−−−−−
= √ 16 + 36 + 144
− −−
= √ 196 = 14
73. (C)
√ 1+√ 1+x 4−√ 2

a = lim
4
x
x→0

√ 1+x 4−1
= lim
x→0 4
x (√ 1+√ 1+x 4+√ 2 )

4
x
= lim
x→0 4
x (√ 1+√ 1+x 4+√ 2 )(√ 1+x 4+1)

Applying limit a =
4√ 2
1

2
sin x
b = lim
√ 2 −√ 1+cos x
x→0
2
(1−cos x)(√ 2 +√ 1+cos x)
= lim
2−(1+cos x)
x→0
– − −−−−−−
b = lim (1 + cos x) (√ 2 + √ 1 + cos x )
x→0
– – –
Applying limits b = 2 (√ 2 + √ 2) = 4√ 2

– 3
Now, ab 3
=
1
× (4√ 2) = 32
4√ 2

74. (B)
1 1
√ 3+x−√ 1+x
1
∫ dx = ∫ dx
√ 3+x+√ 1+x (3+x)−(1−x)
0 0

1 1
1 − −−−− − −−−−
[∫ √ 3 + x dx − ∫ (√ 1 + x ) dx]
2
0 0

3 3 1

(3+x) 2 2(1+x) 2
1
[2 − ]
2 3 3

0
3
1 2 – 2
[ (8 − 3√ 3) − (2 2 − 1)]
2 3 3

1 – –
= [8 − 3√ 3 − 2√ 2 + 1]
3
– 2 – – –
= 3 − √3 − √ 2 = a + b √ 2 + c√ 3
3
2
a = 3, b = − , c = −1
3

2a + 3b − 4c = 6 − 2 + 4 = 8
75. (C)
dx
+ ax = 0
dt
dx
= −a dt
x
dx
∫ = −a ∫ dt
x

ln|x| = −at + c

At t = 0, x = 2

ln 2 = 0 + c

ln x = −at + ln 2
x −at
= e
2
−at
x = 2e . . . . (i)
dy
+ by = 0
dt

dy
= −b dt
y

ln|y| = −bt + λ

t = 0, y = 1

0 = 0 + λ
−bt
y = e . . . . (ii)

According to question
3y (1) = 2x (1)
−b −a
3e = 2 (2e )
a−b 4
e =
3

For x (t) = y (t)


−at −bt
⇒ 2e = e
(a−b)t
2 = e
t
4
2 = ( )
3

log 4 2 = t
3

76. (D)
2
a(7x−12−x )
f (3

) =
2
(for f (x) to be cont.)
b|x −7x+12|

−a (x−3)(x−4) −a

⇒ f (3 ) = ; x < 3 ⇒
b (x−2)(x−4) b

Hencef (3 − −a
) =
b
sin(x−3)
lim ( )

Then f (3 and
+ + x−3
) = 2 x→3
= 2

f (3) = b

Hence f (3) = f (3
+
) = f (3

)
a
⇒ b = 2 = −
b

b = 2, a = −4

Hence only 1 ordered pair (–4, 2).


77. (D)
4, 9, 14, 19, …., up to 25th term
T 25 = 4 + (25 − 1) 5 = 4 + 120 = 124

3, 6, 9, 12, …, up to 37th term


T 37 = 3 + (37 − 1) 3 = 3 + 108 = 111

Common difference of Ist series d = 5 1

Common difference of IInd series d = 3 2

First common term = 9, and


their common difference = 15 (LCM of d and d ) 1 2

then common terms are


9, 24, 39, 54, 69, 84, 99

78. (B)
f : N − {1} → N

f (n) = The highest prime factor of n .


f (2) = 2

f (4) = 2

⇒ many one
4 is not image of any element
⇒ into

Hence many one and into


Neither one-one nor onto.

79. (B)
Equation of CE
y − 1 = − (x − 3)

x + y = 4

orthocentre lies on the line x + y = 4

So, a + b = 4
a

I1 = ∫ x sin(x (4 − x))dx . . . . (i)


b

Using king rule


b

I1 = ∫ (4 − x) sin(x (4 − x))dx . . . (ii)


a

(i) + (ii)
b

2I1 = ∫ 4 sin(x (4 − x))dx


a

2I1 = 4I2

I1 = 2I2
I1
= 2
I2
36I1
= 72
I2
80. (A)
n−1 2 n
C r = (k − 8) C r+1

r + 1 ⩾ 0, r ⩾ 0

r⩾0
n−1
Cr
2
n
= k − 8
Cr+1

r+1 2
= k − 8
n
2
⇒ k − 8 > 0
– –
(k − 2√ 2) (k + 2√ 2) > 0
– –
k ∈ (−∞, −2√ 2) ∪ (2√ 2, ∞) . . . . (i)

r+1
∴ n ⩾ r + 1, ⩽ 1
n
2
⇒ k − 8 ⩽ 1
2
k − 9 ⩽ 0

−3 ⩽ k ⩽ 3 . . . . (ii)

From equation (I) and (II) we get


– –
k ∈ [−3, −3√ 2) ∪ (2√ 2, 3)

81. (29.00)
2x + 3y − 2 = t
dy dt
2 + 3 =
dx dx

4x + 6y − 4 = 2t

4x + 6y − 7 = 2t − 3
dy −(2x+3y−2)
=
dx 4x+6y−7

dt −3t+4t−6 t−6
= =
dx 2t−3 2t−3
2t−3
∫ dt = ∫ dx
t−6

2t−12 9
∫ ( + ) ⋅ dt = x
t−6 t−6

2t + 9 ln(t − 6) = x + c

2 (2x + 3y − 2) + 9 ln(2x + 3y − 8) = x + c

x = 0, y = 3

c = 14

4x + 6y − 4 + 9 ln(2x + 3y − 8) = x + 14

x + 2y + 3 ln(2x + 3y − 8) = 6

α = 1, β = 2, γ = 8

α + 2β + 3γ = 1 + 4 + 24 = 29

82. (202.00)
3 2 ′ ′′ ′′′
f (x) = x + x ⋅ f (1) + x ⋅ f (2) + f (3)
′ 2 ′ ′′
f (x) = 3x + 2xf (1) + f (2)
′′ ′
f (x) = 6x + 2f (1)
′′′
f (x) = 6
′ ′′ ′′′
f (1) = −5, f (2) = 2, f (3) = 6
3 2
f (x) = x + x ⋅ (−5) + x ⋅ (2) + 6
′ 2
f (x) = 3x − 10x + 2

f (10) = 300 − 100 + 2 = 202
83. (48.00)
cos 2x + a sin x = 2a − 7

a (sin x − 2) = 2 (sin x − 2) (sin x + 2)

sin x = 2, a = 2 (sin x + 2)

⇒ a ∈ [2, 6]

p = 2, q = 6
∘ ∘
r = tan 9 + cot 9 − tan 27 − cot 27
1 1
r = −
sin 9⋅cos 9 sin 27⋅cos 27

4 4
= 2[ − ]
√ 5 −1 √ 5 +1

r = 4

p ⋅ q ⋅ r = 2 × 6 × 4 = 48

84. (9.00)
1
p⋅
3 4
8 = 1
+ 2
1− 1
4 (1− )
4

(sum of infinite terms of A.G.P = 1−r


a
+
dr
2
)
(1−r)

4p
⇒ = 4 ⇒ p = 9
9

85. (12.00)
3 4
5 5 1 5 1 5 1
a = P (X = 3) = × × + ( ) ⋅ + ( ) ⋅ +. . .
6 6 6 6 6 6 6
25

216 25 6 25
= 5
= × =
216 1 36
1−
6

5 6
5 1 5 1
P (X ⩾ 6) = ( ) ⩾ + ( ) ⋅ +. . .
6 6 6 6
5
5 1
( ) ⋅
6 6 5
5
= = ( )
5
6
1−
6
5
5
( )
6
25
c = =
3
5 36
( )
6

2 2
5 5
( ) +( )
b+c 6 6

= = 12
a 2
5 1
( ) ⋅
6 6
86. (28.00)
2 0 1
⎡ ⎤
A = ⎢1 1 0 ⎥ , B = [B 1 , B 2 , B 3 ]
⎣ ⎦
1 0 1

x1 x2 x3
⎡ ⎤ ⎡ ⎤ ⎡ ⎤
B 1 = ⎢ y1 ⎥ , B 2 = ⎢ y2 ⎥ , B 3 = ⎢ y3 ⎥

⎣ ⎦ ⎣ ⎦ ⎣ ⎦
z1 z2 z3

2 0 1 x1 1
⎡ ⎤⎡ ⎤ ⎡⎡ ⎤⎤
AB 1 = ⎢ 1 1 0 ⎥ ⎢ y1 ⎥ = ⎢⎢ 0 ⎥⎥
⎣ ⎦⎣ ⎦ ⎣⎣ ⎦⎦
1 0 1 z1 0

x1 = 1, y1 = −1, z 1 = −1

2 0 1 x2 2
⎡ ⎤⎡ ⎤ ⎡ ⎤
AB 2 = ⎢ 1 1 0 ⎥ ⎢ y2 ⎥ = ⎢3⎥
⎣ ⎦⎣ ⎦ ⎣ ⎦
1 0 1 z2 0

x2 = 2, y2 = 1, z 2 = −2

2 0 1 x3 3
⎡ ⎤⎡ ⎤ ⎡ ⎤
AB 3 = ⎢ 1 1 0 ⎥ ⎢ y3 ⎥ = ⎢ 2 ⎥
⎣ ⎦⎣ ⎦ ⎣ ⎦
1 0 1 z3 1

x3 = 2, y3 = 0, z 3 = −1

1 2 2
⎡ ⎤
B = ⎢ −1 1 0 ⎥
⎣ ⎦
−1 −2 −1

α = |B| = 3

β = 1
3 3
α + β = 27 + 1 = 28

87. (2890.00)
f (x) − f (y) ⩾ ln x − ln y + x − y
f (x)−f (y) ln x−ln y
⩾ + 1
x−y x−y

Let x > y
′ − 1
lim f (x ) ⩾ + 1 . . . (1)
x
y→x

Let x < y
′ 1
lim f (x+) ⩽ + 1 . . . . (2)
x
y→x

′ − ′ +
f (x ) = f (x )
′ 1
f (x) = + 1
x

′ 1 2
f ( ) = x + 1
2
x

20 20
2 2
∑ (x + 1) = ∑ x + 20
x=1 x−1

20×21×41
= + 20
6

= 2890
88. (119.00)

1 2

2 2 2
A = ∫ [(8 − 4y ) − (−2y )] dy + ∫ [(8 − 4y ) − (2y − 4)] dy
0 1
3
1
3 3
2y 4y 2
107 m
2
= [8y − ] + [12y − y − ] = =
3 3 12 n
0 1

∴ m + n = 119

89. (5.00)
2 2
x + x + 1 = 0 ⇒ x = ω, ω = α

Let α = ω
Now (1 + α)
7 2
14
= −ω = −ω = 1 + ω

A = 1, B = 1, C = 0

∴ 5 (3A − 2B − c) = 5 (3 − 2 − 0) = 5

90. (5.00)
^ ^ ^ ^ ^ ^
(α i −2 j +2k)⋅(α i +2α j −2k)

cos θ =
√ α2 +4+4√ α2 +4α2 +4

2
α −4α−4
cos θ =
√ α +8√ 5α2 +4
2

2
⇒ α − 4α − 4 > 0
2
⇒ α − 4α + 4 > 8
2
⇒ (α − 2) > 8
– –
⇒ α − 2 > 2√ 2 or α − 2 < −2√ 2
– –
α > 2 + 2√ 2 or α < 2 − 2√ 2

α ∈ (−∞, −0.82) ∪ (4.82, ∞)

Least positive integral value of α ⇒ 5


JEE Main 27-01-2024 (Evening Shift)

Questions

1. The total kinetic energy of 1 mole of oxygen at 27 ∘


C is [Use universal gas constant (R) = 8.31J /mole K

(a) 6232.5 J
(b) 5670.5 J
(c) 5942.0 J
(d) 6845.5 J

2. The equation of state of a real gas is given by (P +


a
) (V − b) = RT , where P, V and T are pressure,
2
V

volume and temperature respectively and R is the universal gas constant. The dimensions of a
2
is similar to
b

that of:
(a) P
(b) R
(c) RT
(d) PV

3. A heavy iron bar of weight 12 kg is having its one end on the ground and the other on the shoulder of a man.
The rod makes an angle 60 with the horizontal, the weight experienced by the man is:

(a) 12 kg
(b) 3 kg

(c) 6√3kg
(d) 6 kg

4. When a polaroid sheet is rotated between two crossed Polaroids then the transmitted light intensity will be
maximum for a rotation of:
(a) 45 ∘

(b) 90 ∘

(c) 60 ∘

(d) 30 ∘

5. Given below are two statement: one is labelled as Assertion (A) and the other is labelled as Reason (R).
Assertion (A): Work done by electric field on moving a positive charge on an equipotential surface is always
zero.
Reason (R): Electric lines of forces are always perpendicular to equipotential surfaces. In the light of the
above statements, choose the most appropriate answer from the options given below:
(a) Both (A) and (R) are correct and (R) is the correct explanation of (A)
(b) (A) is correct but (R) is not correct
(c) (A) is not correct but (R) is correct
(d) Both (A) and (R) are correct but (R) is not the correct explanation of (A)

6. The threshold frequency of a metal with work function 6.63 eV is:


(a) 1.6 × 10 H z
12

(b) 1.6 × 10 H z
15

(c) 16 × 10 H z
12

(d) 16 × 10 H z
15
7. A ball suspended by a thread swings in a vertical plane so that its magnitude of acceleration in the extreme
position and lowest position are equal. The angle (θ) of thread deflection in the extreme position will be:
(a) tan ( )−1 1

2

(b) tan (√2)−1

(c) 2tan
−1
(
√5
1
)

1
(d)
−1
2tan ( )
2

8. A bullet is fired into a fixed target looses one third of its velocity after travelling 4 cm. It penetrates further
m before coming to rest. The value of D is:
−3
D × 10

(a) 3
(b) 2
(c) 4
(d) 32

9. A current of 200μA deflects the coil galvanometer through 60 . The current to cause deflection through
∘ π

10

radian is:
(a) 60μA
(b) 120μA
(c) 180μA
(d) 30μA

10. An object is placed in a medium of refractive index 3. An electromagnetic wave of intensity 6 × 10 W /m 8 2

falls normally on the object and it is absorbed completely. The radiation pressure on the object would be
(speed of light in free space = 3 × 10 m/s):
8

(a) 18N m −2

(b) 2N m −2

(c) 36N m −2

(d) 6N m −2

11. Given below are two statements: one is labelled as Assertion (A) and the other is labelled as Reason (R):
Assertion (A): In Vernier calliper if positive zero error exists, then while taking measurements, the reading
taken will be more than the actual reading.
Reason (R): The zero error in Vernier Calliper might have happened due to manufacturing defect or due to
rough handing.
In the light of the above statements, choose the correct answer from the options given below:
(a) A is true but R is false
(b) Both A and R are correct and R is the correct explanation of A
(c) Both A and R are correct but R is not the correct explanation of A
(d) A is false but R is true

12. During an adiabatic process, the pressure of a gas is found to be proportional to the cube of its absolute
Cp
temperature. The ratio of for the gas is:
Cv

(a) 3

(b) 7

(c) 9

(d) 5

3
13. The atomic mass of C is 12.000000 u and that of C is 13.003354u. The required energy to remove a
6
12
6
13

neutron from C , if mass of neutron is 1.008665u, will be:


6
13

(a) 6.25 MeV


(b) 4.95 MeV
(c) 62.5 MeV
(d) 49.5 MeV

14. Given below are statements:


Statement (I): The limiting force of static friction depends on the area of contact and independent of
materials.
Statement (II): The limiting force of kinetic friction is independent of the area of contact and depends on
materials.
In the light of the above statements, choose the most appropriate answer from the options given below:
(a) Both Statement I and Statement II are correct
(b) Statement I is correct but Statement II is incorrect
(c) Both Statement I and statement II are incorrect
(d) Statement I is incorrect but statement II is correct.

15. The truth table of the given circuit diagram is:

A B Y

0 0 1

(a) 0 1 0

1 0 0

1 1 1

A B Y

0 0 0

(b) 0 1 1

1 0 1

1 1 0

A B Y

0 0 0

(c) 0 1 0

1 0 0

1 1 1

A B Y

0 0 1

(d) 0 1 1

1 0 1

1 1 0
16. Three voltmeters, all having different internal resistances are joined as shown in figure. When some
potential difference is applied across A and B, their readings are V , V and V . Choose the correct option.
1 2 3

(a) V1 + V2 > V3

(b) V1 = V2

(c) V1 + V2 = V3

(d) V1 ≠ V3 − V2

17. Given below are two statements: one is labelled as Assertion (A) and the other is labelled as Reason (R)
Assertion (A): The angular speed of the moon in its orbit about the earth is more than the angular speed of
the earth in its orbit about the sun.
Reason (R): The moon takes lens time taken by the earth to move around the sun
In the light of the above statements, choose the most appropriate answer from the options given below:
(a) Both A and R are correct and R is the correct explanation of A
(b) Both A and R are correct but R is not the correct explanation of A
(c) A is not correct but R is correct
(d) A is correct but R is not correct

18. Primary side of a transformer is connected to 230V, 50 Hz supply. Turns ratio of primary to secondary
winding is 10: 1. Load resistance connected to secondary side is 46Ω. The power consumed in it is:
(a) 11.5 W
(b) 10.0 W
(c) 12.5 W
(d) 12.0 W

19. Wheatstone bridge principle is used to measure the specific resistance (S 1) of given wire, having length L,
2

radius r. If X is the resistance of wire, then specific resistance is; S


1 = X(
πr

L
) . If the length of the wire
gets doubled then the value of specific resistance will be:
S1
(a) 2
S1
(b) 4

(c) S1

(d) 2S1

20. Given below are two statements: One is labelled as Assertion (A) and the other is labelled as Reason (R).
Assertion (A): The property of body, by virtue of which it tends to regain its original shape when the
external force is removed, is Elasticity.
Reason (R): The restoring force depends upon the bonded inter atomic and inter molecular force of solid.
In the light of the above statements, choose the correct answer from the options given below:
(a) A is false but R is true
(b) Both A and R are true but R is not the correct explanation of A
(c) Both A and R are true and R is the correct explanation of A
(d) A is true but R is false
21. A body falling under gravity covers two points A and B separated by 80m in 2s. The distance of upper point
A from the starting point is ____ m (use g = 10ms ). −2

22. The magnetic field at the centre of a wire loop formed by two semicircular wires of radii R = 2π m and 1

= 4π m, carrying current I = 4A as per figure given below is α × 10 T . The value of α is _____.


−7
R 2

(Centre O is common for all segments)

23. Two charges of −4μC and +4μC are placed at the points A (1, 0, 4) m and B (2, −1, 5) m located in an
electric field E ⃗ = 0.20^i V /cm . The magnitude of the torque acting on the dipole is 8√−

α × 10 N m,
−5

where α = ____.

24. The electric potential at the surface of an atomic nucleus (z = 50) of radius 9 × 10 −13
cm is
_____ × 10 V .
6

25. A series LCR circuit with L = 100


mH , C =
10
−3

F and R = 10Ω , is connected across an ac source of


π π

220V, 50 Hz supply. The power factor of the circuit would be _____

26. A ring and a solid sphere roll down the same inclined plane without slipping. They start from rest. The radii
of both bodies are identical and the ratio of their kinetic energies is , where x is _____.
7

27. The reading of pressure metre attached with a closed pipe is 4.5 × 10 N /m . On opening the valve, water
4 2

starts flowing and the reading of pressure metre falls to 2.0 × 10 N /m . The velocity of water is found to
4 2

−−
be √V m/s . The value of V is ____.

28. A closed organ pipe 150cm long gives 7 beats per second with an open organ pipe of length 350cm, both
vibrating in fundamental mode. The velocity of sound is ____ m/s.

29. If Rydberg’s constant is R, the longest wavelength of radiation in Paschen series will be α

7R
, where
α = ____

30. ∘

A parallel beam of monochromatic light of wavelength 5000A is incident normally on a single narrow slit
of width 0.001mm. The light is focused by convex lens on screen, placed on its focal plane. The first
minima will be formed for the angle of diffraction of _____ (degree)
31. Match List - I with List – II

List – I(Reaction) List – II(Reagent)

(I) Na2Cr2O7, H2SO4

(A)

(II) (i) NaOH (ii) CH3Cl

(B)

(III) (i) NaOH, CHCl3 (ii) NaOH (iii) HCl

(C)

(IV) (i) NaOH (ii) CO2 (iii) HCl

(D)

Choose the correct answer from the options given below:


(a) A – II, B – I, C – III, D – IV
(b) A – IV, B – III, C – I, D – II
(c) A – IV, B – I, C – III, D – II
(d) A – II, B – III, C – I, D – IV

32. The molecular formula of second homologue in the homologous series of mono carboxylic acids is
________.
(a) C2H2O2
(b) C2H4O2
(c) C3H6O2
(d) CH2O
33. Identity the incorrect pair from the following:
(a) Photography - AgBr
(b) Wacker process - Pt Cl2
(c) Polythene preparation - TiCl4, Al(CH3)3
(d) Haber process – Iron

34. The incorrect statement regarding conformations of ethane is :


(a) The conformations of ethane are inter-convertible to one-another.
(b) Ethane has infinite number of conformations.
(c) Eclipsed conformation is the most stable conformation.
(d) The dihedral angle in staggered conformation is 60°.

35. The quantity which changes with temperature is:


(a) Molality
(b) Mass percentage
(c) Molarity
(d) Mole fraction

36. The final product A, formed in the following reaction sequence is :

(a)

(b) Ph – CH2 – CH2 – CH2

(c)

(d) Ph – CH2 – CH2 – CH2 – OH


37. Identify B formed in the reaction.
excess NH 3 NaOH

Cl – (CH2)4 – Cl −−−−−−→A −−−→ B + H2O + NaCl

(a)

+ +
(b) ClNH3 – (CH2)4 – NH3Cl–

(c)

(d) H2N – (CH2)4 – NH2

38. Which structure of protein remains intact after coagulation of egg white on boiling?
(a) Quaternary
(b) Tertiary
(c) Secondary
(d) Primary

39. Bond line formula of HOCH(CN)2 is:

(a)

(b)

(c)

(d)
40. Which of the following statements is not correct about rusting of iron?
(a) When pH lies above 9 or 10, rusting of iron does not take place.
(b) Dissolved acidic oxides SO2, NO2 in water act as catalyst in the process of rusting.
(c) Coating of iron surface by tin prevents rusting, even if the tin coating is peeling off.
(d) Rusting of iron is envisaged as setting up of electrochemical cell on the surface of iron object.

41. Identify from the following species in which d2sp3 hybridization is shown by central atom:
(a) [Co(NH3)6]3+
(b) SF6
(c) [Pt(Cl4)]2–
(d) BrF5

42. Choose the correct option having all the elements with d10 electronic configuration from the following:
(a) 28Ni, 24Cr, 26Fe, 29Cu
(b) 29Cu, 30Zn, 48Cd, 47Ag
(c) 27Co, 28Ni, 26Fe, 24Cr
(d) 46Pd, 28Ni, 26Fe, 24Cr

43. Phenolic group can be identified by a positive:


(a) Tollen’s test
(b) Phthalein dye test
(c) Carbylamine test
(d) Lucas test
44. Major product formed in the following reaction is a mixture of:

(a)

(b)

(c)

(d)

45. The order of relative stability of the contributing structure is:

Choose the correct answer from the options given below:


(a) I > II > III
(b) I = II = III
(c) Il > I > III
(d) III > II > I
46. Which of the following cannot function as an oxidising agent?
(a) BrO3–
(b) MnO4–
(c) N3–
(d) SO42–

47. The technique used for purification of steam volatile water immiscible substances is :
(a) distillation
(b) fractional distillation
(c) fractional distillation under reduced pressure
(d) steam distillation

48. Given below are two statements:


Statement (I): Oxygen being the first member of group 16 exhibits only –2 oxidation state.
Statement (II): Down the group 16 stability of +4 oxidation state decreases and +6 oxidation state
increases.
In the light of the above statements, choose the most appropriate answer from the options given below:
(a) Statement I is incorrect but Statement II is correct
(b) Both Statement I and Statement II are incorrect
(c) Both Statement I and Statement II are correct
(d) Statement I is correct but Statement II is incorrect

49. Which among the following halide/s will not show SN1 reaction:
(A) H2C = CH – CH2Cl
(B) CH3 – CH = CH – Cl
(C)

(D)

Choose the most appropriate answer from the options given below :
(a) (B) only
(b) (A) and (B) only
(c) (A), (B) and (D) only
(d) (B) and (C) only
50. Given below are two statements:
Statement (I): In the Lanthanoids, the formation Ce+4 is favoured by its noble gas configuration.
Statement (II): Ce+4 is a strong oxidant reverting to the common + 3 state.
In the light of the above statements, choose the most appropriate answer from the options given below:
(a) Statement I is false but Statement II is true
(b) Statement I is true but Statement II is false
(c) Both Statement I and Statement II are true
(d) Both Statement I and Statement II are false

51. The Spin only magnetic moment value of square planar complex [Pt(NH3)2Cl(NH2CH3)]Cl is _______
B.M. (Nearest integer)
(Given atomic number for Pt = 78)

52. Total number of compounds with Chiral carbon atoms from following is _____

CH3 – CH2 – CH(NO2) – COOH


CH3 – CH2 – CHBr – CH2 – CH3
CH3 – CH(I) – CH2 – NO2
CH3 – CH2 – CH(OH) – CH2OH

53. The hydrogen electrode is dipped in a solution of pH = 3 at 25°C. The potential of the electrode will be -
______ × 10–2 V.
2.303 RT
( = 0.059 V)
F

54. Time required for completion of 99.9% of a First order reaction is ______ times of half life (t1/2) of the
reaction.

55. For a certain thermochemical reaction M → N at T = 400 K, ∆H⊖ = 77.2 kJ mol –1, ∆S = 122 JK–1, log
equilibrium constant (logK) is ______ × 10–1.

56. 9.3 g of aniline is subjected to reaction with excess of acetic anhydride to prepare acetanilide. The mass of
acetanilide produced if the reaction is 100% completed is _______ × 10–1 g.
(Given molar mass in g mol–1 N : 14, O : 16, C : 12, H : 1)

57. Volume of 3M NaOH (formula weight 40 g mol–1) which can be prepared from 84 g of NaOH is ________
× 10–1 dm3.
58. The number of non-polar molecules from the following is ______
HF, H2O, SO2, H2, CO2, CH4, NH3, HCl, CHCl3, BF3

59. Total number of ions from the following with noble gas configuration is ______
Sr2+ (z = 38), Cs+ (z = 55), La2+ (z = 57), Pb2+ (z = 82), Yb2+ (z = 70) and Fe2+ (z = 26)

60. 1 mole of PbS is oxidised by "X" moles of O3 to get "Y" moles of O2. X + Y = ____

61. If lim
3+α sin x+β cos x+log (1−x)
e
=
1
, then 2α − β is equal to :
2 3
3tan x
x→0

(a) 1
(b) 5
(c) 2
(d) 7

62. An urn contains 6 white and 9 black balls. Two successive draws of 4 balls are made without replacement.
The probability, that the first draw gives all white balls and the second draw gives all black balls, is:
(a) 256
5

(b) 715
5

(c) 256
3

(d) 715
3

63. If 2tan 2
θ − 5 sec θ = 1 has exactly 7 solutions in the interval [0, nπ
] , for the least value of n ∈ N, then
2
n


k
k
is equal to:
2
k=1

1
(a)
15
14
(2 − 15)
2
1
(b) 2
15
(2
14
− 14)

1
(c) 13
(2
14
− 15)
2
15
(d) 1 −
13
2

64. Let g (x) = 3f ( ) + f (3 − x) and f


x

3
′′
(x) > 0 for all x . If g is decreasing in (0, α) and
∈ (0, 3)

increasing in (α, 3), then 8α is:


(a) 18
(b) 0
(c) 20
(d) 24

65. The position vectors of the vertices A, B and C of a triangle are 2^i − 3^j + 3k
^
, 2 i + 2 j + 3k and
^ ^ ^

− i + j + 3k respectively. Let l denotes the length of the angle bisector AD of ∠BAC where D is on the
^ ^ ^

line segment BC, then 2l equals: 2

(a) 42
(b) 50
(c) 49
(d) 45
66. π

For 0 < a < 1, the value of the integral ∫ dx

2
1−2a cos x+a
is:
0
π
(a) 1+a
2

π
(b) 1−a
2

(c)
π

2
π−a
2

(d) π+a
π

67. Let the position vectors of the vertices A, B and C of a triangle be 2^i + 2^j + k
^
, i + 2 j + 2k and
^ ^ ^

2 i + j + 2k respectively. Let l , l and l be the lengths of perpendiculars drawn from the orthocenter of
^ ^ ^
1 2 3

the triangle on the sides AB, BC and C A respectively, then l + l + l equals: 1


2
2
2
3
2

(a) 1

(b) 1

(c) 1

(d) 1

68. Let f : R − {
−1
} → R and g : R − {
−5
} → R be defined as f (x) =
2x+3
and g (x) =
|x|+1
. Then,
2 2 2x+1 2x+5

the domain of the function f og is:


(a) R − {− } 5

(b) R − {−
5

2
,−
7

4
}

(c) R − {−
7

4
}

(d) R

69. Let α =
(4!)!
and β =
(5!)!
. Then:
3! 4!
(4!) (5!)

(a) α ∈ N and β ∉ N
(b) α ∉ N and β ∉ N
(c) α ∈ N and β ∈ N
(d) α ∉ N and β ∈ N

70. 8
(x −x )dx
2

The integral ∫ −1 1
is equal to:
12 6 3
(x +3x +1)tan (x + )
x3

∣ ∣
(a) loge ( tan

−1
(x
3
+
1

x
3
) ) + C

1

(b) ∣
loge ( tan
−1 3
(x +
1 ∣
) )
3

+ C
∣ x
3 ∣
3

(c) ∣
loge ( tan
−1 3
(x +
1 ∣
) ) + C
∣ x
3 ∣
1

(d) ∣
loge ( tan
−1 3
(x +
1 ∣
) )
2

+ C
∣ x
3 ∣
71. Let the image of the point (1, 0, 7) in the line x
=
y−1
=
z−2
be the point (α, β, γ) . Then which one of
1 2 3

the following points lies on the line passing through (α, β, γ) and making angles 2π

3
and 3π

4
with y-axis and
z-axis respectively and an acute angle with x -axis?

(a) (3, −4, 3 + 2√2)

(b) (3, 4, 3 − 2√2)

(c) (1, 2, 1 − √2)

(d) (1, −2, 1 + √2)

72. Let A and B be two finite sets with m and n elements respectively. The total number of subsets of the set A
is 56 more than the total number of subsets of B . Then the distance of the point P (m, n) from the point
Q (−2, −3) is:

(a) 8
(b) 4
(c) 6
(d) 10

73. Consider the function f : (0, 2) → R defined by f (x) =


x
+
2

x
and the function g (x) defined by
2

min {f (t)} , 0 < t ⩽ x and 0 < x ⩽ 1


g (x) = {
3
. Then,
+ x, 1 < x < 2
2

(a) g is continuous and differentiable for all x ∈ (0, 2)


(b) g is continuous but not differentiable at x = 1

(c) g is not continuous for all x ∈ (0, 2)

(d) g is neither continuous nor differentiable at x = 1

74. The 20th term from the end of the progression 20, 19 1
, 18
1
, 17
3
, . . . . , −129
1
is:
4 2 4 4

(a) −110
(b) −118
(c) −100
(d) −115

75. Let e be the eccentricity of the hyperbola x


2


y
2

= 1 and e be the eccentricity of of the ellipse


1 2
16 9
2

which passes through the foci of the hyperbola. If e then the length of the
2 y
x
+ 2
= 1, a > b, 1 e2 = 1,
2
a b

chord of the ellipse parallel to the x-axis and passing through (0, 2) is:
(a) 4√–5

(b) 3√5
(c) 8√ 5

(d) 10√ 5

76. Let R be the interior region between the lines 3x − y + 1 = 0 and x + 2y − 5 = 0 containing the origin.
The set of all values of a , for which the points (a , a + 1) lie in R , is:2

(a) (−3, 0) ∪ ( , 1) 2

(b) (−3, −1) ∪ (− , 1) 1

(c) (−3, −1) ∪ ( , 1) 1

(d) (−3, 0) ∪ ( , 1) 1

3
77. Considering only the principal values of inverse trigonometric functions, the number of positive real values
of x satisfying tan (x) + tan (2x) = is −1 −1 π

(a) 1
(b) more than 2
(c) 0
(d) 2

78. If α, β are the roots of the equation, x 2


− x − 1 = 0 and S n = 2023α
n
+ 2024β
n
, then:
(a) S = S + S 12 11 10

(b) S = S + S 11 10 12

(c) 2S = S + S 11 12 10

(d) 2S = S + S 12 11 10

79. If y is the solution curve of the differential equation


= y (x)
2
(x − 4) dy − (y − 3y) dx = 0, x > 2, y (4) =
2
and the slope of the curve is never zero, then the 3

value of y (10) equals:


3

(a) 1

1−(8) 4

3
(b) 1+2√ 2

3
(c) 1−2√ 2

(d) 1

1+(8) 4

80. ∣ 1
3
α +
3

2 2
∣ ∣
The values of α, for which ∣ 1
1
α +
1
∣ = 0, lie in the interval
3 3
∣ ∣
∣ 2α + 3 3α + 1 0 ∣

(a) (−2, 1)

(b) (−3, 0)

(c) (0, 3)
3 3
(d) (−
2
,
2
)

81. x

Let f (x) = ∫ g (t) loge (


1−t

1+t
) dt, where g is a continuous odd function. If
0
π

2
2
then α is equal to _____.
2
x cos x π
∫ (f (x) + ) dx = ( ) − α,
x α
1+e
π

2

82. Let A be a 2 × 2 real matrix and I be the identity matrix of order 2. If the roots of the equation
|A − xI | = 0 be −1 and 3, then the sum of the diagonal elements of the matrix A is _____.
2

83. The lines x−2


=
y
=
z−7
and x+3
=
y+2
=
z+2
intersect at the point P . If the distance of P from the
2 −2 16 4 3 1

line is l, then 14l is equal to _____.


x+1 y−1 z−1 2
= =
2 3 1

84. The coefficient of x 2012


in the expansion of (1 − x) 2008
(1 + x + x )
2
2007
is equal to ____.
85. Let the complex numbers α and 1
lie on the circles |z − z 0|
2
= 4 and |z − z 0|
2
= 16 respectively, where
α

. Then, the value of 100|α| is _____.


2
z0 = 1 + i

86. If the sum of squares of all real values of α , for which the lines 2x − y + 3 = 9, 6x + 3y + 1 = 0 and
αx + 2y − 2 = 0 do not form a triangle is p, then the greatest integer less than or equal to p is ______.

87. If the area of the region {(x, y) : 0 ⩽ y ⩽ min {2x, 6x − x }}


2
is A, then 12A is equal to ____.

88. If the solution curve, of the differential equation dy


=
x+y−2
passing through the point (2, 1) is
dx x−y

then 5β + α is equal to
y−1 1 y−1
−1
tan ( ) − loge (α + ( ) ) = loge |x − 1| ,
x−1 β x−1

89. The mean and standard deviation of 15 observations were found to be 12 and 3 respectively. On rechecking
it was found that an observation was read as 10 in place of 12. If μ and σ denote the mean and variance of
2

the correct observations respectively, then 15 (μ + μ + σ ) is equal to _____.


2 2

90. Consider a circle (x − α) + (y − β) = 50, where α, β > 0 . If the circle touches the line y + x
2 2
= 0 at

the point P , whose distance from the origin is 4√2, then (α + β) is equal to _______. 2
Answer Key

1. A 2. A 3. B 4. A 5. A 6. B
7. D 8. D 9. A 10. D 11. B 12. A
13. B 14. D 15. B 16. C 17. A 18. A
19. C 20. D 21. 45.00 22. 3.00 23. 2.00 24. 8.00
25. 1.00 26. 7.00 27. 50.00 28. 294.00 29. 144.00 30. 30.00
31. B 32. B 33. B 34. C 35. C 36. D
37. D 38. D 39. C 40. C 41. A 42. B
43. B 44. C 45. A 46. C 47. D 48. B
49. A 50. C 51. 0 52. 5 53. 18 54. 10
55. 37 56. 135 57. 7 58. 4 59. 3 60. 8
61. B 62. D 63. C 64. A 65. D 66. B
67. D 68. A 69. C 70. B 71. B 72. D
73. B 74. D 75. D 76. D 77. A 78. A
79. D 80. B 81. 2.00 82. 10.00 83. 108.00 84. 0.00
85. 20.00 86. 32.00 87. 304.00 88. 11.00 89. 2521.00 90. 100.00
Solutions

1. (A)
f
Kinetic energy = 2
nRT

5
= × 1 × 8.31 × 300J
2

6232.5J

2. (A)
a 2
[P ] = [ 2
] ⇒ [a] = [P V ]
V

And [V ] = [b]
2
[a] [P V ]

2
= 2
= [P ]
[b ] [V ]

3. (B)

Torque about O = 0
L ∘
120 ( cos 60 ) − N2 L = 0
2

N2 = 30N

4. (A)
Let I be intensity of unpolarized light incident on first polaroid.
0

st polaroid
I = Intensity of light transmitted from 1
I0
1 =
2

θ be the angle between 1st and 2nd polaroid


ϕ be the angle between 2
nd and 3rd polaroid
st and 3rd polaroid are crossed)
θ + ϕ = 90 (as 1


ϕ = 90 − θ

I2 = Intensity from 2nd polaroid


I0
2 2
I2 = I1 cos θ = cos θ
2

I3 = Intensity from 3rd polaroid


2
I3 = I2 cos ϕ
2 2
I3 = I1 cos θcos ϕ
I0
2 2
I3 = cos θcos ϕ
2

ϕ = 90 − θ
I0 2
2
I3 = cos θsin θ
2
2
I0 2 sin θ cos θ
I3 = [ ]
2 2

I0 2
I3 = sin 2θ
8

I3 will be maximum when sin 2θ = 1



2θ = 90

θ = 45
5. (A)
Electric line of force are always perpendicular to equipotential surface so angle between farce and displacement
will always be 90°. So work done equal to 0.

6. (B)
ϕ 0 = hv 0
−19 −34
6.63 × 1.6 × 10 = 6.63 × 10 v0
−19
1.6×10
v0 =
−34
10
15
v0 = 1.6 × 10 Hz

7. (D)

Loss in kinetic energy = Gain in potential energy


1 2
⇒ mv = mgℓ (1 − cos θ)
2
2
v
⇒ = 2g (1 − cos θ)

Acceleration at lowest point =


2
v

Acceleration at extreme point = g sin θ

Hence,
2
v
= g sin θ

∴ sin θ = 2 (1 − cos θ)
θ 1 −1 1
⇒ tan = ⇒ θ = 2tan ( )
2 2 2

8. (D)
2 2
v − u = 2aS
2
2u 2 −2
( ) = u + 2 (−a) (4 × 10 )
3
2
4u 2 −2
= u − 2a (4 × 10 )
9
2
5u −2
− = −2a (4 × 10 ) . . . . . (1)
9
2
2u
0 = ( ) + 2 (−a) (x)
3
2
4u
− = −2ax . . . . . (2)
9

(1) / (2)
−2
5 4×10
=
4 x
16 −2
x = × 10
5
−2
x = 3.2 × 10 m
−3
x = 32 × 10 m

9. (A)
i ∝ θ (angle of deflection)
i2 θ2 i2 π/10
3
∴ = ⇒ = =
i1 θ1 200μA π/3 10

⇒ i 2 = 60μA
10. (D)
Radiation pressure = I

v
I .μ
=
c
8
6×10 ×3 2
= 8
= 6N /m
3×10

11. (B)
Assertion & Reason both are correct

12. (A)
P ∝ T
3
⇒ PT
−3
= constant
PV
γ
= const
γ
nRT
P( ) = const
P

P
1−γ
T
γ
= const
γ

PT 1−γ
= const
γ
= −3
1−γ

γ = −3 + 3γ

3 = 2γ
3
γ =
2

13. (B)
13 12 1
6C + Energy → 6C , +0 n

Δm = (12.000000 + 1.008665) − 13.003354 = −000531u

∴ Energy required = 0.00531 × 931.5M eV = 4.95M eV

14. (D)
Co-efficient of friction depends on surface in contact So, depends on material of object

15. (B)

¯ ¯
Y = A. B + A . B

This is XOR GATE.

16. (C)
From KVL,
V1 + V2 − V3 = 0 ⇒ V1 + V2 = V3

17. (A)
2π 1
ω = ⇒ ω ∝
T T

T moon = 27 days

T earth = 365 days 4 hour

⇒ ωmoon > ωearth


18. (A)
V1 N1
=
V2 N2
230 10
=
V2 1

V 2 = 23V
2
V2
Power consumed = R
23×23
= = 11.5W
46

19. (C)
As specific resistance does not depends on dimension of wire so, it will not change.

20. (D)
Theory

21. (45.00)

From A → B
1 2
−80 = −v1 t − × 10t
2
1 2
−80 = −2v1 − × 10 × 2
2

−80 = −2v1 − 20

−60 = −2v1

v1 = 30m/s

From O to A
2 2
v = u + 2gS
2
30 = 0 + 2 × (−10) (−S )

900 = 20S

S = 45m
22. (3.00)

μ i μ i
0 π 0 π
( ) ⊗ + ( )⊗
2R 2 2π 2R 1 2π

μ i μ i
0 0
( + )⊗
4R 2 4R 1

−7 −7
4π×10 ×4 4π×10 ×4
+
4×4π 4×2π
−7 −7
= 3 × 10 = α × 10

α = 3

23. (2.00)


τ ⃗ = p⃗ × E


p ⃗ = qℓ

⃗ V V
E = 0.2 = 20
cm m

^ ^ ^
p ⃗ = 4 × ( i − j + k)

^ ^ ^
= (4 i − 4 j + 4k ) μC − m

^ ^ ^ ^ −6
τ ⃗ = (4 i − 4 j + 4k ) × (20 i ) × 10 Nm

^ ^ −5
– −5
= (8k + 8 j) × 10 = 8√ 2 × 10

α = 2

24. (8.00)
kQ
Potential = R
=
k.Ze

R
9 −19
9×10 ×50×1.6×10
= −13 −2
9×10 ×10
6
= 8 × 10 V

25. (1.00)
1 π
Xc = = −3
= 10Ω
ωC 2π×50×10
100 −3
X L = ωL = 2π × 50 × × 10
π

= 10Ω

∵ XC = XL , Hence, circuit is in resonance


∴ power factor = = = 1
R

Z
R

Z
26. (7.00)
In pure rolling work done by friction is zero.
Hence potential energy is converted into kinetic energy. Since initially the ring and the sphere have same
potential energy, finally they will have same kinetic energy too.
∴ Ratio of kinetic energies = 1
7
⇒ = 1 ⇒ x = 7
x

27. (50.00)
Change in pressure = 1

2
ρv
2

4 4 1 3 2
4.5 × 10 − 2.0 × 10 = × 10 × v
2
4 1 3 2
2.5 × 10 = × 10 × v
2
2
v = 50
−−
v = √ 50
−− − −
Velocity of water = √ V = √ 50

V = 50

28. (294.00)

v v
fc = , fo =
4ℓ1 2ℓ2

|fc − fo | = 7
v v
− = 7
4×150 2×350
v v
− = 7
600cm 700cm
v v
− = 7
6m 7m
1
v( ) = 7
42

v = 42 × 7

= 294m/s

29. (144.00)
Longest wavelength corresponds to transition between n = 3 and n = 4
1 2 1 1 2 1 1
= RZ ( − ) = RZ ( − )
λ 2 2 9
3 4 16
2
7RZ
=
9×16

⇒ λ =
144

7R
for Z = 1
∴ α = 144

30. (30.00)
For first minima
a sin θ = λ
−10
λ 5000×10 1
⇒ sin θ = = −6
=
a 2
1×10

⇒ θ = 30
31. (B)
(A) → Kolbe Schmidt Reaction
(B) → Reimer Tiemann Reaction
(C) → Oxidation of phenol to p-benzoquinone
(D) → PhOH + NaOH → H2O + PhO–

32. (B)

33. (B)
The catalyst used in Wacker's process is PdCl2

34. (C)
Eclipsed conformation is the least stable conformation of ethane

35. (C)
Moles of solute
Molarity =
Volume of solution

Since volume depends on temperature, molarity will change upon change in temperature.

36. (D)

37. (D)
38. (D)
Boiling an egg causes denaturation of its protein resulting in loss of its quarternary, tertiary and secondary
structures.

39. (C)
CH (OH) (CN)2 is

40. (C)
As tin coating is peeled off, then iron is exposed to atmosphere.

41. (A)
[Co(NH3)6]3+ – d2sp3 hybridization
BrF5 – sp3d2 hybridization
[Pt(Cl4)]2– – dsp2 hybridization
SF6 – sp3d2 hybridization

42. (B)
[Cr] = [Ar]4s1 3d5
[Cd] = [Kr]5s2 4d10
[Cu] = [Ar]4s1 3d10
[Ag] = [Kr]5s1 4d10
[Zn] = [Ar]4s2 3d10

43. (B)
Carbylamine Test-Identification of primary amines
Lucas Test - Differentiation between 1°, 2° and 3° alcohols
Tollen's Test - Identification of Aldehydes
Phthalein Dye Test - Identification of phenols

44. (C)
45. (A)
I > II > III, since neutral resonating structures are more stable than charged resonating structure. II > III, since
stability of structure with –ve charge on more electronegative atom is higher

46. (C)
In N3– ion 'N' is present in its lowest possible oxidation state, hence it cannot be reduced further because of
which it cannot act as an oxidizing agent.

47. (D)
Steam distillation is used for those liquids which are insoluble in water, containing non-volatile impurities and
are steam volatile.

48. (B)
Statement-I: Oxygen can have oxidation state from –2 to +2, so statement I is incorrect
Statement- II: On moving down the group stability of +4 oxidation state increases whereas stability of +6
oxidation state decreases down the group, according to inert pair effect.
So both statements are wrong.

49. (A)
+

Since CH3 – CH = C H is very unstable, CH3 – CH = CH – Cl cannot give SN1 reaction.

50. (C)
Statement (1) is true, Ce+4 has noble gas electronic configuration.
Statement (2) is also true due to high reduction potential for Ce4+/Ce3+ (+1.74V), and stability of Ce3+, Ce4+
acts as strong oxidizing agent.

51. (0)
Pt2+ (d8)

Pt2+ → dsp2 hybridization and have no unpaired e–s.


∴ Magnetic moment = 0

52. (5)
Chiral carbons are marked by
53. (18)
2H+(aq.) + 2e– → H2(g)
= 0 – 0.059 × 3 = – 0.177 volts. = –17.7 × 10–2 V.

54. (10)
2.303 a 100
( ) log( ) 3
t 99.9% k a−x 100−99.9 log 10 3
= 2.303
= = = = 10
t 1/2 log 2 log 2 0.3
log 2
k

55. (37)
∆Go = ∆Ho – T∆So
= 77.2 × 103 – 400 × 122 = 28400 J
∆Go = –2.303RTlogK
⇒ 28400 = –2.303 × 8.314 × 400 logK
⇒ log K = –3.708 = –37.08 × 10–1

56. (135)

nAcetanilide = nAniline
m 9.3
⇒ =
135 93

⇒ m = 13.5 g

57. (7)
nNaOH (84/40)
−1
M = ⇒ 3 = ⇒ V = 0.7 L = 7 × 10 L
V sol (in L) V

58. (4)
The non-polar molecules are CO2, H2, CH4 and BF3

59. (3)
Noble gas configuration = ns2 np6
[Sr2+] = [Kr]
[Cs+] = [Xe]
[Yb2+] = [Xe] 4f14
[La2+] = [Xe] 5d1
[Pb2+] = [Xe] 4f14 5d10 6s2
[Fe2+] = [Ar] 3d6
60. (8)
PbS + 4O3 → PbSO4 + 4O2
x = 4, y = 4

61. (B)
3+α sin x+β cos x+log (1−x)
e 1
lim =
2 3
3tan x
x→0
3 2 4 2 3
x x x x x
3+α[x− +...]+β[1− + ....]+(−x− − ...)
3! 2! 4! 2 3 1
⇒ lim =
2 3
3tan x
x→0
1 β
2
(3+β)+(α−1)x+(− − )x +...
2 2 2
x 1
⇒ lim × =
2 2 3
3x tan x
x→0
1 β
− −

⇒ β + 3 = 0, α − 1 = 0 and 2

3
2
=
1

⇒ β = −3, α = 1

⇒ 2α − β = 2 + 3 = 5

62. (D)
6 9
C4 C4 3
× =
15 11 715
C4 C4

Hence option (d) is correct.

63. (C)
2
2tan θ − 5 sec θ − 1 = 0
2
⇒ 2sec θ − 5 sec θ − 3 = 0

⇒ (2 sec θ + 1) (sec θ − 3) = 0
1
⇒ sec θ = − ,3
2
1
⇒ cos θ = −2,
3
1
⇒ cos θ =
3

For 7 solutions n = 13
13

So, ∑
k
k
= S (say)
2
k=1

1 2 3 13
S = + 2
+ 3
+. . . . + 13
2 2 2 2
1 1 1 12 13
S = 2
+ 3
+. . . + 13
+ 14
2 2 2 2 2
1
1−
S 1 213 13
⇒ = ⋅ 1
− 14
2 2 2
1−
2
13
2 −1 13
⇒ S = 2. ( 13
) − 13
2 2
64. (A)
g (x) = 3f (
x

3
and f
) + f (3 − x)
′′
(x) > 0 ∀ x ∈ (0, 3)

(x) is increasing function



⇒ f
′ 1 ′ x ′
g (x) = 3 × . f ( ) − f (3 − x)
3 3
′ x ′
= f ( ) − f (3 − x)
3

If g is decreasing in (0, α )

g (x) < 0
′ x ′
f ( ) − f (3 − x) < 0
3
′ x ′
f ( ) < f (3 − x)
3
x
⇒ < 3 − x
3

9
⇒ x <
4

Therefore α =
9

Then 8α = 8 ×
9

4
= 18

65. (D)
AB = 5

AC = 5

∴ D is midpoint of BC
1 3
D( , , 3)
2 2
−−−−−−−−−−−−−−−−−−−−−−−−−−
2 2 2
1 3
∴ l = √(2 − ) + (−3 − ) + (3 − 3)
2 2
−−
45
l = √
2

2
∴ 2l = 45
66. (B)
π
dx
I = ∫ ; 0 < a < 1
2
1−2a cos x+a
0
π
dx
I = ∫
2
1+2a cos x+a
0
π

2 2
2(1+a )
2I = 2 ∫ 2
dx
2 2 2
(1+a ) −4a cos x
0
π

2 2 2
2(1+a ).sec x
⇒ I = ∫ dx
2
2 2 2
(1+a ) .sec x−4a
0
π

2 2 2
2.(1+a ).sec x
⇒ I = ∫ dx
2 2 2
2 2
(1+a ) .tan x+(1−a )
0
π
2
2.sec x
2 dx
1+a2
⇒ I = ∫
2
2 1−a2
0 tan x+( )
1+a2

2 π
⇒ I = [ − 0]
2 2
(1−a )
π
I =
2
1−a

67. (D)
ΔABC is equilateral
Orthocentre and centroid will be same
5 5 5
G( , , )
3 3 3

Mid-point of AB is D ( 3

2
, 2,
3

2
)
−−−−−−−−−−
1 1 1
∴ l1 = √ + +
36 9 36


1
l1 = √ = l2 = l3
6

2 2 2 1
∴ l1 + l2 + l3 =
2
68. (A)
2x+3 1
f (x) = ; x ≠ −
2x+1 2
|x|+1
5
g (x) = , x ≠ −
2x+5 2

Domain of f (g (x))
2g(x)+3
f (g (x)) =
2g(x)+1

|x|+1
x ≠ −
5

2
and 2x+5
≠ −
1

x ∈ R − {−
5

2
} and x ∈ R

∴ Domain will be R − {− 5

2
}

69. (C)
(4!)! (5!)!
α = ,β =
3! 4!
(4!) (5!)

(24)! (120)!
α = ,β =
6 24
(4!) (5!)

Let 24 distinct objects are divided into 6 groups of 4 objects in each group.
No. of ways of formation of group = ∈ N
24!
6
(4!) .6!

Similarly,
Let 120 distinct objects are divided into 24 groups of 5 objects in each group.
(120)!
No. of ways of formation of groups = ∈ N 24
(5!) .24!

70. (B)
8 2
x −x
I = ∫ dx
−1 1
12 6 3
(x +3x +1)tan (x + )
x3

Let tan −1
(x
3
+
1

3
) = t
x

1 2 3
⇒ 2
. (3x − ) dx = dt
4
1 x
3
1+(x + )
x3
6 6
x 3x −3
⇒ . dx = dt
12 6 4
x +3x +1 x
1 dt 1
I = ∫ = ln|t| + C
3 t 3

1 ∣ −1 3 1 ∣
I = ln tan (x + ) + C
3 ∣ x
3 ∣
1

∣ −1 3 1 ∣ 3

I = ln tan (x + ) + C
∣ x
3 ∣

Hence option (b) is correct


71. (B)
x y−1 z−2
L1 = = = = λ
1 2 3

M (λ, 1 + 2λ, 2 + 3λ)


−−→
^ ^ ^
P M = (λ − 1) i + (1 + 2λ) j + (3λ − 5) k
−−→
PM is perpendicular to line L 1

−−→
⃗ ⃗^ ^ ^
PM. b = 0 (b = i + 2 j + 3 k )

⇒ λ − 1 + 4λ + 2 = 9λ − 15 = 0

14λ = 14 ⇒ λ = 1

∴ M = (1, 3, 5)

⃗ ⃗
Q = 2M − P

[M is midpoint of P ⃗ & Q

]
⃗ ^ ^ ^ ^ ^
Q = 2 i + 6 j + 10k − i − 7k

⃗ ^ ^ ^
Q = i + 6 j + 3k

∴ (α, β, γ) = (1, 6, 3)

Required line having direction cosine (l, m, n)


2 2 2
l + m + n = 1
2
2
2 1 1
⇒ l + (− ) + (− ) = 1
2 √2

2 1
l =
4

∴ [Line make acute angle with x-axis]


l =
1

Equation of line passing through (1, 6, 3) will be


^ ^ ^ 1 ^ 1 ^ 1 ^
r ⃗ = ( i + 6 j + 3k ) + μ ( i − j − k)
2 2 √2

Option (b) satisfying for μ = 4

72. (D)
m n
2 − 2 = 56
n m−n 3
2 (2 − 1) = 2 × 7

2
n
and 2
= 2
3
− 1 = 7
m−n

⇒ n = 3 and 2 = 8
m−n

⇒ n = 3 and m − n = 3

⇒ n = 3 and m = 6

P (6, 3) and Q (−2, −3)


−− −−−− − −−
2 2
P Q = √8 + 6 = √ 100 = 10
73. (B)
x 2
f : (0, 2) → R; f (x) = +
2 x
′ 1 2
f (x) = −
2 2
x

∴ f (x) is decreasing in domain

x 2
+ 0 < x ⩽ 1
2 x
g (x) = {
3
+ x 1 < x < 2
2

74. (D)
1 1 3 1
20, 19 , 18 , 17 , . . . . , −129
4 2 4 4

This is A.P. with common difference


1 3
d 1 = −1 + = −
4 4
1 1
−129 , . . . , 19 , 20
4 4

This is also A.P. a = −129


1

4
and d =
3

Required term = −129


1

4
+ (20 − 1) (
3

4
)

1 3
= −129 − + 15 − = −115
4 4
75. (D)
2 2
x y 5
H : − = 1, e1 =
16 9 4
4
∴ e1 e2 = 1 ⇒ e2 =
5

Also, ellipse is passing through (±5, 0)


∴ a = 5 and b = 3
2 2
x y
E : + = 1
25 9

5√ 5
End point of chord are (± 3
, 2)

10√ 5
∴ LP Q =
3

76. (D)
2
P (a , a + 1)

L1 = 3x − y + 1 = 0

Origin and P lies same side w.r.t. L 1

⇒ L1 (0) . L1 (P ) > 0
2
∴ 3 (a ) − (a + 1) + 1 > 0

2
⇒ 3a − a > 0
1
a ∈ (−∞, 0) ∪ ( , ∞) . . . . (1)
3

Let L : x + 2y − 5 = 0
2

Origin and P lies same side w.r.t. L 2

⇒ L2 (0) . L2 (P ) > 0
2
⇒ a + 2 (a + 1) − 5 < 0
2
⇒ a + 2a − 3 < 0

⇒ (a + 3) (a − 1) < 0

∴ a ∈ (−3, 1) . . . (2)

Intersection of (1) and (2)


1
a ∈ (−3, 0) ∪ ( , 1)
3
77. (A)
−1 −1 π
tan x + tan 2x = ; x > 0
4
−1 π −1
⇒ tan 2x = − tan x
4

Taking tan both sides


1−x
⇒ 2x =
1+x
2
⇒ 2x + 3x − 1 = 0
−3±√ 9−8 −3±√ 17
x = =
8 8

Only possible x
−3+√ 17
=
8

78. (A)
2
x − x − 1 = 0
n n
Sn = 2023α + 2024β
n−1 n−1 n−2 n−2
Sn−1 + Sn−2 = 2023α + 2024β + 2023α + 2024β
n−2 n−2
= 2023α [1 + α] + 2024β [1 + β]
n−2 2 n−2 2
= 2023α [α ] + 2024β [β ]
n n
= 2023α + 2024β

Sn−1 + Sn−2 = Sn

Put n = 12

S11 + S10 = S12

79. (D)
2 2
(x − 4) dy − (y − 3y) dx = 0
dy dx
⇒ ∫ = ∫
2 2
y −3y x −4

y−(y−3)
1 dx
⇒ ∫ dy = ∫
3 y(y−3) x −4
2

1 1 x−2
⇒ (ln|y − 3| − ln|y|) = ln∣

∣ + C

3 4 x+2
y−3 x−2
1 ∣ ∣ 1
⇒ ln = ln∣

∣ + C

3 ∣ y ∣ 4 x+2

At x = 4, y =
3

2
1
∴ C = ln 3
4
y−3 x−2
1 ∣ ∣ 1 1
∴ ln = ln∣

∣ +
∣ ln(3)
3 ∣ y ∣ 4 x+2 4

At x = 10
y−3
1 ∣ ∣ 1 2 1
ln = ln∣

∣ +
∣ ln(3)
3 ∣ y ∣ 4 3 4
3
y−3 dy
∣ ∣
ln = ln 2 4 , ∀ x > 2, < 0
∣ y ∣ dx

As y (4) =
3

2
⇒ y ∈ (0, 3)
1

−y + 3 = 8 4 .y
3
y =
1

1+8 4
80. (B)
3 3
∣ 1 α + ∣
2 2
∣ ∣
1 1
∣ 1 α + ∣ = 0
3 3
∣ ∣
∣ 2α + 3 3α + 1 0 ∣

7α −7
⇒ (2α + 3) { } − (3α + 1) { } = 0
6 6

7α 7
⇒ (2α + 3) . + (3α + 1) . = 0
6 6
2
⇒ 2α + 3α + 3α + 1 = 0
2
⇒ 2α + 6α + 1 = 0
−3+√ 7 −3−√ 7
⇒ α = ,
2 2

Hence option (b) is correct.

81. (2.00)
x
1−t
f (x) = ∫ g (t) ln( )dt
1+t
0
−x
1−t
f (−x) = ∫ g (t) ln( )dt
1+t
0
x
1+y
f (−x) = − ∫ g (−y) ln( )dy
1−y
0
x
1−y
= − ∫ g (y) ln(
1+y
)dy (g is odd)
0

f (−x) = −f (x) ⇒ f is also odd


Now,
π

2
2
x cos x
I = ∫ (f (x) + x
) dx . . . . (1)
1+e
π

2

2
2 x
x e cos x
I = ∫ (f (−x) + x
) dx . . . . (2)
1+e
π

2

π π

2 2

2 2
2I = ∫ x cos x dx = 2 ∫ x cos x dx
π
− 0
2

π 2

2 2
I = (x sin x) − ∫ 2x sin x dx
0
0
π
2
π 2
= − 2(−x cos x + ∫ cos x dx)
4 0
2 2 2
π π π
= − 2 (0 + 1) = − 2 ⇒ ( ) − 2
4 4 2

∴ α = 2
82. (10.00)
|A − xI | = 0

Roots are –1 and 3


Sum of roots = tr (A) = 2
Product of roots = |A| = −3
a b
Let A = [ ]
c d

We have a + d = 2

ad − bc = −3
2
a b a b a + bc ab + bd
2
A = [ ] × [ ] = [ ]
2
c d c d ac + cd bc + d

We need a 2
+ bc + bc + d
2

2 2
= a + 2bc + d
2
= (a + d) − 2ad + 2bc

= 4 − 2 (ad − bc)

= 4 − 2 (−3)

= 4 + 6

= 10

83. (108.00)
x−2 y z−7
= = = λ
2 −2 16
x+3 y+2 z+2
= = = k
4 3 1

⇒ λ + 2 = 4k − 3

−λ = 3k − 2

⇒ k = 1, λ = −1

8λ + 7 = k − 2

∴ P = (1, 1, −1)

Projection of 2^i − 2k
^
on 2^i + 3^j + k
^
is
4−2 2
= =
√ 4+9+1 √ 14

2 4 108
∴ l = 8 − =
14 14
2
⇒ 14l = 108
84. (0.00)
2007 2007
2
(1 − x) (1 − x) (1 + x + x )
2007
3
(1 − x) (1 − x )
2007 2007 3
(1 − x) ( C0 − C 1 (x ) +. . .)

General term
r 2007 3r
(1 − x) ((−1) Cr x )
r r
2007 3r 2007 3r+1
(−1) Cr x − (−1) Cr x

3r = 2012
2012
r ≠
3

3r + 1 = 2012

3r = 2011
2011
r ≠
3

Hence there is no term containing x 2012


.
So coefficient of x = 0
2012

85. (20.00)
2
|z − z 0 | = 4

⇒ (α − z 0 ) (ᾱ − z̄ 0 ) = 4
2
⇒ αᾱ − αz̄ 0 − z 0 ᾱ + |z 0 | = 4
2
⇒ |α| − αz̄ 0 − z 0 ᾱ = 2 . . . . (1)
2
|z − z 0 | = 16

1 1
⇒ ( − z0 ) ( − z̄ 0 ) = 16
ᾱ α

2
⇒ (1 − ᾱz 0 ) (1 − ᾱz 0 ) = 16|α|
2 2 2
⇒ 1 − ᾱz 0 − ᾱz 0 + |α| |z 0 | = 16|α|
2
⇒ 1 − ᾱz 0 − αz̄ 0 = 14|α| . . . . (2)

From (1) and (2)


2
⇒ 5|α| = 1
2
⇒ 100|α| = 20

86. (32.00)
2x − y + 3 = 0

6x + 3y + 1 = 0

αx + 2y − 2 = 0

Will not form a Δ if αx + 2y − 2 = 0 is concurrent with 2x − y + 3 = 0 and 6x + 3y + 1 = 0 or parallel


to either of them so
Case-1: Concurrent lines
∣ 2 −1 3 ∣
∣ ∣ 4
6 3 1 = 0 ⇒ α =
∣ ∣ 5

∣ α 2 −2 ∣

Case-2: Parallel lines


or − = 2
α −6 α
− =
2 3 2

⇒ α = 4 or α = −4
16
P = 16 + 16 +
25
16
[P ] = [32 + ] = 32
25
87. (304.00)
We have

6
1 2
A = × 4 × 8 + ∫ (6x − x ) dx
2
4
76
A =
3

12A = 304

88. (11.00)
dy x+y−2
=
dx x−y

x = X + h, y = Y + k
dY X+Y
=
dX X−Y

h + k − 2 = 0
}h = k = 1
h − k = 0

Y = vX
2
dv 1+v dv 1+v
v + = ⇒ X − =
dX 1−v dX 1−v
1−v dX
dv =
2 X
1+v
−1 1 2
tan v − ln(1 + v ) = ln|X| + C
2

As curve is passing through (2, 1)


2
y−1 1 y−1
−1
tan ( ) − ln(1 + ( ) ) = ln|x − 1|
x−1 2 x−1

∴ α = 1 and β = 2

⇒ 5β + α = 11
89. (2521.00)
Let the incorrect mean be μ and standard deviation be σ ′ ′

We have
∑ xi

μ = = 12 ⇒ ∑ xi = 180
15

As per given information correct ∑ x i = 180 − 10 + 12

⇒ μ (correct mean) =
182

15

Also
−−−−
2
−−−−−
∑ xi
′ 2
σ = √ − 144 = 3 ⇒ ∑ xi = 2295
15

Correct ∑ x = 2295 − 100 + 144


i
2
= 2339

σ (correct variance) =
2 2339 182×182

15 15×15

Required value
2 2
= 15 (μ + μ + σ )

182 182×182 2339 182×182


= 15 ( + + − )
15 15×15 15 15×15
182 2339
= 15 ( + )
15 15

= 2521

90. (100.00)

2 2
S : (x − α) + (y − β) = 50

CP = r

∣ α+β ∣ –
= 5√ 2
∣ √2 ∣
2
⇒ (α + β) = 100
JEE Main 29-01-2024 (Morning Shift)

Questions

1. A biconvex lens of refractive index 1.5 has a focal length of 20 cm in air. Its focal length when immersed in a
liquid of refractive index 1.6 will be:
(a) – 160 cm
(b) + 160 cm
(c) + 16 cm
(d) – 16 cm

2. Two charges of 5Q and -2Q are situated at the points (3a, 0) and (−5a, 0) respectively. The electric flux
through a sphere of radius ‘4a’ having center at origin is:
3Q
(a) ε0

2Q
(b) ε0

5Q
(c) ε0

7Q
(d) ε0

3. Two vessels A and B are of the same size and are at same temperature. A contains 1g of hydrogen and B
contains 1g of oxygen. P and P are the pressures of the gases in A and B respectively, then is:
PA
A B
PB

(a) 8
(b) 4
(c) 32
(d) 16

4. The deflection in moving coil galvanometer falls from 25 divisions to 5 division when a shunt of 24Ω is
applied. The resistance of galvanometer coil will be:
(a) 100Ω
(b) 48Ω
(c) 96Ω
(d) 12Ω

5. The potential energy function (in J) of a particle in a region of space is given as U = (2x + 3y + 2z) .
2 3

Here x, y and z are in meter. The magnitude of x-component of force (in N) acting on the particle at point
P(1, 2, 3)m is:
(a) 6
(b) 4
(c) 8
(d) 2
6. Given below are two statements:
Statement I: If a capillary tube is immersed first in cold water and then in hot water, the height of capillary
rise will be smaller in hot water
Statement II: If a capillary tube is immersed first in cold water and then in hot water, the height of capillary
rise will be smaller in cold water.
In the light of the above statements, choose the most appropriate from the options given below.
(a) Statement I is false but Statement II is true
(b) Both Statement I and Statement II are false
(c) Statement I is true but Statement II is false
(d) Both Statement I and Statement II are true

7. The explosive in Hydrogen bomb is a mixture of 1H


2
,1 H
3
and 3 Li
6
in some condensed form. The chain
reaction is given by
6 1 4 3
3 Li + 0n → 2He + 1H
2 3 4 1
1H + 1H → 2He + 0n

During the explosion the energy released is approximately


[Given: M (Li) = 6.01690amu, M ( H ) = 201471amu, M (1
2
2He
4
) = 4.00388amu and
1amu = 931.5M eV ]

(a) 28.12 MeV


(b) 12.64 MeV
(c) 16.48 MeV
(d) 22.22 MeV

8. If the radius of curvature of the path of two particles of same mass are in the ratio 3 : 4, then in order to have
constant centripetal force, their velocities will be in the ration of:
(a) 1 : √–3

(b) 2 : √3

(c) √3 : 1

(d) √3 : 2

9. A capacitor of capacitance 100μF is charged to a potential of 12V and connected to a 6.4 mH inductor to
produce oscillations. The maximum current in the circuit would be:
(a) 3.2 A
(b) 2.0 A
(c) 1.5 A
(d) 1.2 A

10. A galvanometer having coil resistance 10Ωshows a full scale deflection for a current of 3mA. For it to
measure a current of 8A, the value of the shunt should be:
(a) 4.85 × 10 Ω −3

(b) 3 × 10 Ω −3

(c) 3.75 × 10 Ω −3

(d) 2.75 × 10 Ω −3
11. In the given circuit, the breakdown voltage of the Zener diode is 3.0 V. What is the value of I z?

(a) 7 mA
(b) 3.3 mA
(c) 5.5 mA
(d) 10 mA

12. The resistance R = where V


V
= (200 ± 5) V and I = (20 ± 0.2) A , the percentage error in the
I

measurement of R is:
(a) 3.5%
(b) 5.5%
(c) 3%
(d) 7%

13. The de-Broglie wavelength of an electron is the same as that of a photon. If velocity of electron is 25% of
the velocity of light, then the ratio of K.E. of electron and K.E. of photon will be:
(a) 8

1
1
(b) 1

(c) 1

8
1
(d) 4

14. A block of mass 100kg slides over a distance of 10m on a horizontal surface. If the co-efficient of friction
between the surfaces is 0.4, then the work done against friction (in J) is:
(a) 3900
(b) 4200
(c) 4000
(d) 4500

15. At what distance above and below the surface of the earth a body will have same weight. (take radius of
earth as R).
(a) √ 5 R−R

(b) √– 5R − R

(c) √ 3 R−R

(d) R

16. A body starts moving from rest with constant acceleration covers displacement S in first (p − 1) seconds
1

and S in first p seconds. The displacement S + S will be made in time:


2 1 2

(a) (2p + 1) s
(b) (2p − 1) s
(c) (2p − 2p + 1) s
2

−−−− −− −− −− −−
(d) √(2p − 2p + 1) s
2
17. The electric current through a wire varies with time as I = I + βt , where I
0 0 = 20A and β = 3A/s . The
amount of electric charge crossed through a section of the wire in 20s is:
(a) 1000 C
(b) 1600 C
(c) 80 C
(d) 800 C

18. Match List I with List II

List I List II
→ →
A. ∮ B . dl = μo i c + μo ε o

E
I. Gauss’s law for electricity
dt

→ →
B. ∮ E . dl =

B
II. Gauss’ law for magnetism
dt

→ →
C. ∮ E . dl =
Q
III. Faraday law
ε0

→ −→
D. ∮ B . dA = 0 IV Ampere – Maxwell law

Choose the correct answer from the options given below:


(a) A – I, B – II, C – III, D - IV
(b) A – II, B – III, C – I, D – IV
(c) A – IV, B – I, C – III, D – II
(d) A – IV, B – III, C – I, D – II

19. A thermodynamic system is taken from an original state A to an intermediate state B by a linear process as
shown in the figure. It’s volume is then reduced to the original value from B to C by an isobaric process.
The total work done by the gas from A to B and B to C would be:

(a) 2200 J
(b) 33800 J
(c) 600 J
(d) 1200 J

20. A convex mirror of radius of curvature 30 cm forms an image that is half the size of the object. The object
distance is:
(a) -15 cm
(b) 15 cm
(c) 45 cm
(d) -45 cm
21. A ball rolls off the top of a stairway with horizontal velocity u. The steps are 0.1 m high and 0.1 m wide.
The minimum velocity u with which that ball just hits the step 5 of the stairway will be √− −
x ms where −1

2
x = _____ [use g = 10m/s ] .

22. An electron is moving under the influence of the electric field of a uniformly charged infinite plane sheet S
having surface charge density +σ. The electron at t = 0 is at a distance of 1m from S and has a speed of 1
m∈ 0
m/s. The maximum value of σif the electron strikes S at t = 1s is α [ e
]
m
C

2
, the value of α is _____.

23. In a test experiment on a model aeroplane in wind tunnel, the flow speeds on the upper and lower surfaces
of the wings are 70ms and 65ms respectively. If the wing area is 2m , the lift of the wing is ____ N.
−1 −1 2

(Given density of air = 1.2kgm ) −3

24. The magnetic potential due to a magnetic dipole at a point on its axis situated at a distance of 20 cm from its
μ
center is 1.5 × 10 T m. The magnetic moment of the dipole is _____ Am . (Given:
−5 2
= 10 T mA ) 4π
o −7 −1

25. A 16Ω wire is bend to form a square loop. A 9V battery with internal resistance 1Ω is connected across one
of its sides. If a 4μF capacitor is connected across one of its diagonals, the energy stored by the capacitor
will be μJ , where x = _____.
x

26. When a hydrogen atom going from n = 2 to n = 1 emits a photon, its recoil speed is x

5
m/s . Where x =
_____. (Use, mass of hydrogen atom = −27
1.6 × 10 kg )

27. When the displacement of a simple harmonic oscillator is one third of its amplitude, the ratio of total energy
to kinetic energy is , where x = _____.
x

28. A square loop of side 10cm and resistance 0.7Ω is placed vertically in east-west plane. A uniform magnetic
field of 0.20T is set up across the plane in north east direction. The magnetic field is decreased to zero in 1s
at a steady rate. Then, magnitude of induced emf is √− −
x × 10 V . The value of x is _____
−3

29. In a double slit experiment shown in figure, when light of wavelength 400nm is used, dark fringe is
observed at P. If D = 0.2 m, the minimum distance between the slits S and S is _____ m
1 2

30. A cylinder is rolling down on an inclined plane of inclination 60 . It’s acceleration during rolling down will

be x
m/s , where x = _____ (use g = 10m/s ).
2 2

√3
31. Appearance of blood red colour, on treatment of the sodium fusion extract of an organic compound with
FeSO4 in presence of concentrated H2SO4 indicates the presence of element/s
(a) Br
(b) S
(c) N and S
(d) N

32. The arenium ion which is not involved in the bromination of Aniline is ______

(a)

(b)

(c)

(d)
33. The final product A farmed in the following multistep reaction sequence is

(a)

(b)

(c)

(d)

34. The correct set of four quantum numbers for the valence electron of rubidium atom (Z = 37) is:
(a) 5, 0, 0, +
1

(b) 5, 1, 0, +
1

(c) 5, 0, 1, +
1

(d) 5, 1, 1, +
1

35. The difference in energy between the actual structure and the lowest energy resonance structure for the
given compound is
(a) ionization energy
(b) electromeric energy
(c) hyperconjugation energy
(d) resonance energy
36. Given below are two statements : one is labelled as Assertion A and the other is labelled as Reason R :
Assertion A: The first ionisation enthalpy decreases across a period.
Reason R: The increasing nuclear charge outweighs the shielding across the period.
In the light of the above statements, choose the most appropriate from the options given below :
(a) A is true but R is false
(b) A is false but R is true
(c) Both A and R are true and R is the correct explanation of A
(d) Both A and R are true but R is not the correct explanation of A

37. The major product(P) in the following reaction is

(a)

(b)

(c)

(d)
38. Chlorine. undergoes disproportionation in alkaline medium as shown below:
a Cl2(g) + b OH–(aq) → c ClO–(aq) + d Cl–(aq) + e H2O(l)
The values of a, b, c and d in a balanced redox reaction are respectively :
(a) 2, 4, 1 and 3
(b) 2, 2, 1 and 3
(c) 1, 2, 1 and 1
(d) 3, 4, 4 and 2

39. Which of the following is not correct?


(a) ∆G is positive for a non-spontaneous reaction
(b) ∆G is positive for a spontaneous reaction
(c) ∆G is negative for a spontaneous reaction
(d) ∆G is zero for a reversible reaction

40. Type of amino acids obtained by hydrolysis of proteins is :


(a) α
(b) β
(c) δ
(d) 𝛾

41. KMnO4 decomposes on heating at 513K to form O2 along with


(a) MnO2 & K2O2
(b) K2MnO4 & Mn
(c) K2MnO4 & MnO2
(d) Mn & KO2

42. Given below are two statements:


Statement I: The electronegativity of group 14 elements from Si to Pb, gradually decreases.
Statement II: Group 14 contains non-metallic, metallic, as well as metalloid elements.
In the light of the above statements, choose the most appropriate from the options given below:
(a) Both Statement I and Statement II are true
(b) Statement I is true but Statement II is false
(c) Statement I is false but Statement II is true
(d) Both Statement I and Statement II are false

43. In chromyl chloride test for confirmation of Cl– ion. a yellow solution is obtained. Acidification of the
solution and addition of amyl alcohol and 10% H2O tums organic layer blue indicating formation of
chromium pentoxide. The oxidation state of chromium in that is
(a) +5
(b) +3
(c) +6
(d) +10

44. In which one of the following metal carbonyls, CO forms a bridge between metal atoms?
(a) [Ru3(CO)12]
(b) [Os3(CO)12]
(c) [Co2(CO)8]
(d) [Mn2(CO)10]
45. Identify the incorrect pair from the following:
(a) Cryolite – Na3AlF6
(b) Fluorspar – BF3
(c) Carnallite – KCl · MgCl2 · 6H2O
(d) Fluoroapatite – 3 Ca3(PO4)2 · CaF2

46. Given below are two statements : one is labelled as Assertion A and the other is labelled as Reason R :
Assertion A: Aryl halides cannot be prepared by replacement of hydroxyl group of phenol by halogen atom.
Reason R: Phenols react with halogen acids violently.
In the light of the above statements. choose the most appropriate from the options given below:
(a) A is false but R is true
(b) Both A and R are true but R is not the correct explanation of A
(c) Both A and R are true and R is the correct explanation of A
(d) A is true but R is false

47. The interaction between π bond and lone pair of electrons present on an adjacent atom is responsible for
(a) Electromeric effect
(b) Hyperconjugation
(c) Inductive effect
(d) Resonance effect

48. In alkaline medium. MnO oxidises I– to


(a) IO–
(b) IO −

(c) I2
(d) IO −

4
49. Identify product A and product B:

(a)

(b)

(c)

(d)
50. Match List I with List II

LIST I (Substances) LIST II (Element Present)


A. Ziegler catalyst I. Rhodium
B. Blood Pigment II. Cobalt
C. Wilkinson catalyst III. Iron
D. Vitamin B12 IV. Titanium

Choose the correct answer from the options given below:


(a) A-III, B-II, C-IV, D-I
(b) A-II, B-III, C-IV, D-I
(c) A-II, B-IV, C-I, D-III
(d) A-IV, B-III, C-I, D-II

51.

Consider the given reaction. The total number of oxygen atom/s present per molecule of the product (P) is
____

52. The number of species from the following which are paramagnetic and with bond order equal to one is
_____
H2, He , O , N , O , F2, Ne , B2
+

2
+

2
2−

2
2−

2
+

53. Number of compounds among the following which contain sulphur as heteroatom is ______
Furan, Thiophene, Pyridine, Pyrrole, Cysteine, Tyrosine

54. Number of compounds with one lone pair of electrons on central atom amongst following is ______
O3, H2O, SF4, ClF3, NH3, BrF5, XeF4

55. The osmotic pressure of a dilute solution is 7 × 105 Pa at 273K. Osmotic pressure of the same solution at
283 K is _____ × 104 Nm–2.

56. The mass of zinc produced by the electrolysis of zinc sulphate solution with a steady current of 0.015 A for
15 minutes is ___ × 10–4 g.
(Atomic mass of zinc = 65.4 amu)

57. For a reaction taking place in three steps at same temperature, overall rate constant K =
K1 K2
. If Ea1, Ea2
K3

and Ea3 are 40, 50 and 60 kJ/mol respectively, the overall Ea is _______ kJ/mol.
58. A solution of H2SO4 is 31.4% H2SO4 by mass and has a density of 1.25g/mL.
The molarity of the H2SO4 solution is ___ M (nearest integer)
[Given molar mass of H2SO4 = 98g mol–1]

59. From the compounds given below, number of compounds which give positive Fehling's test is _____
Benzaldehyde, Acetaldehyde, Acetone, Acetophenone, Methanal, 4-nitrobenzalclehyde, cyclohexane
carbaldehyde.

60. For the reaction N2O4(g) ⇌ 2NO2(g), Kp = 0.492 attn at 300K. Kc for the reaction at same temperature is
___ × 10–2.
(Given: R = 0.082 L atm mol–1 K–1)

61. A function y = f (x) satisfies f (x) sin 2x + sin x − (1 + cos 2


x) f

(x) = 0 with condition f (0) .
= 0

Then, f ( ) is equal to
π

(a) −1
(b) 1
(c) 0
(d) 2

62. Let a,⃗ b ⃗ and c ⃗ be three non-zero vectors such that b ⃗ and c ⃗ are non-collinear. If a⃗ + 5b ⃗ is collinear with
c ,⃗ b + 6c ⃗ is collinear with a ⃗ and a ⃗ + αb + βc ⃗ = 0 , then α + β is equal to
⃗ ⃗

(a) 35
(b) −30
(c) 30
(d) 25

63. In an A.P., the sixth term a 6 = 2 . If the product a 1 a4 a5 is the greatest, then the common difference of the
A.P. is equal to
3
(a) 2

(b) 8

5
2
(c) 3

(d) 5

64. π

2
2

If the value of the integral then the value of a is


2
x cos x 1+sin x π
∫ ( + ) dx = (π + a) − 2,
1+π
2 sin x 2023 4
π 1+e

2

(a) 3
(b) 2
(c) − 3

(d) 3

2
65. 2 + 2x, −1 ⩽ x < 0 −x, −3 ⩽ x ⩽ 0
If f (x) = { x
; g (x) = { , then range of (f og) (x) is
1 − , 0 ⩽ x ⩽ 3 x, 0 < x ⩽ 1
3

(a) [0, 1)

(b) [0, 1]

(c) [0, 3)

(d) (0, 1]

66. Let P QR be a triangle with R (−1, 4, 2). Suppose M (2, 1, 2) is the mid point of P Q . The distance of the
y y+3
centroid of ΔP QR from the point of intersection of the lines and is
x−2 z+3 x−1 z+1
= = = =
0 2 −1 1 −3 1

(a) 69
−−
(b) √99
(c) 9
−−
(d) √69

67. Consider the function f : [ , 1] → R defined by f (x) = 4√–


1
2x
3

− 3√ 2x − 1 .
2

Consider the statements


(I) The curve y = f (x) intersects the x-axis exactly at one point.
(II) The curve y = f (x) intersects the x-axis at x = cos . π

12

Then
(a) Both (I) and (II) are correct.
(b) Only (I) is correct.
(c) Only (II) is correct.
(d) Both (I) and (II) are incorrect.

68. Let (5, ) be the circumcenter of a triangle with vertices A (a, −2) , B (a, 6) and C ( , −2). Let α denote
a a

4 4

the circumradius, β denote the area and gg denote the perimeter of the triangle. Then α + β + γ is
(a) 53
(b) 30
(c) 62
(d) 60

69. If in a G.P. of 64 terms, the sum of all the terms is 7 times the sum of the odd terms of the G.P., then the
common ratio of the G.P. is equal to
(a) 5
(b) 4
(c) 6
(d) 7

70. Let O be the origin and the position vectors of A and B be 2^i + 2^j + k
^
and 2^i + 4^j + 4k
^
respectively. If
the internal bisector of ∠AOB meets the line AB at C , then the length of OC is
3−−
(a) √34 2
3−−
(b) √31 2
2−−
(c) √31 3
2−−
(d) √34 3
71. (2 +2
x −x
) tan x√ tan
−1 2
(x −x+1)

Suppose f (x) =
2
3
. Then the value of f ′
(0) is equal to
(7x +3x+1)

(a) 0
π
(b) 2

(c) √−−
π

(d) π

72. For x ∈ (−
π
,
π
), if y (x) = ∫
cosec x+sin x

2
dx, and lim y (x) = 0 then y ( π
) is equal to
2 2 cosec x sec x+tan xsin x − 4
π
x→( )
2

1 1
(a) √2
tan
−1
(−
2
)

(b) 1

2
tan
−1
(
1

√2
)

(c) tan
−1
(
√2
1
)

(d) −
√2
1
tan
−1
(
√2
1
)

73. Let R be a relation on Z × Z defined by (a, b) R (c, d) if and only if ad − bc is divisible by 5. Then R is
(a) Reflexive and symmetric but not transitive
(b) Reflexive but neither symmetric nor transitive
(c) Reflexive and transitive but not symmetric
(d) Reflexive, symmetric and transitive

74. Let A be a square matrix such that AA T


= I . Then 1
A [(A + A
T
)
2
+ (A − A
T
2
) ] is equal to
2

(a) A
2
+ A
T

(b) A
3
+ I

(c) A
2
+ I

(d) A
3
+ A
T

75. If α, − π

2
< α <
π

2
is the solution of 4 cos θ + 5 sin θ = 1, then the value of tan α is
(a) √ 10−10

12

(b) 10−√ 10

(c) √ 10−10

(d) 10−√ 10

12

76. A fair die is thrown until 2 appears. Then the probability, that 2 appears in even number of throws, is
(a) 5

11

(b) 5

(c) 1

(d) 6

11
77. In a ΔABC , suppose y = x is the equation of the bisector of the angle B and the equation of the side AC
is 2x − y = 2 . If 2AB = BC and the points A and B are respectively (4, 6) and (α, β), then α + 2β is
equal to
(a) 39
(b) 48
(c) 45
(d) 42

78. ⎡
1 0 0

Let A and |2A| where α, β . Then a value of α is
3 21
= ⎢0 α β⎥ = 2 ∈ Z

⎣ ⎦
0 β α

(a) 3
(b) 17
(c) 5
(d) 9

79. ⎛ (
π
)
3


2
1

lim
π


1
2
∫ cos(t 3 )dt⎟

is equal to
π
x→ (x− ) 3
2 x
⎝ 2 ⎠

(a) 3π

(b) 3π

8
2

(c) 3π

(d)
2

80. If z = 1
− 2i is such that |z + 1| = αz + β (1 + i) ,
−−−
i = √ −1 and α, β ∈ R , then α + β is equal to
2

(a) −4
(b) 3
(c) 2
(d) −1

81. If the points of intersection of two distinct conics x 2


+ y
2
= 4b and x
2

+
y
2

= 1 lie on the curve


2
16 b

y
3
= 3x ,
2
then 3√ 3 times the area of the rectangle formed by the intersection points is _____.

82. Let f (x) = 2


x
− x , x ∈ R.
2
If m and n are respectively the number of points at which the curves
y = f (x) and y = f

(x) intersect the x -axis, then the value of m + n is _____.

83. All the letters of the word “GTWENTY” are written in all possible ways with or without meaning and these
words are written as in a dictionary. The serial number of the word “GTWENTY” is _____.

84. Equation of two diameters of a circle are 2x − 3y = 5 and 3x − 4y = 7. The line joining the points
, −4) and (− , 3) intersects the circle at only one point P (α, β) . Then, 17β − α is equal to
22 1
(−
7 7

85. If 11
C1
+
11
C2
+. . . +
11
C9
=
n
with gcd (n, m) = 1 , then n + m is equal to _____.
2 3 10 m
86. Let α, β be the roots of the equation x 2
− x + 2 = 0 with Im(α) > Im(β) . Then α 6
+ α
4
+ β
4
− 5α
2
is
equal to _____.

87. If the solution curve y = y (x) of the differential equation (1 + y 2


) (1 + loge x) dx + xdy = 0, x > 0
3
α−tan( )

passes through the point (1, 1) and y (e) then α + 2β is


2

= ,
3
β+tan( )
2

88. If the mean and variance of the data 65, 68, 58, 44, 48, 45, 60, α, β, 60 where α > β , are 56 and 66.2
respectively, then α + β is equal to _____.
2 2

89. A line with direction ratios 2, 1, 2 meets the lines x = y + 2 = z and x + 2 = 2y = 2z respectively at the
points P and Q . If the length of the perpendicular from the point (1, 2, 12) to the line P Q is l, then l is 2

____.

90. The area (in sq. units) of the part of the circle x + y = 169 which is below the line 5x − y
2 2
= 13 is
) , where α, β are coprime numbers. Then α + β is equal to ____.
πα 65 α −1 12
− + sin (
2β 2 β 13
Answer Key

1. A 2. C 3. D 4. C 5. B 6. C
7. D 8. D 9. C 10. C 11. C 12. A
13. C 14. C 15. A 16. D 17. A 18. D
19. C 20. A 21. 2 22. 8 23. 810 24. 6
25. 81 26. 17 27. 9 28. 2 29. 0.20 30. 10
31. C 32. D 33. C 34. A 35. D 36. B
37. D 38. C 39. B 40. A 41. C 42. C
43. C 44. C 45. B 46. D 47. D 48. B
49. D 50. D 51. 1 52. 1 53. 2 54. 4
55. 73 56. 46 57. 30 58. 4 59. 3 60. 2
61. B 62. A 63. B 64. A 65. B 66. D
67. A 68. A 69. C 70. D 71. C 72. D
73. A 74. D 75. A 76. A 77. D 78. C
79. C 80. B 81. 432.00 82. 5.00 83. 553.00 84. 2.00
85. 2041.00 86. 13.00 87. 3.00 88. 6344.00 89. 65.00 90. 171.00
Solutions

1. (A)
μ1 = 1.5

μm = 1.6

fa = 20cm
f (μ −1)μ
As f
m
=
1

(μ −μ
m

)
a 1 m

f (1.5−1)1.6
m
=
20 1.5−1.6

fm = −160 cm

2. (C)

5Q
5Q charge is inside the spherical region flux through sphere = ε0

3. (D)
PA V A n A RT A
=
PB V B n B RT B

Given V = A
VB

and T = T A B
1

PA 2
= 1
= 16
PB
32

4. (C)
Let x = current/division

After applying shunt

5x × G = 20x × 24

G = 4 × 24 = 96Ω

5. (B)
Given U = 2x
2
+ 3y
3
+ 2z
∂U
Fx = − = −4x
∂x

At x = 1 magnitude of F is 4N. x
6. (C)
Surface tension will be less as temperature increases.
2T cos θ
h =
ρgr

Height of capillary rise will be smaller in hot water and larger in cold water.

7. (D)
6 1 4 3
3 Li + 0n → 2He + 1H
2 3 4 1
1H + 1H → 2He + 0n

6 2 4
3
Li + 1 H → 2 (2 H e )

Energy released in process


2
Q = Δmc
2 4
Q = [M (Li) + m (1 H ) − 2 × M (2 H e )] × 931.5M eV

Q = [6.01690 + 2.01471 − 2 × 4.00388] × 931.5M eV

Q = 22.216M eV

Q = 22.22M eV

8. (D)
Given m and
r1 3
1 = m2 =
r2 4

As centripetal force F =
2
mv

In order to have constant (same in this question) centripetal force


F1 = F2
2 2
m1 v m2 v
1 2
=
r1 r2

v1

r
− √3
1
⇒ = √ =
v2 r2 2

9. (C)
By energy conservation
1 2 1 2
CV = LImax
2 2


C
Imax = √ V
L
−−−−− −
−6
100×10
= √ −3
× 12
6.4×10

12 3
= = = 1.5A
8 2

10. (C)
Given G = 10Ω

Ig = 3mA

I = 8A

In case of conversion of galvanometer into ammeter.

We have I gG = (I − Ig ) S
Ig G
S =
I −Ig
−3
(3×10 )10
−3
S = = 3.75 × 10 Ω
8−0.003
11. (C)

V z = 3V

Let potential at B = 0V
Potential at E (V ) = 10V E

V C = V A = 3V

IZ + II = I
10−3 7
I = = A
1000 1000
3
I1 = A
2000

Therefore, I
7−1.5
z = = 5.5mA
1000

12. (A)
V
R =
I

According to error analysis


dR dV dI
= +
R V I
dR 5 0.2
= +
R 200 20
dR 7
=
R 200
dR 7
%error × 100 = × 100 = 3.5%
R 200

13. (C)
For photon
hc hc
Ep = ⇒ λp =
λp Ep

For electron
h hv e
λe = =
me v e 2Ke

Given v e = 0.25c
h×0.25c hc
λe = =
2Ke 8Ke

Also λ p = λe
hc hc
=
Ep 8Ke

Ke 1
=
Ep 8

14. (C)
Given m = 100 kg
s = 10m

μ = 0.4

As f = μmg = 0.4 × 100 × 10 = 400N


Now W = f . s = 400 × 10 = 4000J
15. (A)

2
gR
gp = 2
(R+h)

h
gq = g (1 − )
R

gp = gq

g h
2
= g (1 − )
h R
(1+ )
R

2
h h
(1 − 2
) (1 + ) = 1
R R

Take h

R
= x

So x 3
− x + x
2
= 0
√ 5 −1
x =
2
R –
h = (√ 5 − 1)
2

16. (D)
S in first (p − 1) sec
1

S in first p sec
2

1 2
S1 = a(p − 1)
2
1 2
S2 = a(p)
2
1 2
S1 + S2 = at
2
2 2 2
(p − 1) + p = t
−−−−−−−−−−
2
t = √2p + 1 − 2p

17. (A)
Given that
Current I = I0 + βt

I0 = 20A

β = 3A/s

I = 20 + 3t
dq
= 20 + 3t
dt
q 20

∫ dq = ∫ (20 + 3t) dt
0 0

20 20

q = ∫ 20dt+ ∫ 3tdt
0 0
20
2
3t
q = [20t + ] = 1000C
2
0
18. (D)
Ampere – Maxwell law
→ dϕ
⃗ E
→ ∮ B . dl = μ0 i c + μ0 ω0
dt
→ dϕ
Faraday law → ⃗
∮ E . dl =
dt
B

−→
Q
Gauss’s law for electricity → ⃗
∮ E . dA =
ε0

−→
Gauss’s law for magnetism → ⃗
∮ B . dA = 0

19. (C)

Work done AB = (8000 + 6000) Dyne/cm


1

2
2 3
× 4m
2 3
= (6000Dyne/cm ) × 4m

Work done BC = − (4000 Dyne/cm ) × 4m 2 3

Total work done = 2000 Dyne/cm × 4m 2 3

3 1 N 3
= 2 × 10 × 5
× 4m
2
10 cm

−2 N 3
= 2 × 10 × −4
× 4m
2
10 m
2
= 2 × 10 × 4N m = 800J

20. (A)

Given R = 30 cm
R
f = = +15cm
2

Magnification (m) = ± 1

For convex mirror, virtual image is formed for real object.


Therefore, m is +ve
1 f
=
2 f −u

u = −15cm
21. (2)

The ball needs to just cross 4 steps to just hit 5th step
Therefore, horizontal range (R) = 0.4m
R = u.t
Similarly, in vertical direction
1 2
h = gt
2
1 2
0.4 = gt
2
2
1 0.4
0.4 = g( )
2 u
2
u = 2

u = √ 2m/s

Therefore, x = 2.

22. (8)
σe
u − 1m/s; a = −
2ε0 m

t = 1s

S = −1m

Using S = ut +
1

2
at
2

1 σe 2
−1 = 1 × 1 − × × (1)
2 2ε0 m
ε0 m
∴ σ = 9
e

∴ α = 8

23. (810)
1 2 2
F = ρ (v − v )A
2 1 2
1 2 2
F = × 1.2 × (70 − 65 ) × 2
2

= 810N

24. (6)
μ
0 M
V =
4π 2
r
−5 −7 M
⇒ 1.5 × 10 = 10 × 2
−2
(20×10 )

−5 −4
1.5×10 ×20×20×10
⇒ M = −7
10

M = 1.5 × 4 = 6
25. (81)

V V 9 9
I = I = = 12×4
=
R eq R eq 4
1+
12+4

9 4 9
I1 = × =
4 16 16
9 9
V A − V B = I1 × 8 = × 8 = V
16 2
1 81
∴ U = × 4 × μJ
2 4
81
∴ U = μJ
2

∴ x = 81

26. (17)

ΔE = 10.2eV

Recoil speed (v) =


ΔE

mc
10.2eV
=
−27 8
1.6×10 ×3×10
−19
10.2×1.6×10
= −27 8
1.6×10 ×3×10
17
v = 3.4m/s = m/s
5

Therefore, x = 17

27. (9)
Let total energy E =
1

2
KA
2

2 2
1 A KA E
U = K( ) = =
2 3 2×9 9

E 8E
KE = E − =
9 9

Ratio Total

KE
= 8E
E
=
9

8
9

x = 9
28. (2)

⃗ 2
^
A = (0.1) j

⃗ 0.2 ^ 0.2 ^
B = i + j
√2 √2

Magnitude of induced emf



Δϕ ⃗
B .A −0 – −3
e = = = √ 2 × 10 V
Δt 1

29. (0.20)
Path difference for minima at P
−−− −−−− λ
2√D 2 + d 2 − 2D =
2
−−− −−−− λ
∴ √D 2 + d 2 − D =
4
−−− −−−− λ
∴ √D 2 + d 2 = + D
4
2
2 2 2 λ Dλ
⇒ D + d = D + +
16 2
2
2 Dλ λ
⇒ d = +
2 16
−9 −14
2 0.2×400×10 4×10
⇒ d = +
2 4
2 −10
⇒ d ≈ 400 × 10
−5
∴ d = 20 × 10

⇒ d = 0.20mm

30. (10)

g sin θ
For rolling motion, a = I cm
1+
M R2

g sin θ
a = 1
1+
2
√3
2×10×
2
=
3
10
=
√3

Therefore x = 10
31. (C)
+
H

Fe2+ −−−−−−−→ Fe+3


Conc.H 2 SO4

SCN

Fe+3 −−−→ Fe(SCN) (blood red colour)


3
Appearance of blood red colour indicates presence of both nitrogen and sulphur

32. (D)
Since −N¨
H group is o/p directing hence arenium ion will not be formed by attack at meta position
2

i.e.

33. (C)

34. (A)
Rb = [Kr]5s1
n=5
l=0
m=0
s = +½ or –½

35. (D)
The difference in energy between the actual structure and the lowest energy resonance structure for the given
compound is known as resonance energy.

36. (B)
First ionisation energy increases along the period. Along the period Z increases which outweighs the shielding
effect
37. (D)

38. (C)

⇒ Cl2 + 2O
¯
H(aq) → Cl– + ClO–+H2O

39. (B)
(∆G)P,T = (+)ve for non-spontaneous process

40. (A)
Proteins are natural polymers composed of α-amino acids which are connected by peptide linkages.
Hence proteins upon acidic hydrolysis produce α-amino acids.

41. (C)
Δ

KMnO4 −→ K2MnO4 + MnO2 + O2


42. (C)
Gr-14, EN
C, 2.5
Si, 1.8
Ge, 1.8
Sn, 1.8
Pb, 1.9
The electronegativity values for elements from Si to Pb are almost same. So Statement I is false.

43. (C)
44. (C)
1)

2)

3)

4)

45. (B)
Fluorspar is CaF2
46. (D)
Assertion (A): Given statement is correct because in phenol hydroxyl group cannot be replaced by halogen atom.
Reason (R):

Given reason is false


Hence Assertion (A) is correct but Reason (R) is false

47. (D)
It is a type of conjugation responsible for resonance.

48. (B)
alkaline medium
− − − −
2MnO + H2 O + I −−−−−−−−−→ 2MnO2 + 2OH + IO
4 3

49. (D)

50. (D)
Ziegler catalyst → Titanium
Blood pigment → Iron
Wilkinson catalyst → Rhodium
Vitamin B12 → Cobalt

51. (1)

Hence total number of oxygen atom present per molecule

is 1
52. (1)

Magnetic behaviour Bond order


H2 Diamagnetic 1
He +

2
Paramagnetic 0.5
O +

2
Paramagnetic 2.5
N 2−

2
Paramagnetic 2
O 2−

2
Diamagnetic 1
F2 Diamagnetic 1
Ne +

2
Paramagnetic 0.5
B2 Paramagnetic 1

53. (2)

54. (4)

55. (73)
π = CRT
π1 T1
⇒ =
π2 T2
5
π 1 T2 7×10 ×283 4 −2
⇒ π2 = = = 72.56 × 10 Nm
T1 273
56. (46)
Zn+2 + 2e– → Zn
W=Z×i×t
65.4
= × 0.015 × 15 × 60
2×96500

= 45.75 × 10–4 gm

57. (30)
(E a + E a − E a )
1 2 3
K1 .K2 A 1 .A 2 −
K = = .e RT

K3 A3

(E a + E a − E a )
1 2 3
A1 A2 −
−Ea /RT
A. e = .e RT

A3

Ea = Ea1 + Ea2 − Ea3 = 40 + 50 – 60 = 30 kJ / mole.

58. (4)
31.4
( )
nsolute 98

M = × 1000 = × 1000 = 4.005 ≈ 4


V 100
( )
1.25

59. (3)
Acetaldehyde (CH3CHO), Methanal(HCHO), and cyclohexane carbaldehyde

60. (2)
Δng
KP = KC . (RT)

ng = 1
KP 0.492 −2
⇒ Kc = = = 2 × 10
RT 0.082×300

61. (B)
dy sin 2x
− ( ) y = sin x
dx 2
1+cos x

I.F. = 1 + cos x
2

2
y ⋅ (1 + cos x) = ∫ (sin x) dx

= − cos x + C

x = 0, C = 1
π
y( ) = 1
2

62. (A)

a ⃗ + 5 b = λc ⃗


b = 6 c ⃗ = μa ⃗

Eliminating a⃗
⃗ 6 1 ⃗
λc ⃗ − 5 b = c⃗ + b
μ μ
−1
∴ μ = , μ = −30
5

α = 5, β = 30
63. (B)
a 6 = 2 ⇒ a + 5d = 2

a 1 a 4 a 5 = a (a + 3d) (a + 4d)

= (2 − 5d) (2 − 2d) (2 − d)
2 2 3
f (d) = 8 − 32d + 34d − 20d + 30d − 10d

f (d) = −2 (5d − 8) (3d − 2)

8
d =
5

64. (A)
π

2
2 2
x cos x 1+sin x
I = ∫ ( x
+ ) dx
1+π sin x 2023
π 1+e

2

2
2 2
x cos x 1+sin x
I = ∫ ( + ) dx
1+π −x sin (−x) 2023
π 1+

2

On Adding, we get
π

2 2
2I = ∫ (x cos x + 1 + sin x) dx
π

2

On solving
2
π 3π
I = + − 2
4 4

a = 3

65. (B)
1 + 2g (x) , −1 ⩽ g (x) < 0 . . . . (1)
f (g (x)) = {
g(x)
1 − , 0 ⩽ g ⩽ 3 . . . . (2)
3

By (1) x ∈ ϕ
And by (2) x ∈ [−3, 0] and x ∈ [0, 1]

Range of f (g (x)) is [0, 1]

66. (D)
Centroid G divides MR in 1 : 2

G (1, 2, 2)

Point of intersection A of given lines is (2, −6, 0)


− −
AG = √ 69
67. (A)
– 2 –
f

(x) = 12√ 2x − 3√ 2 ⩾ 0 for [ 1

2
, 1]
1
f ( ) < 0
2

f (1) = 0 ⇒ (a) is correct


– 3
f (x) = √ 2 (4x − 3x) − 1 = 0

Let cos α = x,
π π
cos 3α = cos ⇒ α =
4 12
π
x = cos
12

(b) is correct.

68. (A)
a a
A (a, −2) , B (a.6) , C ( , −2) , O (5, )
4 4

AO = BO
2 2 2 2
a a
(a − 5) + ( + 2) = (a − 5) + ( − 6)
4 4

a = 8

AB = 8, AC = 6, BC = 10

α = 5, β = 24, γ = 24

69. (C)
2 3 63
a + ar + ar + ar +. . . . +ar
2 4 62
= 7 (a + ar + ar +. . . +ar )
64 64
a(1−r ) 7a(1−r )
⇒ =
1−r 1−r2

r = 6

70. (D)

length of OC
√ 136 2√ 34
= =
3 3

71. (C)
f (h)−f (0)

f (0) = lim
h
h→0
h −h −1 2
(2 + 2 ) tan h√ tan (h −h+1)−0

= lim
2 3
h→0 (7h +3h+1) h



= √π
72. (D)
2
(1+sin x) cos x
y (x) = ∫ 4
dx
1+sin x

Put sin x = t
1
(t− )
2 t
1+t 1 −1
= ∫ 4
dt = tan + C
t +1 √2 √2
π
x = ,t = 1
2

∴ C = 0

π 1 −1 −1
y( ) = tan ( )
4 √2 √2

73. (A)
(a, b) R (a, b)as ab − ab = 0
Therefore reflexive
Let (a, b) R (c, d) ⇒ ad − bc is divisible by 5
⇒ bc − ad is divisible by 5 ⇒ (c, d) R (a, b)

Therefore symmetric
Relation not transitive as (3, 1) R (10, 5) and (10, 5) R (1, 1) but (3, 1) is not related to (1, 1)

74. (D)
T T
AA = I = A A

On solving given expression, we get


2 2
1 2 T T 2 T T
A [A + (A ) + 2AA + A + (A ) − 2AA ]
2

2
2 T
= A [A + (A ) ]

3 T
= A + A

75. (A)
4 + 5 tan θ = sec θ

Squaring: 24tan 2
θ + 40 tan θ + 15 = 0
−10±√ 10
tan θ =
12

and tan θ is rejected


10+√ 10
= −( )
12

(a) is correct.

76. (A)
3 5
Required probability = 5

6
×
1

6
+ (
5

6
) ×
1

6
+ (
5

6
) ×
1

6
+. . . .
5

1 6 5
= × 25
=
6 11
1−
36
77. (D)

AD : DC = 1 : 2
4−α 10
=
6−α 8

α = β

α = 14 and β = 14

78. (C)
2 2
|A| = α − β
3 21 4
|2A| = 2 ⇒ |A| = 2
2 2
α − β = 16

(α + β) (α − β) = 16 ⇒ α = 4 or 5

79. (C)
Using L’hospital rule
2
0−cos x×3x
= lim
π − π
2(x− )
x→
2 2

π
sin(x− )
2 2

= lim ×
π − π 4
x→ 2(x− )
2 2

2

=
8

80. (B)
1
z = − 2i
2

|z + 1| = αz + β (1 + i)
3 α
∣ − 2i∣
∣ 2
∣ = 2
− 2αi + β + βi

3 α
∣ − 2i∣
∣ 2
∣ = ( 2
+ β) + (β − 2α) i
−−−−−
β = 2α and α

2
+ β = √
9

4
+ 4

α + β = 3

81. (432.00)
Putting y = 3x in both the conics
2 2

We get x = b and +
2
= 1
b

16
3

⇒ b = 4, 12 (b = 4 is rejected because curves coincide)

∴ b = 12

Hence points of intersection are


−−
(±√ 12 , ± 6) ⇒ area of rectangle = 432
82. (5.00)

∴ m = 3
′ x
f (x) = 2 ln 2 − 2x = 0
x
2 ln 2 = 2x

∴ n = 2

m + n = 5

83. (553.00)
Words starting with E = 360
Words starting with GE = 60
Words starting with GN = 60
Words starting with GTE = 24
Words starting with GTN = 24
Words starting with GTT = 24
GTWENTY = 1
Total = 553

84. (2.00)
Centre of circle is (1, −1)

Equation of AB is 7x − 3y + 10 = 0 . . . . (i)

Equation of CP is 3x + 7y + 4 = 0 . . . . (ii)

Solving (i) and (ii)


−41 1
α = ,β =
29 29

∴ 17β − α = 2
85. (2041.00)
9
11
Cr

r+1
r=1

9
1 12
= ∑ C r+1
12
r=1
1 12 2035
= [2 − 26] =
12 6

∴ m + n = 2041

86. (13.00)
6 4 4 2
α + α + β − 5α
4 4 2 2
= α (α − 2) + α − 5α + (β − 2)
5 4 2 2
= α − α − 5α + β − 4β + 4
3 4 2
= α (α − 2) − α − 5α + β − 2 − 4β + 4
3 2
= −2α − 5α − 3β + 2
2
= −2α (α − 2) − 5α − 3β + 2
2
= −7α + 4α − 3β + 2

= −7 (α − 2) + 4α − 3β + 2

= −3α − 3β + 16 = −3 (1) + 16 = 13

87. (3.00)
1 ln x dy
∫ ( + ) dx + ∫ = 0
x x 1+y
2

2
(ln x)
−1
ln x + + tan y = C
2

Put x = y = 1
π
∴ C =
4
2
(ln x)
−1 π
⇒ ln x + + tan y =
2 4

Put x = e
3
1−tan
π 3 2
⇒ y = tan( − ) = 3
4 2
1+tan
2

∴ α = 1, β = 1

⇒ α + 2β = 3

88. (6344.00)
x̄ = 56
2
σ = 66.2
2 2
α +β +25678 2
⇒ − (56) = 66.2
10
2 2
∴ α + β = 6344
89. (65.00)
Let P (t, t − 2, t) and Q (2s − 2, s, s)
D.R’s of PQ are 2, 1, 2
2s−2−t s−t+2 s−t
= =
2 1 2

⇒ t = 6 and s = 2
⇒ P (6, 4, 6) and Q (2, 2, 2)
x−2 y−2 z−2
PQ : = = = λ
2 1 2

Let F (2λ + 2, λ + 2, 2λ + 2)

A (1, 2, 12)

−→ −
−→
AF ⋅ P Q = 0

∴ λ = 2
− −
So, F(6, 4, 6) and AF = √ 65

90. (171.00)

12
−−−−−−−
Area = ∫ √169 − y 2 dy −
1

2
× 25 × 5
−13

π 169 65 169 −1 12
= × − + sin
2 2 2 2 13

∴ α + β = 171
JEE Main 29-01-2024 (Evening Shift)

Questions

1. An electric field is given by (6^i + 5^j + 3k


^
) N /C . The electric flux through a surface area 30 i m
^ 2
lying in
YZ-plane (in SI unit) is:
(a) 60
(b) 150
(c) 180
(d) 90

2. The bob of a pendulum was released from a horizontal position. The length of the pendulum is 10m. If it
dissipates 10% of its initial energy against air resistance, the speed with which the bob arrives at the lowest
point is:
[Use g = 10ms ] −2


(a) 5√6ms −1

(b) 2√– 5ms


−1


(c) 5√5ms −1

(d) 6√– 5ms


−1

3. In the given circuit, the current in resistance R is:


3

(a) 2.5 A
(b) 2A
(c) 1A
(d) 1.5 A

4. Given below are two statements:


Statement I: Most of the mass of the atom and all its positive charge are concentrated in a tiny nucleus and
the electrons revolve around it, is Rutherford’s model.
Statement II: An atom is a spherical cloud of positive charges with electrons embedded in it, is a special
case of Rutherford’s model.
In the light of the above statements, choose the most appropriate from the options given below.
(a) Statement I is true but Statement II is false
(b) Both statement I and Statement II are false
(c) Statement I is false but Statement II is true
(d) Both Statement I and Statement II are true
5. A stone of mass 900g is tied to a string and moved in a vertical circle of radius 1 m making 10 rpm. The
tension in the string. When the stone is at the lowest point is (if π = 9.8 and g = 9.8m/s ):
2 2

(a) 9.7 N
(b) 9.8 N
(c) 17.8 N
(d) 8.82 N

6. A wire of length L and radius r is clamped at one end. If its other end is pulled by a force F, its length
increases by l. If the radius of the wire and the applied force both are reduced to half of their original values
keeping original length constant, the increase in length will become:
(a)
3

2
times
(b) 4 times
(c) 3 times
(d) 2 times

7. A small liquid drop of radius R is divided into 27 identical liquid drops. If the surface tension is T, then the
work done in the process will be:
(a) 8πR T 2

(b) πR T
1

8
2

(c) 4πR T 2

(d) 3πR T 2

8. A plane electromagnetic wave of frequency 35MHz travels in free space along the X-direction. At a
particular point (in space and time) E ⃗ = 9.6^jV /m. The value of magnetic field at this point is:
(a) 3.2 × 10 ^i T−8

(b) 3.2 × 10 k ^
−8
T

(c) 9.6 × 10 kT ^
−8

(d) 9.6^jT

9. A particle is moving in a straight line. The variation of position ‘x’ as a function of time ‘t’ is given as
x = (t − 6t + 20t + 15) m. The velocity of the body when its acceleration becomes zero is:
3 2

(a) 10 m/s
(b) 6 m/s
(c) 4 m/s
(d) 8 m/s

10. Two sources of light emit with a power of 200W. The ratio of number of photons of visible light emitted by
each source having wavelengths 300nm and 500nm respectively, will be:
(a) 1 : 5
(b) 3 : 5
(c) 5 : 3
(d) 1 : 3
11. N moles of a polyatomic gas (f = 6) must be mixed with moles of a monoatomic gas so that the mixture
behaves as a diatomic gas. The value of N is:
(a) 6
(b) 3
(c) 2
(d) 4

12. In an a.c. circuit, voltage and current are given by:


V = 100 sin(100t)V and

)mA respectively.
π
I = 100 sin(100t +
3

The average power dissipated in one cycle is:


(a) 25 W
(b) 5 W
(c) 2.5 W
(d) 10 W

13. Two particles X and Y having equal charges are being accelerated through the same potential difference.
Thereafter they enter normally in a region of uniform magnetic field and describes circular paths of radii R 1

and R respectively. The mass ratio of X and Y is:


2
2

(a) (
R2
)
R1

R2
(b) (
R1
)

(c) (
R1
)
R2

R1
(d) (
R2
)
14. The truth table for given circuit is:

A B Y

0 0 1

(a) 0 1 1

1 0 1

1 1 0

A B Y

0 0 0

(b) 0 1 1

1 0 0

1 1 1

A B Y

0 0 0

(c) 0 1 0

1 0 0

1 1 1

A B Y

0 0 1

(d) 0 1 0

1 0 1

1 1 0

15. If the distance between object and its two times magnified virtual image produced by a curved mirror is 15
cm, the focal length of the mirror must be:
(a) 12 cm
(b)
10
cm
3

(c) 15 cm
(d) – 10 cm

16. A planet take 200 days to complete one revolution around the sun. If the distance of the planet from sun is
reduced to one fourth of the original distance, how many dys will it take to complete one revolution:
(a) 25
(b) 100
(c) 20
(d) 50
17. In Young’s double slit experiment, light from two identical sources are superimposing on a screen. The path
difference between the two lights reaching at a point on the screen is . The ratio of intensity of fringe at

this point with respect to the maximum intensity of the fringe is:
1
(a) 2

(b) 3

4
1
(c) 3
1
(d) 4

18. A physical quantity Q is found to depend on quantities a, b, c by the relation Q = 3

. The percentage
4
a b

2
c

error in a, b and c are 3% , 4% and 5% respectively. Then the percentage error in Q is:
(a) 43 %
(b) 34 %
(c) 14 %
(d) 66 %

19. The temperature of a gas having 2.0 × 10 25


molecules per cubic meter at 1.38 atm (Given,
k = 1.38 × 10
−23
JK ) is:
−1

(a) 200 K
(b) 300 K
(c) 100 K
(d) 500 K

20. A bob of mass ‘m’ is suspended by a light string of length ‘L’. It is imparted a minimum horizontal velocity
at the lowest point A such that it just completes half circular reaching the top most position B. The ratio of
(K.E)
kinetic energies (K.E)
A
is :
B

(a) 2:5
(b) 3:2
(c) 5:1
(d) 1:5

21. Two metallic wires P and Q have same volume and are made up of same material. If their area of cross
sections are in the ratio 4 : 1 and force F is applied to P, and extension of Δl is produced. The force which
1

is required to produce same extension in Q is F . 2


F1
The value of F2
is _____.
22. A simple harmonic oscillator has an amplitude A and time period 6π second. Assuming the oscillation starts
√3
from its mean position, the time required by it to travel from x = A to x = A will be s , where
2
π

x = ____.

23. A charge of 4.0μC is moving with a velocity of 4.0 × 10 6


ms
−1
along the positive y-axis under a magnetic
field B⃗ of strength (2k
^
) T . The force acting on the charge is x i N . The value of x is ____.
^

24. A horizontal straight wire 5m long extending from east to west falling freely at right angle to horizontal
component of earths magnetic field 0.60 × 10 W bm . The instantaneous value of emf induced in the
−4 −2

wire when its velocity is 10ms is _____ ×10 V


−1 −3

25. In the given figure, the charge stored in 6μF capacitor, when points A and B are joined by a connecting
wire is _____ μC .

26. Hydrogen atom is bombarded with electrons accelerated through a potential difference of V, which causes
excitation of hydrogen atoms. If the experiment is being performed at T = 0K, the minimum potential
difference needed to observe any Balmer series lines in the emission spectra will be V , where α = ____.
α

10

27. In the given circuit, the current flowing through the resistance 20Ω is 0.3A, while the ammeter reads 0.9A.
The value of R is _____ Ω .
1

28. A particle is moving in a circle of radius 50 cm in such a way that at any instant the normal and tangential
components of it’s acceleration are equal. If its speed at t = 0 is 4m/s, the time taken to complete the first
revolution will be [1 − e ] s, where α = ____.
1

α
−2π


29. A body of mass 5kg moving with a uniform speed 3√2ms in X – Y plane along the line y
−1
= x + 4 . The
angular momentum of the particle about the origin will be ____ kgm s . 2 −1
30. ∘

In a sling slit diffraction pattern, a light of wavelength 6000A is used. The distance between the first and
third minima in the diffraction pattern in found to be 3mm when the screen in placed 50 cm away from slits.
The width of the slit is ____ 10 m .
−4

31. Chromatographic technique/s based on the principle of differential adsorption is/are


A. Column chromatography
B. Thin layer chromatography
C. Paper chromatography
Choose the most appropriate answer from the options given below:
(a) C only
(b) B only
(c) A & B only
(d) A only

32. Given below are two statements:


Statement I: Fluorine has most negative electron gain enthalpy in its group.
Statement II: Oxygen has least negative electron gain enthalpy in its group.
In the light of the above statements, choose the most appropriate from the options given below.
(a) Both Statement I and Statement II are false
(b) Statement I is false but Statement II is true
(c) Statement I is true but Statement II is false
(d) Both Statement I and Statement II are true

33. Match List I with List II

List I(Spectral Series for Hydrogen) List II(Spectral Region/Higher Energy State)
A. Lyman I. Infrared region
B. Balmer II. UV region
C. Paschen III. Infrared region
D. Pfund IV. Visible region

Choose the correct answer from the options given below :-


(a) A-I, B-III, C-II, D-IV
(b) A-II, B-III, C-I, D-IV
(c) A-I, B-II, C-III, D-IV
(d) A-II, B-IV, C-III, D-I
34. Match List I with List II

List I (Bio Polymer) List II (Monomer)


A. Starch I. nucleotide
B. Cellulose II. α-glucose
C. Nucleic acid III. β-glucose
D. Protein IV. α-amino acid

Choose the correct answer from the options given below :-


(a) A-IV, B-II, C-I, D-III
(b) A-II, B-III, C-I, D-IV
(c) A-I, B-III, C-IV, D-II
(d) A-II, B-I, C-III, D-IV

35. Phenol treated with chloroform in presence of sodium hydroxide, which further hydrolysed in presence of
an acid results
(a) Salicyclic acid
(b) Benzene-1,2-diol
(c) Benzene-1,3-diol
(d) 2-Hydroxybenzaldehyde

36. Identify the reagents used for the following conversion

(a) A = DIBAL-H, B = NaOH(alc), C = Zn/HCl


(b) A = DIBAL-H, B = NaOH(aq), C = NH2–NH2/KOH, ethylene glycol
(c) A = LiAlH4, B = NaOH(aq), C = NH2–NH2/KOH, ethylene glycol
(d) A = LiAlH4, B = NaOH(alc), C = Zn/HCl
37. Which of the following reaction is correct?

(a)

(b)

o
HNO2 ,0 C

(c) CH3CH2CH2NH2 −−−−−−→ CH3CH2OH + N2 + HCl


H2 O

(d) C2H5CONH2 + Br2 + NaOH → C2H5CH2NH2 + Na2CO3 + NaBr + H2O


38. Which one of the following will show geometrical isomerism?

(a)

(b)

(c)

(d)
39. According to IUPAC system, the compound

is named as
(a) 1-Hydroxyhex-2-ene
(b) Cyclohex-1-en-2-ol
(c) Cyclohex-1-en-3-ol
(d) Cyclohex-2-en-1-ol

40. Anomalous behaviour of oxygen is due to its


(a) Large size and high electronegativity
(b) Small size and high electronegativity
(c) Large size and low electronegativity
(d) Small size and low electronegativity

41. The correct IUPAC name of K2MnO4 is


(a) Potassium tetraoxidomanganese (VI)
(b) Potassium tetraoxopermanganate (VI)
(c) Potassium tetraoxidomanganate (VI)
(d) Dipotassium tetraoxidomanganate (VII)

42. Alkyl halide is converted into alkyl isocyanide by reaction with


(a) KCN
(b) NaCN
(c) AgCN
(d) NH4CN

43. Match List I with List II

List I (Compound) List II (pKa value)


A. Ethanol I. 10.0
B. Phenol II. 15.9
C. m-Nitrophenol III. 7.1
D. p-Nitrophenol IV. 8.3

Choose the correct answer from the options given below :-


(a) A-II, B-I, C-IV, D-III
(b) A-I, B-II, C-III, D-IV
(c) A-III, B-IV, C-I, D-II
(d) A-IV, B-I, C-II, D-III
44. Which of the following statements are correct about Zn, Cd and Hg?
A. They exhibit high enthalpy of atomization as the d-subshell is full.
B. Zn and Cd do not show variable oxidation state while Hg shows +I and +II.
C. Compounds of Zn, Cd and Hg are paramagnetic in nature.
D. Zn, Cd and Hg are called soft metals.
Choose the most appropriate from the options given below:
(a) B, C only
(b) C, D only
(c) B, D only
(d) A, D only

45. The ascending acidity order of the following H atoms is

(a) A<B<D<C
(b) D<C<B<A
(c) A<B<C<D
(d) C<D<B<A

46. The element having the highest first ionization enthalpy is


(a) Si
(b) N
(c) Al
(d) C

47. On passing a gas, ‘X’, through Nessler’s reagent, a brown precipitate is obtained.
The gas ‘X’ is
(a) H2S
(b) NH3
(c) Cl2
(d) CO2

48. Which of the following acts as a strong reducing agent?


(Atomic number: Ce = 58, Eu = 63, Gd = 64, Lu = 71)
(a) Gd3+
(b) Eu2+
(c) Lu3+
(d) Ce4+
49. The product A formed in the following reaction is:

(a)

(b)

(c)

(d)

50. A reagent which gives brilliant red precipitate with Nickel ions in basic medium is
(a) sodium nitroprusside
(b) meta-dinitrobenzene
(c) dimethyl glyoxime
(d) neutral FeCl3

51. The oxidation number of iron in the compound formed during brown ring test for NO ion is _____.

52. The total number of anti bonding molecular orbitals, formed from 2s and 2p atomic orbitals in a diatomic
molecule is _____________.

53. The total number of molecules with zero dipole moment among CH4, BF3, H2O, HF, NH3, CO2 and SO2 is
_________.

54. The total number of ‘Sigma’ and Pi bonds in 2-formylhex-4-enoic acid is ____.

55. Standard enthalpy of vapourisation for CCl is 30.5 kJ mol–1. Heat required for vapourisation of 284 g of
4
CCl4 at constant temperature is ____kJ.
(Given molar mass in g mol–1 ; C = 12, Cl = 35.5)
56. If 50 mL of 0.5 M oxalic acid is required to neutralise 25 mL of NaOH solution, the amount of NaOH in 50
mL of given NaOH solution is _____ g.

57. A constant current was passed through a solution of AuCl ion between gold electrodes. After a period of

10.0 minutes, the increase in mass of cathode was 1.314 g. The total charge passed through the solution is
_________ × 10–2 F.
(Given atomic mass of Au = 197)

58. Molality of 0.8 M H SO solution (density 1.06 g cm–3) is _____×10–3 m.


2 4

59. The following concentrations were observed at 500 K for the formation of NH3 from N2 and H2. At
equilibrium : [N2] = 2 × 10–2 M, [H2] = 3 ×10–2 M and
[NH3] = 1.5 × 10–2M. Equilibrium constant for the reaction is _______.

60. The half-life of radioisotopic bromine - 82 is 36 hours. The fraction which remains after one day is
___________ ×10–2.
(Given antilog 0.2006 = 1.587)

61. 2 1 2 1 2 0
⎡ ⎤ ⎡ ⎤
Let A = ⎢6 2 11 ⎥ and P = ⎢5 0 2 ⎥ . The sum of the prime factors of ∣∣P −1
AP − 2I ∣
∣ is equal
⎣ ⎦ ⎣ ⎦
3 3 2 7 1 5

to
(a) 26
(b) 66
(c) 27
(d) 23

62. If sin( y
) = loge |x| + is the solution of the differential equation x cos(
α y
)
dy
= y cos(
y
) + x and
x 2 x dx x

y (1) =
π

3
, then α 2
is equal to
(a) 4
(b) 12
(c) 9
(d) 3

63. Let r and θ respectively be the modulus and amplitude of the complex number z = 2 − i (2 tan

), then
8

(r, θ) is equal to

(a) (2 sec , ) 3π

8

(b) (2 sec

8
,

8
)

(c)
11π 11π
(2 sec , )
8 8

(d)
5π 3π
(2 sec , )
8 8
64. If the mean and variance of five observations are 24
and 194
respectively and the mean of the first four
5 25

observations is 7

2
, then the variance of the first four observations is equal to
(a) 105

(b) 4

(c) 5

(d) 77

12

65. Let P (3, 2, 3) , Q (4, 6, 2) and R (7, 3, 2) be the vertices of ΔP QR . Then, the angle ∠QP R is
π
(a) 6

(b) cos ( ) −1 1

18
π
(c) 3

(d) cos ( ) −1 7

18

66. The sum of the solutions x ∈ R of the equation


3
3 cos 2x+cos 2x
= x
3 2
− x + 6 is
6 6
cos x−sin x

(a) 0
(b) 3
(c) −1
(d) 1

67. Number of ways of arranging 8 identical books into 4 identical shelves where any number of shelves may
remain empty is equal to
(a) 18
(b) 15
(c) 12
(d) 16

68. 3 3

− −−−− −−−−−− −−− − −− −−−−−− −−−− −


If ∫ where C is the
sin 2 x+cos 2 x
dx = A√ cos θ tan x − sin θ + B√ cos θ − sin θ cot x + C ,
3 3
√ sin xcos x sin(x−θ)

integration constant, then AB is equal to


(a) 4 cosec (2θ)
(b) 2 sec θ
(c) 4 sec θ
(d) 8 cosec (2θ)

69. −
−→ −
−→ −
−→
Let OA = a,⃗ OB = 12a⃗ + 4b ⃗ and OC = b ,⃗ where O is the origin. If S is the parallelogram with adjacent
sides OA and OC , then is equal to ____.
area of the quadrilateral OABC

area of S

(a) 10
(b) 8
(c) 7
(d) 6
70. Let y = loge (
1−x
2

) , −1 < x < 1 . Then at x =


1
, the value of 225 (y ′
− y
′′
) is equal to
2 2
1+x

(a) 742
(b) 746
(c) 732
(d) 736

71. If log a, log b, log c are in A.P. and log


e e e e
a − loge 2b, loge 2b − loge 3c, loge 3c − loge a are also in an A.P.,
then a : b : c is equal to
(a) 25 : 10 : 4
(b) 9 : 6 : 4
(c) 6 : 3 : 2
(d) 16 : 4 : 1

72. Let A be the point of intersection of the lines 3x + 2y = 14, 5x − y = 6 and B be the point of intersection
of the lines 4x + 3y = 7, 6x + y = 5. The distance of the point P (5, −2) from the line AB is
(a) 8
(b)
13

(c) 5

(d) 6

73. An integer is chosen at random from the integers 1, 2, 3, . . . . , 50. The probability that the chosen integer is
a multiple of atleast one of 4, 6 and 7 is
(a) 21

50

(b) 8

25

(c) 9

50

(d)
14

25

74. The function f (x) =


2
x
, x ∈ R − {−2, 8}
x −6x−16

(a) decreases in (−∞, −2) ∪ (−2, 8) ∪ (8, ∞)


(b) increases in (−∞, −2) ∪ (−2, 8) ∪ (8, ∞)
(c) decreases in (−∞, −2) and increases in (8, ∞)
(d) decreases in (−2, 8) and increases in (−∞, −2) ∪ (8, ∞)

75. If R is the smallest equivalence relation on the set {1, 2, 3, 4} such that {(1, 2) , (1, 3)} ⊂ R, then the
number of elements in R is ______.
(a) 8
(b) 15
(c) 10
(d) 12

76. Let x = (m, n are co-prime and natural numbers) be a solution of the equation cos(2sin x) = and
m −1 1

n 9

let α, β (α > β) be the roots of the equationmx − nx − m + n = 0. Then the point (α, β) lies on the
2

line
(a) 5x + 8y = 9
(b) 3x + 2y = 2
(c) 3x − 2y = −2
(d) 5x − 8y = −9
77. Let a unit vector u
^ = ^ ^ ^
x i + y j + zk makes angles π
,
π
and 2π
with the vectors 1 ^
i +
1 ^
k,
1 ^
j +
1 ^
k
2 3 3 √2 √2 √2 √2

2
and √2
1 ^
i +
1

√2
^
j respectively. If v ⃗ = 1

√2
^
i +
1

√2
^
j +
1

√2
^
k, then |u
^ − v|⃗ is equal to
11
(a) 2

(b) 9
(c) 7
(d) 5

78. If each term of a geometric progression a , a , a . . . . with a = 1 2 3 1


1
and a ≠ a is the arithmetic mean of
2 1
8

the next two terms and S = a + a +. . . . . +a , then S − S


n 1 2 n 20 18 is equal to
(a) 2 15

(b) 2 18

(c) −2 15

(d) −2 18

79. The function f (x) = 2x + 3(x) , x ∈ R, has 2

(a) exactly one point of local maxima and no point of local minima
(b) exactly one point of local maxima and exactly one point of local minima
(c) exactly one point of local minima and no point of local maxima
(d) exactly two points of local maxima and exactly one point of local minima

80. The distance of the point (2, 3) from the line 2x − 3y + 28 = 0, measured parallel to the line

√ 3x − y + 1 = 0, is equal to

(a) 3 + 4√2

(b) 4 + 6√3

(c) 4√2

(d) 6√3

81. Let the area of the region {(x, y) : 0 ⩽ x ⩽ 3, 0 ⩽ y ⩽ min {x


2
+ 2, 2x + 2}} be A. Then 12A is equal
to _____.

82. Remainder when 64 32


32

is divided by 9 is equal to _____.

83. Let O be the origin, and M and N be the points on the lines x−5
=
y−4
=
z−5
and x+8
=
y+2
=
z+11

4 1 3 12 5 9
−−→ −→

respectively such that M N is the shortest distance between the given lines. Then OM ⋅ ON is equal to
_____.

84. Let for any three distinct consecutive terms a, b, c of an A.P., the lines ax + by + c = 0 be concurrent at
the point P and Q (α, β) be a point such that the system of equations x + y + z = 6, 2x + 5y + αz = β
and x + 2y + 3z = 4, has infinitely many solutions. Then (P Q) is equal to _____.
2

85. Let the slope of the line 4x + 5y + 3 = 0 be 27r 1 +


9r2
for some r 1, r2 ∈ R . Then
2

x
2

lim (∫
3r
2
x
8t
dt) is equal to _____.
x→3 2 3
−r2 x −r1 x −3x
3 2
86. Let the set C = {(x, y) ∣
2 y
∣ x − 2 = 2023, xy ∈ N} . Then ∑ (x + y) is equal to _____.
(x,y) ∈ C

87. π

3
− −−−−− −− – –
If ∫ √ 1 − sin 2x dx = α + β√ 2 + γ√ 3 , where α, β and γ are rational numbers, then 3α + 4β − γ is
π

equal to ____.

88. Let α, β be the roots of the equation x − √–


2
6x + 3 = 0 such that Im(α) > Im(β) . Let a, b be integers not
−−

divisible by 3 and n be a natural number such that √ −1 . Then n + a + b
99
α 98 n
+ α = 3 (a + ib) , i =
β

is equal to _____.

− −−−−−−−−−−−−−−−−−−−−−−−−−
89. 
2
2
 2r [(f (r)) −f (x)f (r)] f(r)

Let f (x) = lim {


2 2
3
− r e r } be differentiable in (−∞, 0) ∪ (0, ∞) and f (1) = 1.
r −x
⎷r→x

Then the value of ea, such that f (a) = 0 , is equal to ____.

90. Let P (α, β) be a point on the parabola y = 4x . If P also lies on the chord of the parabola x
2 2
= 8y whose
mid point is (1, ) , then (α − 28) (β − 8) is equal to ____.
5

4
Answer Key

1. C 2. D 3. C 4. A 5. B 6. D
7. A 8. B 9. D 10. B 11. D 12. C
13. C 14. B 15. D 16. A 17. A 18. B
19. D 20. C 21. 16 22. 2 23. 32 24. 3
25. 36 26. 121 27. 30 28. 8 29. 60 30. 2
31. C 32. B 33. D 34. B 35. D 36. B
37. A 38. B 39. D 40. B 41. C 42. C
43. A 44. C 45. D 46. B 47. B 48. B
49. C 50. C 51. 1 52. 4 53. 3 54. 21
55. 56 56. 4 57. 2 58. 815 59. 417 60. 63
61. A 62. D 63. A 64. C 65. C 66. C
67. B 68. D 69. B 70. D 71. B 72. D
73. A 74. A 75. C 76. A 77. D 78. C
79. B 80. B 81. 164.00 82. 1.00 83. 9.00 84. 113.00
85. 12.00 86. 46.00 87. 6.00 88. 49.00 89. 2.00 90. 192.00
Solutions

1. (C)
⃗ ^ ^ ^
E = 6 i + 5 j + 3k

⃗ ^
A = 30 i

⃗ A ⃗
ϕ = E.

^ ^ ^ ^
ϕ = (6 i + 5 j + 3k ) . (30 i )

ϕ = 6 × 30 = 180

2. (D)

ℓ = 10m

Initial energy = mgℓ

So, mgℓ =9

10
1

2
mv
2

9 1 2
⇒ × 10 × 10 = v
10 2
2
v = 180
−−− –
v = √ 180 = 6√ 5m/s

3. (C)

R eq = 2Ω + 2Ω + 1Ω = 5Ω
V 10
i = = = 2A
R eq 5

Current in resistance R 3 = 2 × (
4

4+4
) = 2 ×
4

8
= 1A

4. (A)
According to Rutherford atomic model, most of mass of atom and all its positive charge is concentrated in tiny
nucleus & electron revolve around it.
According to Thomson atomic model, atom is spherical cloud of positive charge with electron embedded in it.
Hence,
Statement I is true but statement II false.
5. (B)
Given that

900 9
m = 900gm = kg = kg
1000 10

r = 1m
2π(10)
2πN π
ω = = = rad/ sec
60 60 3
2
T − mg = mrω
2
T = mg + mrω
2
9 9 π
= × 9.8 + × 1( )
10 10 3
2
9 π
= 8.82 + ×
10 9

= 8.82 + 0.98 = 9.80N

6. (D)
Stress
Y =
Strain
F

πr 2
Y = ℓ

2 ℓ
F = Y πr × . . . . . (i)
L
F /2

πr 2/4

Y =
Δℓ

L
2
Δℓ πr
F = Y × 2 ×
L 4

From (i)
2
2 ℓ Δℓ πr
Y πr = Y
L L 2

Δℓ = 2ℓ

7. (A)
Volume constant
4 3 4 3
πR = 27 × × πr
3 3
3 3
R = 27r

R = 3r
R
r =
3
2
2 R
r =
9

Work done = T . ΔA
2 2
= 27T (4πr ) − T 4πR
2
R 2
= 27T 4π − 4πR T
9
2
= 8πR T
8. (B)
E
= C
B
E 8
= 3 × 10
B
E 9.6
B = 8
= 8
3×10 3×10
−8
B = 3.2 × 10 T

^ ^
B = v
^ × E

^ ^ ^
= i × j = k

So, B⃗ = 3.2 × 10
−8 ^
kT

9. (D)
3 2
x = t − 6t = 20t + 15
dx 2
= v = 3t − 12t + 20
dt
dv
= a = 6t − 12
dt

When a = 0
6t − 12 = 0; t = 2 sec

At t = 2 sec
2
v = 3(2) − 12 (2) + 20 = 8m/s

10. (B)
hc
n1 × = 200
λ1

hc
n2 × = 200
λ2
n1 λ1 300
= =
n2 λ2 500
n1 3
=
n2 5

11. (D)
n 1 f +n 2 f
1 2
feq =
n 1 +n 2

For diatomic gas f eq = 5


(N )(6)+(2)(3)
5 =
N +2

5N + 10 = 6N + 6

N = 4

12. (C)
P avg = V rms Irms cos(Δϕ)
−3
100 100×10 π
= × × cos( )
√2 √2 3
4
10 1 −3
= × × 10
2 2
10
= = 2.5W
4

13. (C)
√ 2m(KE)
p √ 2mqV
mv
R = = = =
qB qB qB qB


R ∝ √m
2
m ∝ R
2
m1 R1
= ( )
m2 R2
14. (B)

¯¯¯¯
Y = A. B + A . B
¯¯¯¯
= (A + A ) . B

Y = 1.B

Y = B

15. (D)

−v
m = 2 =
u
−(15−u)
2 =
−u

2u = 15 − u

3u = 15 ⇒ u = 5cm

v = 15 − u = 15 − 5 = 10cm
1 1 1
= +
f v u

1 1 1−2 −1
= + = =
10 (−5) 10 10

f = −10cm

16. (A)
2 3
T ∝ r
2 2
T T
1 2

3
= 3
r r
1 2
2 2
(200) T
2
= 3
3
r r
( )
4

200×200 2
= T
4×4×4 2

200
T2 =
4×2

T 2 = 25 days
17. (A)

Δx =
4
2π 2π 7λ 7π
ϕ = Δx = × =
λ λ 4 2

ϕ
2
I = Imax cos ( )
2

I ϕ 7π 7π
2 2 2
= cos ( ) = cos ( ) = cos ( )
Imax 2 2×2 4

2 π
= cos (2π − )
4
2 π 1
= cos ( ) =
4 2

18. (B)
4 3
a b
Q =
2
c
ΔQ Δa Δb Δc
= 4 + 3 + 2
Q a b c

ΔQ Δa Δb Δc
× 100 = 4 ( × 100) + 3 ( × 100) + 2 ( × 100)
Q a b c

% error in Q = 4 × 3% + 3 × 4% + 2 × 5%

= 12% + 12% + 10% = 34%

19. (D)
P V = nRT
N
PV = RT
NA

N = Total no. of molecules


N
P = kT
V
5 25 −23
1.38 × 101 × 10 = 2 × 10 × 1.38 × 10 × T
5 2
1.01 × 20 = 2 × 10 × T
3
1.01×10
T = ≈ 500K
2

20. (C)
Apply energy conservation between A & B
1 2 1 2
mV = mV + mg (2L)
2 L 2 H
−−−
∵ VL = √5gL
−−
So, V H = √gL
1 2
m(√ 5gL)
(K.E) A 5
2
= =
1 2 1
(K.E)
B m(√ gL)
2
21. (16)
F /A
Stress Fℓ
Y = = =
Strain Δℓ/ℓ AΔℓ

Fℓ
Δℓ =
AY
V
V = Aℓ ⇒ ℓ =
A
FV
Δℓ =
2
A Y

Y & V is same for both the wires


F
Δℓ ∝
2
A
2
Δℓ1 F1 A
2
= ×
2
Δℓ2 A F2
1

Δℓ 1 = Δℓ 2
2 2
F1 A = F2 A
2 1
2
F1 A 2
1 4
= 2
= ( ) = 16
F2 A 1
2

22. (2)

From phasor diagram particle has to move from P to Q in a circle of radius equal to amplitude of SHM.
√ 3A

2 √3
cos ϕ = =
A 2
π
ϕ =
6

Now, π
= ωt
6
π 2π
= t
6 6π
π
t =
2

So, x = 2

23. (32)
Q = 4μC , v ⃗ = 4 × 10 ^
6
jm/s

⃗ ^
B = 2k T

⃗ ⃗
F = q (v ⃗ × B )

−6 6 ^ ^
= 4 × 10 (4 × 10 j × 2k )

−6 6 ^
= 4 × 10 × 8 × 10 i

⃗ ^
F = 32 i N

x = 32
24. (3)
−4 2
B H = 0.60 × 10 W b/m

Induced emf e = B H vℓ
−4 −3
= 0.60 × 10 × 10 × 5 = 3 × 10 V

25. (36)
At steady state, capacitor behaves as an open circuit and current flows in circuit as shown in the diagram.

R eq = 9Ω
9V
i = = 1A

ΔV 6Ω = 1 × 6 = 6V

V A = 3V

So, potential difference across 6μF is 6V


Hence Q = C ΔV
−6
= 6 × 6 × 10 C

= 36μC

26. (121)
For minimum potential difference electron has to make transition from n = 3 to n = 2 state but first electron has
to reach to n = 3 state from ground state. So, energy of bombarding electron should be equal to energy difference
of n = 3 and n = 1 state.
1
ΔE = 13.6 [1 − ] e = eV
2
3
13.6×8
= V
9

V = 12.09V ≈ 12.1V

So, α = 121
27. (30)

Given, i 1 = 0.3A, i 1 + i 2 + i 3 = 0.9A

So, V AB
= i 1 × 20Ω = 20 × 0.3V = 6V
6V 2
i2 = = A
15Ω 5
9
i1 + i2 + i3 = A
10
3 2 9
+ + i3 =
10 5 10
7 9
+ i3 =
10 10

i 3 = 0.2A

So, i 3 × R 1 = 6V

(0.2) R 1 = 6
6
R1 = = 30Ω
0.2

28. (8)
|a C
⃗ | = |a t
⃗ |
2
v dv
=
r dt
v t
dv dt
⇒ ∫ = ∫
v
2 r
4 0
v
−1 t
⇒ [ ] =
v r
4
−1 1
⇒ + = 2t
v 4
4 ds
⇒ v = =
1−8t dt
t s
dt
4∫ = ∫ ds
1−8t
0 0

(r = 0.5m , s = 2πr = π)
t
[ℓn(1−8t)]
0
4 × = π
−8

ℓn (1 − 8t) = −2π
−2π
1 − 8t = e
−2π 1
t = (1 − e ) s
8

So, α = 8

29. (60)
y − x − 4 = 0

d1 is perpendicular distance of given line from origin.


∣ −4 ∣ –
d1 = ∣ ∣ ⇒ 2√ 2m
2 2
∣ √ 1 +1 ∣
– –
So, ∣∣L⃗ ∣∣ 2
= mvd 1 = 5 × 3√ 2 × 2√ 2 kgm /s

2
= 60kgm /s
30. (2)
For nth minima
b sin θ = nλ

(λ is small so sin θ is small, hence sin θ ≃ tan θ )


b tan θ = nλ
y
b = nλ
D

⇒ yn =
nλD

b
(position of nth minima)

B → 1st minima, A → 3rd minima


3λD λD
y3 = , y1 =
d d
2λD
Δy = y3 − y1 =
b
−10
−3 2×6000×10 ×0.5
3 × 10 =
b
−10
2×6000×10 ×0.5
b = −3
3×10
−4
b = 2 × 10 m

x = 2

31. (C)
Memory Based

32. (B)
Statement-1 is false because chlorine has most negative electron gain enthalpy in its group.

33. (D)
A – II, B – IV, C – III, D – I
Fact based.

34. (B)
A-II, B-III, C-I, D-IV
Fact based.
35. (D)

It is Reimer Tiemann Reaction

36. (B)

37. (A)

38. (B)
Due to unsymmetrical

39. (D)
Cyclohex-2-en-1-ol

40. (B)
Fact Based.

41. (C)
K2MnO4
2+x–8=0
⇒ x = +6
O.S. of Mn = +6
IUPAC Name = Potassium tetraoxidomanganate(VI)
42. (C)
Covalent character of AgCN.

43. (A)
Ethanol → 15.9
Phenol → 10
M-Nitrophenol → 8.3
P-Nitrophenol → 7.1

44. (C)
(A) Zn, Cd, Hg exhibit lowest enthalpy of atomization in respective transition series.
(C) Compounds of Zn, Cd and Hg are diamagnetic in nature.

45. (D)

Stability of conjugate base α acidic strength


C<D<B<A

46. (B)
Al < Si < C < N ; IE1 order.

47. (B)
Nessler’s Reagent Reaction :

48. (B)
Eu+2 → Eu+3 + 1e–
[Xe]4f76s0 [Xe] 4f66s0

49. (C)

50. (C)
Ni2+ + 2dmg– → [Ni(dmg)2]
Rosy red/Bright Red precipitate
51. (1)
[Fe(H2O)5(NO)]2+,
Oxidation no. of Fe = +1

52. (4)
Antibonding molecular orbital from 2s = 1
Antibonding molecular orbital from 2p = 3
Total = 4

53. (3)
Molecules with zero dipole moment = CO2, CH4, BF3

54. (21)

21 bonds

55. (56)
∆Hvapo CCl4 = 30.5 kJ / mol
Mass of CCl4 = 284 gm
Molar mass of CCl4 = 154 g/mol
Moles of CCl4 = 284

154
= 1.844 mol
o
∆Hvap for 1 mole = 30.5 kJ/mol
∆Hvapo for 1.844 mol = 30.5 × 1.844
= 56.242 kJ

56. (4)
Equivalent of Oxalic acid = Equivalents of NaOH
50 × 0.5 × 2 = 25 × M × 1
MNaOH = 2M
WNaOH in 50ml = 2 × 50 × 40 × 10–3 g = 4g

57. (2)
W charge
=
E 1 F

1.314 Q
=
197 1 F

Q = 2 × 10–2 F

58. (815)
M×1000
m =
dsol ×1000−M×Molar mass solute

815 × 10–3 m
59. (417)
2 2
−2
[NH 3 ] (1.5×10 )
KC = 3
= 3
= 417
−2 −2
[N 2 ][H 2 ] (2×10 )×(3×10 )

60. (63)
Half life of bromine – 82 = 36 hours
0.693
t1/2 =
K
0.693 −1
K = = 0.01925 hr
36

1st order reaction kinetic equation


2.303 a
t = log
K a−x
a t×K
log = (t = 1 day = 24 hr)
a−x 2.303
−1
a 24hr×0.01925hr
log =
a−x 2.303
a
log = 0.2006
a−x
a
= anti log(0.2006)
a−x
a
= 1.587
a−x

If a = 1
1

1−x
= 1.587 ⇒ 1 – x = 0.6301 = Fraction remain after one day.

61. (A)
−1 −1 −1

∣P AP − 2I ∣
∣ = ∣
∣P AP − 2P P∣

−1
= ∣
∣P (A − 2I ) P ∣

−1
= ∣
∣P ∣
∣ |A − 2I | |P |

= |A − 2I |

∣ 0 1 2 ∣
∣ ∣
= 6 0 11 = 69
∣ ∣
∣ 3 3 0 ∣

So, Prime factor of 69 is 3 & 23


So, sum = 26

62. (D)
Differential equation:
y dy y
x cos = y cos + x
x dx x

y dy
cos [x − y] = x
x dx

Divide both sides by x 2

dy
x −y
y dx 1
cos [ ] =
x x
2 x

y
Let x
= t

dt 1
cos t ( ) =
dx x

1
cos t dt = dx
x

Integrating both sides


sin t = ln|x| + c
y
sin = ln|x| + c
x
√3
Using y (1) = π

3
, we get c =
2

So, α = √3 ⇒ α
2
= 3
63. (A)
z = 2 − i (2 tan

8
) = x + iy (let)
−−−−−− y
2
r = √x + y
2
&θ = tan
−1

x
−−−−−−−−−−−−−−
2
2 5π
r = √(2) + (2 tan )
8
5π 3π
= ∣
∣ 2 sec
∣ = ∣ 2 sec(π −
∣ ∣ )∣

8 8

= 2 sec
8

−2 tan

&θ = tan
−1
(
2
8
)

−1 5π
= tan (tan(π − ))
8

=
8

64. (C)
¯ 24 2 194
X = ; σ =
5 25

Let first four observation be x 1, x2 , x3 , x4

Here,
x 1 +x 2 +x 3 +x 4 +x 5 24
= . . . . (1)
5 5

Also,
x 1 +x 2 +x 3 +x 4 7
=
4 2

⇒ x1 + x2 + x3 + x4 = 14

Now from eqn (1)


x5 = 10

Now, σ 2
=
194

25
2 2 2 2 2
x1 +x 2 +x 3 +x 4 +x 5 576 194
− =
5 25 25
2 2 2 2
⇒ x1 + x2 + x3 + x4 = 54

Now, variance of first 4 observations


4 4 2
2
∑ xi ⎛ ∑ xi ⎞

Var =
i=1 i=1

− ⎜ ⎟
4 4
⎝ ⎠

54 49 5
= − =
4 4 4

65. (C)

Direction ratio of PR = (4, 1, -1)


Direction ratio of PQ = (1, 4, -1)
∣ ∣
Now, cos θ
4+4+1
=
∣ √ 18⋅√ 18 ∣
π
θ =
3
66. (C)
3
3 cos 2x+cos 2x 3 2
6
= x − x + 6
6
cos x−sin x
2
cos 2x(3+cos 2x)
3 2
⇒ 2
= x − x + 6
2
cos 2x(1−sin xcos x)

2
4(3+cos 2x)
3 2
⇒ 2
= x − x + 6
(4−sin 2x)

2
4(3+cos 2x)
3 2
⇒ = x − x + 6
(3+cos 2 2x)

3 2 2
x − x + 2 = 0 ⇒ (x + 1) (x − 2x + 2) = 0

So, sum of real solutions = −1

67. (B)
3 Shelf empty: (8, 0, 0, 0) → 1 way
(7, 1, 0, 0) ⎤

(6, 2, 0, 0) ⎥
2 shelf empty: ⎥ →
⎥ 4 ways
(5, 3, 0, 0) ⎥

(4, 4, 0, 0)

(6, 1, 1, 0) (3, 3, 2, 0)

1 shelf empty: (5, 2, 1, 0) (4, 2, 2, 0) ⎥ → 5 ways



(4, 3, 10)

(1, 2, 3, 2) (5, 1, 1, 1) ⎤

(2, 2, 2, 2) ⎥
0 Shelf empty: ⎥ → 5

ways
(3, 3, 1, 1) ⎥


(4, 2, 1, 1)

Total = 15 ways

68. (D)
3 3

sin 2 x+cos 2 x
∫ dx
3 3
√ sin xcos x sin(x−θ)

3 3

sin 2 x+cos 2 x
I = ∫ dx
3 3
√ sin xcos x(sin x cos θ−cos x sin θ)

1 3

sin 2 x cos 2 x
= ∫ 3
dx + ∫ 3
dx
2 2
sin 2 xcos x√ tan x cos θ−sin θ sin xcos 2 x√ cos θ−cot x sin θ
2 2
sec x cosec x
= ∫ dx + ∫ dx
√ tan x cos θ−sin θ √ cos θ−cot x sin θ

I = I1 + I2 . . . . .{Let}

For I 1, let tan x cos θ − sin θ = t


2

2 2t dt
sec x dx =
cos θ

For I 2, let cos θ − cot x sin θ = z


2

2 2z dz
cosec x dx =
sin θ

I = I1 + I2
2t dt 2z dz
= ∫ + ∫
cos θ t sin θ z
2t 2z
= +
cos θ sin θ
− −−−− −−−−−− −−− − −− −−−−−− −−−− −
= 2 sec θ√ tan x cos θ − sin θ + 2 cosec θ√ cos θ − cot x sin θ

Comparing
AB = 8 cosec 2θ
69. (B)

Area of parallelogram, S = ∣∣a⃗ × b∣∣⃗


Area of quadrilateral = Area (ΔOAB) + Area (ΔOBC )
1 ∣ ⃗ ∣ ∣ ⃗ ⃗ ∣
= { a ⃗ × (12a ⃗ + 4b) + b × (12a ⃗ + 4b) }
2 ∣ ∣ ∣ ∣

∣ ⃗ ∣
= 8 (a ⃗ × b)
∣ ∣
∣ ⃗ ∣

8 (a× b)
∣ ∣
Ratio = ∣ ⃗ ∣
= 8

(a× b)
∣ ∣

70. (D)
2
1−x
y = loge ( )
1+x 2

dy −4x

= y =
dx 1−x 4

Again,
2 4
d y −4(1+3x )
′′
= y =
2 2
dx (1−x )
4

Again,
4
4(1+3x )
′ ′′ −4x
y − y = +
4 2
1−x 4
(1−x )

At x =
1

2
,

′ ′′ 736
y − y =
225

Thus 225 (y ′
− y
′′
) = 225 ×
736

225
= 736

71. (B)
loge a, loge b, loge c are in A.P.
2
∴ b = ac . . . . (i)

Also
loge (
a

2b
) , loge (
2b

3c
) , loge (
3c

a
) are in A.P.
2
2b a 3c
( ) = ×
3c 2b a

b 3
=
c 2

Putting in eq. (i), b 2


= a ×
2b

3
a 3
=
b 2

a : b : c = 9 : 6 : 4
72. (D)
Solving lines L 1 (3x + 2y = 14) and L 2 (5x − y = 6) to get A (2, 4) and solving lines L3 (4x + 3y = 8) and
L4 (6x + y = 5) to get B ( 1

2
, 2) .
Finding Eqn. of AB : 4x − 3y + 4 = 0

Calculate distance PM
4(5)−3(−2)+4
∣ ∣
⇒ = 6
∣ 5 ∣

73. (A)
Given set = {1, 2, 3, . . .50}
P (A) = Probability that number is multiple of 4

P (B) = Probability that number is multiple of 6

P (C ) = Probability that number is multiple of 7

Now,
12 8 7
P (A) = , P (B) = , P (C ) =
50 50 50

Again
4 1 1
P (A ∩ B) = , P (B ∩ C ) = , P (A ∩ C ) =
50 50 50

P (A ∩ B ∩ C ) = 0

Thus
12 8 7 4 1 1
P (A ∪ B ∪ C ) = + + − − − + 0
50 50 50 50 50 50
21
=
50

74. (A)
x
f (x) =
2
x −6x−16

Now,
2
−(x +16)

f (x) = 2
2
(x −6x−16)


f (x) < 0

Thus f (x) is decreasing in


(−∞, −2) ∪ (−2, 8) ∪ (8, ∞)

75. (C)
Given set {1, 2, 3, 4}
Minimum order pairs are (1, 1), (2, 2), (3, 3), (4, 4), (3, 1), (2, 1), (2, 3), (3,2),
(1, 3), (1, 2)
Thus no. of elements = 10

76. (A)
Assume sin −1
x = θ
1
cos(2θ) =
9
2
sin θ = ±
3

As mand n are co-prime natural numbers,


2
x =
3

i.e., m = 2, n = 3
So, the quadratic equation becomes 2x 2
− 3x + 1 = 0 whose roots are α = 1, β =
1

) lies on 5x + 8y = 9
1
(1,
2
77. (D)
Unit vector u
^ = ^ ^ ^
x i + y j + zk
1 ^ 1 ^ 1 ^ 1 ^
⃗ =
p1 i + k, p 2
⃗ = j + k
√2 √2 √2 √2

1 ^ 1 ^
⃗ =
p3 i + j
√2 √2

Now angle between u


^ and p ⃗
1
=
π

2
x z
^ ⋅ p⃗ = 0 ⇒
u + = 0
1
√2 √2

⇒ x + z = 0 . . . . (i)

Angle between u
^ and p ⃗
2
=
π

3
π
^ ⋅ p ⃗ = |u|⃗ ⋅ |p ⃗ | cos
u 2 2 3
y z 1 1
⇒ + = ⇒ y + z = . . . . (ii)
√2 √2 2 √2

Angle between u
^ and p ⃗
3
=

3

^. p ⃗ = |u
u ^| ⋅ |p ⃗ | cos
3 3 3

x 4 −1 −1
⇒ + = ⇒ x + y = . . . . (iii)
√2 √2 2 √2

From equation (i), (ii) and (iii) we get


−1 1
x = , y = 0, z =
√2 √2

Thus u
−1
^ − v
^ ^ 1 ^ 1 ^ 1 ^ 1 ^
= i + k − i − j − k
√2 √2 √2 √2 √2

−2
^ − v
^ = ^ 1 ^
u i − j
√2 √2

−−−−− 2
2 4 1 5
∴ ^ − v|⃗
|u = (√ + ) =
2 2 2

78. (C)
Let rth term of the GP be ar n−1
.
Given, 2a = a r + a r+1 r+2
n−1 n n+1
2ar = ar + ar
2
= 1 + r
r
2
r + r − 2 = 0

Hence, we get, r = −2 (as r ≠ 1)


So, S − S = (Sum upto 20 terms) – (Sum upto 18 terms) =
20 18 T 19 + T 20
18
T 19 + T 20 = ar (1 + r)

Putting the values a = and r 1

8
= −2 ;
We get T + T = −2
19 20
15
79. (B)
2

f (x) = 2x + 3(x) 3

−1

f (x) = 2 + 2x 3

1
= 2 (1 + )
1

x 3

x 3 +1
= 2( )
1

x 3

So, maxima (M) at x = −1 & minima (m) at x = 0

80. (B)


Writing P in terms of parametric co-ordinates 2 + r cos θ, 3 + r sin θ as tan θ = √3

r √ 3r
P (2 + ,3 + )
2 2

P must satisfy 2x − 3y + 28 = 0

So, 2 (2 + r √ 3r
) − 3 (3 + ) + 28 = 0
2 2

We find r = 4 + 6√ 3

81. (164.00)

2 3
2
A = ∫ (x + 2) dx + ∫ (2x + 2) dx
0 2
41
A =
3

12A = 41 × 4 = 164

82. (1.00)
Let 32 = 32
t
32
32 1 2t 2t
64 = 64 = 8 = (9 − 1)

= 9k + 1

Hence remainder = 1
83. (9.00)
x−5 y−4 z−5
L1 : = = = λ
4 1 3

Drs (4, 1, 3) = b1

M (4λ + 5, λ + 4, 3λ + 5)
x+8 y+2 z+11
L2 : = = = μ
12 5 9

N (12μ − 8, 5μ − 2, 9μ − 11)
−−→
M N = (4λ − 12μ + 12, λ − 5μ + 6, 3λ − 9μ + 16) . . . (1)

Now
∣ ^ ^ ^∣
i j k
∣ ∣
⃗ ⃗ ^ ^
b1 × b2 = ∣ 4 1 3 ∣ = −6 i + 8k . . . . (2)
∣ ∣
∣ 12 5 9∣

Equation (1) and (2)


4λ−12μ+13 λ−5μ+6 3λ−9μ+16
∴ = =
−6 0 8

I and II
λ − 5μ + 6 = 0 . . . . (3)

I and III
λ − 3μ + 4 = 0 . . . (4)

Solve (3) and (4) we get


λ = −1, μ = 1

∴ M (1, 3, 2) , N (4, 3, −2)


−−→ −→

∴ OM ⋅ ON = 4 + 9 − 4 = 9

84. (113.00)
∵ a, b, c are in A.P.

⇒ 2b = a + c ⇒ a − 2b + c = 0

∴ ax + by + c passes through fixed point (1, −2)


∴ P = (1, −2)

For infinite solution,


D = D1 = D2 = D3 = 0

∣ 1 1 1 ∣
∣ ∣
D : 2 5 α = 0
∣ ∣
∣ 1 2 3 ∣

⇒ α = 8

∣ 6 1 1∣
∣ ∣
D1 : β 5 α = 0 ⇒ β = 6
∣ ∣
∣ 4 2 3∣

∴ Q = (8, 6)
2
∴ PQ = 113
85. (12.00)
According to the question,
9r2
27r1 + = −9
2
x
2
∫ 8t dt
3
lim
3r x
x→3 2 2 3
−r2 x −r1 x −3x
2
2

= lim
3r 2
8x
(using LH’ Rule)
x→3 2 2
−2r2 x−3r1 x −3
2

72
=
3r
2
−6r −27r −3
2 1
2

72
=
9r
2
− −27r1 −3
2

72
= = 12
9−3

86. (46.00)
2 y
x − 2 = 2023

⇒ x = 45, y = 1

∑ (x + y) = 46
(x,y) ∈ C

87. (6.00)
π

3
− −−−−− −−
= ∫ √ 1 − sin 2xdx
π

= ∫ |sin x − cos x| dx
π

π π

4 3

= ∫ (cos x − sin x) dx + ∫ (sin x − cos x) dx


π π

6 4
– –
= −1 + 2√ 2 − √ 3
– –
= α + β√ 2 + γ√ 3

α = −1, β = 2, γ = −1

3α + 4β − γ = 6

88. (49.00)
2

x − √ 6x + 6 = 0
√ 6 ±i√ 6 √6
x = = (1 ± i)
2 2
π π
– i – −i
α = √ 3 (e 4 ) , β = √ 3 (e 4 )

99
α 98 98 α
∴ + α = α ( + 1)
β β
98 π
α (α+β)
49 i99 –
= = 3 (e 4 ) × √2
β

49
= 3 (−1 + i)
n
= 3 (a + ib)

∴ n = 49, a = −1, b = 1

∴ n + a + b = 49 − 1 + 1 = 49
89. (2.00)
f (1) = 1, f (a) = 0
2 2 f(r)
2r ( f (r)−f (x)f (r))
2 3
f (x) = lim ( − r e r )
2 2
r→x r −x

2 f(r)
2r f (r) (f (r)−f (x))
3
= lim ( ) − r e r
r+x r−x
r→x
2 f(x)
2x f (x)
2 ′ 3
f (x) = f (x) − x e x

2x
y
dy
2 3
y = xy − x e x

dx
2 y
y dy x
= − e x
x dx y

dy
Put y = vx ⇒
dx
= v +
dv

dx
dv x v
v = v + x − e
dx v
v
dv e −v
= ⇒ e v dv = dx
dx v

Integrating both sides


v
e (x + c) + 1 + v = 0

f (1) = 1 ⇒ x = 1, y = 1
2
⇒ c = −1 −
e

v 2
e (−1 − + x) + 1 + v = 0
e
y
2 y
e x (−1 − + x) + 1 + = 0
e x
2
x = a, y = 0 ⇒ a =
e

ae = 2

90. (192.00)
Parabola is x = 8y 2

Chord with mid point (x 1, y1 ) is T = S1


2
∴ xx1 − 4 (y + y1 ) = x1 − 8y1
5
∴ (x1 , y1 ) = (1, )
4
5 5
⇒ x − 4 (y + ) = 1 − 8 × = −9
4 4

∴ x − 4y + 4 = 0 . . . . (i)

(α, β) lies on (i) & also on y 2


= 4x

∴ α − 4β + 4 = 0 . . . . (ii)

& β = 4α . . . . (iii)
2

Solving (ii) & (iii)


– –
= 8 ± 4√ 3 and α =
2
β 4β − 4 = 28 ± 16√ 3
– – – –
∴ (α, β) = (28 + 16√ 3, 8 + 4√ 3) (28 − 16√ 3, 8 − 4√ 3) &
– –
∴ (α − 28) (β − 8) = (±16√ 3) (±4√ 3)

= 192
JEE Main 30-01-2024 (Morning Shift)

Questions

1. A potential divider circuit is shown in figure. The output voltage V is:


0

(a) 2 mV
(b) 12 mV
(c) 4V
(d) 0.5 V

2. The electric field of an electromagnetic wave in free space is represented as E ⃗ ^


. The
= E0 cos(ωt − kz) i

corresponding magnetic induction vector will be:


(a) B⃗ = E C cos(ωt + kz)^j
0

(b) B⃗ = E C cos(ωt − kz)^j


0

(c) B⃗ = E0

C
cos(ωt + kz) j
^

(d)
E0
^
B⃗ = cos(ωt − kz) j
C

3. A particle is placed at the point A of a frictionless track ABC as shown in figure. IT is gently pushed towards
right. The speed of the particle when it reaches the point B is: (Take g = 10m/s ). 2

(a) √− −
10 m/s

(b) 10 m/s
(c) 20 m/s
(d) 2√− −
10 m/s

4. A Zener diode of breakdown voltage 10V is used as a voltage regulator as shown in the figure. The current
through the Zener diode is:

(a) 30 mA
(b) 20 mA
(c) 50 mA
(d) 0
5. Two insulated circular loop A and B of radius ‘a’ carrying a current of ‘I’ in the anti clockwise direction as shown
in the figure. The magnitude of the magnetic induction at the centre will be:

2μ I
(a) a
0

μ
(b) 2
0 I

√ 2 μ0 I
(c) a
μ0 I
(d) √ 2a

6. Match List – I with List – II.

List – I List – II
A Coefficient of viscosity (I) 2
[M L T
−2
]

B Surface tension (II) 2


[M L T
−1
]

C Angular momentum (III) [M L


−1
T
−1
]

D Rotational kinetic energy (IV) 0


[M L T
−2
]

Choose the correct answer from the options given below:


(a) A – III, B – IV, C – II, D – I
(b) A – I, B – II, C – III, D – IV
(c) A – IV, B – III, C – II, D – I
(d) A – II, B – I, C – IV, D – III

7. All surfaces shown in figure are assumed to be frictionless and the pulleys and the string are light.
The acceleration of the block of mass 2 kg is:

g
(a) 3
g
(b) 4

(c) g
g
(d) 2
8. The gravitational potential at a point above the surface of earth is −5.12 × 10 J /kg and the acceleration due to
7

gravity at that point is 6.4m/s . Assume that the mean radius of earth to be 6400 km. The height of this point
2

above the earth’s surface is:


(a) 1200 km
(b) 1000 km
(c) 1600 km
(d) 540 km

9. A particle of mass m is projected with a velocity ‘u’ making an angle of 30 with the horizontal. The magnitude

of angular momentum of the projectile about the point of projection when the particle is at its maximum height h
is:
3

(a) √3

16
mu

(b) √3

2
mu

(c) Zero
3
mu
(d) √ 2g

10. Two thermodynamical processes shown in the figure. The molar heat capacity for process A and B are C and A

C . The molar heat capacity at constant pressure and constant volume are represented by C
B and C ,
P V

respectively. Choose the correct statement.

(a) C B = ∞, C A = 0

(b) CP > CV > CA = CB

(c) CA = 0 and C B = ∞

(d) CA > CP > CV

11. Primary coil of a transformer is connected to 220V ac. Primary and secondary coil turns of the transforms are
100 and 10 respectively. Secondary coil of transformer is connected to two series resistances shown in figure.
The output voltage (V ) is: 0

(a) 7V
(b) 15V
(c) 22V
(d) 44V

12. An electric toaster has resistance of 60Ω at room temperature (27 C ) . The toaster is connected to a 220V

supply. If the current flowing through it reaches 2.75 A, the temperature attained by toaster is around: (If
/ C)
−4 ∘
α = 2 × 10

(a) 1694 C ∘

(b) 694 C ∘

(c) 1235 C ∘

(d) 1667 C ∘
13. At which temperature the r.m.s. velocity of a hydrogen molecule equal to that of an oxygen molecule at 47 ∘
C?

(a) 80 K
(b) 20 K
(c) -73 K
(d) 4 K

14. A spherical body of mass 100g is dropped from a height of 10m from the ground. After hitting the ground, the
body rebounds to a height of 5m. The impulse of force imparted by the ground to the body is given by: (given,
g = 9.8m/s )
2

(a) 2.39kg ms −1

(b) 43.2 kg ms −1

(c) 4.32kg ms −1

(d) 23.9kg ms −1

15. A series LR circuit connected with an ac source E = (25 sin 100t) V has a power factor of 1
. If the source of
√2

emf is changed to E = (20 sin 2000t) V , the new power factor fo the circuit will be:
1
(a) √2

1
(b) √7

1
(c) √5

1
(d) √3

16. The electrostatic potential due to an electric dipole at a distance ‘r’ varies as:
1
(a) 2
r

(b) r
(c) 1

r
1
(d) r
3

17. Young’s modules of material of a wire of length ‘L’ and cross-sectional area A is Y. If the length of the wire is
doubled and cross-sectional area is halved then Young’s modules will be:
(a) Y
(b)
Y

(c) 4Y
(d) 2Y

18. The work function of a substance is 3.0 eV. The longest wavelength of light that can cause the emission of
photoelectrons from this substance is approximately.
(a) 200 nm
(b) 215 nm
(c) 400 nm
(d) 414 nm

19. The ratio of the magnitude of the kinetic energy to the potential energy of an electron in the 5th excited state of
a hydrogen atom is:
1
(a) 4

(b) 4
1
(c) 2

(d) 1
20. The diffraction pattern of a light of wavelength 400 nm diffracting from a slit of width 0.2 mm is focused on the
focal plane of a convex lens of focal length 100cm. The width of the 1st secondary maxima will be:
(a) 2 cm
(b) 0.2 mm
(c) 2 mm
(d) 0.02 mm

21. Each of three blocks P, Q and R shown in figure has a mass of 3 kg. Each of the wires A and B has cross-
sectional area 0.005cm and Young’s modulus 2 × 10 N m . Neglecting friction, the longitudinal strain on
2 11 −2

wire B is _____ ×10 .(Take g = 10m/s )


−4 2

22. A capacitor of capacitance C and potential V has energy E. It is connected to another capacitor of capacitance
2C and potential 2V. Then the loss of energy is E, where x is ____.
x

23. The horizontal component of earth’s magnetic field at a place is 3.5 × 10 T . A very long straight conductor
−5


carrying current of √2A in the direction from South east to North West is placed. The force per unit length
experienced by the conductor is ______ ×10 N /m −6

24. In a closed organ pipe, the frequency of fundamental note is 30Hz. A certain amount of water is now poured in
the organ pipe so that the fundamental frequency is increased to 110Hz. If the organ pipe has a cross-sectional
area of 2cm , the amount of water poured in the organ tube is _____ g. (Take speed of sound in air is 330m/s).
2

25. The distance between object and its two times magnified real image as produced by a convex lens is 45 cm. The
focal length of the lens used is ____ cm.

26. A electron of hydrogen atom on an excited state is having energy E n = −0.85eV . The maximum number of
allowed transitions to lower energy level is _____.

27.

Consider a disc of mass 5 kg, radius 2 m, rotating with angular velocity of 10 rad/s about an axis perpendicular
to the plane of rotation. An identical disc is kept gently over the rotating disc along the same axis. The energy
dissipated so that both the discs continue to rotate together without slipping is _____ J.

28. The displacement and the increase in the velocity of a moving particle in the time interval of t to (t + 1) s are
125m and 50 m/s, respectively. The distance travelled by the particle in (t + 2) s is _____m.
th
29. Two cells are connected in opposition as shown. Cell E is of 8V emf and 2Ω internal resistance; the cell E is
1 2

of 2V emf and 4Ω internal resistance. The terminal potential difference of cell E is _____ V.
2

30. A ceiling fan having 3 blades of length 80cm each is rotating with an angular velocity of 1200 rpm. The
magnetic field of earth in that region is 0.5 G and angle of dip is 30 . The emf induced across the blades is

V . The value of N is ______.


−5
N π × 10

31. In the given reactions, identify the reagent A and reagent B

(a) A-CrO3, B-CrO2Cl2


(b) A-CrO2Cl2, B-CrO2Cl2
(c) A-CrO3, B-CrO3
(d) A-CrO2Cl2, B-CrO3
32. The final product A, formed in the following multistep reaction sequence is :

(a)

(b)

(c)

(d)

33. What happens to freezing point of benzene when small quantity of napthalene is added to benzene?
(a) Remains unchanged
(b) First decreases and then increases
(c) Decreases
(d) Increases

34. Diamagnetic Lanthanoid ions are:


(a) Nd3+ & Ce4+
(b) Nd3+ & Eu3+
(c) Lu3+ & Eu3+
(d) La3+ & Ce4+
35. Given below are two statements:
Statement (I): The orbitals having same energy are called as degenerate orbitals.
Statement (II): In hydrogen atom, 3p and 3d orbitals are not degenerate orbitals.
In the light of the above statements, choose the most appropriate answer from the options given below:
(a) Both Statement I and Statement II are true
(b) Statement I is false but Statement II is true
(c) Both Statement I and Statement II are false
(d) Statement I is true but Statement II is false

36. The Lassiagne's extract is boiled with dil HNO3 before testing for halogens because,
(a) AgCN is soluble in HNO3.
(b) Silver halides are soluble in HNO3.
(c) Ag2S is soluble in HNO3.
(d) Na2S and NaCN are decomposed by HNO3

37. Following is a confirmatory test for aromatic primary amines. Identify reagent (A) and (B).

(a)

A = NaNO2 + HCl, 0 – 5°C; B =

(b)

A = NaNO2 + HCl, 0 – 5°C; B =

(c)

A = NaNO2 + HCl, 0 – 5°C; B =

(d)

A = HNO3 / H2SO4; B =
38. Which of the following molecule/ species is most stable?

(a)

(b)

(c)

(d)

39. Given below are two statements:


Statement (I): The gas liberated on warming a salt with dil H2SO4, turns a piece of paper dipped in lead
acetate into black, it is a confirmatory test for sulphide ion.
Statement (II): In statement-I the colour of paper turns black because of formation of lead sulphite.
In the light of the above statements, choose the most appropriate answer from the options given below:
(a) Both Statement I and Statement II are false
(b) Statement I is false but Statement II is true
(c) Both Statement I and Statement II are true
(d) Statement I is true but Statement II is false
40. Structure of 4-Methylpent-2-enal is:

(a)

(b)

(c)

(d)

41. Compound A formed in the following reaction reacts with B gives the product C. Find out A and B.

+
(a) A = CH3 – C ≡ C
¯
Na , B = CH3 – CH2 – CH2 – Br
(b) A = CH3 – CH2 – CH3, B = CH3 – C ≡ CH
+
(c) A = CH3 – C ≡ C
¯ N , B = CH – CH – CH
a 3 2 3
(d) A = CH3 – CH = CH2, B = CH3 – CH2 – CH2 – Br

42. Given below are two statements : one is labelled as Assertion (A) and the other is labelled as Reason (R).
Assertion (A): CH2 = CH – CH2 – Cl is an example of allyl halide.
Reason (R): Allyl halides are the compounds in which the halogen atom is attached to sp2 hybridised carbon
atom.
In the light of the above statements, choose the most appropriate answer from the options given below:
(a) Both (A) and (R) are true but (R) is not the correct explanation of (A)
(b) (A) is false but (R) is true
(c) Both (A) and (R) are true and (R) is the correct explanation of (A)
(d) (A) is true but (R) is false
43. Match List - I with List - II.

List – I (Molecule) List – II (Shape)


(A) BrF5 (I) T-shape
(B) H2O (II) See saw
(C) ClF3 (III) Bent
(D) SF4 (IV) Square pyramidal

Choose the correct answer from the options given below:


(a) (A)-(I), (B)-(II), (C)-(IV), (D)-(III)
(b) (A)-(II), (B)-(I), (C)-(III), (D)-(IV)
(c) (A)-(III), (B)-(IV), (C)-(I), (D)-(II)
(d) (A)-(IV), (B)-(III), (C)-(I), (D)-(II)

44.

This reduction reaction is known as :


(a) Etard reduction
(b) Wolff-Kishner reduction
(c) Stephen reduction
(d) Rosenmund reduction

45. Match List - I with List - II.

List – I (Species) List – II(Electronic distribution)


(A) Cr+2 (I) 3d8
(B) Mn+ (II) 3d34s1
(C) Ni+2 (III) 3d4
(D) V+ (IV) 3d54s1

Choose the correct answer from the options given below:


(a) (A)-(II), (B)-(I), (C)-(IV), (D)-(III)
(b) (A)-(III), (B)-(IV), (C)-(I), (D)-(II)
(c) (A)-(IV), (B)-(III), (C)-(I), (D)-(II)
(d) (A)-(I), (B)-(II), (C)-(III), (D)-(IV)
46. Aluminium chloride in acidified aqueous solution forms an ion having geometry.
(a) Octahedral
(b) Square planar
(c) Tetrahedral
(d) Trigonal bipyramidal

47. Choose the correct statements from the following:


(A) Ethane-1, 2-diamine is a chelating ligand.
(B) Metallic aluminium is produced by electrolysis of aluminium oxide in presence of cryolite.
(C) Cyanide ion is used as ligand for leaching of silver.
(D) Phosphine act as a ligand in Wilkinson catalyst.
(E) The stability constants of Ca2+ and Mg2+ are similar with EDTA complexes.
Choose the correct answer from the options given below:
(a) (A), (D), (E) only
(b) (B), (C), (E) only
(c) (C), (D), (E) only
(d) (A), (B), (C) only

48. Example of vinylic halide is:

(a)

(b)

(c)

(d)
49. Given below are two statements: one is labelled as Assertion (A) and the other is labelled as Reason (R).
Assertion (A): There is a considerable increase in covalent radius from N to P. However from As to Bi only a
small increase in covalent radius is observed.
Reason (R): Covalent and ionic radii in a particular oxidation state increases down the group.
In the light of the above statements, choose the most appropriate answer from the options given below:
(a) (A) is true but (R) is false
(b) Both (A) and (R) are true and (R) is the correct explanation of (A)
(c) (A) is false but (R) is true
(d) Both (A) and (R) are true but (R) is not the correct explanation oi (A)

50. Sugar which does not give reddish brown precipitate with Fehling’s reagent, is:
(a) Sucrose
(b) Glucose
(c) Maltose
(d) Lactose

51. The pH at which Mg(OH) [K = 1 × 10–11] begins to precipitate from a solution containing 0.10 M Mg2+ ions
2 sp
is ____

52. The total number of molecular orbitals formed from 2s and 2p atomic orbitals of a diatomic molecule is _____

53. The rate of First order reaction is 0.04 mol L–1 s–1 at 10 minutes and 0.03 mol L–1 s–1 at 20 minutes after
initiation. Half life of the reaction is _______ minutes. (Given log2 = 0.3010,
log3 = 0.4771)

54. The compound formed by the reaction of ethanal with semicarbazide contains ________ number of nitrogen
atoms.

55. If IUPAC name of an element is "Unununnium'' then the element belongs to nth group of Periodic table. The
value of n is _____

56. 2MnO + bI– + cH O → x I + yMnO + zO


− ¯
H
4 2 2 2
If the above equation is balanced with integer coefficients, the value of z is ______

57. 0.05 cm thick coating of silver is deposited on a plate of 0.05 m2 area. The number of silver atoms deposited on
plate are _______ × 1023.
(At mass Ag = 108, d = 7.9 g cm–3)

58. The mass of sodium acetate (CH3COONa) required to prepare 250 mL of 0.35 M aqueous solution is _______
g. (Molar mass of CH3COONa is 82.02 g mol–1)

59. On a thin layer chromatographic plate, an organic compound moved by 3.5 cm, while the solvent moved by 5
cm. The retardation factor of the organic compound is ______ × 10–1.
60.

An ideal gas undergoes a cyclic transformation starting from the point A and coming back to the same point by
tracing the path A → B → C → A as shown in the diagram above. The total work done in the process is
________ J.

61. The area (in square units) of the region bounded by the parabola y 2
= 4 (x − 2) and the line y = 2x − 8, is:
(a) 6
(b) 7
(c) 8
(d) 9

62. Let g : R → R be a non constant twice differentiable function such that g ′


(
1 ′
) = g (
3
) . If a real valued
2 2

function f is defined as f (x) = [g (x) + g (2 − x)] , then 1

2
3 1
(a) f

(
2
) + f

(
2
) = 1

(b) f
′′
(x) = 0 for no x in (0, 1)
(c) (x) = 0 for atleast two x in (0, 2)
′′
f

(d) (x) = 0 for exactly one x in (0, 1)


′′
f

63. If 2sin x + sin 2x cos x + 4 sin x − 4 = 0 has exactly 3 solutions in the interval [0,
3 nπ
],n ∈ N , then the
2

roots of the equation x + nx + (n − 3) = 0 belong to: 2

√ 17 √ 17
(a) (−
2
,
2
)

(b) (0, ∞)

(c) Z

(d) (−∞, 0)

64. Let a⃗ ^ ^ ^
= a1 i + a2 j + a3 k and b ⃗ = ^ ^ ^
b1 i + b2 j + b3 k be two vectors such that |a|⃗ ⃗
= 1, a ⃗ ⋅ b = 2 and ∣∣b∣∣⃗ = 4 .
If c ⃗ = ⃗ ⃗
2 (a ⃗ × b) − 3b, then the angle between b ⃗ and c ⃗ is equal to:

(a) cos
−1
(
√3
2
)

(b) cos
−1
(
2

3
)

(c) cos
−1
(−
1

√3
)

√3
(d) cos
−1
(−
2
)
65. Let y = y (x) be the solution of the differential equation sec x dy + {2 (1 − x) tan x + x (2 − x)} dx = 0

such that y (0) = 2. Then y (2) is equal to:


(a) 2 {sin(2) + 1}
(b) 2
(c) 2 {1 − sin(2)}
(d) 1

66. A line passing through the point A (9, 0) makes an angle of 30 with the positive direction of x-axis. If this line

is rotated about A through an angle of 15 in the clockwise direction, then its equation in the new position is:

x
(a) + y = 9
√ 3 +2
y
(b) √ 3 +2
+ x = 9

x
(c) √ 3 −2
+ y = 9

y
(d) √ 3 −2
+ x = 9

67. If the length of the minor axis of an ellipse is equal to half of the distance between the foci, then the eccentricity
of the ellipse is:
2
(a) √5

(b)
√5

(c) √3

2
1
(d) √3

68. Two integers x and y are chosen with replacement from the set {0, 1, 2, 3, . . . , 10}. Then the probability that
|x − y| > 5, is:

(a) 60

121

(b)
30

121

(c) 31

121

(d) 62

121

69. If the circles (x + 1) 2 2


+ (y + 2)
2
= r and x
2
+ y
2
− 4x − 4y + 4 = 0 intersect at exactly two distinct
points, then
1
(a) 2
< r < 7

(b) 3 < r < 7


(c) 0 < r < 7
(d) 5 < r < 9

70. The maximum area of a triangle whose one vertex is at (0, 0) and the other two vertices lie on the curve
y = −2x + 54 at points (x, y) and (−x, y) , where y > 0, is:
2

(a) 92
(b) 108
(c) 88
(d) 122
71. Consider the system of linear equations x + y + z = 4μ, z + 2y + 2λz = 10μ, x + 3y + 4λ z = μ
2 2
+ 15,

where λ, μ ∈ R. Which of the following statements is NOT correct?


(a) The system is consistent if λ ≠
1

(b) The system has unique solution if λ ≠ and μ ≠ 1, 15


1

(c) The system is inconsistent if λ = and μ ≠ 1


1

(d) The system has infinite number of solutions if λ = and μ = 15


1

72. ∣ 4
2cos x 2sin x
4 2
3 + sin 2x ∣
∣ ∣
If f (x) = ∣ 3 + 2cos4 x 2sin x
4 2
sin 2x ∣ , then 1

5
f

(0) = is equal to:
∣ ∣
4 4 2
∣ 2cos x 3 + 2sin x sin 2x ∣

(a) 1
(b) 2
(c) 0
(d) 6

73. n

The value of is:


3
n
lim ∑
2 2 2 2
n→∞ (n +k )(n +3k )
k=1

13π
(a) 8(4√ 3 +3)

(b) (2√ 3 +3)π

24
π
(c) 8(2√ 3 +3)

(d) 13(2√ 3 −3)π

74. Let (α, β, γ) be the foot of perpendicular from the point (1, 2, 3) on the line x+3
=
y−1
=
z+4
. Then
5 2 3

19 (α + β + γ) is equal to:

(a) 102
(b) 101
(c) 99
(d) 100

75. Let A (2, 3, 5) and C (−3, 4, −2) be opposite vertices of a parallelogram ABC D. If the diagonal
−→

^ ^ ^
BD = i + 2 j + 3k , then the area of the parallelogram is equal to:
− −−
(a)1
√ 586
2
− −−
(b) 1

2
√ 410
− −−
(c) 1

2
√ 306
− −−
(d) 1

2
√ 474
76. Let M denote the median of the following frequency distribution

Class 0–4 4–8 8 – 12 12 – 16 16 – 20


Frequency 3 9 10 8 6

Then 20M is equal to:


(a) 416
(b) 208
(c) 104
(d) 52

77. Let S denote the sum of first n terms of an arithmetic progression. If S


n 20 = 790 and S 10 = 145 , then
S − S is:
15 5

(a) 410
(b) 395
(c) 405
(d) 390

78. If the domain of the function f (x) = cos


−1
(
2−|x|
) + {loge (3 − x)}
−1
is [−α, β) − {γ} , then α + β + γ
4

is equal to:
(a) 11
(b) 8
(c) 12
(d) 9

79. If z = z + iy, xy ≠ 0, satisfies the equation z 2


+ i z̄ = 0, then ∣∣z 2

∣ is equal to:
1
(a) 4

(b) 1
(c) 9
(d) 4

80. x

x ∫ f (t)dt

Let f : [−
π

2
,
π

2
] → R be a differentiable function such that f (0) =
1

2
. If the lim
0

x2
= α, then 8α is
2

x→0 e −1

equal to:
(a) 16
(b) 2
(c) 1
(d) 4

81. 1 1
824

Number of integral terms in the expansion of {7 is equal to _____.


( ) ( )
2 6
+ 11 }

82. Let the latus rectum of the parabola x


2


y
2

= 1 subtend an angle of π
at the centre of the hyperbola. If b is 2

9
2 3
b

equal to m
l −−
(1 + √ n ) , where l and m are co-prime numbers, then l 2
+ m
2
+ n
2
is equal to _____.
83. 10

Let α = 1
2
+ 4
2
+ 8
2
+ 13
2
+ 19
2
+ 26 +. . .
2
upto 10 terms and β = ∑ n
4
. If 4α − β = 55k + 40 , then k
n=1

is equal to _____.

84. Let α, β ∈ N be roots of the equation x 2


− 70x + λ = 0, where λ
,
λ
∉ N . If λ assumes the minimum
2 3

(√ α−1+√ β−1)(λ+35)
possible value, then |α−β|
is equal to:

85. A group of 40 students appeared in an examination of 3 subjects - Mathematics, Physics and Chemistry. It was
found that all students passed atleast one of the subjects, 20 students passes in Mathematics, 25 students passed
in Physics, 16 students passes in Chemistry, atmost 11 students passed in both Mathematics and Physics, atmost
15 students passed in both Physics and Chemistry, atmost 15 students passed in both Mathematics and
Chemistry. The maximum number of students passed in all the three subjects is ______.

86. 9
−−−
The value of 9 ∫ [√
10x

x+1
] dx, where [t] denotes the greatest integer less than or equal to t, is _____.
0

87. Let A = {1, 2, 3, . . . . , 7} and let P (A) denote the power set of A. If the number of functions f : A → P (A)

such that a ∈ f (a) , ∀ a ∈ A is m , m and n ∈ N and m is least, then m + n is equal to _____.


n

−−−−−−−−
88. Let y = y (x) be the solution of the differential equation (1 − x ) dy = [xy + (x + 2) √3 (1 − x ) ] dx,
2 3 2

1, y (0) = 0. If y ( ) = , m and n are co-prime numbers, then m + n is equal to _____.


1 m
−1 < x <
2 n

89. If d is the shortest distance between the lines x + 1


1 = 2y = −12z, x = y + 2 = 6z − 6 and d is the
2

y+8 y−2 32√ 3 d


shortest distance between the lines , then the value of is:
x−1 z−4 x−1 z−6 1
= = , = =
2 −7 5 2 1 −3 d2

90. 1
, |x| ⩾ 2
If the function f (x) = {
|x|
is differentiable on R, then 48 (a + b) is equal to ____.
2
ax + 2b, |x| < 2
Answer Key

1. D 2. D 3. A 4. A 5. D 6. A
7. A 8. C 9. A 10. C 11. A 12. A
13. B 14. A 15. C 16. A 17. A 18. D
19. C 20. C 21. 2 22. 2 23. 35 24. 400
25. 10 26. 6 27. 250 28. 175 29. 6 30. 32
31. A 32. B 33. C 34. D 35. D 36. D
37. B 38. A 39. D 40. D 41. A 42. D
43. D 44. D 45. B 46. A 47. D 48. B
49. D 50. A 51. 9 52. 8 53. 24 54. 3
55. 11 56. 8 57. 11 58. 7 59. 7 60. 200
61. D 62. C 63. D 64. D 65. B 66. D
67. A 68. B 69. B 70. B 71. C 72. C
73. B 74. B 75. B 76. B 77. B 78. A
79. B 80. B 81. 138.00 82. 182.00 83. 353.00 84. 60.00
85. 10.00 86. 155.00 87. 44.00 88. 97.00 89. 16.00 90. 15.00
Solutions

1. (D)
R eq = 4000Ω
4 1
i = = A
4000 1000
1
V 0 = i. R = × 500 = 0.5V
1000

2. (D)
Given E ⃗ ^
= E0 cos(ωt − kz) i
E0
⃗ ^
B = cos(ωt − kz) j
C

^ ^ ^
C = E × B

3. (A)
By COME
K EA + UA = K EB + UB
1 2
0 + mg (1) = mv + mg × 0.5
2
− −
v = √ g = √ 10 m/s

4. (A)

Zener is in breakdown region.


10 1
I3 = =
500 50
10 1
I1 = =
200 20

I2 = I1 − I3
1 1 3
I2 = ( − ) = ( ) = 30mA
20 50 100

5. (D)

√ 2μ I
0
∴ B net =
2a
6. (A)
dA
F = ηA
dy

−2 2 −1
[M LT ] = η [L ] [T ]
−1 −1
η = [M L T ]
−2
[M LT ]
F 0 −2
S. T = = = [M L T ]
ℓ [L]

2 −1
L = mvr = [M L T ]
1 2 2 −2
K. E = Iω = [M L T ]
2

7. (A)

40 − 2T = 4a

T − 14a ⇒ 20 = 12a
5 g
⇒ a = ⇒ 2a =
3 3

8. (C)
GME
−7
− = −5.12 × 10 . . . (i)
R E +h

GME

2
= 6.4 . . . . . (ii)
(R E +h)

By (i) and (ii)


5
⇒ h = 16 × 10 m = 1600km

9. (A)
L = mu cos θH
2 2
u sin θ
= mu cos θ ×
2g
3
3 √3 2 √ 3 mu
mu 1
= × × ( ) =
2g 2 2 16g

10. (C)
For process A
γ
log P = γ log V ⇒ P = V , (γ > 1)

PV
−γ
= Constant
R
CA = CV + . . . . . (i)
1+γ

Likewise for process B → PV


−1
= constant
R
CB = Cv +
1+1
R
CB = Cv + . . . . . (ii)
2

C P = C v + R . . . . . (iii)

By (i), (ii) & (iii)


CP > CB > CA > Cv

11. (A)
ε1 N1 100
= = ⇒ ε 2 = 22V
ε2 N2 10
22
I = 3
= 1mA, V 0 = 7V
22×10
12. (A)
220
R T =27 = 60Ω, R T = = 80Ω
2.75

R = R 0 (1 + αΔT )
−4
80 = 60 [1 + 2 × 10 (T − 27)]

T ≈ 1694 C

13. (B)

−−− −−−−−−
3R(320)
3RT
√ = √
2 32

320
T = = 20K
16

14. (A)
⃗ ⃗ ⃗ − ⃗
I = ΔP = P f Pi

M = 0.1 kg
− −−−−−−− − − −−−−− −−−−
I = ΔP = 0.1 (√ 2 × 9.8 × 5 − (−√ 2 × 9.8 × 10 ))
– −1
= 0.1 (14 + 7√ 2) ≈ 2.39kg ms

15. (C)
E = 25 sin(1000t)
1
cos θ =
√2

LR circuit

Initially ω1 L
R
=
1

tan θ
=
1

tan 45

= 1

X L = ω1 L

ω2 = 2θ 1 , given
ω2 L 2ω1 L

tan θ = =
R R

tan θ = 2
′ 1
cos θ =
√5

16. (A)
V =
kP cos θ

2
r
& can also checked dimensionally.

17. (A)
Young’s modulus depends on the material not length and cross sectional area. So young’s modulus remains same.

18. (D)
For P.E.E.: λ ⩽
hc

We

1240nm−eV
λ ⩽
3eV

λ ⩽ 413.33nm

λ max ≈ 414nm for P.E.E.

19. (C)
1

2
|P E| = K E for each value of n (orbit)
KE 1
∴ =
|P E| 2
20. (C)
Width of 1st secondary maxima = λ

a
.D

Here
−3
a = 0.2 × 10 m
−9
λ = 400 × 10 m
−2
D = 100 × 10

Width of 1st secondary maxima


−9
400×10 −2
= −3
× 100 × 10
0.2×10

= 2mm

21. (2)

10 2
a = m/s
3

30 − T 1 = 3 × a

T 1 = 20N

Strain = Stress

22. (2)
C1 C2
Energy loss = 1 2
(V 1 − V 2 )
2 C1 +C2
2
= .E
3

∴ x = 2

23. (35)
−5
B H = 3.5 × 10 T

F = iℓB sin θ, i = √ 2A
F – −5 1
= iB sin θ = √ 2 × 3.5 × 10 ×
ℓ √2

−6
= 35 × 10 N /m

24. (400)
V 11
= 30 ⇒ ℓ1 = m
4ℓ1 4

V 3
= 110 ⇒ ℓ2 = m
4ℓ2 4

Δℓ = 2m,

Change in volume = AΔℓ = 400cm


3

3
M = 400g; (∵ ρ = 1g/cm )
25. (10)
v
= −2
u

v = −2u . . . . (i)

v − u = 45 . . . . (ii)

⇒ u = −15cm

v = 30cm
1 1 1
= −
f v u

f = +10cm

26. (6)
13.6
En = − = −0.85
2
n

⇒ n = 4

No of transition
n(n−1) 4(4−1)
= = = 6
2 2

27. (250)
→ 2
MR 2
Li = I ωi = . ω = 100kgm /s
2
2
1 MR 2
Ei = . .ω = 500J
2 2
→ −

Li = Lf ⇒ 100 = 2I ωf

ωf = 5 rad/ sec
2
5(2) 2
1
Ef = 2 × . . (5) = 250J
2 2

ΔE = 250J

28. (175)
Considering acceleration is constant
v = u + at
2
u + 50 = u + a ⇒ a = 50m/s
1 2
125 = ut + at
2
a
125 = u +
2

⇒ u = 100m/s
a
∴ Snth = u + [2n − 1]
2

= 175m

29. (6)

8−2 6
I = = = 1A
2+4 6

Applying Kirchhoff’s from C to B


VC − 2 − 4 × 1 = VB

V C − V B = 6V
30. (32)
1 −4
B v = B sin 30 = × 10
4

ω = 2π × f = × 1200 rad/s
60
1 2
ε = B V ωℓ
2
−5
= 32π × 10 V

31. (A)

32. (B)

33. (C)
On addition of naphthalene to benzene there is depression in freezing point of benzene.

34. (D)
Ce : [Xe] 4f15d16s2 ; Ce4+ diamagnetic
La : [Xe] 4f05d16s2 ; La3+ diamagnetic

35. (D)
For single electron species the energy depends upon principal quantum number ‘n’ only. So, statement II is false.
Statement I is correct definition of degenerate orbitals.
36. (D)
If nitrogen or sulphur is also present in the compound, the sodium fusion extract is first boiled with concentrated
nitric acid to decompose cyanide or sulphide of sodium during Lassaigne's test

37. (B)

38. (A)

it is aromatic species.

39. (D)
Na2S + H2SO4 → Na2SO4 + H2S
(CH3COO)2Pb + H2S → PbS + 2CH3COOH
Black lead sulphide

40. (D)
41. (A)

42. (D)

43. (D)

BrF5 Square pyramidal

H2O Bent

ClF3 T-shape

SF4 See saw


44. (D)

It is known as rosenmund reduction that is the partial reduction of acid chloride to aldehyde.

45. (B)
5 1 2+ 4
24Cr → [Ar] 3d 4s ; Cr → [Ar] 3d

25Mn → [Ar] 3d54s2; Mn+ → [Ar] 3d54s1


28Ni → [Ar] 3d84s2; Ni2+ → [Ar] 3d8
23V → [Ar] 3d34s2; V+ → [Ar] 3d34s1

46. (A)
AlCl3 in acidified aqueous solution forms octahedral geometry [Al(H2O)6]3+

47. (D)

Bidentate, chelating
Based on Hall-Heroults process
[Rh(PPh3)3CI] Wilkinson's catalyst
Air

Ag2S + NaCN ⇌ Na[Ag(CN)2] + Na2S


Ca++ ion forms more stable complex with EDTA
48. (B)
Vinyl carbon is sp2 hybridized aliphatic carbon

is vinyl halide

While is aryl halide

and are allyl halide

49. (D)
According to NCERT,
Statement-I : Factual data,
Statement-II is true.
But correct explanation is presence of completely filled d and f-orbitals of heavier members.

50. (A)
Sucrose do not contain hemiacetal group.
Hence it does not give test with Fehling solution.
While all other give positive test with Fehling solution

51. (9)
Precipitation when Qsp = Ksp
[Mg2+][OH–]2 = 10–11
0.1 × [OH–]2 = 10–11 ⇒ [OH–] = 10–5
⇒ pOH = 5
⇒ pH = 9

52. (8)
Two molecular orbitals σ 2s and σ*2s.
Six molecular orbitals σ 2pz and σ*2pz.
π2px, π2py and π*2px, π*2py
53. (24)
0.04 = k[A]0e–k × 10 × 60 ….. (1)
0.03 = k[A]0e–k × 20 × 60 ….. (2)
(1)/(2)
4 600 k(2−1)
= e
3
4 600 k
= e
3
4
ln = 600 k
3
4 ln 2
ln = 600 ×
3 t 1/2

ln 2
t1/2 = 600 4
sec
ln
3

log 2 0.3010
t1/2 = 600 × sec. = 10 × min
log 4−log 3 0.6020−0.477

t1/2 = 24.08 min


Ans. 24

54. (3)

55. (11)
111 belongs to 11th group

56. (8)
Reduction Half
2MnO → 2MnO2

2MnO + 4H2O + 6e– → 2MnO2 + 8OH–


Oxidation Half
2I– → I2 + 2e–
6I– → 3I2 + 6e–
Adding oxidation half and reduction half, net reaction is
2MnO + 6I– + 4H2O → 3I2 + 2MnO2 + 8OH–

⇒z=8
⇒ Ans 8
57. (11)
Volume of silver coating = 0.05 × 0.05 × 10000 = 25 cm3
Mass of silver deposited = 25 × 7.9 g
Moles of silver atoms = 25×7.9

108

Number of silver atoms = × 6.023 × 10


25×7.9

108
23

= 11.01 × 1023
Ans. 11

58. (7)
Moles = Molarity × Volume in litres
= 0.35 × 0.25
Mass = moles × molar mass
= 0.35 × 0.25 × 82.02 = 7.18 g
Ans. 7

59. (7)
Distance travelled by sample/organiccompound
Retardation factor =
Distance travelled by solvent

3.5 −1
= = 7 × 10
5

60. (200)
Work done is given by area enclosed in the P vs V cyclic graph or V vs P cyclic graph.
Sign of work is positive for clockwise cyclic process for V vs P graph.
W = × (30 – 10) × (30 – 10) = 200 kPa – dm3
1

= 200 × 1000 Pa – L = 2 L-bar = 200 J

61. (D)
Let X = x − 2
2
y = 4x, y = 2 (x + 2) − 8
2
y = 4x, y = 2x − 4
4
2
y y+4
A = ∫ −
4 2
−2

= 9

62. (C)
′ 3 ′ 1
′ ′ g ( )−g ( )
g (x)−g (2−x) 2 2
′ ′ 3
f (x) = ,f ( ) = = 0
2 2 2
′ 1 ′ 3
g ( )−g ( )

Also f
2 2
′ 1 ′ 1
( ) = = 0, f ( ) = 0
2 2 2
′ 3 ′ 1
⇒ f ( ) = f ( ) = 0
2 2

⇒ roots in ( , 1) and (1, ) 1

2
3

(x) is zero at least twice in (


′′ 1 3
⇒ f , )
2 2
63. (D)
3 2
2sin x + 2 sin x. cos x + 4 sin x − 4 = 0
3 2
2sin x + 2 sin x. (1 − sin x) + 4 sin x − 4 = 0

6 sin x − 4 = 0
2
sin x =
3

n = 5 (in the given interval)


2
x + 5x + 2 = 0
−5±√ 17
x =
2

Required interval (−∞, 0)

64. (D)
Given |a|⃗ ∣ ∣⃗ ⃗ b⃗ = 2
= 1, b = 4, a .
∣ ∣

⃗ ⃗
c ⃗ = 2 (a ⃗ × b) − 3b

Dot product with a⃗ on both sides


c .⃗ a ⃗ = −6 . . . . (1)

Dot product with b ⃗ on both sides



b. c ⃗ = 48 . . . . (2)
2 2
∣ ∣⃗ ∣ ∣⃗
c .⃗ c ⃗ = 4 a ⃗ × b + 9 b
∣ ∣ ∣ ∣
2 2
2 2 2
⃗ ∣ ∣⃗
|c |⃗ = 4 [|a| |b| ⃗ b)
− (a . ] + 9 b
∣ ∣

2 2
|c |⃗ = 4 [(1) (4) − (4)] + 9 (16)

2
|c |⃗ = 4 [12] + 144
2
|c |⃗ = 48 + 144
2
|c |⃗ = 192

b. c ⃗
∴ cos θ =

∣ ∣
b ∣ ⃗
∣c ∣

∣ ∣

−48
∴ cos θ =
√ 192.4

−48
∴ cos θ =
8√ 3 .4

−3
∴ cos θ =
2√ 3

−√ 3 −√ 3
−1
∴ cos θ = ⇒ θ = cos ( )
2 2

65. (B)
dy
2
= 2 (x − 1) sin x + (x − 2x) cos x
dx

Now integrating on both sides


2
y (x) = ∫ 2 (x − 1) sin x dx + [(x − 2x) (sin x) − ∫ (2x − 2) sin x dx]

2
y (x) = (x − 2x) sin x + λ

y (0) = 0 + λ ⇒ 2 = λ
2
y (x) = (x − 2x) sin x + 2

y (2) = 2
66. (D)


Eqn: y − 0 = tan 15

(x − 9) ⇒ y = (2 − √ 3) (x − 9)

67. (A)
2b = ae
b e
=
a 2
−−−−−
2
e
e = √1 −
4

2
e =
√5

68. (B)
If x = 0, y = 6, 7, 8, 9, 10
If x = 1, y = 7, 8, 9, 10
If x = 2, y = 8, 9, 10
If x = 3, y = 9, 10
If x = 4, y = 10
If x = 5, y = no possible value
Total possible ways = (5 + 4 + 3 + 2 + 1) × 2
= 30

Required probability = 30

11×11
=
30

121

69. (B)
If two circles intersect at two distinct points
⇒ |r1 − r2 | < C 1 C 2 < r1 + r2
−−−−−
|r − 2| < √ 9 + 16 < r + 2

|r − 2| < 5 and r + 2 > 5

−5 < r − 2 < 5

r > 3 . . . . (2)

−3 < r < 7 . . . . (1)

From (1) and (2)


3 < r < 7
70. (B)

∣ 0 0 1 ∣
∣ ∣
Area of Δ =
1

2 ∣
x y 1

∣ −x y 1 ∣
1
⇒ ∣
∣ (xy + xy)∣
∣ = |xy|
2

Area (Δ) = |xy| = ∣


2
∣ x (−2x + 54)∣

d(Δ)

dx
= ∣
2
∣ (−6x + 54)∣
∣ ⇒

dx
= 0 at x = 3

Area = 3 (−2 × 9 + 54) = 108

71. (C)
x + y + z = 4μ, x + 2y + 2λz = 10μ, x + 3y + 4λ z = μ
2 2
+ 15 ,
∣ 1 1 1 ∣
∣ ∣ 2
Δ = 1 2 2λ = (2λ − 1)
∣ ∣
2
∣ 1 3 4λ ∣

For unique solution Δ ≠ 0, 2λ − 1 ≠ 0, (λ ≠


1

2
)

Let Δ = 0, λ = 1

∣ 4μ 1 1∣
∣ ∣
Δ y = 0, Δx = Δz = 10μ 2 1
∣ ∣
2
∣ μ + 15 3 1∣

= (μ − 15) (μ − 1)

For infinite solution λ =


1

2
,μ = 1 or 15

72. (C)
4 4 2
∣ 2cos x 2sin x 3 + sin 2x ∣
∣ ∣
4 4 2
∣ 3 + 2cos x 2sin x sin 2x ∣
∣ ∣
4 4 2
∣ 2cos x 3 + 2sin x sin 2x ∣

R2 → R2 − R1 , R3 → R3 − R1
4 2
∣ 2cos4 x 2sin x 3 + sin 2x ∣
∣ ∣
3 0 −3
∣ ∣
∣ 0 3 −3 ∣

f (x) = 45

f (x) = 0
73. (B)
n
3
n
lim ∑ 2 2
k 3k
n→∞ 4
k=1 n (1+ )(1+ )
n2 n2
n
3
1 n
= lim ∑
n k
2 3k2
n→∞
k=1 (1+ )(1+ )
n2 n2

1
dx
= ∫
1
2 2
3(1+x )( +x )
0 3

1
1 2
(x +1)−(x +
2
)
1 3 3

= ∫ × dx
3 2 1
(1+x 2 )(x 2 + )
0 3

1
1 1 1
= ∫ [ − ] dx
2 2 2
1 1+x
2
0 x +( )
√3

1 – −1
– 1
1 −1
1
= [√ 3tan (√ 3x)] − (tan x)
2 0 2 0

√3 π 1 π π π
= ( ) − ( ) = −
2 3 2 4 2√ 3 8

13π
=
8.(4√ 3 +3)

74. (B)

Let foot P (5k − 3, 2k + 1, 3k − 4)


DR’s → AP: 5k − 4, 2k − 1, 3k − 7
DR’s → Line: 5, 2, 3
Condition of perpendicular lines (25k − 20) + (4k − 2) + (9k − 21) = 0

Then k = 43

38

Then 19 (α + β + γ) = 101

75. (B)
∣−−→ −→
− ∣
Area = ∣ AC × BD∣
∣ ∣

∣ ^
i
^
j
^
k∣
∣ ∣
= ∣ 5 −1 7∣
∣ ∣
∣ 1 2 3∣
1 ∣ ^ ^ ^∣ 1 − −−
= √ 474
2
∣ −17 i + 8 j + 11k ∣ = 2
76. (B)

Class Frequency Cumulative frequency


0-4 3 3
4-8 9 12
8-12 10 22
12-16 8 30
16-20 6 36

N
−C
2
M = 1 + ( )h
f

18−12
M = 8 + × 4
10

M = 10.4

20M = 208

77. (B)
20
S20 = [2a + 19d] = 790
2

2a + 19d = 79 . . . . (1)
10
S10 = [2a + 9d] = 145
2

2a + 9d = 29 . . . . (2)

From (1) and (2), a = −8, d = 5


15 5
S15 − S5 = [2a + 14d] − [2a + 4d]
2 2
15 5
= [−16 + 70] − [−16 + 20]
2 2

= 405 − 10

= 395

78. (A)
2−|x|
∣ ∣
−1 ⩽ ⩽ 1
∣ 4 ∣

∣ 2−|x| ∣
⇒ ⩽ 1
∣ 4 ∣

−4 ⩽ 2 − |x| ⩽ 4

−6 ⩽ − |x| ⩽ 2

−2 ⩽ |x| ⩽ 6

|x| ⩽ 6

⇒ x ∈ [−6, 6] . . . . (1)

Now, 3 − x ≠ 1
And x ≠ 2
and 3 − x > 0
x < 3

From (1), (2) and (3)


⇒ x ∈ [−6, 3) − {2}

α = 6

β = 3

γ = 2

α + β + γ = 11
79. (B)
2
z = −i z̄
2

∣z ∣∣ = |i z̄ |
2
|z| = |z|
2
|z| − |z| = 0

|z| (|z| − 1) = 0

|z| = 0 (not acceptable)


∴ |z| = 1
2
∴ |z| = 1

80. (B)
x

x ∫ f (t)dt
0
lim
2
x→0 e x −1 2
( )×x
x2

∫ f (t)dt
2
x
0 e −1
lim ( lim = 1)
x 2
x→0 x→0 x

f (x)
= lim
1
(using L Hospital)
x→0
1
f (0) =
2
1
α =
2
2
8α = 2

81. (138.00)
1 1 824

General term in expansion of ((7) 2


+ (11) 6
) is
824−r r
824
tr+1 = C r (7) 2
(11) 6

For integral term, r must be multiple of 6.


Hence r = 0, 6, 12, . . . .822

82. (182.00)
LR subtends 60° at centre

2
b
2
∘ a b 1
⇒ tan 30 = = =
ae a e
2
√3
2
√ 3b
⇒ e =
9
2 2 4

Also, e 2
= 1 +
b

9
⇒ 1 +
b

9
=
3b

81
4 2
⇒ b = 3b + 27
4 2
⇒ b − 3b − 27 = 0
2 3 −−
⇒ b = (1 + √ 13 )
2

⇒ l = 3, m = 2, n = 13
2 2 2
⇒ l + m + n = 182
83. (353.00)
2 2 2
α = 1 + 4 + 8 ....
2
t n = an + bn + c

1 = a + b + c

4 = 4a + 2b + c

8 = 9a + 3b + c

On solving we get, a =
1

2
,b =
3

2
, c = −1

10 2
2
n 3n
α = ∑ ( + − 1)
2 2
n=1

10 10
2
2 4
4α = ∑ (n + 3n − 2) , β = ∑ n
n=1 n=1

10
3 2
4α − β = ∑ (6n + 5n + 12n + 4) = 55 (353) + 40
n=1

84. (60.00)
2
x − 70x + λ = 0

α + β = 70

αβ = λ

∴ α (70 − α) = λ

Since, 2 and 3 does not divide λ


∴ α = 5, β = 65, λ = 325

By putting value of α, β, λ we get the required value 60.


85. (10.00)

11 − x ⩾ 0 (Maths and Physics)


x ⩽ 11

x = 11 does not satisfy the data.


For x = 10

Hence maximum number of students passed in all the three subjects is 10.

86. (155.00)
10x 1
= 1 ⇒ x =
x+1 9
10x 2
= 4 ⇒ x =
x+1 3
10x
= 9 ⇒ x = 9
x+1
1 2

⎛ 9 3 9 ⎞
I = 9 ⎜∫ 0 dx + ∫ 1. dx + ∫ 2 dx⎟
⎝0 1 2 ⎠
9 3

= 155

87. (44.00)
f : A → P (A)

a ∈ f (a)

That means ‘a ’ will connect with subset which contain element ‘a ’.


Total options for 1 will be 2 . (Because 2 subsets contains 1)
6 6

Similarly, for every other element


Hence, total is 2 × 2 × 2 × 2 × 2 × 2 × 2 = 2
6 6 6 6 6 6 6 42

Ans. 2 + 42 = 44
88. (97.00)
3 2
(x +2)√ 3(1−x )
dy xy
− =
dx 2 2
1−x 1−x
x
− ∫ 2 −
dx−−−− 1

IF =e 1−x 2 = e 2
ln(1−x )
= √1 − x
2 +

−−−−− –
2 3
y√1 − x = √ 3 ∫ (x + 2) dx

−−−−− – x
4

y√1 − x2 = √ 3 ( + 2x) + c
4

⇒ y (0) = 0

∴ c = 0
1 65 m
y( ) = =
2 32 n

m + n = 97

89. (16.00)
x+1 y z x y+2 z−1
L1 : = = , L2 : = =
1 1 1 1 1 1

2 12 6

d1 = shortest distance between L 1 & L2


⃗ ⃗
∣ ⃗ −a⃗ ).(b ×b )
(a2 1 1 2

= ∣ ∣
∣ ⃗ ⃗ ∣
(b 1 ×b 2 )
∣ ∣ ∣ ∣

d1 = 2
x−1 y+8 z−4 x−1 y−2 z−6
L3 : = = , L4 : = =
2 −7 5 2 1 −3

d2 = shortest distance between L 3 & L4


12
d2 =
√3

Hence =
32√ 3 d1 32√ 3 ×2
= 12
= 16
d2
√3

90. (15.00)
1

⎪ ; x ⩾ 2
⎪ x

2
f (x) = ⎨ ax + 2b; −2 < x < 2


⎪ 1
− ; x ⩽ −2
x

Continuous at x = 2 ⇒ = 1

2
a

4
+ 2b

Continuous at x = −2 ⇒ = 1

2
a

4
+ 2b

Since, it is differentiable at x = 2
1
− = 2ax
2
x

Differentiable at x
−1 −1 3
= 2 ⇒ = 4a ⇒ a = ,b =
4 16 8
JEE Main 30-01-2024 (Evening Shift)

Questions

1. A block of ice at 10 C is slowly heated and converted to steam at 100


∘ ∘
C . Which of the following curves
represent the phenomenon qualitatively.

(a)

(b)

(c)

(d)
2. For the photoelectric effect, the maximum kinetic energy (E ) of the photoelectrons is plotted against the
k

frequency (v) of the incident photons as shown in figure. The slope of the graph gives

(a) Work function of the metal


(b) Charge of electron
(c) Planck’s constant
(d) Ratio of Planck’s constant to electric charge

3. Match List I with List II

List I List II


A. Gauss’s law of magnetostatics I. ⃗
∮ E . da =
1
∫ ρdV
ε0



B. Faraday’s law of electro magnetic induction II. ⃗
∮ B . da = 0

→ −

C. Ampere’s law III. ⃗
∮ E . dl = −
d ⃗
∫ B . da
dt


D. Gauss’s law of electrostatics IV ⃗
∮ B . dl = μ0 I

Choose the correct answer from the options given below:


(a) A – I, B – III, C – IV, D – II
(b) A – III, B – IV, C – I, D – II
(c) A – II, B – III, C – IV, D – I
(d) A – IV, B – II, C – III, D – I

4. A beam of unpolarised light of intensity I is passed through a polaroid A and then through another polaroid
0

B which is oriented so that its principal plane makes an angle of 45 relative to that of A. The intensity of

emergent light is:


I0
(a) 2

(b) I 0

(c) I0

(d) I0

5. Escape velocity of a body from earth is 11.2 km/s. If the radius of a planet be one-third the radius of earth
and mass be one-sixth that of earth, the escape velocity from the planet is:
(a) 8.4 km/s
(b) 7.9 km/s
(c) 11.2 km/s
(d) 4.2 km/s
6. If mass is written as m = kc P
G

1

2 h
1

2
then the value of P will be : (Constants have their usual meaning with
k a dimensionless constant)
1
(a) 2
1
(b) − 3
1
(c) 3

(d) 2

7. Projectiles A and B are thrown angle of 45 and 60 with vertical respectively from top of a 400 m high
∘ ∘

tower. If their ranges and times of flight are same, the ratio of their speeds of projection v : v is: A B

[Take g = 10ms ] −2

(a) √– 2 : 1

(b) 1 : √2

(c) 1 : √3
(d) 1 : 2

8. An electron revolving in nth Bohr orbit has magnitude moment μ n. If μ n αn


x
, the value of x is
(a) 3
(b) 0
(c) 2
(d) 1

9. If the total energy transferred to a surface in time t is 6.48 × 10 J , then the magnitude of the total
5

momentum delivered to this surface for complete absorption will be:


(a) 2.16 × 10 kg m/s
−3

(b) 2.46 × 10 kg m/s


−3

(c) 4.32 × 10 kg m/s


−3

(d) 1.58 × 10 kg m/s


−3

10. In the given circuit, the voltage across load resistance (R L) is:

(a) 14.00 V
(b) 8.50 V
(c) 8.75 V
(d) 9.00 V

11. If three moles of monoatomic gas (γ = ) is mixed with two moles of a diatomic gas (γ
5
=
7
) , the value
3 5

of adiabatic exponent γ for the mixture is


(a) 1.75
(b) 1.52
(c) 1.40
(d) 1.35
12. If 50 Vernier divisions are equal to 49 main scale divisions of a traveling microscope and one smallest
reading of main scale is 0.5 mm, the Vernier constant of travelling microscope is
(a) 0.01 mm
(b) 0.1 mm
(c) 0.1 cm
(d) 0.01 cm

13. In a nuclear fission reaction of an isotope of mass M, three similar daughter nuclei of same mass are
formed. The speed of a daughter nuclei in terms of mass defect ΔM will be:
−−−−
(a) c√
2ΔM

M
2

(b) ΔM c

3
−−−−−
(c) √
2cΔM

M
−−−−
(d) c√
3ΔM

14. A block of mass m is placed on a surface having vertical cross section given by y = . If coefficient of
x
2

friction is 0.5, the maximum height above the ground at which block can be placed without slipping is:
(a) m 1

(b) m 1

(c) m 1

(d) m 1

15. Choose the correct statement for processes A and B shown in figure.

(a) T
γ−1
= k for process A and PV = k for process B.
P
γ−1

(b) P
= k for process B and T = k for process A.
T
γ

(c) PV = k for process B and A.


(d) P V = k for process B and PV = k for process A.

16. When a potential difference V is applied across a wire of resistance R, it dissipates energy at a rate W. If the
wire is cur into two halves and these halves are connected mutually parallel across the same supply, the
energy dissipation rate will become:
(a) 4W
(b) 2W
(c) W 1

(d) W 1

4
17. A block of mass 1 kg is pushed up a surface inclined to horizontal at an angle of 60 by a force of 10N

parallel to the inclined surface as shown in figure. When the block is pushed up by 10m along inclined
surface, the work done against frictional force is:
2
[g = 10m/s ]

(a) 10 J

(b) 5√3J

(c) 5 × 10√3J
(d) 5J

18. A particle of charge ‘-q’ and mass ‘m’ moves in a circle of radius ‘r’ around an infinitely long line charge of
linear charge density ‘+λ’. Then time period will be given as:
(Consider k as Coulomb’s constant)

− −−
(a) T =
1

2kλq

2π m

− −−
m
(b) T = 2πr√
2kλq


− −−
1 m
(c) T =
2πr

2kλq

(d) T
2
=

2kλq
m
r
3

19. Three blocks A, B and C are pulled on a horizontal smooth surface by a force of 80N as shown in figure

The tensions T and T in the string are respectively:


1 2

(a) 60N, 80N


(b) 88N, 96N
(c) 80N, 100N
(d) 40N, 64N

20. An alternating voltage V (t) = 220 sin 100πt voltis applied to a purely resistive load of 50Ω. The time
taken for the current to rise from half of the peak value to the peak value is:
(a) 5 ms
(b) 3.3 ms
(c) 2.2 ms
(d) 7.2 ms

21. Two discs of moment of inertia I = 4kgm and I = 2kgm , about their central axes & normal to their
1
2
2
2

planes, rotating with angular speeds 10 rad/s & 4 rad/s respectively are brought into contact face to face
with their axes of rotation coincident. The loss in kinetic energy of the system in the process is ____ J.
22. A big drop is formed by coalescing 1000 small identical drops of water. If E be the total surface energy of
1

1000 small drops of water and E be the surface energy of single big drop of water, then E : E is x : 1
2 1 2

where x = ____.

23. A point source is emitting sound waves of intensity 16 × 10 W m at the origin. The difference in
−8 −2

intensity (magnitude only) at two points located at a distance of 2m and 4m from the origin respectively will
be _____ ×10 W m −8 −2

24. In an experiment to measure the focal length (f) of a convex lens, the magnitude of object distance (x) and
the image distance (y) are measured with reference to the focal point of the lens. The y-x plot is shown in
figure.
The focal length of the lens is _____ cm.

25. A power transmission line feeds input power at 2.3kV to a step down transformer with its primary winding
having 3000 turns. The output power is delivered at 230V by the transformer. The current in the primary of
the transformer is 5A and its efficiency is 90%. The winding of transformer is made of copper. The output
current of transformer is ____ A.

26. Two identical charged spheres are suspended by strings of equal lengths. The strings make an angle of 37 ∘

with each other. When suspended in a liquid of density 0.7g/cm , the angle remains same. If density of
3

material of the sphere is 1.4g/cm , the dielectric constant of the liquid is _____ (tan 37 = )
3 ∘ 3

27. A vector has magnitude same as that of A ⃗ = 3^i + 4^j and is parallel to B⃗ = 4^i + 3^j. The x and y
components of this vector in first quadrant are x and 3 respectively where x = ____.

28. Two resistance of 100Ω and 200Ω are connected in series with a battery of 4V and negligible internal
resistance. A voltmeter is used to measure voltage across 100Ω resistance, which gives reading as 1V. The
resistance of voltmeter must be _____ Ω.

29. The current of 5A flows in a square loop of sides 1m is placed in air. The magnetic field at the centre of the

loop is X√2 × 10 T . The value of X is ____.
−7

30. A simple pendulum is placed at a place where its distance from the earth’s surface is equal to the radius of
the earth. If the length of the string is 4m, then the time period of small oscillations will be ____ s. [take
2
g = π ms
−2
]
31. The molecule/ion with square pyramidal shape is:
(a) PF5
(b) [Ni(CN)4]2–
(c) PCl5
(d) BrF5

32. Given below are two statements:


Statement-I: Since fluorine is more electronegative than nitrogen, the net dipole moment of NF3 is greater
than NH3.
Statement-II: In NH3, the orbital dipole due to lone pair and the dipole moment of NH bonds are in
opposite direction, but in NF3 the orbital dipole due to lone pair and dipole moments of N-F bonds are in
same direction.

In the light of the above statements. Choose the most appropriate from the options given below.
(a) Both statement I and Statement II are true.
(b) Statement I is true but Statement II is false.
(c) Statement I is false but Statement II is true.
(d) Both Statement I and Statement II are false.

33. A and B formed in the following reactions are:


CrO2Cl2 + 4NaOH → A + 2NaCl + 2H2O
A + 2HCl + 2H2O2 → B + 3H2O
(a) A = Na2Cr2O4, B = CrO4
(b) A = Na2CrO4, B = CrO5
(c) A = Na2Cr2O7, B = CrO5
(d) A = Na2Cr2O7, B = CrO3

34. If a substance ‘A’ dissolves in solution of a mixture of ‘B’ and ‘C’ with their respective number of moles as
nA, nB and nC, mole fraction of C in the solution is:
nB
(a) nA +nB
nC
(b) nA +nB +nC
nC
(c) nA −nB −nC
nC
(d) nA ×nB ×nC

35. Salicylaldehyde is synthesized from phenol, when reacted with


(a) CCl4, NaOH
(b) HCCl3, NaOH
(c) CO2, NaOH

(d)

, NaOH
36. The correct stability order of carbocations is
+ + + +
(a) (CH ) C > (CH ) C H > CH − C H
3 3 3 2 > C H3
3 2

(b)

+ + + +
(c) C H3 > (CH3 ) C H > CH3 − C H2 > (CH3 )3 C
2
+ + +
(d) (CH3 ) C
+
> CH3 − C H2 > (CH3 )2 C H + C H3
3

37. Which among the following purification methods is based on the principle of “Solubility” in two different
solvents?
(a) Sublimation
(b) Differential Extraction
(c) Column Chromatography
(d) Distillation

38. Given below are two statements: One is labelled as Assertion A and the other is labelled as Reason R.
Assertion A: H2Te is more acidic than H2S.
Reason R: Bond dissociation enthalpy of H2Te is lower than H2S.
In the light of the above statements. Choose the most appropriate from the options given below.
(a) Both A and R are true but R is NOT the correct explanation of A.
(b) A is false but R is true.
(c) A is true but R is false.
(d) Both A and R are true and R is the correct explanation of A.

39. Choose the correct statements about the hydrides of group 15 elements.
A. The stability of the hydrides decreases in the order NH3 > PH3 > AsH3 > SbH3 > BiH3
B. The reducing ability of the hydrides increases in the order NH3 < PH3 < AsH3 < SbH3 < BiH3
C. Among the hydrides, NH3 is strong reducing agent while BiH3 is mild reducing agent.
D. The basicity of the hydrides increases in the order NH3 < PH3 < AsH3 < SbH3 < BiH3
Choose the most appropriate from the option given below:
(a) A and D only
(b) B and C only
(c) C and D only
(d) A and B only
40. m–chlorobenzaldehyde on treatment with 50% KOH solution yields

(a)

(b)

(c)

(d)
41. The products A and B formed in the following reaction scheme are respectively.

(a)

(b)

(c)

(d)

42. Product A and B formed in the following set of reactions are:

(a)

(b)

(c)

(d)
43. The orange colour of K2Cr2O7 and purple colour of KMnO4 is due to
(a) d → d transition in K2Cr2O7 and charge transfer transitions in KMnO4.
(b) d → d transition in KMnO4 and charge transfer transitions in K2Cr2O7.
(c) Charge transfer transition in both.
(d) d → d transition in both.

44. Reduction potential of ions are given below:


ClO , Eo = 1.19 V

IO , Eo = 1.65 V

BrO , Eo = 1.74 V

The correct order of their oxidising power is:


(a) BrO > ClO > IO

4

4

(b) ClO > IO > BrO


4

4

(c) BrO > IO > ClO


4

4

(d) IO > BrO > ClO


4

4

45. Alkaline oxidative fusion of MnO2 gives “A” which on electrolytic oxidation in alkaline solution produces
B. A and B respectively are:
(a) MnO and Mn2O7
2−

(b) Mn2O3 and MnO 2−

(c) Mn2O7 and MnO −

(d) MnO 2−

4
and MnO −

46. The coordination geometry around the manganese in decacarbonyldimanganese(0)


(a) Trigonal bipyramidal
(b) Square planar
(c) Square pyramidal
(d) Octahedral

47. Given below are two statements:


Statement – I: Along the period, the chemical reactivity of the element gradually increases from group 1 to
group 18.
Statement – II: The nature of oxides formed by group 1 element is basic while that of group 17 elements is
acidic.
In the light above statements, choose the most appropriate from the questions given below:
(a) Both statement I and Statement II are true.
(b) Statement I is false but Statement II is true.
(c) Both Statement I and Statement II is false.
(d) Statement I is true but Statement II is False.

48. The solution from the following with highest depression in freezing point/lowest freezing point is
(a) 180 g of acetic acid dissolved in benzene
(b) 180 g of benzoic acid dissolved in benzene
(c) 180 g of glucose dissolved in water
(d) 180 g of acetic acid dissolved in water
49. Given below are two statements:
Statement – I: High concentration of strong nucleophilic reagent with secondary alkyl halides which do not
have bulky substituents will follow SN2 mechanism.
Statement – II: A secondary alkyl halide when treated with a large excess of ethanol follows SN1
mechanism.
In the light of the above statements, choose the most appropriate from the questions given below:
(a) Both statement I and Statement II are false.
(b) Statement I is false but Statement II is true.
(c) Both statement I and Statement II are true.
(d) Statement I is true but Statement II is false.

50. IUPAC name of following compound is

(a) 3–Aminobutanenitrile
(b) 2–Aminopentanenitrile
(c) 3–Aminopropanenitrile
(d) 2–Aminobutanenitrile

51. The total number of correct statements, regarding the nucleic acids is_________.
A. RNA is regarded as the reserve of genetic information.
B. DNA molecule self-duplicates during cell division
C. DNA synthesizes proteins in the cell.
D. The message for the synthesis of particular proteins is present in DNA
E. Identical DNA strands are transferred to daughter cells.

52. The pH of an aqueous solution containing 1M benzoic acid (pKa = 4.20) and 1M sodium benzoate is 4.5.
The volume of benzoic acid solution in 300 mL of this buffer solution is _______ mL.
(given: log 2 = 0.3)

53. NO2 required for a reaction is produced by decomposition of N2O5 in CCl4 as by equation
2N2O5(g) → 4NO2(g) + O2(g)
The initial concentration of N2O5 is 3 mol L–1 and it is 2.75 mol L–1 after 30 minutes.
The rate of formation of NO2 is x × 10–3 mol L–1 min–1, value of x is ________. (nearest integer)

54. Total number of species from the following which can undergo disproportionation reaction _______.
H2O2, ClO , P4, Cl2, Ag, Cu+1, F2, NO2, K+

55. 2-chlorobutane + Cl2 → C4H8Cl2(isomers)


Total number of optically active isomers shown by C4H8Cl2, obtained in the above reaction is_______.
56. Number of complexes which show optical isomerism among the following is ________.
cis – [Cr(ox)2Cl2]3–, [Co(en)3]3+,
cis – [Pt(en)2Cl2]2+, cis – [Co(en)2Cl2]+ ,
trans – [Pt(en)2Cl2]2+, trans – [Cr(ox)2Cl2]3–

57. Number of geometrical isomers possible for the given structure is/are ___________.

58. Number of metal ions characterized by flame test among the following is _______.
Sr2+, Ba2+, Ca2+, Cu2+, Zn2+, Co2+, Fe2+

59. Two reactions are given below:


2Fe(s) + O2(g) → Fe2O3(s), ∆Hº = – 822kJ / mol
3

C(s) + O2(g) → CO(g), ∆Hº = – 110 kJ / mol


1

Then enthalpy change for following reaction


3C(s) + Fe2O3(s) → 2Fe(s) + 3CO(g)

60. Number of spectral lines obtained in He+ spectra, when an electron makes transition from fifth excited state
to first excited state will be

61. Let A (α, 0) and B (0, β) be the points on the line 5x + 7y = 50 . Let the point P divide the line segment
2 2
y
AB internally in the ratio 7 : 3. Let 3x − 25 = 0 be a directrix of the ellipse E :
x

a
2
+ = 1 and the
2
b

corresponding focus be S . If from S , the perpendicular on the x-axis passes through P , then the length of
the latus rectum of E is equal to,
(a)
32

(b) 25

(c) 32

(d) 25

62. Let P be a point on the hyperbola H : −


x
2 y
2

= 1, in the first quadrant such that the area of triangle


9 4
−−
formed by P and the two foci of H is 2√13 . Then, the square of the distance of P from the origin is
(a) 22
(b) 18
(c) 20
(d) 26
63. Let f (x) = (x + 3) (x − 2) , x ∈ [−4, 4] . If M and m are the maximum and minimum values of f ,
2 3

respectively in [−4, 4] , then the value of M − m is


(a) 608
(b) 392
(c) 600
(d) 108

64. Let f : R → R be defined as f (x) = ae


2x
+ be
x
+ cx. If f (0) = −1, f

(loge 2) = 21 and
log 4
e

∫ (f (x) − cx) dx =
39

2
, then the value of |a + b + c| equals
0

(a) 12
(b) 10
(c) 8
(d) 16

65. For α, β ∈ (0, ) , let 3 sin(α + β)


π
= 2 sin(α − β) and a real number k be such that tan α = k tan β .
2

Then, the value of k is equal to


2
(a) − 3

(b) −5
2
(c) 3

(d) 5

66. If the domain of the function f (x) = log (


2x+3
) + cos(
2x−1
) is (α, β], then the value of 5β − 4α
e 2
4x +x−3 x+2

is equal to
(a) 9
(b) 11
(c) 10
(d) 12

67. Suppose 2 − p, p, 2 − α, α are the coefficients of four consecutive terms in the expansion of (1 + x) . n

Then the value of p − α + 6α + 2p equals


2 2

(a) 8
(b) 4
(c) 10
(d) 6

68. 2
x + 3x + a , x ⩽ 1
Let a and b be real constants such that the function f defined by f (x) = { be
bx + 2 , x > 1
2

differentiable on R . Then, the value of ∫ f (x) dx equals


−2

(a) 21
(b) 19

(c) 15

(d) 17
69. 2

Let a⃗ and b ⃗ be two vectors such that ∣∣b∣∣⃗ = 1 and ∣∣b ⃗ × a∣∣⃗ = 2 . Then ∣∣(b ⃗ × a)
⃗ − b
∣⃗

is equal to
(a) 4
(b) 1
(c) 5
(d) 3

70. Consider the system of linear equations x + y + z = 5, x + 2y + λ 2


z = 9, x + 3y + λz = μ, where
λ, μ ∈ R. Then, which of the following statement is NOT correct?

(a) System has infinite number of solutions if λ = 1 and μ = 13


(b) System has unique solution if λ ≠ 1 and μ ≠ 13
(c) System is consistent if λ ≠ 1 and μ = 13
(d) System is inconsistent if λ = 1 and μ ≠ 13

71. Let L ^ ^ ^ ^ ^ ^
, ^ ^ ^ ^ ^
: r ⃗ = ( i − j + 2k ) + λ ( i − j + 2k ) , λ ∈ R L2 : r ⃗ = ( j − k ) + μ (3 i + j + pk ) , μ ∈ R,
1

and L 3
^ ^ ^
: r ⃗ = δ (l i + m j + nk ) , δ ∈ R be three lines such that L is perpendicular to L and L is
1 2 3

perpendicular to both L and L . Then, the point which lies on L is


1 2 3

(a) (−1, −7, 4)


(b) (1, −7, 4)
(c) (1, 7, −4)
(d) (−1, 7, 4)

72. Let f : R − {0} → R be a function satisfying f ( x


) =
f (x)
for all x, y, f (y) ≠ 0 . If f ′
(1) = 2024,
y f (y)

then
(a) x f ′
(x) + 2024f (x) = 0

(b) x f ′
(x) − 2024f (x) = 0

(c) x f ′
(x) + f (x) = 2024

(d) x f ′
(x) − 2023f (x) = 0

73. Bag A contains 3 white, 7 red balls and Bag B contains 3 white, 2 red balls. One bag is selected at random
and a ball is drawn from it. The probability of drawing the ball from the bag A, if the ball drawn is white, is
(a) 10
3

(b) 1

(c) 1

3
1
(d) 4

74. If x − y + 2hxy + 2gx + 2f y + c = 0 is the locus of a point, which moves such that it is always
2 2

equidistant from the lines x + 2y + 7 = 0 and 2x − y + 8 = 0, then the value of g + c + h − f equals


(a) 29
(b) 14
(c) 6
(d) 8
75. ⎛
x 0 0

Let R = ⎜0 y 0⎟ be a non-zero 3 × 3 matrix, where
⎝ ⎠
0 0 z

x sin θ = y sin θ (θ +

3
) = z sin(θ +

3
. For a square matrix M , let trace (M )
) ≠ 0, θ ∈ (0, 2π)

denote the sum of all the diagonal entries of M . Then, among the statements:
(I) Trace (R ) = 0
(II) If trace (adj (adj (R))) = 0, then R has exactly one non-zero entry.
(a) Only (I) is true
(b) Neither (I) nor (II) is true
(c) Only (II) is true
(d) Both (I) and (II) are true

76. Let a⃗ ^ ^ ^
= i + α j + βk , α, β ∈ R . Let a vector b ⃗ be such that the angle between a⃗ and b ⃗ is π
2

and ∣∣b∣∣⃗ = 6 .
4
2

If a⃗ ⋅ b ⃗ = 3√ 2 , then the value of (α 2 ∣ ∣
+ β ) a⃗ × b ⃗
2
∣ ∣
is equal to
(a) 75
(b) 90
(c) 95
(d) 85

77. If z is a complex number, then the number of common roots of the equations z 1985
+ z
100
+ 1 = 0 and
+ 2z + 1 = 0, is equal to
3 2
z + 2z

(a) 0
(b) 2
(c) 1
(d) 3

78. Let a and b be two distinct positive real numbers. Let 11th term of a GP, whose first term is a and third term
is b, is equal to p term of another GP, whose first term is a and fifth term is b. Then p is equal to
th

(a) 24
(b) 21
(c) 20
(d) 25

79. Let f : R → R be a function defined by f (x) =


x
1
, and g (x) = f (f (f (f (x)))) . Then,
4 4
(1+x )

√ 2√ 5

18 ∫
2
x g (x) dx is equal to
0

(a) 42
(b) 36
(c) 39
(d) 33
80. Let y = f (x) be a thrice differentiable function in (−5, 5) . Let the tangents to the curve y = f (x) at
(1, f (1)) and (3, f (3)) makes an angles and , respectively with positive x-axis. If
π

6
π

4
3

where α, β are integers, then the value of α + β equals
′ 2 ′′
27 ∫ ((f (t)) + 1) f (t) dt = α + β√ 3
1

(a) 36
(b) 26
(c) −14
(d) −16

81. n
(
n 2
Ck )
n−1
n
Ck
n
Ck+1
Let α = ∑ (
k+1
) and β = ∑ (
k+2
. If 5α
) = 6β , then n equals ______.
k=0 k=0

82. The variance σ of the data 2

xi 0 1 5 6 10 12 17
fi 3 2 3 2 6 3 3

is ______.

83. The area of the region enclosed by the parabola (y − 2) 2


= x − 1 , the line x − 2y + 4 = 0 and the
positive coordinate axes is _____.

84. n

Let S be the sum to n -terms of an arithmetic progression 3, 7, 11, . . . . If 40


n < (
6

n(n+1)
∑ Sk ) < 42 ,
k=1

then n equals _____.

85. The number of real solutions of the equation x (x 2


+ 3 |x| + 5 |x − 1| + 6 |x − 2|) = 0 is ____.

86. The number of symmetric relations defined on the set {1, 2, 3, 4} which are not reflexive is ______.

87. In an examination of Mathematics paper, there are 20 questions of equal marks and the question paper is
divided into three sections: A, B and C . A student is required to attempt total 15 questions taking at least 4
questions from each section. If section A has 8 questions, section B has 6 questions and section C has 6
questions, then the total number of ways a student can select 15 questions is _____.

88. Let a line passing through the point (−1, 2, 3) intersect the lines L 1 :
x−1
=
y−2
=
z+1
at M (α, β, γ)
3 2 −2
2
(α+β+γ)
and L at N (a, b, c) . Then, the value of equals ____.
x+2 y−2 z−1
2 : = =
−3 4 2
−2
(a+b+c)

89. Let Y = Y (x) be a curve lying in the first quadrant such that the area enclosed by the line
Y − y = Y

(x) (X − x) and the co-ordinate axes, where (x, y) is any point on the curve, is always
2

. If Y then 12Y equals _____.


−y


+ 1, Y (x) ≠ 0 (1) = 1, (2)
2Y (x)
90. Consider two circles C : x + y = 25 and C : (x − α) + y = 16, where α ∈ (5, 9). Let the angle
1
2 2
2
2 2

between the two radii (one to each circle) drawn from one of the intersection points of C and C be
1 2

√ 63
. If the length of common chord of C and C is β, then the value of (αβ) equals _____.
−1 2
sin ( ) 1 2
8
Answer Key

1. A 2. C 3. C 4. D 5. B 6. A
7. B 8. D 9. A 10. C 11. B 12. D
13. A 14. C 15. D 16. A 17. D 18. B
19. D 20. B 21. 24 22. 10 23. 3 24. 20
25. 45 26. 2 27. 4 28. 200 29. 40 30. 8
31. D 32. D 33. B 34. B 35. B 36. A
37. B 38. D 39. D 40. A 41. C 42. C
43. C 44. C 45. D 46. D 47. B 48. D
49. C 50. A 51. 3 52. 100 53. 17 54. 6
55. 3 56. 4 57. 4 58. 4 59. 492 60. 10
61. C 62. A 63. A 64. C 65. B 66. D
67. A 68. D 69. D 70. B 71. D 72. B
73. C 74. B 75. B 76. B 77. B 78. B
79. C 80. B 81. 10.00 82. 29.00 83. 5.00 84. 9.00
85. 1.00 86. 960.00 87. 11376.00 88. 196.00 89. 20.00 90. 1575.00
Solutions

1. (A)

2. (C)
K . E. = hf − ϕ

tan θ = h

3. (C)
Maxwell’s eqaution

4. (D)
Intensity of emergent light
I0 ∘ I0
2
cos 45 =
2 4

5. (B)
RE ME
RP = , MP =
3 6
−−−−
2GMe
Ve = √ . . . . . (i)
Re
−−−−−
2GMP
Vp = √ . . . (ii)
RP

Ve –
= √2
Vp

Ve 11.2
VP = = = 7.9km/ sec
√2 √2

6. (A)
1 1
P −
m = kc G 2 h 2

1 1
P −
1 0 0 −1 −1 3 −2 2 2 −1 2
M L T = [LT ] [M L T ] [M L T ]

By comparing P =
1

2
7. (B)

For u A
&uB time of flight and range can not be same.

8. (D)
Magnetic moment = iπr
2

evr
μ =
2
1 2
μ ∝ ( )n
n

μαn

x = 1

9. (A)
5
E 6.48×10 −3
p = = 8
= 2.16 × 10
C 3×10

10. (C)

14
i = = 3.5mA
4

V L = iR L = 3.5 × 2.5 volt

= 8.75 volt

11. (B)
f1 = 3, f2 = 5

n1 = 3, n2 = 2
n 1 f +n 2 f 9+10
1 2 19
fmixture = = =
n 1 +n 2 f 5

2×5 29
γ mixture = 1 + = = 1.52
19 19

12. (D)
50V + S = 49S + S

S = 50 (S − V )

.5 = 50 (S − V )
0.5 1
S − V = = = 0.01mm
50 100
13. (A)
(X) → (Y ) + (Z ) + (P )
M M M
M
3 3 3

2 1 M 2 1 M 2 1 M 2
ΔM c = V + V + V
2 3 2 3 2 3
−−−−
2ΔM
V = c√
M

14. (C)
dy x 1
= tan θ = = μ =
dx 2 2
1
x = 1, y =
4

15. (D)
Steeper curve (B) is adiabatic
Adiabatic ⇒ P V =Const. v

v
Or P ( ) = const. T

P
v

= const.
T
v−1
P

Curve (A) is isothermal


T = const.
PV = const.

16. (A)
2
v
= W . . . . . (i)
R
2
v ′
= W . . . . (ii)
1 R
( )
2 2

From (i) & (ii), we get



W = 4W

17. (D)
Work done again frictional force
= μN × 10

= 0.1 × 5 × 10 = 5J

18. (B)
2kλq
2
= mω r
r
2kλq
2
ω =
2
mr
2 2kλq

( ) =
T 2
mr

− −−
m
T = 2πr√
2kλq
19. (D)
F 80 2
aA = aB = aC = = = 8m/s
5+3+2 10

T 1 = 5 × 8 = 40

T 2 − T 1 = 3 × 8 ⇒ T 2 = 64

20. (B)
Rising half to peak
T
t =
6
2π π π 1
t = = = = = 3.33ms
6ω 3ω 300π 300

21. (24)
I1 ω1 + I2 ω2 = (I1 + I2 ) ω0 (C . O. A. M )

gives ω 0 = 8rad/s
1 2 1 2
E1 = I1 ω + I2 ω = 216J
2 1 2 2
1 2
E2 = (I1 + I2 ) ω = 192J
2 0

∴ ΔE = 24J

22. (10)
4 3 4 3
ρ ( πr ) 1000 = πR ρ
3 3

R = 10r
2
E1 = 1000 × 4πr × S
2
E2 = 4π(10r) S
E1 10
= , x = 10
E2 1

23. (3)
We know for point source, I ∝
1

2
=
K

2
r 4πr

r1 = 2m, r2 = 4m = 2r1
−8 2
I0 = 16 × 10 W /m

At 2m,
−8
I0 16×10
I1 = 2
=
4
(2)
−8
I0
I1 = 4 × 10
−8
&I 2 =
42
=
16×10

16
= 10
−8

−8 2
|I2 − I1 | = 3 × 10 W /m , x = 3
24. (20)
1 1 1
− =
f +20 −(f +20) f

2 1
= f = 20cm
f +20 f

Or x 1 x2 = f
2
gives f = 20cm

25. (45)
P i = 2300 × 5watt

P 0 = 2300 × 5 × 0.9 = 230 × I2

I2 = 45A

26. (2)

T cos θ = mg

T sin θ = F e
Fe
tan θ =
mg
Fe
tan θ = . . . . (i)
ρ Vg
B

Fe
tan θ = k
. . . . . (ii)

(ρ −ρ )V g
B L

From Eq. (i) & (ii)


ρ B V g = (ρ B − ρ L ) kV g

1.4 = 0.7k

k = 2

27. (4)
^ ^
5(4 i +3 j )
¯ ¯∣ ^ ^ ^
N = ∣
∣A ∣ B = = 4i + 3j
5

∴ x = 4

28. (200)

R v 100 200
=
R v +100 3

3R v = 2R v + 200

R v = 200
29. (40)
μ i
0 1 1
B = 4 × ( + )
1 √2 √2
4π( )
2

−7

= 4 × 10 × 5 × 2 × √2
– −7
= 40√ 2 × 10 T

30. (8)
Acceleration due to gravity g
g

=
4
−−
4ℓ
T = 2π√
g

−−−
4×4
T = 2π√
g

4
T = 2π = 8s
π

31. (D)
BrF5

square pyramidal

32. (D)

33. (B)
(A)
CrO2Cl2 + 4NaOH → Na 2 CrO 4 + 2NaCl + 2H2O
(A)

Na2CrO4 + 2H2O2 + 2HCl → CrO + 5 2NaCl + 3H2O


Missing from balanced equation
(B)

34. (B)
nC
Mole fraction of C =
nA +nB +nC

35. (B)
36. (A)
More no. of hyperconjugable Hydrogens, more stable is the carbocations.

37. (B)
Different Extraction
Different layers are formed which can be separated in funnel. (Theory based).

38. (D)
Due to lower Bond dissociation enthalpy of H2Te it ionizes to give H+ more easily than H2S.

39. (D)
On moving down the group, bond strength of M–H bond decreases, which reduces the thermal stability but
increases reducing nature of hydrides, hence A and B are correct statements.

40. (A)
Meta–chlorobenzaldehyde will undergo
Cannizzaro reaction with 50% KOH to give m–chlorobenzoate ion and m–chlorobenzyl alcohol.

41. (C)
42. (C)

43. (C)
K2Cr2O7 → Cr+6 → No d – d transition
KMnO4 → Mn7+ → No d – d transition
are Charge transfer.

44. (C)
Higher the value of +ve SRP (Std. reduction potential) more is tendency to undergo reduction, so better is
oxidising power of reactant.
Hence, ox. Power:- BrO > IO > ClO

4

4

45. (D)
Alkaline oxidative fusion of MnO2:
2MnO2 + 4OH– + O2 → 2MnO 2−

4
+ 2H2O
Electrolytic oxidation of MnO 2−

4
in alkaline medium.
MnO 2−

4
→ MnO + e–

46. (D)
Mn2(CO)10

47. (B)
Chemical reactivity of elements decreases along the period therefore statement – I is false.
Group – 1 elements from basic nature oxides while group – 17 elements form acidic oxides therefore statement –
II is true.
48. (D)
∆Tf is maximum when i × m is maximum.
1) m1 = = 3, i = 1 + α
180

60

Hence
∆Tf = T (1 + α) kf = 3 × 1.86 = 5.58 oC(α << 1)
2) m2 = 180

60
= 3, i = 0.5, ∆Tf = 3

2
× kf’ = 7.68oC
3) m3 = 180

122
= 1.48, i = 0.5, ∆Tf = 1.48

2
× kf’ = 3.8oC
4) m4 = 180

180
= 1, i = 1, ∆Tf = 1 × kf’ = 1.86oC
As per NCERT, kf ' (H2O) = 1.86 k · kg mol–1
kf ' (Benzene) = 5.12 k · kg mol–1

49. (C)
Statement – I: Rate of SN2 ∝ [R–X][Nu–]
SN2 reaction is favoured by high concentration of nucleophile (Nu–) & less crowding in the substrate molecule.
Statement – II: Solvolysis follows SN1 path.
Both are correct Statements.

50. (A)

51. (3)
A. RNA is regarded as the reserve of genetic information. (False)
B. DNA molecule self-duplicates during cell division. (True)
C. DNA synthesizes proteins in the cell. (False)
D. The message for the synthesis of particular proteins is present in DNA. (True)
E. Identical DNA strands are transferred to daughter cells. (True)
52. (100)

1M Benzoic acid + 1M Sodium Benzoate


(Va ml) (Vs ml)
Millimole Va × 1 Vs × 1

pH = 4.5
[salt]
pH = pKa + log
[acid]

Vs
4.5 = 4.2 + log( )
Va

…. (1)
Vs
= 2
Va

Vs + Va = 300 …. (2)
Va = 100 ml

53. (17)
Rate of reaction (ROR)
Δ[N 2 O5 ] [NO2 ] Δ[O2 ]
1 1
= − = =
2 Δt 4 Δt Δt
Δ[N 2 O5 ] (2.75−3)
1 1 −1 −1
ROR = − = − mol L min
2 Δt 2 30
(−0.25)
1 −1 −1
ROR = − mol L min
2 30
1 −1 −1
ROR = mol L min
240
Δ[NO2 ]
Rate of formation of NO2 = Δt
= 4 × ROR
= 4

240
= 16.66 × 10–3 molL–1 min–1 ≃ 17 × 10–3

54. (6)
Intermediate oxidation state of element can undergo disproportionation.
H2O2, ClO , P4, Cl2, Cu+1, NO2

55. (3)
56. (4)
cis – [Cr(ox)2Cl2] 3– → can show optical isomerism
(no POS & COS)
[Co(en)3]3+ → can show (no POS & COS)
cis – [Pt(en)2Cl2]2+ → can show (no POS & COS)
cis – [Co(en)2Cl2]+ → can show (no POS & COS)
trans – [Pt(en)2Cl2]2+ → can’t show (contains POS & COS)
trans – [Cr(ox)2Cl2]3– → can’t show (contains POS & COS)

57. (4)
3 stereocenteres, symmetrical
Total Geometrical isomers → 4. EE, ZZ, EZ (two isomers)

58. (4)
All the following metal ions will respond to flame test.
Sr2+, Ba2+, Ca2+, Cu2+

59. (492)
2Fe(s) + O2(g) → Fe2O3(s), ∆Hº = – 822kJ / mol …. (1)
3

C(s) + O2(g) → CO(g), ∆Hº = – 110 kJ / mol …. (2)


1

3C(s) + Fe2O3(s) → 2Fe(s) + 3CO(g), ∆H3 = ?


(3) = 3 × (2) – (1)
∆H3 = 3 × ∆H2 – ∆H1
= 3(–110) + 822
= 492 kJ/mole

60. (10)
5th excited state ⇒ n1 = 6
1st excited state ⇒ n2 = 2
∆n = n1 – n2 = 6 – 2 = 4
Maximum number of spectral lines
Δn(Δn+1) 4(4+1)
= = = 10
2 2
61. (C)
A = (10, 0) ⎫

50 ⎬ P = (3, 5)
B = (0, ) ⎭

7

ae = 3
a 25
=
e 3

a = 5

b = 4
2

Length of LR =
2b

a
=
32

62. (A)

2 2
x y
− = 1
9 4
2 2
a = 9, b = 4
2
2 2 2 2 b
b = a (e − 1) ⇒ e = 1 +
2
a
2 4 13
e = 1 + =
9 9
−−
√ 13 13 − −
e = ⇒ s 1 s 2 = 2ae = 2 × 3 × √ = 2√ 13
3 3

− −
Area of ΔP S 1 S2 =
1

2
× β × s 1 s 2 = 2√ 13
1 − − − −
⇒ × β × (2√ 13 ) = 2√ 13 ⇒ β = 2
2
2
α
2 β α
2
2

− = 1 ⇒ − 1 = 1 ⇒ α = 18 ⇒ α = 3√ 2
9 4 9
−−−−−−
Distance of P from origin = √α 2 + β 2
− −−−− − −
= √ 18 + 4 = √ 22
63. (A)
′ 2 2 3
f (x) = (x + 3) .3(x − 2) + (x − 2) 2 (x + 3)
2
= 5 (x + 3) (x − 2) (x + 1)

f (x) = 0, x = −3, −1, 2

f (−4) = −216

f (−3) = 0, f (4) = 49 × 8 = 392

M = 392, m = −216

M − m = 392 + 216 = 608

Ans = 3

64. (C)
2x x
f (x) = ae + be + cx

f (0) = −1

a + b = −1
′ 2x x
f (x) = 2ae + be + c

f (ln 2) = 21

8a + 2b + c = 21
ln 4
2x x 39
∫ (ae + be ) dx =
2
0
ln 4
2x
ae x 39
[ + be ] =
2 2
0
a 39
⇒ 8a + 4b − − b =
2 2

15a + 6b = 39

15a − 6a − 6 = 39

9a = 45

⇒ a = 5

b = −6

c = 21 − 40 + 12 = −7

a + b + c − 8

|a + b + c| = 8

65. (B)
3 sin α cos β + 3 sin β cos α

= 2 sin α cos β − 2 sin β cos α

5 sin β cos α = − sin α cos β


1
tan β = − tan α
5

tan α = −5 tan β
66. (D)
and −1
2x+3 2x−1
> 0 ⩽ ⩽ 1
2 x+2
4x +x−3

and
2x+3 3x+1 x−3
> 0 ⩾ 0 & ⩽ 0
(4x−3)(x+1) x+2 x+2

−1
(−∞, −2) ∪ [ , ∞) . . . . (1)
3

(−2, 3] . . . . (2)

−1
[ , 3] . . . . (3)
3

(1) ∩ (2) ∩ (3)


3
( , 3]
4
3
α = , β = 3
4

5β − 4α = 15 − 3 = 12

67. (A)
2 − p, p, 2 − α, α

Binomial coefficients are


n
Cr ,
n
C r+1 ,
n
C r+2 ,
n
C r+3 respectively
n n
⇒ Cr + C r+1 = 2
n+1
⇒ C r+1 = 2 . . . . (1)

Also, n
C r+2 +
n
C r+3 = 2
n+1
⇒ C r+3 = 2 . . . . (2)

From (1) and (2)


n+1 n+1
C r+1 = C r+3

⇒ 2r + 4 = n + 1

n = 2r + 3
2r+4
C r+1 = 2

Data Inconsistent

68. (D)
f is continuous

∴ 4 + a = b + 2

a = b − 2

2x + 3 , k < 1

f (x) = {
b , x > 1

f is differentiable
∴ b = 5

∴ a = 3
1 2
2
∫ (x + 3x + 3) dx + ∫ (5x + 2) dx
−2 1

1 2
3 2 2
x 3x 5x
= [ + + 3x] + [ + 2x]
3 2 2
−2 1

1 3 −8 5
= ( + + 3) − ( + 6 − 6) + (10 + 4 − − 2)
3 2 3 2

3 5
= 6 + + 12 − = 17
2 2
69. (D)
∣ ∣⃗ ∣ ⃗ ∣
b = 1 & b × a⃗ = 2
∣ ∣ ∣ ∣

⃗ ⃗ ⃗ ⃗ ⃗ ⃗
(b × a ) ⋅ b = b ⋅ (b × a ) = 0

2 2 2
∣ ∣ ∣ ⃗ ∣ ∣ ∣⃗
(b ⃗ × a) − b ⃗ = b × a⃗ + b
∣ ∣ ∣ ∣ ∣ ∣

70. (B)
∣ 1 1 1 ∣
∣ 2

1 2 λ = 0
∣ ∣
∣ 1 3 λ ∣
2
⇒ 2λ − λ − 1 = 0
1
λ = 1, −
2

∣ 1 1 5 ∣
∣ 2

2 λ 9 = 0 ⇒ μ = 13
∣ ∣
∣ 3 λ μ∣

Infinite solution λ = 1 & μ = 13


For unique solution λ ≠ 1
For no solution λ = 1 & μ ≠ 13
If λ ≠ 1 and μ ≠ 13
Considering the case when λ = − and μ 1

2
≠ 13 this will generate no solution case

71. (D)
L1 ⊥L2 L3 ⊥L1 , L2

3 − 1 + 2P = 0

P = −1

∣ ^ ^ ^ ∣
i j k
∣ ∣
^ ^ ^
∣ 1 −1 2 ∣ = − i + 7 j + 4k
∣ ∣
∣ 3 1 −1 ∣

∴ (−δ, 7δ, 4δ) will lie on L 3

For δ = 1 the point will be (−1, 7, 4)

72. (B)

f (1) = 2024

f (1) = 1
f (x)
x
f ( ) =
y f (y)

Partially differentiating w. r. t. x
′ x 1 1 ′
f ( ). = f (x)
y y f (y)

y → x

f (x)
′ 1
f (1) . =
x f (x)

′ ′
2024f (x) = xf (x) ⇒ xf (x) − 2024f (x) = 0
73. (C)

E1 : A is selected
E2 : B is selected
E : white ball is drawn
E
P (E).P ( )
E1 E1
P ( ) =
E E E
P (E 1 ).P ( )+P (E 2 ).P ( )
E1 E2
1 3
×
2 10
= 1 3 1 3
× + ×
2 10 2 5

3 1
= =
3+6 3

74. (B)
Locus of point P (x, y) whose distance from
Gives
x + 2y + 7 = 0 & 2x − y + 8 = 0 are equal is
x+2y+7 2x−y+8
= ±
√5 √5
2 2
(x + 2y + 7) − (2x − y + 8) = 0

Combined equation of lines


(x − 3y + 1) (3x + y + 15) = 0
2 2
3x − 3y − 8xy + 18x − 44y + 15 = 0
2 2 8 44
x − y − xy + 6x − y + 5 = 0
3 3

2 2
x − y + 2hxy + 2gx 2 + 2f y + c = 0
4 22
h = , g = 3, f = − ,c = 5
3 3
4 22
g + c + h − f = 3 + 5 − + = 8 + 6 = 14
3 3

75. (B)
2π 4π
x sin θ = y sin(θ + ) = z sin(θ + ) ≠ 0
3 3

⇒ x, y, z ≠ 0

Also,
2π 4π
sin θ + sin(θ + ) + sin(θ + ) = 0 ∀θ ∈ R
3 3
1 1 1
⇒ + + = 0
x y z

⇒ xy + yz + zx = 0

(i) Trace (R) = x + y + z


If x + y + z = 0 and xy + yz + zx = 0

⇒ x = y = z = 0

Statement (i) is False


(ii) adj (adj (R)) = |R| R
trace (adj (adj (R)))

= xyz (x + y + z) ≠ 0

Statement (ii) is also False


76. (B)
2
∣ ∣⃗ –
b = 6; ⃗ ∣ b∣⃗ cos θ = 3√ 2
|a |
∣ ∣ ∣ ∣
2
2
⃗ ∣ ∣⃗ 2
|a | b cos θ = 18
∣ ∣
2
|a⃗| = 6

Also 1 + α 2
+ β
2
= 6
2 2
α + β = 5

To find
2
2
2

2 ∣ ∣⃗ 2
(α + β ) |a | b sin θ
∣ ∣
1
= (5) (6) (6) ( )
2

= 90

77. (B)
z
1985
+ z
100
+ 1 = 0 and z 3
+ 2z
2
+ 2z + 1 = 0
2
(z + 1) (z − z + 1) + 2z (z + 1) = 0
2
(z + 1) (z + z + 1) = 0
2
⇒ z = −1, z = w, w

Now putting z = −1 not satisfy


Now put z = w
1985 100
⇒ w + w + 1
2
⇒ w + w + 1 = 0

Also, z = w
2

3970 200
⇒ w + w + 1
2
⇒ w + w + 1 = 0

Two common root

78. (B)
1st G.P. ⇒ t1 = a, t 3 = b = ar
2
⇒ r
2
=
b

a
5
5
10 2 b
t11 = ar = a( r ) = a ⋅ ( )
a

2nd G.P. ⇒ T 1 = a, T 5 = ar
4
= b
1

4 b b 4
⇒ r = ( ) ⇒ r = ( )
a a

p−1

p−1 b 4
T p = ar = a( )
a

p−1
5
b b 4
t11 = T p ⇒ a( ) = a( )
a a

p−1
⇒ 5 = ⇒ p = 21
4
79. (C)
x
f (x) = 1

4 4
(1+x )
x

f (x) (1+x 4) 4
x
f of (x) = 1
= 1
= 1

4 4 x4 4 4 4
(1+f (x) ) (1+2x )
(1+ )
1+x 4
x
f (f (f (f (x)))) = 1

4 4
(1+4x )

√ 2√ 5
3
x
18 ∫ 1
dx
0 4 4
(1+4x )

Let 1 + 4x 4
= t
4

3 3
16x dx = 4t dt
3
3
18 t dt

4 t
1

3
3
9 t
= ( )
2 3
1
3
= (26) = 39
2

80. (B)
dy

y = f (x) ⇒ = f (x)
dx

dy π 1 1
′ ′
) = f (1) = tan = ⇒ f (1) =
dx 6 √3 √3
(1,f (1))

dy π
′ ′
) = f (3) = tan = 1 ⇒ f (3) = 1
dx 4
(3,f (3))

3
′ 2 ′′

27 ∫ ((f (t)) + 1) f (t) dt = α + β√ 3
1

3
′ 2 ′′
I = ∫ (( f (t)) + 1) f (t) dt
1
′ ′′
f (t) = z ⇒ f (t) dt = dz

z = f (3) = 1
′ 1
z = f (1) =
√3

1
1
3
2 z
I = ∫ (z + 1) dz = ( + z)
3 1
1
√3
√3

1 1 1 1
= ( + 1) − ( ⋅ + )
3 3 3√ 3 √3

4 10 4 10 –
= − = − √3
3 9√ 3 3 27
– 4 10 – –
α + β√ 3 = 27 ( − √ 3) = 36 − 10√ 3
3 27

α = 36, β = −10

α + β = 36 − 10 = 26
81. (10.00)
n
n n
Ck ⋅ Ck n+1
α = ∑ ⋅
k+1 n+1
k=0
n
1 n+1 n
= ∑ C k+1 ⋅ C n−k
n+1
k=0

1 2n+1
α = ⋅ C n+1
n+1
n−1
n
n Ck+1 n+1
β = ∑ Ck ⋅
k+2 n+1
k=0

n−1
1 n n+1
∑ C n−k . C k+2
n+1
k=0

1 2n+1
= . C n+2
n+1
2n+1
β Cn+2 2n+1−(n+2)+1
= =
α 2n+1
Cn+1 n+2

β n 5
= =
α n+2 6

n = 10

82. (29.00)

2
xi fi fi xi fi xi

0 3 0 0
1 2 2 2
5 3 15 75
6 2 12 72
10 6 60 600
12 3 36 432
17 3 51 867
2
∑ fi = 22 ∑ fi xi = 2048

∴ ∑ fi xi = 176
∑ f xi
So x̄ = ∑ f
i
=
176

22
= 8
i

For σ 2 1 2 2
= ∑ fi xi − (x̄)
N
1 2
= × 2048 − (8)
22

= 93.090964

= 29.0909
83. (5.00)
Solving the equations
(y − 2) = x − 1 and x − 2y + 4
2
= 0

x = 2 (y − 2)

2
x
= x − 1
4
2
x − 4x + 4 = 0
2
(x − 2) = 0

x = 2
3

Enclose area (w.r.t y-axis) = ∫ x dy− Area of Δ .


0

3
2 1
= ∫ ((y − 2) + 1) dy − × 1 × 2
2
0

3
2
= ∫ (y − 4y + 5) dy − 1
0

3 3
y
2
= [ − 2y + 5y] − 1
3
0

= 9 − 18 + 15 − 1 = 5

84. (9.00)
Sn = 3 + 7 + 11+. . . . n terms
n 2
= (6 + (n − 1) 4) = 3n + 2n − 2n
2
2
= 2n + n
n n n
2
∑ Sk = 2 ∑ K + ∑ K
k=1 k=1 k=1

n(n+1)(2n+1) n(n+1)
= 2 ⋅ +
6 2

2n+1 1
= n (n + 1) [ + ]
3 2

n(n+1)(4n+5)
=
6
n
6
⇒ 40 < ∑ Sk < 42
n(n+1)
k=1

40 < 4n + 5 < 42

35 < 4n < 37

n = 9
85. (1.00)
x = 0 and x + 3 |x| + 5 |x − 1| + 6 |x − 2|
2
= 0

Here all terms are +ve except at x = 0


So there is no value of x
Satisfies this equation
Only solution x = 0
No of solution 1.

86. (960.00)
2 −n
n

Total number of relation both symmetric and reflexive = 2 2

2
n +n
( )

Total number of symmetric relation = 2 2

⇒ Then number of symmetric relation which are not reflexive


n(n+1) n(n−1)

⇒ 2 2 − 2 2

10 6
⇒ 2 − 2

⇒ 1024 − 64

= 960

87. (11376.00)
If 4 questions from each section are selected remaining 3 questions can be selected either in (1,
1, 1) or (3, 0, 0) or (2, 1, 0)
∴ Total ways =
8 6 6 8 6 6 8 6 6
C ⋅ C ⋅ C +
5 C C ⋅
5 C × 2 +
5 C ⋅
6 C ⋅ C × 2 5 4 5 6 4
8 6 6 8 6 6
+ C4 ⋅ C6 ⋅ C5 × 2 + C7 ⋅ C4 ⋅ C4

= 56 ⋅ 6 ⋅ 6 + 28 ⋅ 6 ⋅ 15 ⋅ 2 + 56 ⋅ 15 ⋅ 2 + 70 ⋅ 6 ⋅ 2 + 8 ⋅ 15 ⋅ 15

= 2016 + 5040 + 1680 + 840 + 1800 = 11376

88. (196.00)
M (3λ + 1, 2λ + 2, −2λ − 1) ∴ α + β + γ = 3λ + 2

N (−3μ − 2, −2μ + 2, 4μ + 1) ∴ a + b + c = −μ + 1

3λ+2 2λ −2λ−4
= =
−3μ−1 −2μ 4μ−2

3λμ + 2μ = 3λμ + λ

2μ = λ

2λμ − λ = λμ + 2μ

λμ = λ + 2μ

⇒ λμ = 2λ

⇒ μ = 2 (λ ≠ 0)

∴ λ = 4

α + β + γ = 14

a + b + c = −1
2
(α+β+γ)
= 196
2
(a+b+c)
89. (20.00)
2
1 −y −y
A = ( ′
+ x) (y − xY /x) = ′
+ 1
2 Y (x) 2Y (x)

′ ′ 2 ′
⇒ (−y + xY (x)) (y − xY (x)) = −y + 2Y (x)
2 ′ ′ 2 ′ 2 2 ′
−y + xyY (x) + xyY (x) − x [Y (x)] = −y + 2Y (x)
2 ′
2xy − x Y (x) = 2
dy 2xy−2
=
dx 2
x
dy 2 −2
− y =
dx x 2
x

I.F. = e
−2 ln x
=
1

2
x
1 2 −3
y ⋅ = x + c
x
2 3

Put x = 1, y = 1
2 1
1 = + c ⇒ c =
3 3
2 1 1 2
Y = ⋅ + X
3 X 3
5
⇒ 12Y (2) = × 12 = 20
3

90. (1575.00)
2 2 2 2
C1 : x + y = 25, C 2 : (x − α) + y = 16

−1 √ 63
θ = sin ( )
8

√ 63
sin θ =
8

β
Area of ΔOAP =
1

2
× α(
2
) =
1

2
× 5 × 4 sin θ

√ 63
⇒ αβ = 40 ×
8
− −
αβ = 5 × √ 63
2
(αβ) = 25 × 63 = 1575
JEE Main- 31-01-2024 (Morning Shift)

Questions

1. The fundamental frequency of a closed organ pipe is equal to the first overtone frequency of an open organ
pipe. If length of the open pipe is 60cm, the length of the closed pipe will be:
(a) 30 cm
(b) 15 cm
(c) 45 cm
(d) 60 cm

2. The parameter that remains the same for molecules of all gases at a given temperature is:
(a) Kinetic energy
(b) Speed
(c) Mass
(d) Momentum

3. A coil is places perpendicular to a magnetic field of 5000T. When the field is changed to 3000 T in 2s, an
induced emf of 22V is produced in the coil. If the diameter of the coil is 0.02m, then the number of turns in
the coil is:
(a) 140
(b) 35
(c) 7
(d) 70

4. An artillery piece of mass M fires a shell of mass M horizontally. Instantaneously after the firing, the ratio
1 2

of kinetic energy of the artillery and that of the shell is:


M2
(a) (M1 +M2 )

M1
(b) (M1 +M2 )

M2
(c) M1
M1
(d) M2

5. In the given arrangement of a doubly inclined plane two blocks of masses M and m are placed. The blocks
are connected by a light string passing over an ideal pully as shown. The coefficient of friction between the
surface of the plane and the blocks is 0.25. The value of m, for which M = 10 kg will move down with an
acceleration of 2m/s , is: (Take g = 10m/s and tan 37 = )
2 2 ∘ 3

(a) 4.5 kg
(b) 2.25 kg
(c) 9 kg
(d) 6.5 kg
6. Two charges q and 3q are separated by a distance ‘r’ in air. At a distance x from charge q, the resultant
electric field is zero. The value of x is:
r
(a) 3(1+√ 3 )

(b) (1+√ 3 )

r
r
(c) (1+√ 3 )

(d) r (1 + √ 3)

7. If the percentage errors in measuring the length and the diameter of a wire are 0.1% each. The percentage
error in measuring its resistance will be:
(a) 0.144 %
(b) 0.3%
(c) 0.2%
(d) 0.1%

8. The given figure represents two isobaric processes for the same mass of an ideal gas, then

(a) P1 = P2

(b) P2 > P1

(c) P1 > P2

(d) P2 ⩾ P1

9. Identify the logic operation performed by the given circuit.

(a) NOR
(b) OR
(c) AND
(d) NAND

10. The relation between time ‘t’ and distance ‘x’ is t 2


= αx + βx, where α and β are constants. The relation
between acceleration (a) and velocity (v) is:
(a) a = −2αv 3

(b) a = −5αv 5

(c) a = −4αv 4

(d) a = −3αv 2
11. A force is represented by F = ax + bt
2
1

Where x = distance and t = time. The dimensions of b

a
are:
(a) [M LT ] −2

(b) [M L T ] 2 −3

(c) [M L T ] 3 −3

(d) [M L T ] −1 −1

12. A rigid wire consists of a semicircular portion of radius R and two straight sections. The wire is partially
immerged in a perpendicular magnetic field B = B ^j as shown in figure. The magnitude force on the wire
0

if it has a current i is:

(a) ^
2iBR j

(b) ^
iBR j

(c) ^
−iBR j

(d) ^
−2iBR j

13. A coin is placed on a disc. The coefficient of friction between the coin and the disc is μ . If the distance of
the coin from the center of the disc is r, the maximum angular velocity which can be given to the disc, so
that the coin does not slip away, is:
−−
μg
(a) √
r
μ

(b) √ rg
μg
(c) r
−−
r
(d) √ μg

14. When a metal surface is illuminated by light wavelength λ , the stopping potential is 8V. When the same
surface is illuminated by light of wavelength 3λ, stopping potential is 2V. The threshold wavelength for this
surface is:
(a) 3λ
(b) 4.5λ
(c) 5λ
(d) 9λ

15. The refractive index of a prism with apex angle A is cot A


. The angle of minimum deviation is:
2

(a) δ = 180 − 3A
m

(b) δ = 180 − 4A
m

(c) δ = 180 − A
m

(d) δ = 180 − 2A
m

16. Four identical particles of mass m are kept at the four corners of square. If the gravitational force exerted on
2√ 2 +1 2

one of the masses by the other masses is ( 32


)
Gm
2
, the length of the sides of the square is
L

(a) 2L
L
(b) 2

(c) 3L
(d) 4L

17. If the wavelength of the first member of Lyman series of hydrogen is λ . The wavelength of the second
member will be
(a) 5
λ
27

(b) 27

5
λ

(c) 27

32
λ

(d) 32

27
λ

18. In a plane EM wave, the electric field oscillates sinusoidal at a frequency of 5 × 10 H z and an amplitude 10

of 50V m . The total average energy density of the electromagnetic field of the wave is: [Use
−1

C /N m ]
−12 2 2
ε 0= 8.85 × 10

(a) 2.212 × 10 J m −10 −3

(b) 1.106 × 10 J m −8 −3

(c) 4.425 × 10 J m −8 −3

(d) 2.212 × 10 J m −8 −3

19. Two conductors have the same resistance at 0 C but their temperature coefficients of resistance are α and

1

α . The respective temperature coefficients for their series and parallel combinations are:
2
α1 +α2
(a) ,α + α
2
1 2

α1 α2
(b) α + α ,
1 2
α1 +α2
α1 +α2 α1 +α2
(c) ,
2 2
α1 +α2
(d) α1 + α2 ,
2
20. A small stell ball is dropped into a long cylinder containing glycerine. Which one of the following is the
correct representation of the velocity time graph for the transit of the ball?

(a)

(b)

(c)

(d)

21. A body starts falling freely from height H hits an inclined plane in its path at height h. As a result of this
perfectly elastic impact, the direction of the velocity of the body becomes horizontal. The value of for H

which the body will take the maximum time to reach the ground is _____.

22. The mass defect in a particular reaction is 0.4g. The amount of energy liberated is n × 10 7
kW h, where n =
_____.
(Speed of light = 3 × 10 m/s)
8

23. Equivalent resistance of the following network is _____ Ω

24. A solid circular disc of mass 50 kg rolls along a horizontal floor so that its center of mass has a speed of 0.4
m/s. The absolute value of work done on the disc to stop it is ____ J.

25. Two waves of intensity ratio 1: 9 cross each other at a point. The resultant intensities at that point, when (a)
waves are incoherent is I (b) Waves are coherent is I and differ in phase by 60 . If then x =
∘ I1 10
1 2 =
I2 x

_____.
26. The depth below the surface of sea to which a rubber ball be taken so as to decreases its volume by 0.02% is
____m.
(Take density of sea water = 10 kgm , Bulk modulus of rubber = 9 × 10 N m , and g = 10ms )
3 −3 8 −2 −2

27. A small square loop of wire of side l is placed inside a large square loop of wire of side L (L = l ) . The
2

loops are coplanar and their centers coincide. The value of the mutual inductance of the system is
H , where x = ______.

− −7
√ x × 10

28. An electron moves through a unform magnetic field B⃗ = B ^i + 2B ^jT . At a particular instant of time, the
0 0

velocity of electron is u ⃗ = 3^i + 5^j m/s . If the magnetic force acting on electron is F ⃗ = 5ek
^
N , where e is

the charge of electron, then the value of B is ____ T. 0

29. A particle performs simple harmonic motion with amplitude A. Its speed is increased to three times at an
instant when its displacement is . The new amplitude of motion is
2A

3
. The value of n is _____.
nA

30. A parallel plate capacitor with plate separation 5 mm is charged up by a battery. It is found that on
introducing a dielectric sheet of thickness 2 mm, while keeping the battery connections intact, the capacitor
draws 25% more charge from the battery than before. The dielectric constant of the sheet is _____.

31. The metals that are employed in the battery industries are
A. Fe
B. Mn
C. Ni
D. Cr
E. Cd
Choose the correct answer from the options given below:
(a) A, B, C and D only
(b) A, B, C, D and E
(c) B, D and E only
(d) B, C and E only

32. Identify the factor from the following that does not affect electrolytic conductance of a solution.
(a) The nature of the electrolyte added.
(b) The nature of the electrode used.
(c) The nature of solvent used.
(d) Concentration of the electrolyte.

33. Integrated rate law equation for a first order gas phase reaction is given by (where Pi is initial pressure and
Pt is total pressure at time t)
(2Pi −Pt )
(a) k =
2.303

t
× log
Pi
2Pi
(b) k =
2.303

t
× log
(2Pi −Pt )

Pi
(c)
2.303
k = × log
t (2Pi −Pt )

Pi
(d) k =
2.303

t
×
(2Pi −Pt )
34. The linear combination of atomic orbitals to form molecular orbitals takes place only when the combining
atomic orbitals
A. have the same energy
B. have the minimum overlap
C. have same symmetry about the molecular axis
D. have different symmetry about the molecular axis
Choose the most appropriate from the options given below:
(a) A, B, C only
(b) A and C only
(c) B and D only
(d) B, C, D only

35. For the given reaction, choose the correct expression of KC from the following:-
Fe3+(aq) + SCN–(aq) ⇌ (FeSCN)2+(aq)
3+ −
[Fe ][SCN ]

(a) KC =
[FeSCN
2+
]

2+ 2
[FeSCN ]
(b) KC =
3+ −
[Fe ][SCN ]
2+
[FeSCN ]

(c) KC =
3+
2 − 2
[Fe ] [SCN ]
2+
[FeSCN ]

(d) KC =
3+ −
[Fe ][SCN ]

36. Given below are two statements:


Statement I: IUPAC name of HO–CH2–(CH2)3–CH2–COCH3 is 7-hydroxyheptan-2-one.
Statement II: 2-oxoheptan-7-ol is the correct IUPAC name for above compound.
In the light of the above statements. choose the most appropriate answer from the options given below:
(a) Both Statement I and Statement II are incorrect.
(b) Statement I is correct but Statement II is incorrect.
(c) Statement I is incorrect but Statement II is correct.
(d) Both Statement I and Statement II are correct.

37. The correct sequence of electron gain enthalpy of the elements listed below is
A. Ar
B. Br
C. F
D. S
Choose the most appropriate from the options given below:
(a) A > D > B > C
(b) A > D > C > B
(c) C > B > D > A
(d) D > C > B > A
38. Match List I with List II

LIST I (Technique) LIST II (Application)


A. Distillation I. Separation of glycerol from spent-lye
B. Fractional distillation II. Aniline – Water mixture
C. Steam distillation III. Separation of crude oil fractions
D. Distillation under reduced pressure IV. Chloroform – Aniline

Choose the correct answer from the options given below:


(a) A-IV, B-I, C-II, D-III
(b) A-IV, B-III, C-II. D-I
(c) A-I. B-II, C-IV, D-III
(d) A-II, B-III. C-I, D-IV

39. The compound that is white in color is


(a) lead iodide
(b) ammonium arsinomolybdate
(c) lead sulphate
(d) ammonium sulphide

40. Match List I with List II

LIST-I LIST-II
A. Glucose/NaHCO3/∆ I. Gluconic acid
B. Glucose/HNO3 II. No reaction
C. Glucose/HI/∆ III. n-hexane
D. Glucose/Bromine water IV. Saccharic acid

Choose the correct answer from the options given below:


(a) A-III, B-II, C-I, D-IV
(b) A-II, B-IV, C-III, D-I
(c) A-I, B-IV, C-III, D-II
(d) A-IV, B-I, C-III, D-II

41. Given below are two statements: One is labelled as Assertion A and the other is labelled as Reason R:
Assertion A: Alcohols react both as nucleophiles and electrophiles.
Reason R: Alcohols react with active metals such as sodium, potassium and aluminum to yield
corresponding alkoxides and liberate hydrogen.
In the light of the above statements, choose the correct answer from the options given below:
(a) Both A and R are true and R is the correct explanation of A.
(b) Both A and R are true but R is NOT the correct explanation of A
(c) A is true but R is false.
(d) A is false but R is true.
42. The product (C) in the below mentioned reaction is:
KOH(alc) HBr Δ

CH3 – CH2 – CH2 – Br −−−−−→A −−→B −−−−−→C


Δ KOH(aq)

(a) Propan-2-ol
(b) Propene
(c) Propan-1-ol
(d) Propyne

43. Identify the mixture that shows positive deviations from Raoult's Law
(a) CHCl3 + (CH3)2CO
(b) (CH3)2CO + C6H5NH2
(c) (CH3)2CO + CS2
(d) CHCl3 + C6H6

44. ‘Adsorption’ principle is used for which of the following purification method?
(a) Chromatography
(b) Extraction
(c) Sublimation
(d) Distillation

45. Identify correct statements from below:


A. The chromate ion is square planar.
B. Dichromates are generally prepared from chromates.
C. The green manganate ion is diamagnetic.
D. Dark green coloured K2MnO4 disproportionates in a neutral or acidic medium to give permanganate.
E. With increasing oxidation number of transition metal, ionic character of the oxides decreases.
Choose the correct answer from the options given below:
(a) A, D, E only
(b) B, C, D only
(c) B, D, E only
(d) A, B, C only

46. Consider the oxides of group 14 elements


SiO2, GeO2, SnO2, PbO2, CO and GeO. The amphoteric oxides are
(a) SiO2, GeO2
(b) SnO2, PbO2
(c) SnO2, CO
(d) GeO, GeO2

47. Give below are two statements:


Statement-I: Noble gases have very high boiling points.
Statement-II: Noble gases are monoatomic gases. They are held together by strong dispersion forces.
Because of this they are liquefied at very low temperature. Hence, they have very high boiling points.
In the light of the above statements. choose the correct answer from the options given below:
(a) Both Statement I and Statement II are false.
(b) Statement I is true but Statement II is false.
(c) Statement I is false but Statement II is true.
(d) Both Statement I and Statement II are true.
48. Given below are two statements: One is labelled as Assertion A and the other is labelled as Reason R:
Assertion A: pKa value of phenol is 10.0 while that of ethanol is 15.9.
Reason R: Ethanol is stronger acid than phenol.
In the light of the above statements, choose the correct answer from the options given below:
(a) A is true but R is false.
(b) Both A and R are true and R is the correct explanation of A.
(c) Both A and R are true but R is NOT the correct explanation of A.
(d) A is false but R is true.

49. A species having carbon with sextet of electrons and can act as electrophile is called
(a) carbon free radical
(b) carbanion
(c) carbocation
(d) pentavalent carbon

50. The correct statements from following are:


A. The strength of anionic ligands can be explained by crystal field theory.
B. Valence bond theory does not give a quantitative interpretation of kinetic stability of coordination
compounds.
C. The hybridization involved in formation of [Ni(CN)4]2– complex is dsp2.
D. The number of possible isomer(s) of cis-[PtCl2(en)2]2+ is one
Choose the correct answer from the options given below:
(a) B, D only
(b) B, C only
(c) A, C only
(d) A, D only

51. The number of species from the following in which the central atom uses sp3 hybrid orbitals in its bonding
is__________.
NH3, SO2, SiO2, BeCl2, CO2, H2O, CH4, BF3

52. Consider the following reaction at 298 K.


O2(g) ⇌ O3(g), KP = 2.47 × 10–29
3

∆rG⊖ for the reaction is _________ kJ. (Given R = 8.314 JK–1 mol–1)

53. Number of moles of methane required to produce 22g CO (g) after combustion is x × 10–2 moles. The value
2
of x is

54.

The total number of hydrogen atoms in product A and product B is__________.


55. The product of the following reaction is P.

The number of hydroxyl groups present in the product P is________.

56. The ionization energy of sodium in kJ mol–1, If electromagnetic radiation of wavelength 242 nm is just
sufficient to ionize sodium atom is______.

57. The 'Spin only’ Magnetic moment for [Ni(NH ) ]2+ is______× 10–1 BM.
3 6
(given = Atomic number of Ni : 28)

58. One Faraday of electricity liberates x × 10–1 gram atom of copper from copper sulphate, x is______.

59. Molar mass of the salt from NaBr, NaNO3, KI and CaF2 which does not evolve coloured vapours on heating
with concentrated H2SO4 is _____ g mol–1,
(Molar mass in g mol–1 : Na : 23, N : 14, K : 39, O : 16, Br : 80, I : 127, F : 19, Ca : 40)

60. Number of alkanes obtained on electrolysis of a mixture of CH3COONa and C2H5COONa is_____.

61. If the system of linear equations


x − 2y + z = 4

2x + αy + 3z = 5

3x − y + βz = 3

has infinitely many solutions, then 12α + 13β is equal to


(a) 60
(b) 64
(c) 58
(d) 54

62. Let a be the sum of all coefficients in the expansion of (1 − 2x + 2x 2


)
2023
2
(3 − 4x
3
+ 2x )
2024
and
x
log(1+t)

⎛∫ dt ⎞
t2024 +1

b = lim ⎜
0

2
. If the equations cx

2
+ dx + e = 0 and 2bx
2
+ ax + 4 = 0 have a common root,
x
x→0
⎝ ⎠

where c, d, e ∈ R , then d : c : e equals


(a) 1 : 1 : 4
(b) 1 : 2 : 4
(c) 2 : 1 : 4
(d) 4 : 1 : 4
63. The solution curve of the differential equation y dx
= x (loge x − loge y + 1) , x > 0, y > 0 passing
dy

through the point (e, 1) is


(a) 2 ∣∣log ∣∣ = y + 1
e y
x

y
(b) ∣ log
∣ e x
∣ = x

(c) ∣ log
∣ e
x

y
∣ = y

y
(d) ∣ log
∣ e x
∣ = y2

64. Two marbles are drawn in succession from a box containing 10 red, 3o white, 20 blue and 15 orange
marbles. With replacement being made after each drawing. Then the probability, that first drawn marble is
red and second drawn marble is white, is
(a) 2

25

(b) 4

75

(c) 2

(d) 4

25

65. Let α, β, γ, δ ∈ Z and let A (α, β) , B (1, 0) , C (γ, δ) and D (1, 2) be the vertices of a parallelogram
−−
ABC D. If AB = √ 10 and the points A and C lie on the line 3y = 2x + 1, then 2 (α + β + γ + δ) is

equal to
(a) 5
(b) 8
(c) 10
(d) 12

66. ⎧

g (x) , x ⩽ 0

Let g (x) be a linear function and f (x) = ⎨


1+x
1

x
, is continuous at x = 0. If

⎪( ) , x > 0
2+x

f

(1) = f (−1) , then the value g (3) is
(a) 1

3
loge (
4

9
) + 1

(b) 1

3
loge (
4
1
)

9e 3

(c) loge (
4

9
) − 1

(d) loge (
4
1
)

9e 3

67. Three rotten apples are accidently mixed with fifteen good apples. Assuming the random variable x to be
the number of rotten apples in a draw of two apples, the variance of x is
(a) 37

153

(b)
47

153

(c)
40

153

(d) 57

153
68. The distance of the point Q (0, 2, −2) from the line passing through the point P (5, −4, 3) and
perpendicular to the lines ^ ^ ^ ^ ^
r ⃗ = (−3 i + 2k ) + λ (2 i + 3 j + 5k ) , λ ∈ R and
^ ^ ^ ^ ^ ^
r ⃗ = ( i − 2 j + k ) + μ (− i + 3 j + 2k ) , μ ∈ R is:
− −
(a) √ 86
−−
(b) √ 20
− −
(c) √ 74
−−
(d) √ 54

69. For α, β, γ ≠ 0 , if sin


−1
α + sin
−1
β + sin
−1
γ = π and (α + β + γ) (α − γ + β) = 3αβ, then γ equals

(a) √3
(b)
√3

2
1
(c) √2

√ 3 −1
(d)
2√ 2

70. The area of the region {(x, y) : y


2
⩽ 4x, x < 4,
xy(x−1)(x−2)
> 0, x ≠ 3} is
(x−3)(x−4)

(a) 64

(b)
16

(c) 8

(d) 32

71. For 0 < c < b < a, let (a + b − 2c) x + (b − c − 2a) x + (c + a − 2b) 2


= 0 and α ≠ 1 be one of its
root. Then, among the two statements
(I) If α ∈ (−1, 0) , then b cannot be the geometric mean of a and c.
(II) If α ∈ (0, 1) , then b may be the geometric mean of a and c.
(a) Neither (I) nor (II) is true
(b) only (I) is true
(c) Both (I) and (II) are true
(d) only (II) is true

72. ∣ x
3
2x
2
+ 1 1 + 3x ∣
∣ ∣
If 2
f (x) = ∣ 3x + 2 2x
3
x + 6 ∣ for all x ∈ R , then 2f (0) + f ′
(0) is equal to
∣ ∣
3 2
∣ x − x 4 x − 2 ∣

(a) 42
(b) 48
(c) 18
(d) 24

73. The sum of the series 1


2 4
+
2
2 4
+
3
2 4
+. . . up to 10-terms is
1−3⋅1 +1 1−3⋅2 +2 1−3⋅3 +3

(a) −
55

109

(b) 45

109

(c) −
45

109

(d) 55

109
74. If f (x) = 4x+3
,x ≠
2
and (f of ) (x) = g (x) , where g : R − {
2
} → R − {
2
}, then (gogog) (4) is
6x−4 3 3 3

equal to
(a) 4
(b) 19

20

(c) −
19

20

(d) −4

75. If the foci of a hyperbola are same as that of the ellipse x


2

+
y
2

= 1 and the eccentricity of the hyperbola


9 25

– −

is 15

8
times the eccentricity of the ellipse, then the smaller focal distance of the point (√2, 14

3

2

5
) on the
hyperbola, is equal to


(a) 14√
2

5

4



(b) 7√
2

5
+
8



(c) 7√
2

5

8



(d) 14√
2

5

16

76. Let S be the set of positive integral values of a for which 2


ax +2(a+1)x+9a+4
< 0, ∀x ∈ R. Then, the
2
x −8x+32

number of element in S is:


(a) ∞
(b) 3
(c) 0
(d) 1

77. Let y = y (x) be the solution of the differential equation


dy
=
(tan x)+y
, x ∈ (0,
π
) satisfying
dx sin x(sec x−sin x tan x) 2

the condition y ( π

4
) = 2. Then, y ( π

3
) is
(a)
√3
(2 + loge 3)
2

(b) √ 3 (1 + 2loge 3)

(c) √ 3 (2 + loge 3)
– –
(d) √ 3 (2 + loge √ 3)

78. Let a⃗ ^ ^ ^
= 3 i + j − 2k ,
⃗ ^ ^ ^
b = 4 i + j + 7k and c ⃗ = ^ ^ ^
i − 3 j + 4k be three vectors. If a vectors p ⃗ satisfies

p⃗ × b = c⃗ × b

and p ⃗ ⋅ a⃗ = 0, then p ⃗ ⋅ (^i − ^j − k
^
) is equal to

(a) 24
(b) 28
(c) 32
(d) 36
79. If one of the diameters of the circle x + y − 10x + 4y + 13 = 0 is a chord of another circle C , whose
2 2

center is the point of intersection of the lines 2x + 3y = 12 and 3x − 2y = 5, then the radius of the circle
C is:

(a) 3√2
(b) 4
(c) 6
(d) √− −
20

80. 2|sin x|
e −2|sin x|−1
lim
2
x
x→0

(a) is equal to 2
(b) does not exist
(c) is equal to 1
(d) is equal to −1

81. Let f
x

: R → R be a function defined by f (x) =


4 +2
4
x
and
f (1−a) f (1−a)

M = ∫ xsin
4
(x (1 − x)) dx, N = ∫ sin
4
(x (1 − x)) dx; a ≠
1

2
. If αM = βN , α, β ∈ N , then
f (a) f (a)

the least value of α 2


+ β
2
is equal to _____.

82. Let S = (−1, ∞) and f : S → R be defined as


x
11
.
t 5 7 12 61
f (x) = ∫ (e − 1) (2t − 1) (t − 2) (t − 3) (2t − 10) dt
−1

Let p = Sum of squares of the values of x, where f (x) attains local maxima on S , and q = Sum of the
values of x, where f (x) attains local minima on S . Then, the value of p + 2q is _____. 2

83. Let Q and R be the foot of perpendiculars from the point P (a, a, a) on the lines x = y, z = 1 and
x = −y, z = −1 respectively. If ∠QP R is a right angle, then 12a is equal to ____.
2

84. 5

In the expansion of (1 + x) (1 − x 2
) (1 +
3

x
+
3

x
2
+
x
1

3
) ,x ≠ 0 , the sum of the coefficients of x and 3

x
−13
is equal to _____.

85. The total number of words (with or without meaning) that can be formed out of the letters of the word
‘DISTRIBUTION’ taken four at a time, is equal to ____.

86. Let a⃗ and b ⃗ be two vectors such that |a|⃗ ∣ ∣⃗


= 1, b = 4, and a⃗ ⋅ b ⃗ = 2. If c ⃗ = ⃗
(2a ⃗ × b) − 3b

and the angle
∣ ∣

between b ⃗ and c ⃗ is α , then 192sin 2


α is equal to ____.

87. Let A = {1, 2, 3, 4} and R = {(1, 2) , (2, 3) , (1, 4)} be a relation on A. Let S be the equivalence relation
on A such that R ⊂ S and the number of elements in S is n . Then, the minimum value of n is _____.

88. Let the foci and length of the latus rectum of an ellipse x
2

+
y
2

= 1, a > b be (±5, 0) and √50 ,


−−
2 2
a b
2 2
y
respectively. Then, the square of the eccentricity of the hyperbola x
2

2 2
= 1 equals
b a b
89. π

2
11 5
1


If the integral 525 ∫ is equal to (n√2 − 64), then n is equal to ____.
2

sin 2xcos 2 x(1 + cos 2 x) dx


0

90. If α denotes the number of solutions of |1 − i| x


= 2
x
and gb = (
|z|
), where
arg(z)

1−√ πi √ π−i −−−


then the distance of the point (α, β) from the line
π 4
z = (1 + i) [ + ] , i = √ −1,
4 √ π+i 1+√ πi

4x − 3y = 7 is
Answer Key

1. B 2. A 3. D 4. C 5. A 6. C
7. B 8. C 9. B 10. A 11. C 12. D
13. A 14. D 15. D 16. D 17. C 18. B
19. C 20. C 21. 2 22. 1 23. 1 24. 6
25. 13 26. 18 27. 128 28. 5 29. 7 30. 2
31. D 32. B 33. C 34. B 35. D 36. B
37. A 38. B 39. C 40. B 41. B 42. A
43. C 44. A 45. C 46. B 47. A 48. A
49. C 50. B 51. 4 52. 163 53. 50 54. 10
55. 0 56. 494 57. 28 58. 5 59. 78 60. 3
61. C 62. A 63. C 64. B 65. B 66. D
67. C 68. C 69. B 70. D 71. C 72. A
73. A 74. A 75. C 76. C 77. D 78. C
79. C 80. A 81. 5.00 82. 27.00 83. 12.00 84. 118.00
85. 3734.00 86. 48.00 87. 16.00 88. 51.00 89. 176.00 90. 3.00
Solutions

1. (B)

λ λ
= L1 2 ( ) = λ
4 2

2v
v = f λ f2 =
2L2
v
v = f1 (4L1 ) f2 =
L2
v
f1 =
4L
1

f1 = f2
v v
=
4L1 L2

⇒ L2 = 4L1

60 = 4 × L1

L1 = 15cm

2. (A)
f
KE = kT
2

Conceptual

3. (D)
Δϕ
ε = N ( )
t

Δϕ = (ΔB) A

B i = 5000T ,

B f = 3000T

d = 0.02m

r = 0.01m

Δϕ = (ΔB) A
2
= (2000) π(0.01) = 0.2π

Δϕ 0.2π
ε = N ( ) ⇒ 22 = N ( )
t 2

N = 70

4. (C)
|p ⃗ | = |p ⃗ |
1 2
2

same
p
KE = ;p
2M
1
KE ∝
m
2
KE 1 p /2M1 M2
= =
KE 2 2 M1
p /2M2
5. (A)

For M block
∘ ∘
10g sin 53 − μ (10g) cos 53 − T = 10 × 2

T = 80 − 15 − 20

T = 45N

For m block
∘ ∘
T − mg sin 37 − μmg cos 37 = m × 2

45 = 10m

m = 4.5kg

6. (C)


(E net ) = 0
p

kq k.3q
= 2
2
x (r−x)

2 2
(r − x) = 3x

r − x = √ 3x
r
x =
√ 3 +1

7. (B)
ρL
R =
2
d
π
4

ΔR ΔL 2Δd
= +
R L d
ΔL

L
= 0.1% and Δd

d
= 0.1%

ΔR
= 0.3%
R

8. (C)
P V = nRT

nR
V = ( )T
P

Slope = nR

Slope ∝ 1

(Slope) > (Slope)


2 1

P2 < P1

9. (B)
¯¯¯¯¯¯¯¯¯¯ ¯¯¯¯ ¯¯¯¯
¯¯¯¯ ¯¯¯¯ ¯¯¯¯ ¯¯¯¯
Y = A. B = A + B = A + B

(De-Morgan’s law)
10. (A)
t = αx
2
+ βx (differentiating wrt time)
dt
= 2αx + β
dx
1
= 2αx + β
v

(Differentiating wrt time)


1 dv dx
− = 2α
2 dt dt
v
dv 3
= −2αv
dt

11. (C)
1
2
F = ax + bt 2

[F ]
1 −1 −2
[a] = = [M L T ]
2
[x ]
5
[F ]
1 1 −
[b] = 1
= [M L T 2 ]

[t 2 ]

2 2 −5
2 [M L T ]
b 1 3 −3
[ ] = 1 −1 −2
= [M L T ]
a
[M L T ]

12. (D)

Note: Direction of magnetic field is in +k


^

⃗ ⃗ ⃗
F = iℓ × B

ℓ = 2R

⃗ ^
F = −2iRB j

13. (A)

N = mg
2
f = mω r

f = μN
2
μmg = mrω
− −
μg
ω = √
r
14. (D)
E = ϕ + Kmax
hc
ϕ =
λ0

Kmax = eV 0
hc hc
8e = − . . . . . (i)
λ λ0

hc hc
2e = − . . . . (ii)
3λ λ0

On solving (i) & (ii)


λ 0 = 9λ

15. (D)
A+δm
sin( )
2

μ =
A
sin
2

A+δm
A
sin( )
cos 2
2
=
A A
sin sin
2 2

π A A+δm
sin( − ) = sin( )
2 2 2

π A A δm
− = +
2 2 2 2

δm = π − 2A

16. (D)

– ′
F net = √ 2F + F

and F
2 2
Gm ′ Gm
F = =
2 2
a
(√ 2 a)

– Gm
2
Gm
2

F net = √2 +
2 2
a 2a

2√ 2 +1 2 2 2√ 2 +1
Gm Gm
( ) 2
= ( )
32 2 2
L a

a = 4L

17. (C)
2
1 13.6z 1 1
= [ 2
− 2
] . . . . (i)
λ hc 1 2

2
1 13.6z 1 1

= [ 2
− 2
] . . . . (ii)
λ hc 1 3

On dividing (i) & (ii)


′ 27
λ = λ
32

18. (B)
1 2
UE = ∈0 E
2
1 −12 2
UE = × 8.85 × 10 × (50)
2
−8 3
1.106 × 10 J /m
19. (C)
Series:
R eq = R 1 + R 2

2R (1 + α eq Δθ) = R (1 + α 1 Δθ) + R (1 + α 2 Δθ)

2R (1 + α eq Δθ) = 2R (α 1 + α 2 ) RΔθ
α1 +α2
a eq =
2

Parallel:
1 1 1
= +
R eq R1 R2

1 1 1
R
= +
R(1+α1 Δθ) R(1+α2 Δθ)
(1+αeq Δθ)
2

2 1 1
= +
1+αeq Δθ 1+α1 Δθ 1+α2 Δθ

2 1+α2 Δθ+1+α1 Δθ
=
1+αeq Δθ (1+α1 Δθ)(1+α2 Δθ)

2 [(1 + α 1 Δθ) (1 + α 2 Δθ)]

= [2 + (α 1 + α 2 ) Δθ] [1 + α eq Δθ]

= 2 [1 + α 1 Δθ + α 2 Δθ + α 1 α 2 Δθ]
2
= 2 + 2α eq Δθ + (α 1 + α 2 ) Δθ + α eq (α 1 + α 2 ) Δθ

Neglecting small terms


2 + 2 (α 1 + α 2 ) Δθ = 2 + 2α eq Δθ + (α 1 + α 2 ) Δθ

(α 1 + α 2 ) Δθ = 2α eq Δθ
α1 +α2
α eq =
2

20. (C)

mg − F B − F v = ma
4 3 4 3 dv
(ρ π r ) g − (ρ L πr ) g − 6πηrv = m
3 3 dt
5πηr
Let 4

3m
3
πr g (ρ − ρ L ) = K1 and m
= K2

dv
= K1 − K2 v
dt
v t
dv
∫ = ∫ dt
K 1 −K 2 v
0 0
1 v
− ℓn[K1 − K2 v] = t
K2 0

K 1 −K 2 v
ℓn ( ) = −K 2 t
K1

−K 2 t
K1 − K2 v = K1 e
K1
−K 2 t
v = [1 − e ]
K2
21. (2)

Total time of flight = T


−− −−−−−
2(H −h)
2h
T = √ + √
g g

For max. time = dT

dh
= 0


2 −1 1
√ ( + ) = 0
g 2√ H −h 2√ h
−−−−− −

√H − h = √h
H H
h = ⇒ = 2
2 h

22. (1)
2
E = Δmc
2
−3 8
= 0.4 × 10 × (3 × 10 )

7
= 3600 × 10 kW s
7
3600×10 7
= kW h = 1 × 10 kW h
3600

23. (1)

6Ω is short circuit

1
R eq = 3 × = 1Ω
3
24. (6)
Using work energy theorem
1 2 1 2
W = ΔK E = 0 − ( mv + Iω )
2 2
2
1 2 K
W = 0 − mv (1 + 2
)
2 R
1 2 1
= − × 50 × 0.4 (1 + ) = −6J
2 2

Absolute work = +6J

W = −6J |W | = 6J

25. (13)
For incoherent wave I 1 = IA + IB ⇒ I1 = I0 + 9I0

I1 = 10I0
−−−−
For coherent wave I 2 = IA + IB + 2√IA IB cos 60

−−−
2 1
I2 = I0 + 9I0 + 2√9I . = 13I0
0 2

I1 10
=
I2 13

26. (18)
−ΔP
β =
ΔV

ΔV
ΔP = −β
V
ΔV
ρgh = −β
V
3 8 0.02
10 × 10 × h = −9 × 10 × −( )
100

⇒ h = 18m

27. (128)

Flux linked for inner loop.


2
ϕ = B center . ℓ
μ i
0 2
= 4 × L
(sin 45 + sin 45) ℓ

2

– μ i
0 2
ϕ = 2√ 2 ℓ
πL
2
ϕ 2√ 2 μ ℓ
0 – μ
0
M = = = 2√ 2
i πL π
– 4π −7
= 2√ 2 × 10
π
– −7
= 8√ 2 × 10 H
− −− −7
= √ 128 × 10 H

x = 128
28. (5)
⃗ ⃗
F = q (v ⃗ × B )

^ ^ ^ ^ ^
5ek = e (3 i + 5 j) × (B 0 i + 2B 0 j)

^ ^ ^
5ek = e (6B 0 k − 5B 0 k )

⇒ B 0 = 5T

29. (7)
−−−−−−−
v = ω√A2 − x2

At x =
2A

3
−−−−−−−−−−
2
2A √ 5 Aω
v = ω√A2 − ( ) =
3 3

New Amplitude = A

−−− −−−−−−−−−
2

– ′
2 2A
V = 3v = √ 5Aω = ω√(A ) − ( )
3

′ 7A
A =
3

30. (2)
Without dielectric
A∈ 0
Q = V
d

With dielectric
A∈ 0 V
Q = t
d−t+
K

Given
A∈ 0 V A∈ 0 V

t
= (1.25)
d
d−t+
K
2
⇒ 12.5 (3 + ) = 5
K

⇒ K = 2

31. (D)
Mn, Ni and Cd metals used in battery industries.

32. (B)
Conductivity of electrolytic cell is affected by concentration of electrolyte, nature of electrolyte and nature of
solvent.

33. (C)
A→B+C
Pi, 0, 0
Pi – x, x, x
P t = Pi + x
P i – x = Pi – P t + Pi
= 2Pi – Pt
2.303 Pi
k = × log
t (2Pi −Pt )
34. (B)
* Molecular orbital should have maximum overlap
* Symmetry about the molecular axis should be similar

35. (D)
2+
[FeSCN ]
Products ion conc.
KC = = −
Reactants ion conc. 3+
[Fe ][SCN ]

36. (B)
7-Hydroxyheptan-2-one is correct IUPAC name

37. (A)
Element, ∆egH(kJ/mol)
F, –333
S, –200
Br, –325
Ar, +96

38. (B)
Fact (NCERT)

39. (C)
Lead sulphate-white
Ammonium sulphide-soluble
Lead iodide-Bright yellow
Ammonium arsinomolybdate-yellow

40. (B)
NaHCO3

Glucose −−−−−→ no reaction


Δ
HNO3

Glucose −−−→ saccharic acid


Δ
HI

Glucose −→ n-hexane
Δ
Br 2

Glucose −→
− Gluconic acid
Δ

41. (B)
As per NCERT, Assertion (A) and Reason (R) is correct but Reason (R) is not the correct explanation.
42. (A)

43. (C)
(CH3)2CO + CS2 Exhibits positive deviations from Raoult’s Law

44. (A)
Principle used in chromotography is adsorption.

45. (C)
A. CrO42– is tetrahedral
B. 2Na2CrO4 + 2H+ → Na2Cr2O7 + 2Na+ + H2O
C. As per NCERT, green manganate is paramagnetic with 1 unpaired electron.
D. Statement is correct
E. Statement is correct

46. (B)
SnO2 and PbO2 are amphoteric.

47. (A)
Statement I and II are False
Noble gases have low boiling points
Noble gases are held together by weak dispersion forces.

48. (A)
Phenol is more acidic than ethanol because conjugate base of phenoxide is more stable than ethoxide.

49. (C)

Six electron species


50. (B)
B. VBT does not explain stability of complex
C. Hybridisation of [Ni(CN)4]–2 is dsp2.

51. (4)
NH3 → sp3
SO2 → sp2
SiO2 → sp3
BeCl2 → sp
CO2 → sp
H2O → sp3
CH4 → sp3
BF3 → sp2

52. (163)
3

2
O2(g) ⇌ O3(g), KP = 2.47 × 10–29
∆rG⊖ = –RT ln KP
= – 8.314 × 10–3 × 298 × ln (2.47 × 10–29)
= –8.314 × 10–3 × 298 × (–65.87)
= 163.19 kJ

53. (50)
CH4(g) + 2O2(g) → CO2(g) + 2H2O(l)
22
nCO = = 0.5 moles
2 44

So moles of CH4 required = 0.5 moles


i.e. 50 × 10–2 mole
x = 50

54. (10)

Total number of hydrogen atom in A and B is 10

55. (0)
Product benzene has zero hydroxyl group.
56. (494)
1240 1240
E = eV = eV
λ(nm) 242

= 5.12 eV
= 5.12 × 1.6 × 10–19
= 8.198 × 10–19 J/atom
= 494 kJ/mol

57. (28)
NH3 act as WFL with Ni2+
Ni2+ = 3d8

No. of unpaired electron = 2


−−−−−−− –
μ = √n (n + 2) = √ 8 = 2.82 BM

= 28.2 × 10–1 BM
x = 28

58. (5)
Cu2+ + 2e– → Cu
2 Faraday → 1 mol Cu
1 Faraday → 0.5 mol Cu deposit
0.5 mol = 0.5 g atom = 5 × 10–1
x=5

59. (78)
CaF2 does not evolve any gas with concentrated H2SO4.
NaBr → evolve Br2
NaNO3 → evolve NO2
KI → evolve I2

60. (3)

CH3COONa → C H3

C2H5COONa → C 2H5

2C 2H5 → CH3 – CH2 – CH2 – CH3


2CH → CH3 – CH3


3

∙ ∙

C H3 + C 2H5 → CH3 – CH2 – CH3


61. (C)
∣ 1 −2 1 ∣
∣ ∣
D = 2 α 3
∣ ∣
∣ 3 −1 β ∣

= 1 (αβ + 3) + 2 (2β − 9) + 1 (−2 − 3α)

= αβ + 3 + 4β − 18 − 2 − 3α

For infinite solutions D = 0, D 1 = 0, D 2 = 0 and D 3 = 0

D = 0

αβ − 3α + 4β = 17 . . . . (1)

∣ −4 −2 1 ∣
∣ ∣
D1 = 5 α 3 = 0
∣ ∣
∣ 3 −1 β ∣

∣ 1 −4 1 ∣
∣ ∣
D2 = 2 5 3 = 0
∣ ∣
∣ 3 3 β ∣

⇒ 1 (5β − 9) + 4 (2β − 9) + 1 (6 − 15) = 0

13β − 9 − 36 − 9 = 0

13β = 54, β =
54

13
put in (1)
54 54
α − 3α + 4 ( ) = 17
13 13

54α − 39α + 216 = 221


1
15α = 5 ⇒ α =
3

Now, 12α + 13β = 12.


1

3
+ 13.
54

13

= 4 + 54 = 58

62. (A)
Put x = 1

∴ a = 1
x
ln(1+t)
∫ dt
1+t2024
0
b = lim
2
x→0 x

Using L’ HOPITAL Rule


ln(1+x)
1 1
b = lim × =
(1+x
2024
) 2x 2
x→0

Now,
2 2
cx + dx + e = 0, x + x + 4 = 0

(D < 0)
c d e
∴ = =
1 1 4
63. (C)
dx x x
= (ln( ) + 1)
dy y y

Let x

y
= t ⇒ x = ty

dx dt
= t + y
dy dy

dt
t + y = t (ln(t) + 1)
dy

dt dt dy
y = t ln(t) ⇒ =
dy t ln(t) y

dt dy
⇒ ∫ = ∫
t.ln(t) y

dp dy
⇒ ∫ = ∫
p y

Let ln t = p
1
dt = dp
t

⇒ ln p = ln y + c

ln(ln t) = ln y + c

x
ln(ln( )) = ln y + c
y

At x = e, y = 1
e
ln(ln( )) = ln(1) + c ⇒ c = 0
1

∣ x ∣
ln ln( ) = ln y
∣ y ∣

∣ x ∣ ln y
ln( ) = e
∣ y ∣

∣ x ∣
ln( ) = y
∣ y ∣

64. (B)
Probability of drawing first red and then white = 10

75
×
30

75
=
4

75

65. (B)

Let E is mid point of diagonals


α+γ
& 1+1 β+δ 2+0
= =
2 2 2 2

α + γ = 2 & β + δ = 2

2 (α + β + γ + δ) = 2 (2 + 2) = 8
66. (D)
Let g (x) = ax + b
Now function f (x) is continuous at x = 0

∴ lim f (x) = f (0)


+
x→0
1

1+x x

⇒ lim ( ) = b
2+x
x→0

⇒ 0 = b

∴ g (x) = ax

Now, for x > 0


1 1
−1
1+x x 1+x x 1+x
′ 1 1 1
f (x) = ⋅ ( ) ⋅ 2
+ ( ) ⋅ ln( ) ⋅ (− )
x 2+x 2+x 2+x x
2
(2+x)

′ 1 2 2
∴ f (1) = − ⋅ ln( )
9 3 3

And f (−1) = g (−1) = −a


2 2 1
∴ a = ln( ) −
3 3 9
2 1
∴ g (3) = 2 ln( ) −
3 3

4
= ln( 1
)

9⋅e 3

67. (C)
3 bad apples, 15 good apples.
Let X be no of bad apples
15
C2
Then P (X = 0) =
18
C2
=
105

153
3 15
C1 × C1 45
P (X = 1) = =
18 153
C2
3
C2 3
P (X = 2) = =
18 153
C2
105 45 3 51 1
E (X) = 0 × + 1 × + 2 × = =
153 153 153 153 3

2 2
V ar (X) = E (X ) − (E (X))
2
105 45 3 1
= 0 × + 1 × + 4 × − ( )
153 153 153 3
57 1 40
= − =
153 9 153
68. (C)
A vector in the direction of the required line can be obtained by cross product of
∣ ^ ^ ^
i j k∣
∣ ∣
∣ 2 3 5∣
∣ ∣
∣ −1 3 2∣

^ ^ ^
= −9 i − 9 j + 9k

Required line,
^ ^ ^ ′ ^ ^ ^
r ⃗ = (5 i − 4 j + 3k ) + λ (−9 i − 9 j + 9k )

^ ^ ^ ^ ^ ^
r ⃗ = (5 i − 4 j + 3k ) + λ ( i + j − k )

Now distance of (0, 2, −2)

P.V. of P ^ ^ ^
≡ (5 + λ) i + (λ − 4) j + (3 − λ) k

−→
^ ^ ^
AP = (5 + λ) i + (λ − 6) j + (5 − λ) k

−→
^ ^ ^
AP ⋅ ( i + j − k ) = 0

5 + λ + λ − 6 − 5 + λ = 0

λ = 2

∣−−
→∣
− −−−−−−−−
∣AP ∣ = √ 49 + 16 + 9
∣ ∣

∣−−
→∣
− −
∣AP ∣ = √ 74
∣ ∣

69. (B)
Let sin −1
α = A, sin
−1 −1
β = B, sin γ = C

A + B + C = π
2 2
(α + β) − γ = 3αβ
2 2 2
α + β − γ = αβ
2 2 2
α +β −γ
1
=
2αβ 2
1
⇒ cos C =
2

sin C = γ
−−−−− 1
2
cos C = √1 − γ =
2
√3
γ =
2
70. (D)
2
y ⩽ 4x, x < 4
xy(x−1)(x−2)
> 0
(x−3)(x−4)

Case-I: y > 0
x(x−1)(x−2)
> 0
(x−3)(x−4)

x ∈ (0, 1) ∪ (2, 3)

Case-II: y < 0
x(x−1)(x−2)
< 0, x ∈ (1, 2) ∪ (3, 4)
(x−3)(x−4)

Area = −−
2 ∫ √ x dx
0

3 4
2 32
= 2 ⋅ [x 2 ] =
3 3
0

71. (C)
2
f (x) = (a + b − 2c) x + (b + c − 2a) x + (c + a − 2b)

f (x) = a + b − 2c + b + c − 2a + c + a − 2b = 0

f (1) = 0
c+a−2b
∴ α ⋅ 1 =
a+b−2c
c+a−2b
α =
a+b−2c

If, −1 < α < 0


c+a−2b
−1 < < 0
a+b−2c

b + c < 2a and b > a+c

therefore, b cannot be G.M. between a and c.


If, 0 < α < 1
c+a−2b
0 < < 1
a+b−2c

b > c and b < a+c

Therefore, b may be the G.M. between a and c.


72. (A)
∣ 0 1 1 ∣
∣ ∣
f (0) = 2 0 6 = 12
∣ ∣
∣ 0 4 −2 ∣
2 3 2 3 2
∣ 3x 4x 3 ∣ ∣ x 2x + 1 1 + 3x ∣ ∣ x 2x + 1 1 + 3x ∣
∣ ∣ ∣ ∣ ∣ ∣

f (x) = ∣ 3x2 + 2 2x x
3
+ 6∣ + ∣ 6x 2 3x
2
∣ + ∣ 3x2 + 2 2x
3
x + 6 ∣
∣ ∣ ∣ ∣ ∣ ∣
∣ x3 − x 4 x
2
− 2∣ ∣ x3 − x 4
2
x − 2 ∣ ∣ 3x2 − 1 0 2x ∣

∣ 0 0 3 ∣ ∣ 0 1 1 ∣ ∣ 0 1 1 ∣
′ ∣ ∣ ∣ ∣ ∣ ∣
∴ f (0) = 2 0 6 + 0 2 0 + 2 0 6
∣ ∣ ∣ ∣ ∣ ∣
∣ 0 4 −2 ∣ ∣ 0 4 −2 ∣ ∣ −1 0 0 ∣

= 24 − 6 = 18

∴ 2f (0) + f (0) = 42

73. (A)
General term of the sequence,
r
Tr =
2 4
1−3r +r
r
Tr =
4 2 2
r −2r +1−r
r
Tr = 2
2 2
(r −1) −r
r
Tr =
2 2
(r −r−1)(r +r−1)
1 2 2
[(r +r−1)(r −r−1)]
2
Tr =
2 2
(r −r−1)(r +r−1)

1 1 1
= [ − ]
2 2 2
r −r−1 r +r−1

Sum of 10 terms,
10
1 1 1 −55
∑ Tr = [ − ] =
2 −1 109 109
r=1

74. (A)
4x+3
f (x) =
6x−4
4x+3
4( )+3
6x−4 34x
g (x) = = = x
4x+3 34
6( )−4
6x−4

g (x) = x

∴ g (g (g (4))) = 4
75. (C)
2 2
x y
+ = 1
9 25

a = 3, b = 5
−−−−−
9 4
e = √1 − =
25 5

∴ foci = (0, ± be) = (0, ± 4)


4 15 3
∴ eH = × =
5 8 2

Let equation hyperbola


2 y2
x
2
− 2
= −1
A B

∴ B ⋅ eH = 4
8
∴ B =
3

2 2 2 64 9
∴ A = B (e H − 1) = ( − 1)
9 4
2 80
∴ A =
9
2 2
x y
∴ − = −1
80 64

9 9

Directrix: y = ±
B

eH
= ±
16

9


3 ∣ 14 2 16 ∣
PS = e ⋅ PM = ⋅ √ −
2 ∣ 3 5 9 ∣


2 8
= 7√ −
5 3

76. (C)
2
ax + 2 (a + 1) x + 9a + 4 < 0 ∀ x ∈ R

∴ a < 0
77. (D)
dy sin x+y cos x
=
dx 1 sin x
sin x.cos x( −sin x⋅ )
cos x cos x

sin x+y cos x


= 2
sin x(1−sin x)

dy
2
= sec x + y.2 (cosec 2x)
dx
dy
2
− 2 cosec (2x) . y = sec x
dx
dy
+ p. y = Q
dx

I.F. = e ∫ p dx = e ∫ −2 cosec (2x) dx

Let 2x = t
dx
2 = 1
dt
dx
2 = 1
dt
dt
dx =
2

− ∫ cosec(t)dt
= e
t
− ln∣tan ∣
∣ ∣
= e 2

− ln|tan x| 1
= e =
|tan x|

Y (IF) = ∫ Q (IF) dx + C
1 2 1
⇒ y = ∫ sec x ⋅ + c
|tan x| |tan x|

y.
1

|tan x|
= ∫
dt

|t|
+ c for tan x = t

1
y. = ln|t| + c
|tan x|

y = |tan x| (ln|tan x| + c)

Put x =
π

4
,y = 2

2 = ln 1 + c ⇒ c = 2

y = |tan x| (ln|tan x| + 2)
π – –
y( ) = √ 3 (ln √ 3 + 2)
3

78. (C)
⃗ ⃗ ⃗
p⃗ × b − c⃗ × b = 0

⃗ × b ⃗ = 0⃗
(p ⃗ − c )

⃗ ⃗
p ⃗ − c ⃗ = λb ⇒ p ⃗ = c ⃗ + λb

Now, p .⃗ a⃗ = 0 (given)
So, c .⃗ a⃗ + λa.⃗ b ⃗ = 0
(3 − 3 − 8) + λ (12 + 1 − 14) = 0

λ = −8


p ⃗ = c ⃗ − 8b

^ ^ ^
p ⃗ = −31 i − 11 j − 52k

So, p .⃗ (^i − ^j − k
^
)

= −31 + 11 + 52

= 32
79. (C)

2x + 3y = 12

3x − 2y = 5

13x = 39

x = 3, y = 2

Center of given circle is (5, −2)


− −−−−−−−−
Radius √25 + 4 − 13 = 4
−−−−− –
∴ C M = √ 4 + 16 = 5√ 2
− −−−−−
∴ C P = √ 16 + 20 = 6

80. (A)
2|sin x|
e −2|sin x|−1
lim
2
x
x→0
|sin x| 2
e −2|sin x|−1
sin x
lim 2
×
2
|sin x| x
x→0

Let |sin x| = t
2t 2
e −2t−1 sin x
lim 2
× lim
2
t x
t→0 x→0
2t
2e −2
= lim × 1 = 2 × 1 = 2
2t
t→0

81. (5.00)
f (a) + f (1 − a) = 1

f (1−a)
4
M = ∫ (1 − x) . sin x (1 − x) dx

f (a)

M = N − M ⇒ 2M = N

α = 2; β = 1

82. (27.00)
′ x 11 5 7 12 61
f (x) = (e − 1) (2x − 1) (x − 2) (x − 3) (2x − 10)

Local minima at x = 1

2
,x = 5

Local maxima at x = 0, x = 2
1 11
∴ p = 0 + 4 = 4, q = + 5 =
2 2

Then p 2
+ 2q = 16 + 11 = 27
83. (12.00)
x y z−1
= = = r → Q (r, r, 1)
1 1 0
x y z+1
= = = k → R (k, −k, −1)
1 −1 0
¯¯¯¯¯¯¯¯
^ ^ ^
P Q = (a − r) i + (a − r) j + (a − 1) k

a = r + a − r = 0

2a = 2r → a = r

−→
^ ^ ^
P R = (a − k) i + (a + k) j + (a + 1) k

a − k − a − k = 0 ⇒ k = 0

As, P Q⊥P R
(a − r) (a − k) + (a − r) (a + k) + (a − 1) (a + 1) = 0

a = 1 or − 1
2
12a = 12

84. (118.00)
5
2 3 3 1
(1 + x) (1 − x ) (1 + + + )
x x
2
x
3

5
3
2 1
= (1 + x) (1 − x ) ((1 + ) )
x

2 15
(1+x) (1−x)(1+x)
=
15
x
17 17
(1+x) −x(1+x)
=
15
x

= Coeff (x ) in the expansion ≈ coeff (x


3 18
) in (1 + x) 17 17
− x(1 + x)

= 0 − 1

= −1

= Coeff (x −13
) in the expansion ≈ coeff (x 2
) in (1 + x)
17
− x(1 + x)
17

17 17
= ( ) − ( )
2 1

= 17 × 8 − 17

= 17 × 7

= 119

Hence answer = 119 − 1 = 118

85. (3734.00)
We have III, TT, D, S, R, B, U, O, N
Number of words with selection (a, a, a, b)
8 4!
= C1 × = 32
3!

Number of words with selection (a, a, b, b)


4!
= = 6
2!2!

Number of words with selection (a, a, b, c)


2 8 4!
= C1 × C2 × = 672
2!

Number of words with selection (a, b, c, d)


9
= C 4 × 4! = 3024

∴ Total = 3024 + 672 + 6 + 32

= 3734
86. (48.00)
2
⃗ ⃗ ⃗ ∣ ∣⃗
b ⋅ c ⃗ = (2a ⃗ × b) ⋅ b − 3 b
∣ ∣
2
∣ ∣⃗ ∣ ∣⃗
b |c |⃗ cos α = −3 b
∣ ∣ ∣ ∣

|c |⃗ cos α = −12, as ∣∣b∣∣⃗ = 4


a⃗ ⋅ b = 2
1 π
cos θ = ⇒ θ =
2 3
2
2 ∣ ⃗ ∣⃗
|c |⃗ = (2a ⃗ × b) − 3b
∣ ∣
3
= 64 × + 144 = 192
4
2
2
|c |⃗ cos α = 144
2
192cos α = 144
2
192sin α = 48

87. (16.00)
All elements are included
Answer is 16

88. (51.00)
2
− −
Foci ≡ (± 5, 0) ; 2b

a
= √ 50

5√ 2 a
2
ae = 5; b =
2
5√ 2 a
2 2 2
b = a (1 − e ) =
2
5√ 2
2
⇒ a (1 − e ) =
2

5 5√ 2
2
⇒ (1 − e ) =
e 2
– – 2
⇒ √ 2 − √ 2e = e
– 2 –
⇒ √ 2e + e − √ 2 = 0
– 2 –
⇒ √ 2e + 2e − e − √ 2 = 0
– – –
⇒ √ 2e (e + √ 2) − 1 (1 + √ 2) = 0
– –
⇒ (e + √ 2) (√ 2e − 1) = 0
– 1
∴ e ≠ −√ 2; e =
√2

2 2
x y

2
− 2
= 1
2
b a b

a = 5√ 2

b = 5
2 2 2 2 2
a b = b (e 1 − 1) ⇒ e 1 = 51
89. (176.00)
π
1
2 11 5
2

I = ∫ sin 2x ⋅ (cos x) 2
(1 + (cos x) 2
) dx
0

Put cos = t
2
⇒ sin x dx = −2t dt
1
2 11 −−−−−−
∴ I = 4∫ t ⋅ t √(1 + t5 ) (t) dt
0

1
−−−−−
14
I = 4∫ t √1 + t5 dt

Put 1 + t 5
= k
2

4
⇒ 5t dt = 2k dk

√2
2
2 2k
∴ I = 4 ∫ (k − 1) ⋅ k dk
5
1

√2
8 6 4 2
I = ∫ k − 2k + k dk
5
1

7 5 3 √2
8 k 2k k
I = [ − + ]
5 7 5 3
1

8 8√ 2 8√ 2 2√ 2 1 2 1
I = [ − + − + − ]
5 7 5 3 7 5 3

8 22√ 2 8
I = [ − ]
5 105 105

∴ 525 ⋅ I = 176√ 2 − 64

90. (3.00)
– x x
(√ 2) = 2 ⇒ x = 0 ⇒ α = 1

4 √ π−πi−i−√ π √ π−i−πi−√ π
π
z = (1 + i) [ + ]
4 π+1 1+π

πi 2 3
= − (1 + 4i + 6i + 4i + 1)
2

= 2πi

β = π = 4
2

Distance from (1, 4) to 4x − 3y = 7 will be 15

5
= 3
JEE Main 31-01-2024 (Evening Shift)

Questions

1. Consider two physical quantities A and B related to each other as E = 2

where E, x and t have


B−x

At

dimensions of energy, length and time respectively. The dimension of AB is


(a) L M T −2 −1 1

(b) L M T 2 −1 1

(c) L M T −2 1 0

(d) L M T 0 −1 1

2. Force between two point charges q and q placed in vacuum at ‘r’ cm apart is F. Force between them when
1 2

placed in a medium having dielectric constant K = 5 at ‘ ’ cm apart will be:


r

(a) F

25

(b) 25F
(c) F

(d) 5F

3. If two vectors A ⃗ and B⃗ having equal magnitude R are inclined at an angle θ, then
∣ ⃗ ⃗∣ –
(a) ∣
A − B = √ 2R sin(

θ

2
)

∣ ⃗ ⃗∣
(b) ∣
A + B = 2R cos(

θ

2
)

∣ ⃗ ⃗∣
(c) ∣
A − B = 2R cos(

θ

2
)

∣ ⃗ ⃗∣
(d) ∣
A + B = 2R sin(

θ

2
)

4. The speed of sound in oxygen at S.T.P. will be approximately:


(given, R = 8.3J K , γ = 1.4) −1

(a) 325 m/s


(b) 310 m/s
(c) 333 m/s
(d) 341 m/s

5. Given below are two statements:


Statement I: Electromagnetic waves carry energy as they travel through space and this energy is equally
shared by the electric and magnetic fields.
Statement II: When electromagnetic waves strike a surface, a pressure is exerted on the surface.
In the light of the above statements, choose the most appropriate answer from the options given below.
(a) Both Statement I and Statement II are correct
(b) Both Statement I and Statement II are incorrect
(c) Statement I is incorrect but Statement II is correct
(d) Statement I is correct but Statement II is incorrect.

6. The mass number of nucleus having radius equal to half of the radius of nucleus with mass number 192 is:
(a) 32
(b) 24
(c) 20
(d) 40
7. By what percentage will the illumination of the lamp decease if the current drops by 20%
(a) 56%
(b) 46%
(c) 26%
(d) 36%

8. The output of the given circuit diagram is –

A B Y

0 0 0

(a) 1 0 0

0 1 1

1 1 0

A B Y

0 0 0

(b) 1 0 0

0 1 0

1 1 1

A B Y

0 0 0

(c) 1 0 0

0 1 0

1 1 0

A B Y

0 0 0

(d) 1 0 1

0 1 1

1 1 0
9. The measured value of the length of a simple pendulum is 20 cm with 2 mm accuracy. The time for 50
oscillations was measured to be 40 seconds with 1 second resolution. From these measurements, the accuracy
in the measurement of acceleration due to gravity is N%. The value of N is:
(a) 6
(b) 4
(c) 8
(d) 5

10. A body of mass 2 kg begins to move under the action of a time dependent force given by
⃗ ^ 2^
F = (6t i + 6t j) N . The power developed by the force at the time t is given by
(a) (6t
4 5
+ 9t ) W

(b) (9t
5 3
+ 6t ) W

(c) (3t
3 5
+ 6t ) W

(d) (9t
3 5
+ 6t ) W

11. The mass of the moon is times the mass of a planet and its diameter is
1
times the diameter of a planet.
1

144 16

If the escape velocity on the planet is v, the escape velocity on the moon will be:
v
(a) 12
v
(b) 3
v
(c) 4
v
(d) 6

12. The resistance per centimetre of a meter bridge wire is r, with XQ resistance in left gap. Balancing length
from left end is at 40 cm with 25Ω resistance in right gap. Now the wire is replaced by another wire of 2r
resistance per centimetre. The new balancing length for same settings will be at
(a) 20 cm
(b) 10 cm
(c) 80 cm
(d) 40 cm

13. A block of mass 5 kg is placed on a rough inclined surface as shown in the figure. If F ⃗ is the force required
1

to just move the block up the inclined plane and F ⃗ is the force required to just prevent the block from
2

sliding down, then the value of ∣∣F ⃗ ∣∣ − ∣∣F ⃗ ∣∣ is: [use g = 10m/s ]
1 2
2

(a) 5√ 3
N
2

(b) 10 N
(c) 50√–3N

(d) 25√3N
14. A small spherical ball of radius r, falling through a viscous medium of negligible density has terminal
velocity ‘v’. Another ball of the same mass but of radius 2r, falling through the same viscous medium will
have terminal velocity:
v
(a) 2

(b) 2v
v
(c) 4

(d) 4v

15. An AC voltage V = 20 sin 200πt is applied to a series LCR circuit which drives a current
). The average power dissipated is:
π
I = 10 sin(200πt +
3

(a) 50W
(b) 173.2W
(c) 21.6 W
(d) 200W

16. In a photoelectric effect experiment a light of frequency 1.5 times the threshold frequency is made to fall on
the surface of photosensitive material. Now if the frequency is halved and intensity is doubled, the number
of photo electrons emitted will be:
(a) Halved
(b) Doubled
(c) Quadrupled
(d) Zero

17. When unpolarized light is incident at an angle of 60 on a transparent medium from air, the reflected ray is

completely polarized. The angle of refraction in the medium is:


(a) 30 ∘

(b) 45 ∘

(c) 60 ∘

(d) 90 ∘

18. A uniform magnetic field of 2 × 10 T acts along positive Y-direction. A rectangular loop of sies 20 cm
−3

and 10 cm with current of 5A is in Y-Z plane The current is in anticlockwise sense with reference to
negative X axis. Magnitude and direction of the torque is:
(a) 2 × 10 N − m along positive Z-direction
−4

(b) 2 × 10 N − m along negative Z-direction


−4

(c) 2 × 10 N − m along positive X-direction


−4

(d) 2 × 10 N − m along positive Y-direction.


−4
19. A light string passing over a smooth light fixed pulley connects two blocks of masses m and m . If the
1 2
g
acceleration of the system is , then the ratio of masses is:
8

(a) 5

(b) 9

(c) 4

(d) 8

20. A gas mixture consists of 8 moles of argon and 6 moles of oxygen at temperature T. Neglecting all
vibrational modes, the total internal energy of the system is:
(a) 21 RT
(b) 27 RT
(c) 29 RT
(d) 20 RT

21. The distance between charges +q and -q is 2l and between +2q and −2q is 4l. The electrostatic potential at
point P at a distance r from center O is −α [ where the value of α is _____. (Use
ql
9
] × 10 V ,
2
r

4πε0
9 2
= 9 × 10 N m C
−2
)

22. A nucleus has mass number A and volume V . Another nucleus has mass number A and volume V . If
1 1 2 2

relation between mass number is A = 4A , then


V2
2 1 = _____.
V1

23. Two circular coils P and Q of 100 turns each have same radius of π cm. The currents in P and R are 1A and
2A respectively. P and Q are placed with their planes mutually perpendicular with their centers coincide.
The resultant magnetic field induction at the center of the coils is √−
x mT , where x = _____.

[Use μ = 4π × 10 T mA ]
0
−7 −1
24. The magnetic flux ϕ (in weber) linked with a closed circuit of resistance 8Ω varies with time (in seconds) as
ϕ = 5t − 36t + 1. The induced current in the circuit at t = 2s is _____A.
2

25. Light from a point source in air falls on a convex curved surface of radius 20 cm and refractive index 1.5. If
the source is located at 100 cm from the convex surface, the image will be formed at ____ cm from the
object.

26. The time period of simple harmonic motion of mass M in the given figure is π√−−−, where the value of α
αM

5k

is _____.

27. Two identical speres each of mass 2 kg and radius 50 cm are fixed at the ends of a light rod so that the
separation between the centers is 150 cm. Then, moment of inertia of the system about an axis
perpendicular to the rod and passing through its middle point is kg m , where the value of x is _____ .
x

20
2

28. Two blocks of mass 2kg and 4 kg are connected by a metal wire going over a smooth pulley as shown in
figure. The radius of wire is 4.0 × 10 m and Young’s modulus of the metal is 2.0 × 10 N /m . The
−5 11 2

longitudinal strain developed in the wire is . The value of α is ____. [Use g = 10m/s ]
1

απ
2

29. A body of mass ‘m’ is projected with a speed ‘u’ making an angle of 45 with the ground. The angular

momentum of the body about the point of projection, at the highest point is expressed as . The value
√ 2 mu

Xg

of ‘X’ is _____.
30. In the following circuit, the battery has an emf of 2V and an internal resistance of 2

3
Ω . The power
consumption in the entire circuit is _____ W.

31. Choose the correct statements from the following


A. Mn2O7 is an oil at room temperature
B. V2O4 reacts with acid to give VO22+
C. CrO is a basic oxide
D. V2O5 does not react with acid
Choose the correct answer from the options given below:
(a) A and C only
(b) A, B and C only
(c) B and C only
(d) A, B and D only

32. Given below are two statements :


Statement I: S8 solid undergoes disproportionation reaction under alkaline conditions to form S2– and
S2O32–
Statement II: ClO4– can undergo disproportionation reaction under acidic condition.
In the light of the above statements, choose the most appropriate answer from the options given below :
(a) Statement I is correct but statement II is incorrect.
(b) Both statement I and statement II are incorrect
(c) Both statement I and statement II are correct
(d) Statement I is incorrect but statement II is correct
33. Identify major product ‘P’ formed in the following reaction.

(a)

(b)

(c)

(d)
34. Identify structure of 2,3-dibromo-1-phenylpentane.

(a)

(b)

(c)

(d)

35. A(g) ⇌ B(g) + C


(g). The correct relationship between KP, α and equilibrium pressure P is
2
1 1

α 2 P 2

(a) KP = 3

(2+α) 2

3 1

α 2 P 2

(b) KP = 1

(2+α) 2 (1−α)
1 1

α 2 P 2

(c) KP = 1

(2+α) 2

1 3

α 2 P 2

(d) KP = 3

(2+α) 2

36. Identify the name reaction.

(a) Etard reaction


(b) Stephen reaction
(c) Gatterman - Koch reaction
(d) Rosenmund reduction
37. Major product of the following reaction is –

(a)

(b)

(c)

(d)

38. The four quantum numbers for the electron in the outer most orbital of potassium (atomic no. 19) are
(a) n = 4, l = 0, m = 0, s = +
1

(b) n = 2, l = 0, m = 0, s = +
1

(c) n = 4, l = 2, m = –1, s = +
1

(d) n = 3, l = 0, m = 1, s = +
1

2
39. The correct order of reactivity in electrophilic substitution reaction of the following compounds is:

(a) A>B>C>D
(b) B>C>A>D
(c) B>A>C>D
(d) D>C>B>A

40. Choose the correct statements from the following


A. All group 16 elements form oxides of general formula EO2 and EO3 where E = S, Se, Te and Po. Both
the types of oxides are acidic in nature.
B. TeO2 is an oxidising agent while SO2 is reducing in nature.
C. The reducing property decreases from H2S to H2 Te down the group.
D. The ozone molecule contains five lone pairs of electrons.
Choose the correct answer from the options given below:
(a) A and D only
(b) B and C only
(c) A and B only
(d) C and D only

41. Consider the following elements.

Which of the following is/are true about A', B', C' and D'?
A. Order of atomic radii: B' < A' < D' < C'
B. Order of metallic character : B' < A' < D' < C'
C. Size of the element : D' < C' < B' < A'
D. Order of ionic radii : B'+ < A'+ < D'+ < C'+
Choose the correct answer from the options given below :
(a) A, B and D only
(b) A and B only
(c) B, C and D only
(d) A only
42. Identify A and B in the following reaction sequence.

(a)

(b)

(c)

(d)
43. The azo-dye (Y) formed in the following reactions
is Sulphanilic acid + NaNO2 + CH3COOH → X

(a)

(b)

(c)

(d)

44. The fragrance of flowers is due to the presence of some steam volatile organic compounds called essential
oils. These are generally insoluble in water at room temperature but are miscible with water vapour in
vapour phase. A suitable method for the extraction of these oils from the flowers is -
(a) steam distillation
(b) crystallisation
(c) distillation under reduced pressure
(d) distillation
45. Given below are two statements :
Statement I: Group 13 trivalent halides get easily hydrolyzed by water due to their covalent nature.
Statement II: AlCl3 upon hydrolysis in acidified aqueous solution forms octahedral [Al(H2O)6]3+ ion.
In the light of the above statements, choose the correct answer from the options given below :
(a) Statement I is false but statement II is true
(b) Statement I is true but statement II is false
(c) Both statement I and statement II are false
(d) Both statement I and statement II are true

46. Given below are two statements :


Statement I: Aniline reacts with con. H2SO4, followed by heating at 453-473 K gives p-aminobenzene
sulphonic acid, which gives blood red colour in the 'Lassaigne's test'.
Statement II: In Friedel - Craft's alkylation and acylation reactions, aniline forms salt with the AlCl3
catalyst. Due to this, nitrogen of aniline aquires a positive charge and acts as deactivating group.
In the light of the above statements, choose the correct answer from the options given below :
(a) Both statement I and statement II are true
(b) Both statement I and statement II are false
(c) Statement I is true but statement II is false
(d) Statement I is false but statement II is true

47. Which of the following is least ionic?


(a) AgCl
(b) KCl
(c) BaCl2
(d) CoCl2

48. Match List I with List II

LIST – I (Complex ion) LIST – II (Electronic Configuration)


A. [Cr(H2O)6]3+ I. t2g2eg0
B. [Fe(H2O)6]3+ II. t2g3eg0
C. [Ni(H2O)6]2+ III. t2g3eg2
D. [V(H2O)6]3+ IV. t2g6eg2

Choose the correct answer from the options given below:


(a) A-IV, B-I, C-II, D-III
(b) A-IV, B-III, C-I, D-II
(c) A-II, B-III, C-IV, D-I
(d) A-III, B-II, C-IV, D-I
49. Select the option with correct property -
(a) [NiCl4]2– diamagnetic, [Ni(CO)4] paramagnetic
(b) [Ni(CO)4] and [NiCl4]2– both diamagnetic
(c) [Ni(CO)4] diamagnetic, [NiCl4]2– paramagnetic
(d) [Ni(CO)4] and [NiCl4]2– both paramagnetic

50. A sample of CaCO3 and MgCO3 weighed 2.21 g is ignited to constant weight of 1.152 g. The composition
of mixture is :
(Given molar mass in g mol–1 CaCO3 : 100, MgCO3 : 84)
(a) 1.187 g CaCO3 + 1.187 g MgCO3
(b) 1.023g CaCO3 + 1.023g MgCO3
(c) 1.187 g CaCO3 + 1.023g MgCO3
(d) 1.023g CaCO3 + 1.187 g MgCO3

51. r = k[A] for a reaction, 50% of A is decomposed in 120 minutes. The time taken for 90% decomposition of
A is ______ minutes.

52. If 5 moles of an ideal gas expands from 10 L to a volume of 100 L at 300 K under isothermal and reversible
condition then work, w, is –x J. The value of x is ______.
(Given R = 8.314 J K–1 mol–1)

53. A compound (x) with molar mass 108 g mol–1 undergoes acetylation to give product with molar mass 192 g
mol–1. The number of amino groups in the compound (x) is ______.

54. Number of isomeric products formed by monochlorination of 2-methylbutane in presence of sunlight is


_______.

55. The values of conductivity of some materials at


298.15 K in Sm–1 are 2.1 × 103,
1.0 × 10–16,1.2 × 10, 3.91,1.5 × 10–2,
1 × 10–7,1.0 × 103. The number of conductors among the materials is ______.

56. Number of moles of H+ ions required by 1 mole of MnO – to oxidise oxalate ion to CO is ____.
4 2

57. In the reaction of potassium dichromate, potassium chloride and sulfuric acid (conc.), the oxidation state of
the chromium in the product is (+)_______.

58. From the vitamins A, B1, B6, B, C, D, E and K, the number vitamins that can be stored in our body is
______.

59. A diatomic molecule has a dipole moment of 1.2 D. If the bond distance is 1Å, then fractional charge on
each atom is ______ × 10–1 esu .
(Given 1D = 10–18 esu cm)
60. The molarity of 1L orthophosphoric acid (H PO ) having 70% purity by weight (specific gravity 1.54 g cm–
3 4
3)is ______M.
(Molar mass of H3PO4 = 98 g mol–1)

61. Consider the function f : (0, ∞) → R defined by f (x) = e . If m and n be respectively the number
−|log x|
e

of points at which f is not continuous and f is not differentiable, then m + n is


(a) 1
(b) 3
(c) 0
(d) 2

62. Let A be a 3 × 3 real matrix such that


1 1 −1 −1 0 0
⎛ ⎞ ⎛ ⎞ ⎛ ⎞ ⎛ ⎞ ⎛ ⎞ ⎛ ⎞
A⎜0⎟ = 2⎜0⎟,A⎜ 0 ⎟ = 4⎜ 0 ⎟,A⎜1⎟ = 2⎜1⎟ .
⎝ ⎠ ⎝ ⎠ ⎝ ⎠ ⎝ ⎠ ⎝ ⎠ ⎝ ⎠
1 1 1 1 0 0

x 1
⎛ ⎞ ⎛ ⎞
Then, the system (A − 3I ) ⎜ y ⎟ = ⎜2⎟ has
⎝ ⎠ ⎝ ⎠
z 3

(a) no solution
(b) exactly two solutions
(c) unique solution
(d) infinitely many solutions

63. The number of solutions, of the equation e sin x


− 2e
− sin x
= 2, is:
(a) 1
(b) more than 2
(c) 0
(d) 2

64. Let a variable line passing through the centre of the circle x + y − 16x − 4y = 0, meet the positive co-
2 2

ordinate axes at the points A and B . Then the minimum value of OA + OB, where O is the origin, is equal
to
(a) 18
(b) 12
(c) 20
(d) 24

65. Let (α, β, γ) be the mirror image of the point (2, 3, 5) in the line x−1
=
y−2
=
z−3
. Then, 2α + 3β + 4γ
2 3 4

is equal to
(a) 33
(b) 31
(c) 32
(d) 34
66. The number of ways in which 21 identical apples can be distributed among three children such that each
child gets at least 2 apples, is
(a) 406
(b) 136
(c) 130
(d) 142

67. If the function f : (−∞, −1] → (a, b] defined by f (x) = e 3


x −3x+1
is one-one and onto, then the distance
of the point P (2b + 4, a + 2) from the line x + e y = 4 is: −3

− −−−−
(a) √1 + e 6

−−− −−
(b) 3√1 + e 6

−−− −−
(c) 4√1 + e 6

−−− −−
(d) 2√1 + e 6

68. If for some m, n; 6 6


C m + 2 ( C m+1 ) +
6
C m+2 >
8
C3 and n+1
P3 :
n
P4 = 1 : 8 , then n
P m+1 +
n+1
Cm

is equal to
(a) 384
(b) 372
(c) 376
(d) 380

69. Let A (a, b) , B (3, 4) and c (−6, −8) respectively denote the centroid, circumcentre and orthocentre of a
triangle. Then, the distance of the point P (2a + 3, 7b + 5) from the line 2x + 3y − 4 = 0 measured
parallel to the line x − 2y − 1 = 0 is
(a)
17√ 5

(b) 15√ 5

(c) 17√ 5

(d) √5

17

70. x x
2
1

Let f , g R be two functions defined by f (x) dt and g (x) .


2
2 −t −t
: (0, ∞) → = ∫ (|t| − t ) e = ∫ t 2 e dt
−x 0
−−−− −−−−
Then, the value of 9 (f (√log e
9 ) + g (√loge 9 )) is equal to
(a) 9
(b) 8
(c) 10
(d) 6

71. If a = sin (sin(5)) and b


−1
= cos
−1
(cos(5)) , then a 2
+ b
2
is equal to
(a) 25
(b) 8π − 40π + 50
2

(c) 4π + 25 2

(d) 4π − 20π + 50
2
72. Let f : R → (0, ∞) be strictly increasing function such that lim
f (7x)
= 1 . Then, the value of
f (x)
x→∞

f (5x)
lim [
f (x)
− 1] is equal to
x→∞

(a) 0
(b) 1
(c) 7

(d) 4

73. Let z and z be two complex numbers such that z


1 2 1 + z2 = 5 and z 1
3
+ z2
3
= 20 + 15i . Then,
∣ equals
4 4

∣z + z ∣
1 2

(a) 75
(b) 30√– 3

(c) 25√– 3
−−
(d) 15√15

74. The shortest distance between lines L and L , where L 1 2 1 :


x−1
=
y+1
=
z+4
and L is the line, passing
2
2 −3 2
y
through the points A (−4, 4, 3) , B (−1, 6, 3) and perpendicular to the line , is
x−3 z−1
= =
−2 3 1
121
(a) √ 221

141
(b) √ 221

24
(c) √ 117

42
(d) √ 117

75. The temperature T (t) of a body at time t = 0 is 160° F and it decreases continuously as per the differential
equation = −K (T − 80) , where K is a positive constant. If T (15) = 120 F , then T (45) is equal to
dT

dt

(a) 95° F
(b) 90° F
(c) 80° F
(d) 85° F

76. Let P be a parabola with vertex (2, 3) and directrix 2x + y = 6 . Let an ellipse E :
x
2

+
y2
= 1, a > b,
2 2
a b

of eccentricity 1

√2
pass through the focus of the parabola P . Then, the square of the length of the latus
rectum of E , is
(a) 347

(b)
656

25

(c) 385

(d) 512

25

77. The area of the region enclosed by the parabolas y = 4x − x


2
and 3y = (x − 4)
2
is equal to
(a) 6
(b) 4
(c) 32

(d) 14

3
78. Let 2nd, 8th and 44th terms of a non-constant A.P. be respectively the 1st, 2nd and 3rd terms of a G.P. If the
first term of the A.P. is 1, then the sum of its first 20 terms is equal to
(a) 960
(b) 990
(c) 980
(d) 970

79. A coin is biased so that a head is twice as likely to occur as a tail. If the coin is tossed 3 times, then the
probability of getting two tails and one head is
(a) 2

27

(b) 1

27

(c) 2

(d) 1

80. Let the mean and the variance of 6 observations a, b, 68, 44, 48, 60 be 55 and 194, respectively. If a > b ,
then a + 3b is
(a) 180
(b) 200
(c) 210
(d) 190

81. Let A (−2, −1) , B (1, 0) , C (α, β) and D (γ, δ) be the vertices of a parallelogram ABC D. If the point C
lies on 2x − y = 5 and the point D lies on 3x − 2y = 6, then the value of |α + β + γ + δ| is equal to
_____.

82. If lim
2 x
ax e −blog (1+x)+cxe
e
−x

= 1, then 16 (a 2
+ b
2 2
+ c ) is equal to ____.
2
x sin x
x→0

83. A line passes through A (4, −6, −2) and B (16, −2, 4). The point P (a, b, c), where a, b, c are non-
negative integers, on the line AB lies at a distance of 21 units, from the point A. The distance between the
points P (a, b, c) and Q (4, −12, 3) is equal to _____.

84. Let A be a 3 × 3 matrix and det (A) = 2 . If n = det (adj (adj (. . . . . (adj A)))), then the remainder

2024 - times

when n is divided by 9 is equal to _____.

85. Let the coefficient of x in the expansion of r

(x + 3)
n−1
+ (x + 3)
n−2
(x + 2) + (x + 3)
n−3 2
(x + 2) +. . . +(x + 2)
n−1
be α . If
r
n

∑ αr = β
n
− γ
n
, β, γ ∈ N , then the value of β 2
+ γ
2
equals _____.
r=0

86. Let a, b, c be the lengths of three sides of a triangle satisfying the condition
(a + b ) x − 2b (a + c) x + (b + c ) = 0. If the set of all possible values of x is the interval (α, β) ,
2 2 2 2 2

then 12 (α + β ) is equal to ____.


2 2
87. Let A = {1, 2, 3, . . . . . , 100}. Let R be a relation on A defined by (x, y) ∈ R if and only if 2x = 3y. Let
R be a symmetric relation on A such that R ⊂ R and the number of elements in R is n . Then, the
1 1 1

minimum value of n is ____.

88. Let a⃗ ^ ^ ^
= 3 i + 2 j + k,
⃗ ^ ^ ^
b = 2 i − j + 3kand c ⃗ be a vector such that
2
2 (a ⃗ × b) + 24 j − 6k and (a ⃗ − b + i ) ⋅ c ⃗ = −3. Then |c |⃗ is equal to ___.
⃗ ⃗ ^ ^ ⃗ ^
(a ⃗ + b) × c ⃗ =

89. ∣ π
2


120

3

x sin x cos x
4 4
dx∣ is equal to ____.
π sin x+cos x
∣ 0

90. Let y = y (x) be the solution of the differential equation


2
sec x dx + (e
2y 2
tan x + tan x) dy = 0, 0 < x <
π

2
,y(
π

4
) = 0 . If y ( π

6
) = α, then e 8α
is equal to
______.
Answer Key

1. B 2. D 3. B 4. B 5. A 6. B
7. D 8. C 9. A 10. D 11. B 12. D
13. C 14. A 15. A 16. D 17. A 18. B
19. B 20. B 21. 27 22. 4 23. 20 24. 2
25. 200 26. 12 27. 53 28. 12 29. 8 30. 3
31. A 32. A 33. C 34. C 35. B 36. C
37. C 38. A 39. B 40. C 41. A 42. D
43. D 44. A 45. D 46. A 47. A 48. C
49. C 50. C 51. 399 52. 28721 53. 2 54. 6
55. 4 56. 4 57. 6 58. 5 59. 0 60. 11
61. A 62. C 63. C 64. B 65. A 66. B
67. D 68. B 69. C 70. B 71. B 72. A
73. A 74. B 75. B 76. B 77. A 78. D
79. C 80. A 81. 32.00 82. 81.00 83. 22.00 84. 7.00
85. 25.00 86. 36.00 87. 66.00 88. 38.00 89. 15.00 90. 9.00
Solutions

1. (B)
2
[B] = L
2 2
x L 1
A = = 2 −2
= −1
tE TML T MT
−1
[A] = M T
2 −1 1
[AB] = [L M T ]

2. (D)
q q
In air F = 1

4π∈ 0
1

r2
2

In medium
1 q q q q
′ 1 2 25 1 2
F = = = 5F
2 2
4π(K∈ 0 ) ′ 4π(5∈ 0 )
(r ) (r)

3. (B)
−−− −−−− −−−− −−−−
The magnitude of resultant vector R = ′
√a 2 + b2 + 2ab cos θ

Here a = b = R
− −−−−−−−−−−−−−−−
Then R = √R + R + 2R cos θ
′ 2 2 2

– −−−− −−−
= R√ 2√ 1 + cos θ
−−−−− −
– 2 θ
= √ 2R√2cos
2

θ
= 2R cos
2

4. (B)

− −
− −−−−−−−−
γRT
1.4×8.3×273
v = √ = √ −3
M 32×10

= 314.8541 ≃ 315m/s

5. (A)
2
1 2 B
ε0 E =
2 2μ
0

∵ E = CB and C =
μ ε0
1

6. (B)
R2
R1 =
2
1 1
R0
R 0 (A1 ) 3
= (A2 ) 3

2
1
A1 = A2
8
192
A1 = = 24
8

7. (D)
2
P = i R
2
P int = I R
int
2
P f inal = (0.8 Iint ) R
Pfinal −Pint
% change in power = Pint
× 100 = (0.64 − 1) × 100 = −36%
8. (C)

If A ¯
= 0; A = 1
¯
A = 1; A = 0
¯
B = 0; B = 1
¯
B = 1; B = 0
¯¯¯¯¯¯¯¯¯¯¯¯¯¯¯¯¯¯¯¯¯¯¯¯¯¯¯¯¯¯¯¯¯¯¯¯¯¯¯¯¯¯¯
¯¯¯¯¯¯¯¯¯¯¯¯¯¯¯
¯ ¯
Y = (A + B ) + (A + B) = (1 + 1) = 0

9. (A)



T = 2π√
g

2
4π ℓ
g = 2
T
Δg Δℓ 2ΔT
= +
g ℓ T
0.2 1
= + 2( )
20 40
0.3
=
20

Percentage change = 0.3

20
× 100 = 6%

10. (D)
⃗ ^ 2^
F = (6t i + 6t j) N

⃗ ^ 2^
F = ma ⃗ = (6t i + 6t j)


F ^ 2^
a⃗ = = (3t i + 3t j)
m

t
2
3t ^ 3^
v ⃗ = ∫ a dt
⃗ = i + t j
2
0

⃗ 3 5
P = F . v ⃗ = (9t + 6t ) W

11. (B)
−−−−
2GM
V escape = √
R
−−−−
2GM
V Planet = √ = V
R
−−−−−− −−−−
2GM ×16 1 2GM
V Moon = √ = √
144R 3 R

V Planet V
V Moon = =
3 3
12. (D)

25 X
= . . . . . (i)
rℓ1 rℓ2

25 X

= ′
. . . . (ii)
2rℓ 1 2rℓ 2

From (i) and (ii)



ℓ = ℓ2 = 40cm
2

13. (C)
fK = μmg cos θ
50×√ 3
= 0.1 ×
2

= 2.5√ 3N

F 1 = mg sin θ + fK

= 25 + 2.5√ 3

F 2 = mg sin θ − fK

= 25 − 2.5√ 3

∴ F 1 − F 2 = 5√ 3N

14. (A)
Since density is negligible hence Buoyancy force will be negligible
At terminal velocity. M g = 6πηrv
V ∝ (as mass is constant)
1

Now,

v r

=
v r

r = 2r

So, v ′
=
v

2
15. (A)
< P >= I V cos ϕ
20 10 ∘
= × × cos 60
√2 √2

= 50W

16. (D)
f
Since < f i.e. the incident frequency is less than threshold frequency. Hence thee will be no emission of
2
0

photoelectrons.
⇒Current = 0

17. (A)
By Brewster’s law

At complete reflection refracted ray and reflected ray are perpendicular.

18. (B)

⃗ ⃗
M = iA

^
= 5 × (0.2) × (0.1) (− i )

= 0.1 (−^
i)

⃗ ⃗ ^ −3 ^
τ ⃗ = M × B = 0.1 (− i ) × (2 × 10 ) ( j)

−4 ^
= 2 × 10 (−k ) N − m

19. (B)
(m1 −m2 )g g
a = =
(m1 +m2 ) 8

8m1 − 8m2 = m1 + m2

7m1 = 9m2
m1 9
=
m2 7

20. (B)
U = nC V T

⇒ U = n1 C V 1 T + n2 C V 2 T
3R 5R
⇒ 8 × × T + 6 × × T = 27RT
2 2
21. (27)

9
⃗ r⃗
Kp . 9×10 (6qℓ)

V = = cos(120 )
3 2
r r

qℓ
9 2 −2
= − (27) ( ) × 10 N m c
2
r

⇒ α = 27

22. (4)
For a nucleus
Volume: V = 4

3
πR
3

R = R 0 (A) 3

4 3
V = πR A
4 0

V2 A2
⇒ = = 4
V1 A1

23. (20)

μ N i1 μ ×1×100
0 0 −3
BP = = = 2 × 10 T
2r 2π
μ N i2 μ ×2×100
0 0 −3
BQ = = = 4 × 10 T
2r 2π
−−−−−−−−
2 2
B net = √B + B
P Q

− −
= √ 20 mT

x = 20

24. (2)

ε = −( ) = 10t − 36
dt

At t = 2, ε = 16V
ε 16
i = = = 2A
R 8
25. (200)

μ μ μ −μ
2 1 2 1
− =
v u R
1.5 1 1.5−1
− =
v −100 20

v = 100cm

Distance from object


= 100 + 100 = 200cm

26. (12)
2k.k 5k
k eq = + k =
3k 3

−−
keq
Angular frequency of oscillation (ω) = √
m
−−

5k
(ω) = √
3m
−−

Period of oscillation (τ ) =

ω
= 2π√
3m

5k
−−−
12m
= π√
5k

27. (53)

2 2 2
I = ( mR + md ) × 2
5
2 2
2 1 3 53 2
I = 2( × 2 × ( ) + 2 × ( ) ) = kg − m
5 2 4 20

X = 53

28. (12)
2m1 m2 80
T = ( )g = N
m1 +m2 3

2 −10 2
A = πr = 16π × 10 m

Strain = Δℓ


=
F

AY
=
T

AY
80

3 1
= −10 11
=
16π×10 ×2×10 12π

α = 12
29. (8)

2 2
u sin θ
L = mu cos θ
2g

3 1
= mu ⇒ x = 8
4√ 2 g

30. (3)
4
R eq = Ω
3
2
V 4
∴ P = = = 3W
R eq 4/3

31. (A)
(A) Mn2O7 is green oil at room temperature.
(B) V2O4 dissolve in acids to give VO2+ salts.
(C) CrO is basic oxide
(D) V2O5 is amphoteric it reacts with acid as well as base.

32. (A)
S1: S8 + 12 OH⊖ → 4S2– + 2S2O32– + 6H2O
S2: ClO4⊖ cannot undergo disproportionation reaction as chlorine is present in it’s highest oxidation state.

33. (C)
34. (C)

2, 3-dibromo -1-phenylpentane

35. (B)
A(g) ⇌ B(g) + (g) C

t = teq (1 – α), α, α

2
α

α (1−α)
2
PB = . P, PA = . P , PC = .P
α α α
(1+ ) (1+ ) (1+ )
2 2 2

2
PB .P
C
KP =
PA
3 1

α 2 P 2

KP = 1

(2+α) 2 (1−α)

36. (C)

Gatterman-Koch reaction

37. (C)

38. (A)
2 2 6 2 6 1
19K 1s , 2s , 2p , 3s , 3p , 4s .
Outermost orbital of potassium is 4s orbital
n = 4, l = 0, m = 0, s = + . 1

39. (B)
–CH3 shows +M and +I .
–Cl shows +M and –I but inductive effect dominates.
–NO2 shows –M and –I.
Electrophilic substitution α −M and −I
1

α + M and +I
Hence, order is B > A > C > D.
40. (C)
(A) All group 16 elements form oxides of the EO2 and EO3 type where E = S, Se, Te or Po.
(B) SO2 is reducing while TeO2 is an oxidising agent.
(C) The reducing property increases from H2S to H2Te down the group.
(D)

have six lone pairs.

41. (A)
In general along the period from left to right, size decreases and metallic character decrease.
In general down the group, size increases and metallic character increases.
B' < A'(size) C' > A'(size)
D' < C'(size) D' > B'(size)
B' < A' (metallic character)
D' < C' (metallic character)
B'+ < A'+ (size)
D'+ < C'+ (size)
∴ C statement is incorrect.

42. (D)
43. (D)

This is known as Griess-Ilosvay test.

44. (A)
Steam distillation technique is applied to separate substances which are steam volatile and are immiscible with
water.

45. (D)
In trivalent state most of the compounds being covalent are hydrolysed in water. Trichlorides on hydrolysis in
water form tetrahedral [M(OH)4]– species, the hybridisation state of element M is sp3.
In case of aluminium, acidified aqueous solution forms octahedral [Al(H2O)6]3+ ion.

46. (A)
47. (A)
AgCl < CoCl2 < BaCl2 < KCl (ionic character)
Reason: Ag+ has pseudo inert gas configuration.

48. (C)
[Cr(H2O)6]3+ Contains Cr3+ : [Ar] 3d3 : t2g3eg0
[Fe(H2O)6]3+ Contains Fe3+ : [Ar] 3d5 : t2g3eg2
[Ni(H2O)6]2+ Contains Ni2+ : [Ar] 3d8 : t2g6eg2
[V(H2O)6]3+ Contains V3+ : [Ar] 3d2 : t2g2eg0

49. (C)
[Ni(CO)4], diamagnetic, sp3 hybridisation, number of unpaired electrons = 0
[NiCl4]2–, paramagnetic, sp3 hybridisation, number of unpaired electrons = 2

50. (C)
Δ

CaCO3(s) −→ CaO(s) + CO2(g)


Δ

MgCO3(s) −→ MgO(s) + CO2(g)


Let the weight of CaCO3 be x gm
∴ weight of MgCO3 = (2.21 – x) gm
Moles of 3 CaCO3 decomposed = moles of CaO Formed
x

100
= moles of CaO formed
∴ weight of CaO formed = × 56 x

100

Moles of MgCO3 decomposed = moles of MgO formed


(2.21−x)

84
= moles of MgO formed
∴ weight of MgO formed=
2.21−x
× 40
84
2.21−x x
⇒ × 40 + × 56 = 1.152
84 100

∴ x = 1.1886 g = weight of CaCO3 & weight of 3 MgCO3 = 1.0214 g

51. (399)
r = k[A]
So, order of reaction = 1
t1/2 = 120min
For 90% completion of reaction
2.303 a
⇒ k = log( )
t a−x
0.693 2.303 100
⇒ = log
t 1/2 t 10

∴ t = 399 min.
52. (28721)
It is isothermal reversible expansion, so work done negative
V2
W = –2.303 nRT log( V1
)

= –2.303 × 5 × 8.314 × 300 log( 100

10
)

= –28720.713 J
≡ –28721 J

53. (2)

Gain in molecular weight after acylation with one –NH2 group is 42.
Total increase in molecular weight = 84
∴ Number of amino group in x = =2
84

42

54. (6)

∴ Number of isomeric products = 6

55. (4)
Conductivity (S m–1)
2.1 × 103, 1.2 × 10, 3.91, 1 × 103
are conductors at 298.15K
1 × 10–16 Insulator at 298.15 K
1.5 × 10–2, 1 × 10–7
Semiconductor at 298.15 K
Therefore, number of conductors is 4.

56. (4)
2MnO4– + 5C2O42– + 16H+ → 2Mn2+ + 10CO2 + 8H2O
∴ Number of moles of H+ ions required by 1 mole of MnO4– to oxidise oxalate ion to CO2 is 8
57. (6)
K2Cr2O7(s) + 4KCl(s) + 6H2SO4 conc. → 2CrO2Cl2(g) + 6KHSO4 + 3H2O
This reaction is called chromyl chloride test.
Here oxidation state of Cr is +6.

58. (5)
Vitamins A, D, E, K and B12 are stored in liver and adipose tissue.

59. (0)
μ = 1.2 D = q × d
⇒ 1.2 × 10–10 esu Å = q × 1Å
∴ q = 1.2 × 10–10 esu

60. (11)
Specific gravity (density) = 1.54 g/cc.
Volume = 1L = 1000 ml
Mass of solution = 1.54 × 1000 = 1540 g
% purity of H2SO4 is 70%
So weight of H3PO4 = 0.7 × 1540 = 1078 g
Mole of H3PO4 = 1078

98
= 11
Molarity = = 11 11

1 L

61. (A)
f : (0, ∞) → R
−|log x|
f (x) = e e

1
; 0 < x < 1
− ln x
1 e
f (x) = = {
|ln x| 1
e
; x ⩾ 1
ln x
e
1
⎧ 1
= x; 0 < x < 1

⎨ x

⎩ 1
; x ⩾ 1
x

m = 0 (No point at which function is not continuous)


n = 1 (Not differentiable)
62. (C)
x1 y1 z1 ⎤

Let A = ⎢ x2 y2 z2 ⎥
⎣ ⎦
x3 y3 z3

1 2
⎡ ⎤ ⎡ ⎤
Given A ⎢ 0 ⎥ = ⎢0⎥ . . . . (1)

⎣ ⎦ ⎣ ⎦
1 2

x1 + z 1 2
⎡ ⎤ ⎡ ⎤
∴ ⎢ x2 + z 2 ⎥ = ⎢ 0 ⎥
⎣ ⎦ ⎣ ⎦
x3 + z 3 2

∴ x1 + z 1 = 2 . . . . (2)

x2 + z 2 = 0 . . . . (3)

x3 + z 3 = 0 . . . (4)

−1 −4
⎡ ⎤ ⎡ ⎤
Given A ⎢ 0 ⎥ = ⎢ 0 ⎥
⎣ ⎦ ⎣ ⎦
1 4

−x1 + z 1 4
⎡ ⎤ ⎡ ⎤
∴ ⎢ −x2 + z 2 ⎥ = ⎢ 0 ⎥
⎣ ⎦ ⎣ ⎦
−x3 + z 3 4

⇒ −x1 + z 1 = −4 . . . (5)

−x2 + z 2 = 0 . . . . (6)

−x3 + z 3 = 4

0 0
⎡ ⎤ ⎡ ⎤
Given A ⎢ 1 ⎥ = ⎢2⎥

⎣ ⎦ ⎣ ⎦
0 0

y1 0
⎡ ⎤ ⎡ ⎤
∴ ⎢ y2 ⎥ = ⎢ 2 ⎥
⎣ ⎦ ⎣ ⎦
y3 0

∴ y1 = 0, y2 = 2, y3 = 0

∴ from (2), (3), (4), (5), (6) and (7)


x1 = 3x, x2 = 0, x3 = −1

y1 = 0, y2 = 2, y3 = 0

z 1 = −1, z 2 = 0, z3 = 3

3 0 −1
⎡ ⎤
∴ A = ⎢ 0 2 0 ⎥
⎣ ⎦
−1 0 3

x −1
⎡ ⎤ ⎡ ⎤
∴ Now (A − 3I ) ⎢ y ⎥ = ⎢ 2 ⎥
⎣ ⎦ ⎣ ⎦
z 3

0 0 −1 x −1
⎡ ⎤⎡ ⎤ ⎡ ⎤
∴ ⎢ 0 −1 0 ⎥⎢ y ⎥ = ⎢ 2 ⎥
⎣ ⎦⎣ ⎦ ⎣ ⎦
−1 0 0 z 3

−z 1
⎡ ⎤ ⎡ ⎤

⎢ −y ⎥ = ⎢ 2 ⎥
⎣ ⎦ ⎣ ⎦
−x 3

[z = −1] , [y = −2] , [x = −3]


63. (C)
Take e sin x
= t (t > 0)
2
⇒ t − = 2
t
2
t −2
⇒ = 2
t
2
⇒ t − 2t − 2 = 0
2
⇒ t − 2t + 1 = 3
2
⇒ (t − 1) = 3

⇒ t = 1 ± √3

⇒ t = 1 ± 1.73

⇒ t = 2.73 or −0.73 (rejected as t )


> 0
sin x
⇒ e = 2.73
sin x
⇒ loge e = loge 2.73

⇒ sin x = loge 2.73 > 1

So no solution

64. (B)
(y − 2) = m (x − 8)

⇒ x -intercept
−2
⇒ ( + 8)
m

⇒ y -intercept
⇒ (−8m + 2)
−2
⇒ OA + OB = + 8 − 8m + 2
m
′ 2
f (m) = − 8 = 0
2
m
2 1
⇒ m =
4
−1
⇒ m =
2

−1
⇒ f ( ) = 18
2

⇒ Minimum = 18

65. (A)


−→
∵ P R⊥ (2, 3, 4)

−→
∴ P R. (2, 3, 4) = 0

(α − 2, β − 3, γ − 5) . (2, 3, 4) = 0

⇒ 2α + 3β + 4γ = 4 + 9 + 20 = 33

66. (B)
After giving 2 apples to each child 15 apples left now 15 apples can be distributed in 15+3−1
C2 =
17
C2 ways
17×16
= = 136
2
67. (D)
3
x −3x+1
f (x) = e
3
′ x −3x+1 2
f (x) = e ⋅ (3x − 3)
3
x −3x+1
= e .3 (x − 1) (x + 1)

For f ′
(x) ⩾ 0

∴ f (x) is increasing function


−∞
∴ a = e = 0 = f (−∞)
−1+3+1 3
b = e = e = f (−1)

P (2b + 4, a + 2)
3
∴ P (2e + 4, 2)

3 −3
(2e +4)+2e −4 −−−−−
d = = 2√1 + e 6
√ 1+e−6

68. (B)
6 6 6 8
C m + 2 ( C m+1 ) + C m+2 > C3
7 7 8
C m+1 + C m+2 > C3
8 8
C m+2 > C3

∴ m = 2

And n−1
P3 :
n
P4 = 1 : 8
(n−1)(n−2)(n−3)
1
=
n(n−1)(n−2)(n−3) 8

∴ n = 8
n n+1 8 9
∴ P m+1 + Cm = P3 + C2
9×8
= 8 × 7 × 6 +
2

= 372

69. (C)
A (a, b) , B (3, 4) , C (−6, 8)

⇒ a = 0, b = 0 ⇒ P (3, 5)

Distance from P measured along x − 2y − 1 = 0

⇒ x = 3 + r cos θ, y = 5 + r sin θ

where tan θ =
1

r (2 cos θ + 3 sin θ) = −17

∣ −17√ 5 ∣ 17√ 5
⇒ r = =
∣ 7 ∣ 7
70. (B)
x
2
2 −t
f (x) = ∫ (|t| − t ) e dt
−x
2
′ 2 −x
⇒ f (x) = 2. (|x| − x ) e . . . (1)
2
x
1
−t
g (x) = ∫ t 2 e dt
0
2
′ −x
g (x) = xe (2x) − 0
2 2 2
′ ′ −x 2 −x 2 −x
f (x) + g (x) = 2xe − 2x e + 2x e

Integrating both sides w.r.t. x


α
2
−x
f (x) + g (x) = ∫ exe dx

0
2
x = t

√α
α
−t −t √
⇒ ∫ e dt = [−e ]
0
0
−1
(log (9) )+1
e
= −e
1
⇒ 9 (f (x) + g (x)) = (1 − )9 = 8
9

71. (B)
−1
a = sin (sin 5) = 5 − 2π

and b = cos
−1
(cos 5) = 2π − 5
2 2 2 2
∴ a + b = (5 − 2π) + (2π − 5)
2
= 8π − 40π + 50

72. (A)
f : R → (0, ∞)
f (7x)
lim = 1
f (x)
x→∞

∵ f is increasing
∴ f (x) < f (5x) < f (7x)
f (x) f (5x) f (7x)
∵ < <
f (x) f (x) f (x)

f (5x)
1 < lim < 1
f (x)
x→∞

f (5x)
∴ [ − 1]
f (x)

⇒ 1 − 1 = 0
73. (A)
z1 + z2 = 5
3 3
z1 + z2 = 20 + 15i
3 3
z1 + z2 = 125 − 3z 1 . z 2 (5)

⇒ 20 + 15i = 125 − 15z 1 z 2

⇒ 3z 1 z 2 = 25 − 4 − 3i

⇒ 3z 1 z 2 = 21 − 3i

⇒ z1 . z2 = 7 − i
2
⇒ (z 1 + z 2 ) = 25
2 2
⇒ z1 + z2 = 25 − 2 (7 − i)

⇒ 11 + 2i
2
2 2
(z 1 + z2 ) = 121 − 4 + 44i

4 4 2
⇒ z1 + z2 + 2(7 − i) = 117 + 44i
4 2
⇒ z1 + z2 = 117 + 44i − 2 (49 − 1 − 14i)
4 4
⇒ ∣
∣z1 + z2 ∣∣ = 75

74. (B)
x+4 y−4 z−3
L2 = = =
3 2 0
∣ x 2 −x 1 y −y z2 −z1 ∣
2 1
∣ ∣
∣ 2 −3 2 ∣
∣ ∣

S.D. =
∣ 3 2 0 ∣

∣ ⃗ ⃗
∣n1 ×n2 ∣

∣ 5 −5 −7 ∣
∣ ∣
∣ 2 −3 2 ∣
∣ ∣
∣ 3 2 0 ∣
=
∣ ⃗ ⃗
∣n1 ×n2 ∣

141
=
∣ ^ ^ ^∣
−4 i +6 j +13k
∣ ∣

141
=
√ 16+36+169

141
=
√ 221

75. (B)
dT
= −k (T − 80)
dt

T t
dT
∫ = ∫ −K dt
(T −80)
160 0
T
[ln|T − 80|] = −kt
160

ln|T − 80| − ln 80 = −kt


T −80
ln∣

∣ = −kt

80
−kt
T = 80 + 80e
−k.15
120 = 80 + 80e
40 −k15 1
= e =
80 2
−k.45
∴ T (45) = 80 + 80e
3
−k.15
= 80 + 80(e )
1
= 80 + 80 ×
8

= 90
76. (B)

Slope of axis = 1

2
1
y − 3 = (x − 2)
2

⇒ 2y − 6 = x − 2

⇒ 2y − x − 4 = 0

2x + y − 6 = 0

4x + 2y − 12 = 0

α + 1.6 = 4 ⇒ α = 2.4

β + 2.8 = 6 ⇒ β = 3.2

Ellipse passes through (2.4, 3.2)


2 2
24 32
( ) ( )
10 10

⇒ + 2
= 1 . . . (1)
2
a b
2 2

Also 1 − b

a
2
=
1

2
=
b

a
2
=
1

2
2 2
⇒ a = 2b

Put in (1) ⇒ b
2
=
328

25

2 2 2
2b 4b 2 1 328 656
⇒ ( ) = × b = 4 × × =
a a
2 2 25 25

77. (A)

4
∣ (x−4)
2 ∣
Area = ∣∫ [(4x − x ) −
2

3
] dx∣
∣1 ∣
3 4
∣ (x−4) ∣
Area =
2 3
4x x
∣ − − ∣
3
∣ 2 9 ∣1
64 64 4 1 27
= ∣
∣( − − + − )∣

2 3 2 3 9

⇒ (27 − 21) = 6
78. (D)
1 + d, 1 + 7d, 1 + 43d are in GP
2
(1 + 7d) = (1 + d) (1 + 43d)
2 2
1 + 49d + 14d = 1 + 44d + 43d
2
6d − 30d = 0

d = 5
20
S20 = [2 × 1 + (20 − 1) × 5]
2

= 10 [2 + 95]

= 970

79. (C)
Let probability of tail is 1

⇒ Probability of getting head =


2

∴ Probability of getting 2 tails and 1 head


1 2 1
= ( × × ) × 3
3 3 3
2
= × 3
27
2
=
9

80. (A)
a, b, 68, 44, 48, 60

Mean = 55 a > b
Variance = 194 a + 3b
a+b+68+44+48+60
= 55
6

⇒ 220 + a + b = 330

∴ a + b = 110 . . . . (1)

Also,
2
(x i−x̄ )
∑ = 194
n
2 2 2 2 2 2
⇒ (a − 55) + (b − 55) + (68 − 55) + (44 − 55) + (48 − 55) + (60 − 55) = 194 × 6
2 2
⇒ (a − 55) + (b − 55) + 169 + 121 + 49 + 25 = 1164
2 2
⇒ (a − 55) + (b − 55) = 1164 − 364 = 800
2 2
a + 3025 − 110a + b + 3025 − 110b = 800
2 2
⇒ a + b = 800 − 6050 + 12100
2 2
a + b = 6850 . . . . (2)

Solve (1) & (2);


a = 75, b = 35

∴ a + 3b = 75 + 3 (35) = 75 + 105 = 180


81. (32.00)

α−2 β−1 γ+1


δ
P ≡ ( , ) ≡ ( , )
2 2 2 2

γ+1 β−1
and
α−2 δ
= =
2 2 2 2

⇒ α − γ = 3 . . . . (1) , β − δ = 1 . . . . (2)

Also, (γ, δ) lies on 3x − 2y = 6

3γ − 2δ = 6 . . . . (3)

and (α, β) lies on 2x − y = 5

⇒ 2α − β = 5 . . . . . (4)

Solving (1) , (2), (3), (4)


α = −3, β = −11, γ = −6, δ = −12

|α + β + γ + δ| = 32

82. (81.00)
2 3 2 3
2 x x x x
a (1 + x + + + … . .) − b (x − + − … … .)
2! 3! 2 3

2 3
x x
+cx(1−x+ − +…….)
x! 3!

lim sin x
3
x→0 x ⋅
x

b 2 b c 3
(c−b)x+( −c+a)x +(a− + )x +……
2 3 2

= lim = 1
3
x→∞ x

b
c − b = 0, − c + a = 0
2
b c 3 3
a − + = 1 a = b = c =
3 2 4 2
2 2 2 9 9 9
a + b + c = + +
16 4 4
2 2 2
16 (a + b + c ) = 81

83. (22.00)
x−4 x+6 z+2
= =
12 4 6
x−4 y+6 z+2

6
= 2
= 3
= 21
7 7 7

6 2 3
(21 × + 4, × 21 − 6, × 21 − 2)
7 7 7

= (22, 0, 7) = (a, b, c)
−−−−−−−−−−−−
∴ √ 324 + 144 + 16 = 22
84. (7.00)
|A| = 2
2024
(n−1)
adj (adj (adj. . . . . (a))) = |A|

2024 times
2024
2
= |A|
2024
2
= 2
2024 674 674
2 2 2022
2 = (2 ) 2 = 4(8) = 4(9 − 1)

2024
⇒ 2 ≡ 4 ( mod 9)

⇒ 2
2024
≡ 9m + 4, m ← even
3m
9m+4 3
2 ≡ 16.(2 ) ≡ 16 ( mod 9)

≡ 7

85. (25.00)
n−1 n−2 n−3
(x + 3) + (x + 3) (x + 2) + (x + 3)
2 n−1
(x + 2) + … … . +(x + 2)
n−1 n−2 n−3 2 n−1
∑ αr = 4 + 4 × 3 + 4 × 3 …… + 3
2 n−1
n−1 3 3 3
= 4 [1 + + ( ) … . +( ) ]
4 4 4
n
3
1−( )
n−1 4

= 4 × 3
1−
4
n n n n
= 4 − 3 = β − γ

β = 4, γ = 3
2 2
β + γ = 16 + 9 = 25

86. (36.00)
2 2 2 2 2
(a + b )x − 2b(a + c)x + b + c = 0
2 2 2 2 2 2
⇒ a x − 2abx + b + b x − 2bcx + c = 0
2 2
⇒ (ax − b) + (bx − c) = 0

⇒ ax − b = 0, bx − c = 0

⇒ a + b > c b + c > a c + a > b


2
a + ax > bx ax + bx > a ax + a > ax
2 2 2
a + ax > ax ax + ax > a x − x + 1 > 0
2 2
x − x − 1 < 0 x + x − 1 > 0 always true
1−√ 5 1+√ 5
< x <
2 2
−1−√ 5 −1+√ 5
x < , or x >
2 2

√ 5 −1 √ 5 +1
⇒ < x <
2 2

√ 5 −1 √ 5 +1
⇒ α = ,β =
2 2
2 2
(√ 5 −1) +(√ 5 +1)
2 2
12 (α + β ) = 12 ( ) = 36
4

87. (66.00)
R = {(3, 2) , (6, 4) , (9, 6) , (12, 8) , . . . (99, 66)}

n (R) = 33

∴ 66
88. (38.00)
⃗ ⃗ ^ ^
(a ⃗ + b) × c ⃗ = 2(a ⃗ × b) + 24 j − 6k

^ ^ ^ ^ ^ ^ ^ ^
(5 i + j + 4k ) × c ⃗ = 2(7 i − 7 j − 7k ) + 24 j − 6k

∣ ^ ^ ^∣
i j k
∣ ∣
^ ˆ ˆ
∣ 5 1 4 ∣ = 14 i + 10 j − 20k
∣ ∣
∣x y z∣
ˆ ˆ ˆ ˆ ˆ ˆ
⇒ i (z − 4y) − j(5z − 4x) + k (5y − x) = 14 i + 10 j − 20k

z − 4y = 14, 4x − 5z = 10, 5y − x = −20



(a − b + i) ⋅ c = −3

(2ˆ ˆ ˆ
i + 3 j − 2k ) ⋅ c = −3

2x + 3y − 2z = −3

∴ x = 5, y = −3, z = 2
2
|c |⃗ = 25 + 9 + 4 = 38

89. (15.00)
π
2
x sin x⋅cos x
∫ 4
dx
4
sin x+cos x
0
π

sin x⋅cos x 2 2
= ∫ 4
(x − (π − x) ) dx
4
sin x+cos x
0
π

2 2
sin x⋅cos x(2πx−π )
= ∫ 4 4
sin x+cos x
0
π π

2 2

x sin x cos x 2 sin x cos x


= 2π ∫ 4
dx − π ∫ 4
dx
4 4
sin x+cos x sin x+cos x
0 0
π π

2 2

π sin x cos x 2 sin x cos x


= 2π ⋅ ∫ 4
dx − π ∫ 4
dx
4 sin x+cos x
4 4
sin x+cos x
0 0
π

2
2
π sin x cos x
= − ∫ 4
dx
2 sin x+cos x
4

0
π

2
2
π sin x cos xdx
= − ∫ 2
2 1−2sin x×cos x
2

0
π

2
2
π sin 2x
= − ∫ 2
dx
2 2−sin 2x
0
π

2
2
π sin 2x
= − ∫ dx
2 2
1+cos 2x
0

Let cos 2x = t
90. (9.00)
2 dx 2y 2
sec x + e tan x + tan x = 0
dy

2 dx dt
(Put tan x = 1 ⇒ sec x = )
dy dy

dt 2y 2
+ e × t + t = 0
dy

dt 2 2y
+ t = −t . e
dy

1 dt 1 −2y
+ = −e
2
dy t
t

1 −1 dt du
(Put = u = )
t 2
t dy dy

−du 2y
+ u = −e
dy

du 2y
− u = e
dy

− ∫ dy −y
I. F . = e = e
−y −y 2y
ue = ∫ e × e dy
1 −y y
× e = e + c
tan x
π
x = , y = 0, c = 0
4
π
x = ,y = α
6
– −α α
√ 3e = e + 0


e = √3

e = 9
JEE Main 01-02-2024 (Morning Shift)

Questions

1. The dimensional formula of angular impulse is:


(a) [M LT ] −1

(b) [M L T ] 2 −1

(c) [M L T ] −2 −1

(d) [M L T ] 2 −2

2. Two moles a monatomic gas is mixed with six moles of a diatomic gas. The molar specific heat of the
mixture at constant volume is:
(a) R 5

2
3
(b) R 2

(c) R 7

(d) R 9

3. In the given circuit if the power rating of Zener diode is 10mW, the value of series resistance R to regulate
s

the input unregulated supply is:

(a) 1kΩ

(b) 10Ω

(c) 10kΩ

(d) 5kΩ

4. The minimum energy required by a hydrogen atom in ground state to emit radiation in Balmer series is
nearly:
(a) 13.6 eV
(b) 12.1 eV
(c) 1.9 eV
(d) 1.5 eV

5. o

A monochromatic light of wavelength 6000A is incident on the single slit of width 0.01 mm. If the
diffraction pattern is formed at the focus of the convex lens of focal length 20 cm, the linear width of the
central maximum is:
(a) 60 mm
(b) 12 mm
(c) 120 mm
(d) 24 mm
6. Two identical capacitors have same capacitance C. One of them is charged to the potential V and other to the
potential 2V. The negative ends of both are connected together. When the positive ends are also joined
together, the decrease in energy of the combined system is:
(a) C V
3

4
2

1
(b) C V
4
2

1
(c) C V
2
2

(d) 2C V 2

7. With rise in temperature, the Young’s modulus of elasticity:


(a) Remains unchanged
(b) Increases
(c) Decreases
(d) Changes erratically

8. The reading in the ideal voltmeter (V) shown in the given circuit diagram is:

(a) 0V
(b) 3V
(c) 10V
(d) 5V

9. A parallel plate capacitor has a capacitance C = 200pF . It is connected to 230V ac supply with an angular
frequency 300 rad/s. The rms value of conduction current in the circuit and displacement current in the
capacitor respectively are:
(a) 13.8μA and 13.8μA
(b) 1.38μA and 1.38μA
(c) 13.8μA and 138μA
(d) 14.3μA and 143μA

10. 10 divisions on the main scale of a Vernier calliper coincide with 11 divisions on the Vernier scale. If each
division on the main scale is of 5 units, the least count of the instrument is:
1
(a) 2

(b) 5

11

(c) 50

11

(d) 10

11
11. Consider a block and trolley system as shown in figure. If the coefficient of kinetic friction between the
trolley and the surface is 0.04, the acceleration of the system in ms is: (Consider that the string is
−2

massless and un stretchable and the pully is also massless and frictionless):

(a) 3
(b) 1.2
(c) 4
(d) 2

12. A particle moving in a circle of radius R with uniform speed takes time T to complete one revolution. If this
particle is projected with the same speed at an angle θ to the horizontal, the maximum height attained by it
is equal to 4R. The angel of projection θ is then given by:
1
2

(a) −1
sin [
2gT
]
2

2
π R
1

(b) cos
−1
[
πR
2
]
2

2gT
1

(c) −1
sin [
π
2
R
2
]
2

2gT
1
2

(d) cos
−1
[
2gT
]
2

2
π R

13. The de Broglie wavelengths of a proton and an α particle are λ and 2λ respectively. The ratio of the
velocities of proton and α particle will be:
(a) 1 : 8
(b) 1 : 2
(c) 4 : 1
(d) 8 : 1

14. In series LCR circuit, the capacitance is changed from C to 4C. To keep the resonance frequency
unchanged, the new inductance should be:
(a) Increased by 2L
(b) Increased to 4L
(c) Reduced by L 1

(d) Reduced by L 3

15. The pressure and volume of an ideal gas are related as P V 3

2 = K (Constant). The work done when the gas


is taken from state A (P , V , T ) to state B (P , V , T ) is:
1 1 1 2 2 2

(a) 2 (P V − P V )
2 2 1 1

(b) 2 (P V − P V )
1 1 2 2
−− −−
(c) 2 (√P V − √P V )1 1 2 2
−− −−
(d) 2 (P √V − P √V )
2 2 1 1
16. A galvanometer has a resistance of 50Ω and it allows maximum current of 5mA. It can be converted into
voltmeter to measure upto 100V by connecting in series a resistor of resistance:
(a) 5975Ω
(b) 19950Ω
(c) 19500Ω
(d) 20050Ω

17. A simple pendulum of length 1m has a wooden bob of mass 1 kg. It is struck by a bullet of mass 10 kg −2

moving with a speed of 2 × 10 ms . The bullet gets embedded into the bob. The height to which the bob
2 −1

rises before swinging back is. (use g = 10m/s ) 2

(a) 0.30 m
(b) 0.20 m
(c) 0.40 m
(d) 0.35 m

18. If R is the radius of the earth and the acceleration due to gravity on the surface of earth is g = 2
π m/s ,
2

then the length of the second’s pendulum at a height h = 2R from the surface of earth will be:
(a) m 2

(b) 8

9
m

(c) 4

9
m

(d) 1

9
m

19. The radius (r), length (l) and resistance (R) of a metal wire was measured in the laboratory as
r = (0.35 ± 0.05) cm

R = (100 ± 10) ohm

l = (15 ± 0.2) cm

The percentage error in resistivity of the material of the wire is:


(a) 37.3%
(b) 25.6%
(c) 39.9%
(d) 35.6%

20. A ball of mass 0.5 kg is attached to a string of length 50 cm. The ball is rotated on a horizontal circular path
about its vertical axis. The maximum tension that the string can bear is 400N. The maximum possible value
of angular velocity of the ball in rad/s is:
(a) 20
(b) 1600
(c) 40
(d) 1000

21. Two identical charged spheres are suspended by strings of equal lengths. The strings make an angle θ with
each other. When suspended in water the angle remains the same. If density of the material of the sphere is
1.5 g/cc, the dielectric constant of water will be ____ (Take density of water = 1 g/cc)

22. The current in a conductor is expressed as I = 3t + 4t , where I is in Ampere and is in second. The
2 3

amount of electric charge that flows through a section of the conductor during t = 1s to t = 2s is ______ C.
23. A plane is in level flight at constant speed and each of its two wings has an area of 40m . If the speed of the
2

air is 180km/h over the lower wing surface and 252 km/h over the upper wing surface, the mass of the
plane is _____ kg. (Take air density to be 1kg m and g = 10ms )
−3 −2

24. A particle is moving in one dimension (along x axis) under the action of a variable force. It’s initial position
was 16m right of origin. The variation of its position (x) with time (t) is given as x = −3t + 18t + 16t,
3 2

where x is in m and t is in s. The velocity of the particle when its acceleration becomes zero is ______ m/s.

25. A regular polygon of 6 sides is formed by bending a wire of length 4π meter. If an electric current of 4π√–
3

A is flowing through the sides of the polygon, the magnetic field at the centre of the polygon would be
T . The value of x is _____.
−7
x × 10

26. The identical spheres each of mass 2M are placed at the corners of a right angled triangle with mutually
perpendicular sides equal to 4M each. Taking point of intersection of these two sides as origin, the
magnitude of position vector of the centre of mass of the system is , where the value of x is ____.
4√ 2

27. A rectangular loop of sides 12 cm and 15 cm, with its sides parallel to the x-axis and y-axis respectively,
moves with a velocity of 5 cm/s in the positive x axis direction, in a space containing a variable magnetic
field in the positive z direction. The field has a gradient of 10 T /cm along the negative x direction and it
−3

is decreasing with time at the rate of 10 T /s . If the resistance of the loop is 6mΩ, the power dissipated by
−3

the loop as heat is ____ ×10 W .


−9

28. A tuning fork resonates with a sonometer wire of length 1 m stretched with a tension of 6N. When the
tension in the wire is changed to 54 N, the same tuning fork produces 12 beats per second with it. The
frequency of the tuning fork is _____ Hz.

29. The distance between object and its 3 times magnified virtual image as produced by a convex lens is 20 cm.
The focal length of the lens used is _____ cm.

30. The radius of a nucleus of mass number 64 is 4.8 fermi. Then the mass number of another nucleus having
radius of 4 fermi is , where x is _____.
1000

31. Choose the correct option for free expansion of an ideal gas under adiabatic condition from the following:
(a) q ≠ 0, ΔT = 0, w = 0
(b) q = 0, ΔT = 0, w = 0
(c) q = 0, ΔT < 0, w ≠ 0
(d) q = 0, ΔT ≠ 0, w = 0

32. Given below are two statements:


Statement (I): Aminobenzene and aniline are same organic compounds
Statement (II): Aminobenzene and aniline are different organic compounds.
In the light of the above statements, choose the most appropriate answer from the options given below:
(a) Both Statement I and Statement II are correct
(b) Statement I is incorrect but Statement II is correct
(c) Both Statement I and Statement II are incorrect
(d) Statement I is correct but Statement II is incorrect
33. Identify A and B in the following sequence of reaction

(a)

(b)

(c)

(d)

34. Match List – I with List – II.

List – I (Reactions) List – II (Reagents)

A (I) C H 3 M gBr, H 2 O

B C 6 H 5 C OC 6 H 5 → C 6 H 5 C H 2 C 6 H 5 (II) Z n (H g) and conc. HCl


C C 6 H 5 C H O → C 6 H 5 C H (OH ) C H 3 (III) N aBH 4 , H
+

D (IV) DI BAL − H , H 2 O

Choose the correct answer from the options given below:


(a) A − I I I , B − I V , C − I I , D − I
(b) A − I I I , B − I V , C − I , D − I I
(c) A − I V , B − I I , C − I , D − I I I
(d) A − I V , B − I I , C − I I I , D − I
35. Ionic reactions with organic compounds proceed through:
(A) Homolytic bond cleavage
(B) Heterolytic bond cleavage
(C) Free radical formation
(D) Primary free radical
(E) Secondary free radical
Choose the correct answer from the options given below:
(a) B only
(b) C only
(c) A only
(d) D and E only

36. Given below are two statements: one is labelled as


Assertion (A) and the other is labelled as Reason (R).
Assertion (A): Haloalkanes react with KCN to form alkyl cyanides as a main product while with AgCN
form isocyanide as the main product.
Reason (R): KCN and AgCN both are highly ionic compounds.
In the light of the above statement, choose the most appropriate answer from the options given below:
(a) A is correct but R is not correct
(b) Both A and R are correct and R is the correct explanation of A
(c) A is not correct but R is correct
(d) Both A and R are correct but R is not the correct explanation of A.

37. Which of the following compound will most easily be attacked by an electrophile?

(a)

(b)

(c)

(d)

38. If one strand of a DNA has the sequence ATGCTTCA, sequence of the bases in complementary strand is:
(a) TACGAAGT
(b) CATTAGCT
(c) GTACTTAC
(d) ATGCCGACT
39. Given below are two statements:
Statement (I): A solution of [N i(H O) ] is green in colour
2+
2
6

Statement (II): A solution of [N i(C N ) ] is colourless.


2−

In the light of the above statements, choose the most appropriate answer the options given below:
(a) Both Statement I and Statement II are incorrect
(b) Both Statement I and Statement II are correct
(c) Statement I is correct but Statement II is incorrect
(d) Statement I is incorrect but Statement II is correct

40. In acidic medium K 2 C r2 O 7 shows oxidising action as represented in the half-reaction:


2− + −
C r2 O + XH + Ye → 2A + Z H 2 O
7

X, Y, Z and A are respectively are:


(a) 14, 6, 7 and C r 3+

(b) 8, 6, 4 and C r O 2 3

(c) 14, 7, 6 and C r 3+

(d) 8, 4, 6 and C r O 2 3

41. Given below are two statements: one is labelled as Assertion (A) and the other is labelled as Reason (R)
Assertion (A): P H has lower boiling point than N H .
3 3

Reason (R): In In liquid state N H molecules are associated through vander waal’s forces, but P H
3 3

molecules are associated through hydrogen bonding.


In the light of the above statements, choose the most appropriate answer from the options given below:
(a) A is not correct but R is correct
(b) Both A and R are correct and R is the correct explanation of A
(c) Both A and R are correct but R is not the correct explanation of A
(d) A is correct but R is not correct.

42. Which of the following reactions are disproportionation reactions?


(A) C u → C u + C u
+ 2+

(B) 3M nO + 4H → 2M N O + M nO + 2H O
2−

4
+ −

4
2 2

(C) 2K M nO → K M nO + M nO + O
4 2 4 2 2

(D) 2M N O + 3M n + 2H O → 5M nO + 4H

4
2+
2 2
+

Choose the correct answer from the options given below:


(a) A, B, C
(b) A, B
(c) B, C, D
(d) A, D

43. Arrange the bonds in order of increasing ionic character in the molecules. LiF , K 2 O, N2 S O 2 and C I F 3 :

(a) LiF < K O < C lF < S O < N


2 3 2 2

(b) N < S O < C lF < K O < LiF


2 2 3 2

(c) N < C lF < S O < K O < LiF


2 3 2 2

(d) C lF < N < S O < K O < LiF


3 2 2 2

44. We have three aqueous solutions of N aC l labelled as ‘A’, ‘B’ and ‘C’ with concentration 0.1 M, 0.01 M and
0.001 M, respectively. The value of van’ t Hoff factor (i) for these solutions will be in the order:
(a) i < i < i
A C B

(b) i < i < i


A B C

(c) i > i > i


A B C

(d) i = i = i
A B C
45. According to the wave-particle duality of matter by de-Broglie, which of the following graph plot presents
most appropriate relationship between wavelength of electron (λ) and momentum of electron (p)?

(a)

(b)

(c)

(d)

46. Given below are two statements:


Statement (I): The N H group in Aniline is ortho and para directing and a powerful activating group.
2

Statement (II): Aniline does not undergo Friedel-Craft’s reaction (Alkylation and acylation).
In the light of the above statements, choose the most appropriate answer from the options given below:
(a) Both Statement I and Statement II are incorrect
(b) Both Statement I and Statement II are correct
(c) Statement I is correct but Statement II is incorrect
(d) Statement I is incorrect but Statement II is correct

47. In case of isoelectronic species the size of F , N e and N a is affected by:


− +

(a) None of the factors because their size is the same


(b) Electron-electron interaction in the outer orbitals
(c) Nuclear charge (z)
(d) Principal quantum number (n)
48. In Kjeldahl’s method for estimation of nitrogen, C uS O acts as: 4

(a) Reducing agent


(b) Catalytic agent
(c) Oxidising agent
(d) Hydrolysis agent

49. Which of the following complex is homoleptic?


(a) [N i(C N ) ]
2−

(b) [N i(N H ) C l ]
3
2
2
+
(c) [C o(N H ) C l ]
3
4
2
+
(d) [F e(N H ) C l ]
3 4
2

50. Given below are two statements:


Statement (I): Potassium hydrogen phthalate is a primary standard for standardisation of sodium hydroxide
solution.
Statement (II): In this titration phenolphthalein can be used as indicator.
In the light of the above statements, choose the most appropriate answer from the options given below:
(a) Statement I is correct but Statement II is incorrect
(b) Both Statement I and Statement II are correct
(c) Statement I is incorrect but Statement II is correct
(d) Both Statement I and Statement II are incorrect

51. The lowest oxidation number of an atom in a compound A 2B is -2. The number of electrons in its valence
shell is _____

52. The potential for the given half cell at 298 K is (−) _____10
−2
V .
+ −
2H + 2e → H 2 (g)
(aq)

+
[H ] = 1M , P H = 2atm
2

(Given : 2.303Rt/F = 0.06V , log 2 = 0.3)

53. Total number of deactivating groups in aromatic electrophilic substitution reaction among the following is
____.

54. Consider the following reaction:


3P bC l2 + 2(N H 4 ) P O 4 → P b3 (P O 4 ) + 6N H 4 C l
3 2

If 72 mmol of P bC l is mixed with 50 mmol of (N H


2 4) P
3
O4 , then amount of P b 3 (P O4 )
2
formed is ……
mmol. (Nearest integer)

55. The ratio of 14

in a piece of wood is
C 1
part that of atmosphere. If half life of 14
C is 5730 years, the age of
12 8
C

wood sample is ____ years.


56. The number of white coloured salts, among the following is _____.
(a) S rS O , (b) M g (N H ) P O , (c) BaC rO , (d) M n(OH )
4 4 4 4
2

(e) P bS O , (f) P bC rO , (g) AgBr, (h) P bl


4 4 2

(i) C aC O , (j) [F e(OH ) (C H C OO)]


2 4
2
3

57. Number of optical isomers possible for 2-chlorobutane ____

58. The number of molecules/ion/s having trigonal bipyramidal shape is _____.


2−
P F 5 , BrF 5 , P C l5 , [P t C l4 ] , BF 3 , F e(C O)
5

59. Among the following oxides of p-block elements, number of oxides having atmospheric nature is _____.
C l2 O 7 , C o, P bO 2 , N2 O, N O, Al2 O 3 , S iO 2 , N2 O 5 , S nO 2

60. Ka for C H C OOH is 1.8 × 10


3
−5
and K for N H
b 4 OH is 1.8 × 10 −5
. The pH of ammonium acetate
solution will be _____.

61. If the system of equations


2x + 3y − z = 5

x + αy + 3z = −4

3x − y + βz = 7

has infinitely many solutions, then 13αβ is equal to _____.


(a) 1110
(b) 1220
(c) 1120
(d) 1210

62. π

The value of the integral ∫ 4


x dx

4
equals:
sin (2x)+cos (2x)
0
2

(a) √ 2π

16
2

(b) √ 2π

8
2

(c) √ 2π

64
2

(d) √ 2π

32

63. loge x , x > 0 x , x ⩾ 0


Let f : R → R and g : R → R be defined as f (x) = {
−x
and g (x) = {
x
.
e , x ⩽ 0 e , x < 0

Then, gof : R → R is:


(a) one-one but not onto
(b) onto but not one-one
(c) both one-one and onto
(d) neither one-one nor onto
64. If the shortest distance between the lines x−λ
=
y−2
=
z−1
and
x−√ 3
=
y−1
=
z−2
is 1, then the sum of
−2 1 1 1 −2 1

all possible values of λ is:


(a) 0

(b) −2√3

(c) 3√3
(d) 2√– 3

65. Let a⃗ ^ ^ ^
= −5 i + j − 3k ,
⃗ ^ ^ ^
b = i + 2 j − 4k and c ⃗ = ⃗ ^ ^ ^
(((a ⃗ × b) × i ) × i ) × i . Then,
^ ^ ^
c ⃗ ⋅ (− i + j + k ) is equal to:
(a) −13

(b) −15

(c) −12

(d) −10

66. If n is the number of ways five different employees can sit into four indistinguishable offices where any
office may have any number of persons including zero, then n is equal to:
(a) 47
(b) 51
(c) 53
(d) 43


67. √2 1 1 0
If A = [ –],B = [ ] , C = ABA
T
and X = A
T
C
2
A , then det X is equal to:
−1 √2 1 1

(a) 243
(b) 27
(c) 729
(d) 891

68. Let x
2

+
y
2

= 1, a > b be an ellipse, whose eccentricity is 1


and the length of the latus rectum is √14 .
−−
2 2
a b √2

2 2
y
Then the square of the eccentricity of x

2
a
− 2
= 1 is:
b

(a) 3
(b) 7

(c) 5

(d) 3

69. The area enclosed by the curves xy + 4y = 16 and x + y = 6 is equal to:


(a) 32 − 30log 2 e

(b) 28 − 30log 2 e

(c) 30 − 32log 2 e

(d) 30 − 28log 2 e
70. Let y = y (x) be the solution of the differential equation
dy
= 2(x + y)
3
− x (x + y) − 1, y (0) = 1 .
dx
2

Then, ( 1
+ y(
1
)) equals:
√2 √2

4
(a) 4+√ e

1
(b) 2−√ e

2
(c) 1+√ e

3
(d) 3−√ e

71. Let the median and the mean deviation about the median of 7 observation 170, 125, 230, 190, 210, a, b be
170 and respectively. Then the mean deviation about the mean of these 7 observations is:
250

(a) 28
(b) 30
(c) 32
(d) 31

72. Let 3, a, b, c be in A.P. and 3, a − 1, b + 1, c + 9 be in G.P. Then, the arithmetic mean of a, b and c is:
(a) 11
(b) −4
(c) 13
(d) −1

73. If tan A =
1
, tan B =
√x
and tan C = (x
−3 −2
+ x + x
−1
)
1

2
, 0 < A, B, C <
π
, then
2 √ x 2 +x+1 2
√ x(x +x+1)

A + B is equal to:
(a) 2π − C

(b) C

(c) π

2
− C

(d) π − C

74. A bag contains 8 balls, whose colours are either white or black. 4 balls are drawn at random without
replacement and it was found that 2 balls are white and other 2 balls are black. The probability that the bag
contains equal number of white and black balls is:
(a) 2

(b) 1

(c) 1

(d) 2

75. Let C : x + y = 4 and C : x + y − 4λx + 9 = 0 be two circles. If the set of all values of λ so that
2 2 2 2

the circles C and C intersect at two distinct points, is R − [a, b] , then the point (8a + 12, 16b − 20) lies

on the curve:
(a) x + 2y − 5x + 6y = 3
2 2

(b) 5x − y = −11
2

(c) 6x + y = 42
2 2

(d) x − 4y = 7
2 2

76. For 0 < θ < , if the eccentricity of the hyperbola x
π 2
− y
2 2
cosec θ = 5 is √7 times eccentricity of the
2

ellipse x cosec θ + y = 5, then the value of θ is:


2 2 2

π
(a) 3
π
(b) 4
π
(c) 6

(d) 5π

12

77. If 5f (x) + 4f ( 1
) = x
2
− 2, ∀x ≠ 0 and y 2
= 9x f (x) , then y is strictly increasing in:
x

(a) (0,
1

√5
) ∪ (
√5
1
, ∞)

(b) (−
1

√5
, 0) ∪ (
1

√5
, ∞)

(c) (−
1

√5
, 0) ∪ (0,
1

√5
)

(d) (−∞,
1

√5
) ∪ (0,
1

√5
)

– –
78. Let S = {z ∈ C : |z − 1| and (√ 2 − 1) (z + z̄ ) − i (z − z̄ ) = 2√ 2} . Let z 1
, z2 ∈ S be such that

. Then equals:
2
|z 1 | = max |z| ∣
∣ √ 2z 1 − z 2 ∣

z∈s

(a) 2
(b) 1
(c) 3
(d) 4

– – x – – x
79. Let S = {x ∈ R : (√ 3 + √ 2) + (√ 3 − √ 2) = 10} . Then the number of elements in S is:
(a) 2
(b) 1
(c) 0
(d) 4

80. ⎧

a−b cos 2x
; x < 0
⎪ x
2

Let f : R → R be defined as: f (x) = ⎨


x
2
+ cx + 2 ; 0 ⩽ x ⩽ 1
.



2x + 1 ; x > 1

If f is continuous everywhere in R and m is the number of points where f is NOT differential then
m + a + b + c equals:

(a) 2
(b) 1
(c) 3
(d) 4

81. Let A = {1, 2, 3, . . . , 20} . Let R and R two relation on A such that
1 2

R 1 = {(a, b) : b is divisible by a}

R 2 = {(a, b) : a}is an integral multiple of b}.


Then, number of elements in R − R is equal to _____. 1 2
82. Let P and Q = {z ∈ C : z (1 + i) + z̄ (1 − i)
= {z ∈ C : |z + 2 − 3i| ⩽ 1} ⩽ −8} . Let in

P ∩ Q, |z − 3 + 2i| be maximum and minimum at z and z respectively. If |z
2 2
1 2 1| + 2|z 2 | = α + β√ 2,

where α, β are integers, then α + β equals ______.

83. π

2
8√ 2 cos x dx –
If ∫
sin x 4
= απ + βloge (3 + 2√ 2) , where α, β are integers, then α 2
+ β
2
equals _____.
π (1+e )(1+sin x)

2

84. If x = x (t) is the solution of the differential equation (t + 1) dx


4
= (2x + (t + 1) dt) , x (0) = 2, then,
x (1) equals ____.

85. Let 3, 7, 11, 15, . . . . , 403 and 2, 5, 8, 11, . . . . , 404 be two arithmetic progressions. Then the sum, of the
common terms in them, is equal to ____.

86. cos
−1 2
(1−{x} )sin
−1
(1−{x})

Let {x} denote the fractional part of x and f (x) = 3


,x ≠ 0 . If L and R
x−{x}

respectively denotes the left hand limit and the right hand limit of f (x) at x = 0, then 32

2
(L
2 2
+ R ) is
π

equal to _____.

87. Let the line L : √–2x + y = α pass through the point of the intersection P (in the first quadrant) of the

circle x + y = 3 and the parabola x = 2y. Let the line L touch two circles C and C of equal radius is
2 2 2
1 2

2√ 3. If the centres Q and Q of the circles C and C lie on the y-axis, then the square of the area of the
1 2 1 2

triangle P Q Q is equal to _____.


1 2

88. If the coefficient of x 30


in the expansion of (1 + 1
6
2
7
) (1 + x ) (1 − x ) ; x ≠ 0
3
8
is α , then |α| equals
x

_____.

89. Let the line of the shortest distance between the lines
^ ^ ^ ^ ^ ^
L1 : r ⃗ = ( i + 2 j + 3k ) + λ ( i − j + k ) and L 2
^ ^ ^ ^ ^ ^
: r ⃗ = (4 i + 5 j + 6k ) + μ ( i + j − k ) intersect
L1 and L at P and Q respectively. If (α, β, γ) is the mid point of the line segment P Q, then
2

2 (α + β + γ) is equal to _____.

90. The number of elements in the set S = {(x, y, z) : x, y, z ∈ Z , x + 2y + 3z = 42, x, y, z ⩾ 0} equals


______.
Answer Key

1. B 2. D 3. A 4. B 5. D 6. B
7. C 8. A 9. A 10. B 11. D 12. A
13. D 14. D 15. A 16. B 17. B 18. D
19. C 20. C 21. 3 22. 22 23. 9600 24. 52
25. 72 26. 3 27. 216 28. 6 29. 15 30. 27
31. B 32. D 33. B 34. C 35. A 36. A
37. A 38. A 39. B 40. A 41. D 42. B
43. B 44. B 45. D 46. B 47. C 48. B
49. A 50. B 51. 6 52. 1 53. 2 54. 24
55. 17190 56. 5 57. 2 58. 3 59. 3 60. 7
61. C 62. D 63. D 64. D 65. C 66. B
67. C 68. D 69. C 70. B 71. B 72. A
73. B 74. A 75. C 76. A 77. B 78. A
79. A 80. A 81. 46.00 82. 36.00 83. 8.00 84. 14.00
85. 6699.00 86. 18.00 87. 72.00 88. 678.00 89. 21.00 90. 169.00
Solutions

1. (B)
Angular impulse = Change in angular momentum.
[Angular impulse] = [Angular momentum] = [mvr] = [M L T
2 −1
]

2. (D)
n1 C +n 2 C
V V
1 2
CV =
n 1 +n 2
3 5
2× R+6× R
2 2
=
2+6
9
= R
4

3. (A)

Pd across R s

V 1 = 8 − 5 = 3V

Current through the load resistor


5
I = = 5mA
3
1×10

Maximum current through Zener diode


10
Iz max.
= = 2mA
5

And minimum current through Zener diode


Iz min = 0

∴ Is max. = 5 + 2 = 7mA

And R
V1 3
s min
= = kΩ
Is max 7

Similarly
Is min
= 5mA

And R
V1 3
s max = = kΩ
Is min 5

3 3
∴ kΩ < R s < kΩ
7 5

4. (B)
Transition from n = 1 to n = 3
ΔE = 12.1eV

5. (D)
Linear width
−7
2λd 2×6×10 ×0.2
W = = −5
a
1×10
−2
= 2.4 × 10 = 24mm
6. (B)
q C V +2C V
net
VC = =
Cnet 2C

3V
VC =
2

Loss of energy
2
1 2 1 2 1 3V
= CV + C (2V ) − 2C ( )
2 2 2 2

2
CV
= ( )
4

7. (C)
Conceptual questions

8. (A)
E eq 8×5
i = =
req 8×0.2

I = 25A

V = E − ir

= 5 − 0.2 × 25 = 0

9. (A)
V −12
I = = 230 × 300 × 200 × 10 = 13.8μA
XC

10. (B)
10M S D = 11V S D
10
1 V SD = MSD
11

LC = 1M S D − 1V S D
10
= 1M S D − MSD
11
1M SD 5
= = units
11 11

11. (D)
fk = μN = 0.04 × 20g = 8Newton
60−8 2
a = = 2m/s
26

12. (A)
2πR
= V
T
2

Maximum height H
2
v sin θ
=
2g
2 2
4π R 2
4R = 2
sin θ
T 2g
−−−−
2
2gT
sin θ = √
2
π R

1
2
−1 2gT 2

θ = sin ( )
2
π R

13. (D)
h h h
λ = = ⇒ v =
p mv mλ
vp mα λα
= ×
vα mp λp

= 4 × 2 = 8
14. (D)

ω = ω
1 1
=
′ ′
√L C √ LC

′ ′
∴ L C = LC

L (4C ) = LC
′ L
L =
4

∵ Inductance must be decreased by 3L

15. (A)
For P V = Constant x

If work done by gas is asked then


nRΔT
W =
1−x

Here x =
3

2
P2 V 2 −P1 V 1
∴ W = 1

2

. …. Option (1) is correct


= 2 (P 1 V 1 − P 2 V 2 )

If work done by external is asked then W = −2 (P 1 V1 − P2 V2 ) ...... Option (2) is correct.

16. (B)

V 100
R = − Rg = −3
− 50
Ig 5×10

= 20000 − 50 = 19950Ω

17. (B)

mu = (M + m) V
−2 2
10 × 2 × 10 ≅1 × V

V ≅ 2m/s
2
V
h = = 0.2m
2g

18. (D)
′ GM e 1
g = 2
= g
9
(3R)
−−

T = 2π√ ′
g

Since the time period of second pendulum is 2 sec.


T = 2 sec



2 = 2π√ 9
g

1
ℓ = m
9
19. (C)
ρ
ρ = R

Δρ
ΔR Δr Δℓ
= + 2 +
ρ R r ℓ
10 0.05 0.2
= + 2 × +
100 0.35 15
1 2 1
= + +
10 7 75
Δρ
= 39.9%
ρ

20. (C)
2
T = mω ℓ
2
400 = 0.5ω × 0.5

ω = 40rad/s

21. (3)

In air tan θ F q
= =
2 mg 4πε0 r mg
2

′ 2

In water tan θ F q
= ′
=
2 mg 2
4πε0 εr r mg
eff

Equate both equations


1
ε 0 g = ε 0 ε r g [1 − ]
1.5

εr = 3

22. (22)
2 2
2 3
q = ∫ idt = ∫ (3t + 4t ) dt
1 1
2
3 4
q = (t + t )∣
∣ 1

q = 22C

23. (9600)
2
A = 80m

Using Bernoulli equation .


1 2 2
A (P 2 − P 1 ) = ρ (V − V )A
2 1 2
1 2 2
mg = × 1 (70 − 50 ) × 80
2

mg = 40 × 200 = 9600kg
24. (52)
3 2
x = 3t + 18t + 16t
2
v = −9t + 36 + 16

a = −18t + 36

a = 0 at t = 2s
2
v = −9(2) + 36 × 2 + 16

v = 52m/s

25. (72)

μ I
0 ∘ ∘
B = 6( ) (sin 30 + sin 30 )
4πr

−7
10 ×4π√ 3
= 6
√ 3×4π
( )
2×6

−7
= 72 × 10 T

26. (3)

⃗ +m r ⃗ +m r ⃗
Position vector r ⃗
m1 r 1 2 2 3 3

C OM
=
m1 +m2 +m3

^ ^
2M ×0+2M +4 i +2M ×4 j
⃗ OM =
rC
6M
4 ^ 4 ^
r⃗ = i + j
3 3

4√ 2
|r |⃗ =
3

x = 3
27. (216)

B0 is the magnetic field at origin


−3
dB 10
= −2
dx 10
B x
−1
∫ dB = − ∫ 10 dx
B0 0

−1
B − B 0 = −10 x
x
B = (B 0 − )
10

Motional emf in AB = 0
Motional emf in CD = 0
Motional emf in AD = ε = 1 B 0 ℓv

Magnetic field on rod BC B


−2
(−12×10 )
= (B 0 − )
10

−2

Motional emf in BC = ε 0 = (B 0 +
12×10

10
)ℓ × v

−7
ε eq = ε 2 − ε 1 = 300 × 10 V

For time variation


′ dB −7
(ε eq ) = A = 60 × 10 V
dt
′ −7
(ε eq ) = ε eq + (ε eq ) = 360 × 10 V
net
2
(εeq )
Power = R
net
= 216 × 10
−9
W

28. (6)


1 T
f = √
2L μ


− −−
1 6 1 54
f1 = √ , f2 = √
2 μ 2 μ

f
1 1
=
f 3
2

f2 − f1 = 12

f1 = 6H Z
29. (15)

v = 3u

v − u = 20cm

2u = 20cm

u = 10cm
1 1 1
− =
(−30) (−10) f

f = 15cm

30. (27)
1/3
R = R0 A
3
R ∝ A
3
4.8 64
( ) =
4 A
64 3
= = (1.2)
A
64
= 1.44 × 1.2
A
64 1000
A = =
1.44×1.2 x
144×12
x = = 27
64

31. (B)
During free expansion of an ideal gas under adiabatic conditions q = 0, ΔT = 0, w = 0

32. (D)
Aniline is also known as amino benzene.

33. (B)

34. (C)
DIBAL - H,H 2 O

C H 3 (C H 2 ) C OOC 2 H 5 −−−−−−−−−→ C H 3 (C H 2 ) C H O
5 5
Zn(H g)&C onc.H C l

C 6 H 5 C OC 6 H 5 −−−−−−−−−−−→ C 6 H 5 C H 2 C 6 H 5
C H 3 M gBr

C6 H 5 C H O −−−−−−→ C 6 H 5 C H (OH ) C H 3
H2 O
+
N aBH 4 ,H

C H 3 C OC H 2 C OOC 2 H 5 −−−−−−−→ C H 3 C H (OH ) C H 2 C OOC 2 H 5

35. (A)
Heterolytic cleavage of Bond lead to formation of ions.
36. (A)

AgCN is mainly covalent in nature and nitrogen is available for attack, so alkyl isocyanide is formed as main
product.

37. (A)
Higher the electron density in the benzene ring more easily it will be attacked by an electrophile. Phenol has the
highest electron density amongst all the given compound.

38. (A)
Adenine base pairs with thymine with 2 hydrogen bonds and cytosine base pairs with guanine with 3 hydrogen
bonds.

39. (B)
Green colour solution due to d-d transition.
2+
[N i(H 2 O) ] →
6

is diamagnetic and it is colourless.


2−
[N i(C N ) ] →
4

40. (A)
The balanced reaction is,
2− + − 3+
C r2 O + 14H + 6e → 2C r + 7H 2 O
7

X = 14

Y = 6

A = 7

41. (D)
Unlike N H , P H molecules are not associated through hydrogen bonding in liquid state. That is why the
3 3

boiling point of P H is lower than N H .


3 3

42. (B)
When a particular oxidation state becomes less stable relative to other oxidation state, one lower, one higher, it is
said to undergo disproportionation.
+ 2+
Cu → Cu + Cu
2− + −
3M nO + 4H → 2M nO + M nO 2 + 2 H 2 O
4 4

43. (B)
Increasing order of ionic character
N2 < S O 2 < C lF 3 < K2 O < LiF

Ionic character depends upon difference of electronegativity (bond polarity).


44. (B)

Slat Values of i (For different conc. Of a salt)


0.1 M 0.01 M 0.001 M
NACl 1.87 1.94 1.94

i approach 2 as the solution become very dilute.

45. (D)
h 1
λ = [λ ∝ ]
p p

⇒ λp = h (Constant)
So, the plot is a rectangular hyperbola.

46. (B)
The N H group in Aniline is ortho and para directing and a powerful activating group as N H has strong +M
2 2

effect.
Aniline does not undergo Friedel-Craft’s reaction (alkylation and acylation) as Aniline will form complex with
AlC l which will deactivate the benzene ring.
3

47. (C)
In F , N e, N a all have 1s
− + 2 2 6
, 2s , 2p configuration. They have different size due to the difference in nuclear
charge.

48. (B)
Kjeldahl’s method is used for estimation of Nitrogen where CuSO4 acts as a catalyst.

49. (A)
In Homoleptic complex all the ligand attached with the central atom should be the same. Hence [N i(C N ) 4
]
2−
is
a homoleptic complex.

50. (B)
Statement (I): Potassium hydrogen phthalate is a primary standard for standardization of sodium hydroxide
solution as it is economical and its concentration does not changes with time.
Phenophthalein can acts as indicator in acid base titration as it shows colour in pH range 8.3 to 10.1.

51. (6)
A2 B → 2A
+
+ B
−2
has complete octet in its dianionic form, thus in its atomic state it has 6 electrons in
,B
−2

its valence shell. As it has negative charge, it has acquired two electrons to complete its octet.
52. (1)
P
o 0.06 H2
E = E + − log 2
H /H 2 2 +
[H ]

0.06 2
E = 0.00 − log 2
2
[1]

−2
E = −0.03 × 0.3 = −0.9 × 10 V

53. (2)

54. (24)
Limiting Reagent is P bC l 2

mmol of P b (P O ) formed =
mmol of P bC l2 reacted
3 4
= 24 mmol
2 3

55. (17190)
14 12
( C/ C)
atmosphere
λt = ln
12
14
( C/ C)
wood sample

As per the question,


14 12
( C/ C)
atmosphere 1
=
14
12 8
( C/ C)
wood sample

So, λt = ln 8
ln 2
t = ln 8
t 1

t = 3 × t1/2 = 17190 years

56. (5)
(a) S rS O - White 4

(b) M g (N H ) P O - White 4 4

(c) BaC rO - Yellow 4

(d) M n(OH ) - White 2

(e) P bS O - White 4

(f) P bC rO - Yellow 4

(g) AgBr - Pale yellow


(h) P bl - Yellow 2

(i) C aC O - White 2 4

(j) [F e(OH ) (C H C OO)]- Brown Red 2


3
57. (2)

There is one chiral centre present in given compound.


So, total optical isomers = 2.

58. (3)
Trigonal bipyramidal
P F 5 , P C l5 , F e(C O) ;
5

BrF 5 ; Square pyramidal


; Square planar
−2
[P tC l ]
4

BF ; Trigonal planar
3

59. (3)
Acidic oxide: C l O , S iO , N O
2 7 2 2 5

Neutral oxide: C O, N O, N O 2

Amphoteric oxide: Al O , S nO , P bO2 3 2 2

60. (7)
pKw +pKa −pKb
pH =
2

pK a = pK b
pKw
⇒ pH = = 7
2

61. (C)
Using family of planes
2x + 3y − z − 5 = k 1 (x + αy + 3z + 4) + k 2 (3x − y + βz − 7)

2 = k 1 + 3k 2 , 3 = k 1 α − k 2 − 1 = 3k 1 + βk 2 , −5 = 4k 1 − 7k 2

On solving we get
13 −1 −16
k2 = , k1 = , α = −70, β =
19 19 13

−16
13αβ = 13 (−70) ( ) = 1120
13
62. (D)
π

4
x dx
∫ 4 4
sin (2x)+cos (2x)
0

Let 2x = t then dx =
1

2
dt
π

1 t dt
I = ∫ 4
4 4
sin t+cos t
0
π
π
2 ( −t)dt
1 2

I = ∫
4 4 π π
sin ( −t)+cos 4 ( −t)
0 2 2

2 π
dt
1 2
I = ∫ − I
4 4 4
sin t+cos t
0
π

π dt
2I = ∫ 4
8 sin t+cos t
4

0
π

2
4
π sec t dt
2I = ∫ 4
8 tan t+1
0

Let tan t = y then sec 2


tdt = dy
∞ 2
(1+y )dy
π
2I = ∫
8 4
1+y
0
1
∞ 1+
π y2
= ∫ 1
dy
16 2
y +
0 y2

Put y − 1

y
= p

π dp
I = ∫
6 2
2
p +(√ 2 )
−∞

π p
−1
= [tan ( )]
16√ 2 √2
−∞
2
π
I =
16√ 2
63. (D)
f (x) , f (x) ⩾ 0
g (f (x)) = {
f (x)
e , f (x) < 0
−x
⎧ e , (−∞, 0]

ln x
g (f (x)) = ⎨ e , (0, 1)


ln x, [1, ∞)

Graph of g (f (x))
g (f (x)) ⇒ Many one into

64. (D)

Passing points of lines L 1 & L2 are (λ, 2, 1) & (√ 3, 1, 2)

∣ √ 3 −λ −1 1 ∣
∣ ∣
∣ −2 1 1 ∣
∣ ∣
∣ 1 ∣
S.D. =
1 −2

∣ ^ ^ ^ ∣
i j k
∣ ∣
∣ ∣
−2 1 1
∣ ∣
∣ 1 −2 1 ∣

∣ √ 3 −λ ∣
1 =
∣ √3 ∣

λ = 0, λ = 2√ 3

65. (C)
^ ^ ^
a⃗ = −5 i + j − 3k

⃗ ^ ^ ^
b = i + 2 j − 4k

⃗ ^ ^ ⃗ ⃗ ^
(a ⃗ × b) × i = (a ⃗ ⋅ i ) b − (b ⋅ i ) a ⃗


= −5b − a ⃗

⃗ ⃗ ^ ^
= (((−5b − a ) × i) × i)

^ ^ ^ ^
= ((−11 j + 23k ) × i ) × i

^ ^ ^
⇒ (11k + 23 j) × i

^ ^
⇒ (11 j − 23k )

^ ^ ^
c .⃗ (− i + j + k ) = 11 − 23 = −12
66. (B)
Total ways to partition 5 into 4 parts are :
5, 0, 0, 0 ⇒ 1 way

= 5 ways
5!
4, 1, 0, 0 ⇒
4!

3, 2, 0, 0 ⇒
5!

3!2!
= 10 ways
2, 2, 0, 1 ⇒
5!

2!2!2!
= 15 ways
2, 1, 1, 1 ⇒
5!

3
= 10 ways
2!(1!) 3!

3, 1, 1, 0 ⇒
5!

3!2!
= 10 ways
Total ⇒ 1 + 5 + 10 + 15 + 10 + 10 = 51 ways

67. (C)

√2 1
A = [ ] ⇒ det (A) = 3

−1 √2

1 0
B = [ ] ⇒ det (B) = 1
1 1

Now C = ABA
T
⇒ det (C ) = (det (A))
2
× det (B)

|C | = 9

Now |X| = ∣
T 2
∣ A C A∣

T 2
= ∣
∣A ∣∣ |C | |A|
2 2
= |A| |C |

= 9 × 81

= 729

68. (D)
−−−−−
2 2
1 b 1 b
e = = √1 − ⇒ = 1 −
2 2 2
√2 a a
2
2b
= 14
a
− −−−−
2 −−−−− −

b 1 3
e H = √1 + = √1 + = √
2 2 2
a

2 3
(e H ) =
2
69. (C)
xy + 4y = 16

y (x + 4) = 16 . . . . (1)

x + y = 6

x + y = 6 . . . . (2)

On solving, (1) & (2)


We get x = 4, x = −2

Area = ∫ (6 − x) − (
16

x+4
) dx
−2

= 30 − 32 ln 2

70. (B)
dy 3
= 2x(x + y) − x (x + y) − 1
dx

x + y = t
dt 3
− 1 = 2xt − xt − 1
dx
dt
3
= x dx
2t −t

t dt
4 2
= x dx
2t − t

Let t 2
= z
dz
∫ = ∫ x dz
2
2(2z −z)

dz
∫ = ∫ x dx
1
4z(z− )
2

1
∣ z− ∣
2 2
ln∣ ∣ = x + k
z
∣ ∣
1
z =
2−√ e

71. (B)
Median = 170 ⇒ 125, a, b, 170, 190, 210, 230
Mean deviation about
Median = 0+45+60+20+40+170−a+170−b
=
7
205

⇒ a + b = 300

Mean = 170+125+230+190+210+a+b

7
= 175

Mean deviation
About mean =
50+175−a+175−b+5+15+35+55
= 30
7
72. (A)
3, a, b, c → A.P. ⇒ 3, 3 + d, 3 + 2d, 3 + 3d

3, a − 1, b + 1, c + 9 → G.P ⇒ 3, 2 + d, 4 + 2d, 12 + 3d

A = 3 + d

b = 3 + 2d

c = 3 + 3d
2
(2 + d) = 3 (4 + 2d)

s = 4, −2

If d = 4, G. P ⇒ 3, 6, 12, 24

a = 7

b = 11

c = 15
a+b+c
= 11
3

73. (B)
Finding tan(A + B) we get
tan A+tan B
⇒ tan(A + B) =
1−tan A tan B
1 √x
+

√x(x 2+x+1) √x 2+x+1

= 1
1−
x 2+x+1

(1+x)(√ x 2 +x+1)
⇒ tan(A + B) =
(x 2 +x)(√ x)

(1+x)(√ x 2 +x+1)

(x 2 +x)(√ x)

√ x 2 +x+1
tan(A + B) = = tan C
x√ x

A + B = C

74. (A)
2W 2B
P (4W 4B)×P ( )
4W 4B 4W 4B 2W 2B
P ( ) = +. . . . +P (6W 2B) × P ( )
2W 2B 2W 2B 2W 2B 6W 2B
P (2W 6B)×P ( )+P (3W 5B)×P ( )
2W 6B 3W 5B

4 4
1 C × C
2 2
×
5 8C
4
=
2C × 6C 3C × 5C 6C × 2C
1 2 2 1 2 2 1 2 2
× + × +...+ ×
5 8C 5 8C 5 8C
4 4 4

2
=
7
75. (C)
2 2
x + y = 4

C (0, 0) , r1 = 2
−−−−−−
′ 2
C (2λ, 0) , r2 = √4λ − 9

|r1 − r2 | < C C < |r1 + r2 |
− −−−−− −−−−−−
∣ 2 − √4 2 − 9 ∣ < |2λ| < 2 + √4 2 − 9
∣ λ ∣ λ

2
−−−−−− 2
4 + 4λ − 9 − 4√4λ 2 − 9 < 4λ

True λ ∈ R . . . . (1)
2
−−−−−− 2
4λ < 4 + 4λ − 9 + 4√4λ 2 − 9
−−−−−−
5 < 4√4λ 2 − 9 λ
2

9
and 4
25 2 3 3
< 4λ − 9 λ ∈ (−∞, − ] ∪ [ , ∞)
16 2 2
169 2
< λ
64
13 13
λ ∈ (−∞, − ) ∪ ( , ∞) . . . . (2)
18 8

From (1) and (2)


13 13 13 13
λ ∈ (−∞, − ) ∪ ( , ∞) ⇒ R − [− , ]
8 8 8 8

As per question a = − and b = 13

8
13

∴ Required point is (−1, 6) with satisfies option (c).

76. (A)
− −−−−−−
2
e h = √1 + sin θ
− −−−−−−
2
e c = √1 − sin θ

e h = √ 7e c
2 2
1 + sin θ = 7 (1 − sin θ)
2 6 3
sin θ = =
8 4
√3
sin θ =
2
π
θ =
3

77. (B)
1 2
5f (x) + 4f ( ) = x − 2, ∀ x ≠ 0 . . . . (1)
x

Substitute x →
1

x
1 1
5f ( ) + 4f (x) = − 2 . . . . (2)
x x
2

On solving (1) and (2)


4 2
5x −2x −4
f (x) =
2
9x
2
y = 9x f (x)
dy
3
= 20x − 4x
dx

for strictly increasing


dy
> 0
dx
2
4x (5x − 1) > 0

1 1
x ∈ (− , 0) ∪ ( , ∞)
√5 √5
78. (A)
Let Z = x + iy
Then (x − 1) + y = 1 . . . . (1)
2 2

– –
& (√2 − 1) (2x) − i (2iy) = 2√2
– –
⇒ (√ 2 − 1) x + y = √ 2 . . . . (2)

Solving (1) & (2) we get


Either x = 1 or x = 1

2−√ 2
. . . . (3)

On solving (3) with (2) we get


For x = 1 ⇒ y = 1 ⇒ z = 1 + i 2


& for x =
1
⇒ y = √2 −
1
⇒ z 1 = (1 +
1
) +
i

2−√ 2 √2 √2 √2

Now
2
– 2 ∣ 1 – ∣

∣ √ 2z 1 − z 2 ∣
∣ = ( + 1) √ 2 + i − (1 + i)
∣ √2 ∣
– 2
= (√ 2)

= 2

79. (A)
– – x – – x
(√ 3 + √ 2) + (√ 3 − √ 2) = 10
– – x
Let (√3 + √2) = t
1
t + = 10
t
2
t − 10t + 1 = 0
10±√ 100−4 –
t = = 5 ± 2√ 6
2
– – x – – 2
(√ 3 + √ 2) = (√ 3 + √ 2)

x = 2or x = −2
Number of solutions = 2
80. (A)
At x = 1, f (x) is continuous therefore,
− +
f (1 ) = f (1) = f (1 )

f (1) = 3 + c . . . . (1)
+
f (1 ) = lim 2 (1 + h) + 1
h→0
+
f (1 ) = lim 3 + 2h = 3 . . . . (2)
h→0

from (1) & (2)


c = 0

At x = 0, f (x) is continuous therefore,


− +
f (0 ) = f (0) = f (0 ) . . . . (3)
+
f (0) = f (0 ) = 2 . . . . (4)

f (0

) has to be equal to 2
a−b cos(2h)
lim 2
h→0 h

2 4
4h 16h
a−b{1− + +...}
2! 4!
lim
2
h→0 h

2 2 4
a−b+b{2h − h ...}
3

lim 2
h→0 h

for limit to exist a − b = 0 and limit is 2b . . . . (5)

from (3), (4) & (5)


a = b = 1

checking differentiability at x = 0
1−cos 2h
−2

LHD:
2
h
lim
−h
h→0
2 4
4h 16h 2
1−(1− + ....)−2h
2! 4!

lim = 0
3
h→0 −h
2
(0+h) +2−2
RHD: lim
h
= 0
h→0

Function is differentiable at every point in its domain


∴ m = 0

m + a + b + c = 0 + 1 + 1 + 0 = 2

81. (46.00)
n (R 1 ) = 20 + 10 + 6 + 5 + 4 + 3 + 2 + 2 + 2 + 2 + 1+. . . . +1

10 times

n (R 1 ) = 66

R 1 ∩ R 2 = {(1, 1) (2, 2) , . . (20, 20)}

n (R 1 ∩ R 2 ) = 20

n (R 1 − R 2 ) = n (R 1 ) − n (R 1 ∩ R 2 )

= n (R 1 ) − 20

= 66 − 20

R 1 − R 2 = 46 pairs
82. (36.00)

Clearly for the shaded region z1 is the intersection of the circle and the line passing through P (L1 ) and z is
2

intersection of line L & L . 1 2

Circle: (x + 2) + (y − 3) = 1
2 2

L1 : x + y − 1 = 0

L2 : x − y + 4 = 0

On solving circle & L we get 1

1 1
z 1 : (−2, ,3 + )
√2 √2

On solving L and z is intersection of line L & L


1 2 1 2

We get
−3 5
z2 : ( , )
5 2

2 2 –
|z 1 |
+ 2|z 2 | = 14 + 5√ 2 + 17

= 31 + 5√ 2

So α = 31, β = 5

α + β = 36
83. (8.00)
π

2
8√ 2 cos x
I = ∫ 4
dx
sin x
π (1+e )(1+sin x)

2

Apply king
π

2 sin x
8√ 2 cos x(e )
I = ∫ 4
dx . . . . (2)
sin x
π (1+e )(1+sin x)

2

Adding (1) & (2)


π

2
8√ 2 cos x
2I = ∫ 4
dx
1+sin x
π

2

2
8√ 2 cos x
I = ∫ dx
4
1+sin x
0

Put sin x = t
1
8√ 2
I = ∫ dx
4
1+t
0
1 1
1 1+ 1−
– t2 t2
I = 4√ 2 ∫ ( − ) dt
2 1 2 1
t + t +
0 t2 t2

1 1
1 (1+ ) (1− )
– t2 t2
I = 4√ 2 ∫ − dt
2 2
1 1
0 (t− ) +2 (t+ ) −2
t t

Let t − 1

t
= z &t+ 1

t
= k

0 2
– dz dk
= 4√ 2 [ ∫ − ∫ 2
]
2
z +2 k −2
−∞ ∞

0 2
– 1 −1 z 1 k−√ 2
= 4√ 2[ tan ] − [ ln( )]
√2 √2 2√ 2 k+√ 2
−∞ ∞

– π 1 2−√ 2
= 4√ 2 [ − [ln ]]
2√ 2 2√ 2 2+√ 2

= 2π + 2 ln(3 + 2√ 2)

α = 2

β = 2

84. (14.00)
4
(t + 1) dx = (2x + (t + 1) ) dt

4
2x+(t+1)
dx
=
dt t+1
dx 2x 3
− = (t + 1)
dt t+1
2

I.F. =
− ∫ dt −2 ln(t+1) 1
e t+1
= e = 2
(t+1)

x 1 3

2
= ∫ 2
(t + 1) dt + c
(t+1) (t+1)
2
(t+1)
x
2
= + c
2
(t+1)

3
⇒ c =
2
4
(t+1) 2
3
x = + (t + 1)
2 2

Put, t = 1
3
x = 2 + 6 = 14
85. (6699.00)
3, 7, 11, 15, ….., 403
2, 5, 8, 11, …., 404
LCM (4, 3) = 12
11, 23, 35,….. let (403)
403 = 11 + (n − 1) × 12
392
= n − 1
12

33 ⋅ 66 = n

Sum = 33

2
(22 + 32 × 12)

= 6699

86. (18.00)
Finding right hand limit
lim f (x) = lim f (0 + h)
+
x→0 h→0

= lim f (h)
h→0
−1 2 −1
cos (1−h )sin (1−h)
= lim 2
h→0 h(1−h )

−1 2
cos (1−h ) −1
sin 1
= lim ( )
h 1
h→0

Let cos −1 2
(1 − h ) = θ ⇒ cos θ = 1 − h
2

π θ
= lim
2 √ 1−cos θ
θ→0
π 1
= lim
2
θ→0 1−cos θ

2
θ

π 1
=
2 1

2

π
R =
√2

Now finding left hand limit


L = lim f (x)

x→0

= lim f (−h)
h→0
2 −1
−1
cos (1−{−h} )sin (1−{−h})

= lim 3
h→0 {−h}−{−h}

−1 2 −1
cos (1−(−h+1) )sin (1−(−h+1))

= lim
3
h→0 (−h+1)−(−h+1)

−1 2 −1
cos (−h +2h)sin h
= lim
2
h→0 (1−h)(1−(1−h) )

−1
π sin h
= lim ( )
2 2
h→0 (1−(1−h) )

−1
π sin h
= lim ( 2
)
2 −h +2h
h→0
−1
π sin h 1
= lim ( )( )
2 h −h+2
h→0
π
L =
4
2 2
32 2 2 32 π π
(L + R ) = ( + )
π
2 π 2 16

= 16 + 2

= 18
87. (72.00)
x
2
+ y
2
= 3 and x 2
= 2y
2
y + 2y − 3 = 0 ⇒ (y + 3) (y − 1) = 0

y = −3 y = 1 or
– –
y = 1, x = √ 2 ⇒ P (√ 2, 1)

p lies on the line



√ 2x + y = α
– –
√ 2 (√ 2) + 1 = α

α = 3

For circle C 1

Q lies on y axis
1

Let Q (0, α) coordinates


1

R = 2√ 3 (Given)
1

Line L act as tangent


Apply p = r (condition of tangency)
∣ α−3 ∣ –
⇒ = 2√ 3
∣ √3 ∣

⇒ |α − 3| = 6

α − 3 = 6 or α − 3 = −6

⇒ α = 9 or α = −3

∣ √2 1 1∣
1 ∣ ∣
ΔP Q1 Q2 = 0 9 1
2 ∣ ∣
−3 ∣ 0 1∣
1 – –
= ( √ 2 (12)) = 6 √ 2
2
2
(ΔP Q1 Q2 ) = 72
88. (678.00)
6 7 8
2 3
(x+1) (1+x ) (1−x )
Coeff. of x 30
in x
6

7 8
Coeff. of x in (1 + x)36 6 2 3
(1 + x ) (1 − x )

General term
6 7 8 r3 r1 +2r2 +3r3
C r1 C r2 C r3 (−1) x

r1 + 2r2 + 3r3 = 36

Case-I:

r1 r2 r3

0 6 8
2 5 8
4 4 8
6 3 8

r1 + 2r2 = 12 (Taking r 3 = 8 )
Case-II:

r1 r2 r3

1 7 7
3 6 7
5 5 7

r1 + 2r2 = 15 (Taking r 3 = 7 )
Case-III:

r1 r2 r3

4 7 6
6 6 6

r1 + 2r2 = 18 (Taking r 3 = 6 )
Coeff.
= 7 + (15 × 21) + (15 × 35) + (35) − (6 × 8) − (20 × 7 × 8) − (6 × 21 × 8) + (15 × 28) + (7 × 28)

= −678 = α

|α = 678|
89. (21.00)

⃗ ^ ^ ^
b = i − j + k (DR’s of L ) 1

i + j − k (DR’s of L )
⃗ ^ ^ ^
d = 2

∣ ^ ^ ^ ∣
i j k
∣ ∣
⃗× ⃗ = ∣ ∣
b d 1 −1 1
∣ ∣
∣ 1 1 −1 ∣

= 0^ ^ ^
i + 2 j + 2k (DR's of Line perpendicular to L and L ) 1 2

DR of AB line
= (0, 2, 2) = (3 + μ − λ, 3 + μ + λ, 3 − μ − λ)
3+μ−λ 3+μ+λ 3−μ−λ
= = =
0 2 2

Solving above equation we get μ = −


3

2
and λ =
3

Point A = (
5

2
,
1

2
,
9

2
)

5 7 15
B = ( , , )
2 2 2

Point of AB = (
5

2
, 2, 6) = (α, β, γ)

2 (α + β + γ) = 5 + 4 + 12 = 21

90. (169.00)
x + 2y + 3z = 42, x, y, z ⩾ 0

z = 0, x + 2y = 42 ⇒ 22

z = 1, x + 2y = 39 ⇒ 20

z = 2, x + 2y = 36 ⇒ 19

z = 3, x + 2y = 33 ⇒ 17

z = 4, x + 2y = 30 ⇒ 16

z = 5, x + 2y = 27 ⇒ 14

z = 6, x + 2y = 24 ⇒ 13

z = 7, x + 2y = 21 ⇒ 11

z = 8, x + 2y = 15 ⇒ 8

z = 9, x + 2y = 15 ⇒ 8

z = 10, x + 2y = 12 ⇒ 7

z = 11, x + 2y = 9 ⇒ 5

z = 12, x + 2y = 6 ⇒ 4

z = 13, x + 2y = 3 ⇒ 2

z = 14, x + 2y = 0 ⇒ 1

Total: 169
JEE Main- 01-02-2024 (Evening Shift)

Questions

1. A light planet is revolving around a massive star in a circular orbit of radius R with a period of revolution T.
3

If the force of attraction between planet and star is proportional to R −


2
then choose the correct option:
(a) T ∝ R
5
2
2

(b) T ∝ R
7
2
2

(c) T ∝ R
2 3

(d) T ∝ R
3
2
2

2. Monochromatic light of frequency 6 × 10 H z is produced by a laser. The power emitted is 2 × 10


14 −3
W .
How many photons per second on an average, are emitted by the source?
(Given h = 6.63 × 10 J s) −34

(a) 7 × 10 16

(b) 5 × 10 15

(c) 6 × 10 15

(d) 9 × 10 18

3. Match List – I with List – II.

List – I (Number) List – II (Significant figure)


A 1001 I 3
B 010.1 II 4
C 100.100 III 5
D 0.0010010 IV 6

Choose the correct answer from the options given below:


(a) A – IV, B – III, C – I, D – II
(b) A – III, B – IV, C – II, D – I
(c) A – II, B – I, C – IV, D – III
(d) A – I, B – II, C – III, D – IV

4. Train A is moving along two parallel rail tracks towards north with speed 72 km/h and train B is moving
towards south with speed 108 km/h. Velocity of train B with respect to A and velocity of ground with respect
to B are (in ms ): −1

(a) – 50 and – 30
(b) – 30 and 50
(c) 50 and – 30
(d) – 50 and 30
5. A diatomic gas (γ = 1.4) does 200 J of work when it is expanded isobarically. The heat given to the gas in
the process is:
(a) 850 J
(b) 800 J
(c) 700 J
(d) 600 J

6. A body of mass 4 kg experiences two forces F ⃗ 1


^ ^ ^
= 5 i + 8 j + 7k and F ⃗
2
^ ^ ^
= 3 i − 4 j − 3k . The acceleration
acting on the body is:
(a) 4^i + 2^j + 2k
^

(b) 2^i + ^j + k
^

(c) 2^i + 3^j + 3k


^

(d) −2^i − ^j − k^

7. In a metre-bridge when a resistance in the left gap is 2Ω and unknown resistance in the right gap, the balance
length is found to be 40cm. On shunting the unknown resistance with 2Ω, the balance length changes by:
(a) 62.5 cm
(b) 22.5 cm
(c) 65 cm
(d) 20 cm

8. If frequency of electromagnetic wave is 60 MHz and it travels in air along z direction then the corresponding
electric and magnetic field vectors will be mutually perpendicular to each other and the wavelength of the
wave (in m) is:
(a) 10
(b) 2.5
(c) 5
(d) 2

9. In an ammeter, 5% of the main current passes through the galvanometer. If resistance of the galvanometer is
G, the resistance of ammeter will be:
(a) G

199

(b) 199G
(c) 200G
(d) G

20

10. A big drop is formed by coalescing 1000 small droplets of water. The surface energy will become:
(a) 1

10
th

(b) 100 times


(c) 1

100
th

(d) 10 times

11. A cricket player catches a ball of mass 120g moving with 25 m/s speed. If the catching process is completed
in 0.1 s then the magnitude of force exerted by the ball on the hand of player will be (in SI unit):
(a) 25
(b) 30
(c) 24
(d) 12
12. A disc of radius R and mass M is rolling horizontally without slipping with speed v. It then moves up an
inclined smooth surface as shown in figure. The maximum height that the disc can go up the incline is:

(a) 2

3
v

g
2

(b) 1

2
v

g
2

(c) v

g
2

(d) 3

4
v

13. A galvanometer (G) of 2Ω resistance is connected in the given circuit. The ratio of charge stored in C and
1

C is:
2

1
(a) 2
2
(b) 3

(c) 1
(d) 3

14. If the root mean square velocity of hydrogen molecule at a given temperature and pressure is 2km/s, the root
mean square velocity of oxygen at the same condition in km/s is:
(a) 1.5
(b) 1.0
(c) 0.5
(d) 2.0

15. Form the statements given below:


(A) The angular momentum of an electron in nth orbit is an integral multiple of h.
(B) Nuclear forces do not obey inverse square law.
(C) Nuclear forces are spin dependent.
(D) Nuclear forces are central and charge independent
(E) Stability of nucleus is inversely proportional to the value of packing fraction.
Choose the correct answer from the options given below:
(a) A, C, D, E only
(b) B, C, D, E only
(c) A, B, C, D only
(d) A, B, C, E only
16. To measure the temperature coefficient of resistivity α of a semiconductor, an electrical arrangement shown
in the figure is prepared. The arm BC is made up of the semiconductor. The experiment is being conducted
at 25 C and resistance of the semiconductor arm is 3mΩ. Arm BC is cooled at a constant rate of 2 C /s . If
∘ ∘

the galvanometer G shows no deflection after 10s, then α is:


(a) −1 × 10
−2
C
−1


(b) −1.5 × 10
−2
C
−1


(c) −2.5 × 10
−2
C
−1

(d) −2 × 10
−2

C
−1

17. A microwave of wavelength 2.0 cm falls normally on a slit of width4.0 cm. The angular spread of the
central maximum of the diffraction pattern obtained on a screen 1.5 m away from the slit, will be:
(a) 60 ∘

(b) 30 ∘

(c) 45 ∘

(d) 15 ∘

18. A transformer has an efficiency of 80% and works at 10V and 4kW. If the secondary voltage is 240V, then
the current in the secondary coil is:
(a) 13.33 A
(b) 1.33 A
(c) 1.50 A
(d) 15.1 A

19. Conductivity of a photodiode starts changing only if the wavelength of incident light is less than 660 nm.
The band gap of photodiode is found to be ( ) eV . The value of X is: (Given,
X

h = 6.6 × 10
−34 −19
J s, e = 1.6 × 10 C )
(a) 11
(b) 15
(c) 13
(d) 21

20. C1 and C are two hollow concentric cubes enclosing charges 2Q and 3Q respectively as shown in figure.
2

The ratio of electric flux passing through C and C is: 1 2

(a) 2:5
(b) 5:2
(c) 3:2
(d) 2:3
21. One end of a metal wire is fixed to a ceiling and a load of 2 kg hangs from the other end. A similar wire is
attached to the bottom of the load and another load of 1 kg hangs from this lower wire. Then the ratio of
longitudinal strain of upper wire to that of the lower wire will be_____ .
[Area of cross section of wire = 0.005cm , Y = 2 × 10 N m and g = 10ms ]
2 11 −2 −2

22. A uniform rod AB of mass 2 kg and length 30 cm at rest on a smooth horizontal surface. An impulse of
force 0.2Ns is applied to end B. The time taken by the rod to turn through at right angle will be s, where xπ

= ___.

23. Suppose a uniformly charged wall provides a uniform electric field of 2 × 10 N /C normally. A charged
4

particle of mass 2 g being suspended through a silk thread of length 20 cm and remain stayed at a distance
of 10cm from the wall. Then the charge on the particle will be μC where x = ______.[Use
1

√x

g = 10m/s
2
]

24. ∘

In Young’s double slit experiment, monochromatic light of wavelength 5000A is used. The slits are 1.0 mm
apart and screen is placed at 1.0 m away from slits. The distance from the centre of the screen where
intensity becomes half of the maximum intensity for the first time is ______ ×10 m −6

25. A particular hydrogen – like ion emits the radiation of frequency 3 × 10 H z when it makes transition from
15

n = 2 to n = 1. The frequency of radiation emitted in transition from n = 3 to n = 1 is × 10 H z, when x


x

9
15

= ______.

26. A coil of 200 turns and area 0.29m is rotated at half a revolution per second and is placed in unform
2

magnetic field of 0.01T perpendicular to axis of rotation of the coil. The maximum voltage generated in the
coil is 2π

β
volt. The value of β is _____.

27. A mass m is suspended from a spring of negligible mass and the system oscillates with a frequency f . The 1
f
frequency of oscillations if a mass 9m is suspended from the same spring is f . The value of
2 is ______.
f
1

28. A particle initially at rest starts moving from reference point x = 0 along x – axis, with velocity v that varies
as v = 4√− −
x m/s . The acceleration of the particle is _____ ms
−2
.

29. A moving coil galvanometer has 100 turns and each turn has an area of 2.0cm . The magnetic field
2

produced by the magnet is 0.01 T and the deflection in the coil is 0.05 radian when a current of 10mA is
passed through it. The torsional constant of the suspension wire is x × 10 N − m/rad. The value of x is
−5

_____.

30. In an electrical circuit drawn below the amount of charge stored in the capacitor is _____ μC .
31. Given below are two statements:
Statement (I): S iO and GeO are acidic while SnO and PbO are amphoteric in nature.
2 2

Statement (II): Allotropic forms of carbon are due to property of catenation and pπ − dπ bond formation.
In the light of the above statements, choose the most appropriate answer from the options given below:
(a) Both Statement I and Statement II are false
(b) Statement I is false but Statement II is true
(c) Both Statement I and Statement II are true
(d) Statement I is true but Statement II is false

32. The transition metal having highest 3rd ionisation enthalpy is:
(a) Mn
(b) V
(c) Cr
(d) Fe

33. The functional group that shows negative resonance effect is:
(a) −OH
(b) −OR
(c) −N H 2

(d) −C OOH

34. The Strongest reducing agent among the following is:


(a) P H 3

(b) S bH 3

(c) BiH 3

(d) N H 3

35. Solubility of calcium phosphate (molecular mass M) in water is W per 100 ml. at 25
g

C . Its solutions
product at 25 C will be approximately.

(a) 3
10 (
W

M
)

(b) 7
10 (
W

M
)

(c) 5
10 (
W
)
M

(d) 7
10 (
W

M
)
36. Match List – I with List – II.

List – II (Compound) List – II (Use)


A. Carbon tetrachloride (I) Paint remover
B. Methylene chloride (II) Refrigerators and air conditioners
C. DDT (III) Fire extinguisher
D. Freons (IV) Non-Biodegradable insecticide

Choose the correct answer from the options given below:


(a) A − I V , B − I I I , C − I I , D − I
(b) A − I , B − I I , C − I I I , D − I V
(c) A − I I I , B − I , C − I V , D − I I
(d) A − I I , B − I I I , C − I , D − I V

37. Given below are two statements:


Statement (I): Both metals and non-metals exist in p and d-block elements
Statement (II): Non-metals have higher ionisation enthalpy and higher electronegativity than the metals.
In the light of the above statements, choose the most appropriate answer from the options given below:
(a) Statement I is true but Statement II is false
(b) Statement I is false but Statement II is true
(c) Both Statement I and Statement II are false
(d) Both Statement I and Statement II are true

38. Lassaigne's test is used for detection of :


(a) Nitrogen and Sulphur only
(b) Nitrogen, Sulphur and Phosphorous Only
(c) Phosphorous and halogens only
(d) Nitrogen, Sulphur, phosphorous and halogens

39. Given below are two statements:


Statement (I): Dimethyl glyoxime forms a six-membered covalent chelate when treated with N iC l solution
2

in presence of N H OH 4

Statement (II): Prussian blue precipitate contains iron both in (+2) and (+3) oxidation states.
In the light of the above statements, choose the most appropriate answer from the options given below:
(a) Statement I is false but Statement II is true
(b) Both Statement I and Statement II are false
(c) Statement I is true but Statement II is false
(d) Both Statement I and Statement II are true

40. The set of meta directing functional groups from the following sets is:
(a) −C N , −N H , −N H R, −OC H
2 3

(b) −N O , −C H O, −S O H , −C OR
2 3

(c) −C N , −C H O, −N H C OC H , −C OOR
3

(d) −N O , −N H , −C OOH , −C OOR


2 2
41. Which of the following compounds show colour due to d-d transition?
(a) K C rO
2 4

(b) K C r O
2 2 7

(c) K M nO 4

(d) C uS O .5H O 4 2

42. The number of radial node/s for 3p orbital is:


(a) 2
(b) 4
(c) 3
(d) 1

43. Given below are statements:


Statement (I): A π bonding MO has lower electron density above and below inter-nuclear axis.
Statement (II): The π antibonding MO has a node between the nuclei:

In the light of the above statements, choose the most appropriate answer from the options given below:
(a) Statement I is true but Statement II is false
(b) Statement I is false but Statement II is true
(c) Both Statement I and Statement II are false
(d) Both Statement I and Statement II are true

44. Which among the following has highest boiling point?


(a) C H C H C H C H − OH
3 2 2 2

(b) C H C H C H C H
3 2 2 3

(c) C H C H C H C H O
3 2 2

(d) H C − O − C H
5 2 2 5

45. Select the compound from the following that will show intramolecular hydrogen bonding.
(a) H O2

(b) N H 3

(c)

(d) C 2 H 5 OH
46. Match List – I with List – II

List – I (Reactants) List – II (Product)


A Phenol, Z n/Δ (I) Salicylaldehyde
B Phenol, C O , N aOH , H C l (II) Salicylic acid
2

C Phenol, C O , N aOH , H C l (III) Benzene


2

D Phenol, Conc. H N O 3 (IV) Picric acid

Choose the correct answer from the options given below:


(a) A − I V , B − I , C − I I , D − I I I
(b) A − I V , B − I I , C − I , D − I I I
(c) A − I I I , B − I , C − I I , D − I V
(d) A − I I I , B − I V , C − I , D − I I

47. Given below are two statements: one is labelled as Assertion A and the other is labelled as Reason R.
Assertion (A): In aqueous solutions C r is reducing while M n is oxidising in nature.
2+ 3+

Reason (R): Extra stability to half filled electronic configuration is observed than incompletely filled
electronic configuration.
In the light of the above statements, choose the most appropriate answer from the options given below:
(a) Both A and R are true and R is the correct explanation of A
(b) A is true but R is false
(c) A is false but R is true
(d) Both A and R are true but R is not the correct explanation of A

48.

Acid D formed in above reaction is:


(a) Oxalic acid
(b) Succinic acid
(c) Gluconic acid
(d) Malonic acid

49. [C o(N H 3 ) ]
3+
and [C oF ] are respectively known as:
6
3−

(a) Outer orbital complex, Inner orbital complex


(b) Spin paired complex, spin free complex
(c) Spin free complex, spin free complex
(d) Inner orbital complex, Spin paired complex.
50. In the given reactions identify A and B

(a) A : 2 – Pentyne, B : Cis – 2 – butene


(b) A : n – Pentane, B : Trans – 2 - butene
(c) A : 2 – Pentyne, B : Trans – 2 - butene
(d) A : n – Pentane, B : Cis – 2 – Butene

51. The amount of electricity in Coulomb required for the oxidation of 1 mol of H 2O to O is _______ ×10
2
5
C

52. The number of tripeptides formed by three different amino acids using each amino acid once is

53. 10 mL of gaseous hydrocarbon on combustion given 4 mL of C O (g) and 50 mL of water vapour. Total
2

number of carbon and hydrogen atoms in the hydrocarbon is _____

54. Mass of ethylene glycol (antifreeze) to be added to 18.6 kg of water to protect the freezing point at −24 ∘
C

is ______ kg (Molar mass in g mol for ethylene glycol 62, K of water = 1.86K kg mol )
−1
f
−1

55. Following Kjeldah’s method. 1g of organic compound released ammonia, that neutralised 10mL of
2M H S O The percentage of nitrogen in the compound is _____ %.
2 4

56. The following data were obtained during the first thermal decomposition of a gas A at constant volume:
A (g) → 2B (g) + C (g)

S.No Time/s Total pressure/ (atom)


1 0 0.1
2. 115 0.28

The rate constant of the reaction is_____ × 10 −2


s
−1
(nearest integer)

57. Consider the following redox reaction:


− + 2+
M nO + H + H 2 C2 O 4 ⇌ M n + H2 O + C O 2
4

The standard reduction potential are given as below (E +

red
) :
o 2+
E −
/M n = +1.51V
M nO
4

o
E /H 2 C 2 O 4 = −0.49V
C O2

If the equilibrium constant of the above reaction is given as K eq


x
= 10 , then the value of x = ____ (nearest
integer)

58. Total number of isomeric compounds (including stereoisomers) formed by monochlorination of 2-


methylbutane is _____.
59. For a certain reaction at 300K, K = 10, then ΔG for the same reaction is _____ ×10 ∘ −1
kJ mol
−1
. (Given
R = 8.314J K mol ) −1 −1

60. Number of compounds which give reaction with Hinesburg’s reagent is _____

61. Let Ajay will not appear in JEE exam with probability p =
2
, while both Ajay and Vijay will appear in the
7

exam with probability q =


1

5
. Then the probability, that Ajay will appear in the exam and Vijay will not
appear is:
(a) 18

35

(b) 35
9

(c) 35
3

(d) 24

35

62. x − 1, x is even
Let f (x) = { ,x ∈ N . If for some a ∈ N , f (f (f (a))) = 21, then
2x, x is odd
3
|x|
lim

{
a
− [
x

a
]} , where [t] denotes the greatest integer less than or equal to t, is equal to:
x→a

(a) 144
(b) 169
(c) 225
(d) 121

63. The number of solutions of the equation 4sin 2 3


x − 4cos x + 9 − 4 cos x = 0; x ∈ [−2π, 2π] is:
(a) 1
(b) 0
(c) 2
(d) 3
64. Let α be a non-zero real number. Suppose f : R → R is a differentiable function such that f (0) = 2 and
f (x) = 1 . If f (x) = αf (x) + 3, for all x ∈ R , then f (−log 2) is equal to _____.

lim e
x→−∞

(a) 5
(b) 9
(c) 7
(d) 3

65. Let P and Q be the points on the line =


x+3
=
y−4
which are at a distance of 6 units from the point
z+1

8 2 2

R (1, 2, 3). If the centroid of the triangle P QR is (α, β, γ) , then α + β + γ is:2 2 2

(a) 24
(b) 18
(c) 26
(d) 36

66. Let f (x) = ∣∣2x + 5 |x| − 3∣∣ , x ∈ R. If m and n denote the number of points where f is not continuous
2

and not differentiable respectively, then m + n is equal to:


(a) 2
(b) 0
(c) 3
(d) 5

67. Let α and β be the roots of the equation px 2


+ qx − r = 0, where p ≠ 0. If p, q and r be the consecutive
terms of a non constants G.P. and + = , then the value of (α − β) is:
1 1 3 2

α β 4

(a) 20

(b) 9
(c) 8
(d) 80

68. Let P be a point on the ellipse x


+
2 y
2

= 1. Let the line passing through P and parallel to y-axis meet the
9 4

circle x + y = 9 at point Q such that P and Q are on the same side of the x-axis. Then, the eccentricity
2 2

of the locus of the point R on P Q such that P R : RQ = 4 : 3 as P moves on the ellipse is:
(a) 11

19

(b) 13

21

(c)
√ 13

(d)
√ 139

23

69. If the mirror image of the point P (3, 4, 9) in the line


x−1
=
y+1
=
z−2
is (α, β, γ) , then 14 (α + β + γ)
3 2 1

is:
(a) 132
(b) 108
(c) 138
(d) 102
70. Consider a ΔABC where A (1, 3, 2) , B (−2, 8, 0) and C (3, 6, 7). If the angle bisector of ∠BAC meets

−→ −
−→
the line BC at D, then the length of the projection of the vector AD on the vector AC is:
39
(a) 2√ 38
−−
(b) √19
(c) √ 38

(d)
37

2√ 38

71. If the domain of the function f (x) =


√ x 2 −25
+ log10 (x
2
+ 2x − 15) is (−∞, α) ∪ [β, ∞), then
2
(4−x )

α
2
+ β
3
is equal to:
(a) 150
(b) 140
(c) 125
(d) 175

72. Let the locus of the midpoints of the chords of the circle x 2 2
+ (y − 1) = 1 drawn from the origin intersect
the line x + y = 1 at P and Q . Then, the length of P Q is:
(a) 1
(b) 1

2
1
(c) √2

(d) √2

73. Let S denote the sum of the first n terms of an arithmetic progression. If S
n 10 = 390 and the ratio of the
tenth and the fifth terms is 15 : 7, then S − S is equal to: 15 5

(a) 790
(b) 690
(c) 800
(d) 890

74. 10 10

Consider 10 observations x such that and where α, β


2
1, x2 , . . . , x10 ∑ (xi − α) = 2 ∑ (xi − β) = 40,
i=1 i=1
β
are positive integers. Let the mean and variance of the observations be 6

5
and 84

25
respectively. Then α
is
equal to:
(a) 1
(b) 2
(c) 5

(d) 3

75. 1 1

The value of ∫ 3
(2x
2
− 3x + x + 1)
3
dx is equal to:
0

(a) 0
(b) 2
(c) 1
(d) −1
76. If z is a complex number such that |z| ⩾ 1, then the minimum value of ∣∣z + 1
(3 + 4i)∣
∣ is:
2

(a) 2
(b) 3
(c) 5

2
3
(d) 2

77. Consider the relation R and R defined as aR b ⇔ a + b = 1 for all a, b


1 2 1
2 2
∈ R and
(a, b) R (c, d) ⇔ a + d = b + c for all (a, b) , (c, d) ∈ N × N . Then
2

(a) Only R is an equivalence relation


1

(b) Only R is an equivalence relation


2

(c) Neither R nor R is an equivalence relation


1 2

(d) R and R both are equivalence relations


1 2

78. π

3

If ∫ 4
cos x dx = aπ + b√ 3 , where a and b are rational numbers, then 9a + 8b is equal to:
0
3
(a) 2

(b) 2
(c) 3
(d) 1

79. Let the system of equations x + 2y + 3z = 5, 2x + 3y + z = 9, 4x + 3y + λz = μ have infinite


number of solutions. Then λ + 2μ is equal to:
(a) 17
(b) 28
(c) 22
(d) 15

80. Let m and n be the coefficients of seventh and thirteenth terms respectively in the expansion of
18
1
1

(
1

3
x 3 +
1
2
) . Then ( n

m
) 3
is:
2x 3

(a) 9

(b) 4

(c) 1

(d) 1

81. If three successive terms of a G.P. with common ratio r (r > 1) are the lengths of the sides of a triangle and
[r] denotes the greatest integer less than or equal to r, then 3 [r] + [−r] is equal to ____.

82. The sum of squares of all possible values of k, for which area of the region bounded by the parabolas
= kx and ky = (y − x) is maximum, is equal to ____.
2 2
2y

83. Let a⃗ ^ ^ ^
= i + j + k, and c ⃗ = 4^i + c ^j + c k
⃗ ^ ^ ^
b = − i − 8 j + 2k
^ be three vectors such that
2 3

⃗ ⃗ If the angle between the vector c ⃗ and the vector 3^


b × a⃗ = c ⃗ × a. i + 4 j + k is θ, then the greater integer
^ ^

less than or equal to tan θ is _____. 2


84. 2
(√ x+1)(x −√ x)
If y =
x√ x+x+√ x
+
1

15
2
(3cos x − 5) cos x,
3
then 96y ′
(
π

6
) is equal to _____.

85. Let ABC be an isosceles triangle in which A is at (−1, 0) , ∠A =



, AB = AC and B is on the positive
3
4

-axis. If and the line BC intersects the line y at (α, β), then is ______.
β
x BC = 4√ 3 = x + 3
2
α

86. Three points O (0, 0) , P (a, a ) , Q (−b, b ) , a > 0, b > 0, are on the parabola y = x . Let S be the
2 2 2
1

area of the region bounded by the line P Q and the parabola, and S be the area of the triangle OP Q. If the 2

minimum value of is , gcd (m, n) = 1 , then m + n is equal to ____.


S1 m

S2 n

87. Let A = I − 2M M , where M is a real matrix of order 2 × 1 such that the relation M M = I holds.
2
T T
1

If λ is a real number such that the relation AX = λX holds for some non-zero real matrix X of order
2 × 1, then the sum of squares of all possible values of λ is equal to _____.

88. If dx
=
1+x−y
2

, x (1) = 1, then 5x (2) is equal to ____.


dy y

89. The lines L , L , . . . , L are distinct. For n = 1, 2, 3, . . . , 10 all the lines L


1 2 20
are parallel to each other 2n−1

and all the lines L pass through a given point P . The maximum number of points of the intersection of
2n

pairs of lines from the set {L , L , . . . . , L } is equal to _____.


1 2 20

90. x 12

Let f : (0, ∞) → R and f (x) = ∫ tf (t) dt . If F 2


(x ) = x
4
+ x
5
, then ∑ f (r )
2
is equal to ____.
0 r=1
Answer Key

1. A 2. B 3. C 4. D 5. C 6. B
7. B 8. C 9. D 10. A 11. B 12. B
13. A 14. C 15. D 16. A 17. B 18. A
19. B 20. A 21. 3 22. 4 23. 3 24. 125
25. 32 26. 5 27. 3 28. 8 29. 4 30. 60
31. D 32. A 33. D 34. C 35. D 36. C
37. B 38. D 39. A 40. B 41. D 42. D
43. B 44. A 45. C 46. C 47. A 48. B
49. B 50. C 51. 2 52. 6 53. 14 54. 15
55. 56 56. 2 57. 338 58. 6 59. 57 60. 5
61. A 62. A 63. B 64. B 65. B 66. C
67. D 68. C 69. B 70. D 71. A 72. C
73. A 74. B 75. A 76. D 77. B 78. B
79. A 80. A 81. 1.00 82. 8.00 83. 38.00 84. 105.00
85. 36.00 86. 7.00 87. 2.00 88. 5.00 89. 101.00 90. 219.0
Solutions

1. (A)
GM m 2
F = 3
= mω R

R 2

ω
2

1

5
∵ T =

ω
So
R 2

5
2
T ∝ R 2

2. (B)
P = nhv
−3
P 2×10 15
n = = = 5 × 10
−34 14
hv 6.63×10 ×6×10

3. (C)
Theoretical.

4. (D)

V A = 20m/s

V B = −30m/s

Velocity of B w.r.t. A
V B/A = −50m/s

Velocity of ground w.r.t. B


V G/B = 30m/s

5. (C)
2 2
γ = 1 + = 1.4 ⇒ = 0.4
f f

⇒ f = 5

W = nRΔT = 200J
f +2
Q = ( ) nRΔT
2

7
= × 200 = 700J
2

6. (B)
Net force = ^ ^ ^
8 i + 4 j + 4k

= 2^ ^ ^
F
a⃗ = i + j + k
m
7. (B)

First case 2

40
=
X

60
⇒ X = 3Ω

In second case X ′
=
2×3

2+3
= 1.2Ω

2 1.2
=
ℓ 100−ℓ

200 − 2ℓ = 1.2ℓ
200
ℓ = = 62.5cm
3.2

Balance length changes by 22.5 cm.

8. (C)
8
c 3×10
λ = = = 5m
6
f 60×10

9. (D)

Is S = Ig G
95 5I
IS = G
100 100
G
S =
19
2
G

SG 19
RA = =
S+G 20G

19

G
RA =
20

10. (A)
Let’s say radius of small droplets is r and that of big drop is R
4 3 4 3
πR = 1000 πr
3 3

R = 10r
2
Ui = 1000 (4πr S )
2
Uf = 4πR S
2
= 100 (4πr S )
1
Uf = Ui
10

11. (B)
Δp
F av =
Δt
0.12×25
= = 30N
0.1
12. (B)
Only the translational kinetic energy of disc changes into gravitational potential energy. And rotational KE
remains unchanged as there is no friction.
1 2
mv = mgh
2
2
v
h =
2g

13. (A)

In steady state
Req = 12Ω
6
I = = 0.5A
12

P.D across C 1 = 3V

P.D across C 2 = 4V

q1 = C 1 V 1 = 12μC

q2 = C 2 V 2 = 24μC
q
1 1
=
q 2
2

14. (C)

−−−
3RT
V rms = √
M
−−
− −−
V1 M2 2 32
= √ ⇒ = √
V2 M1 V2 2

V 2 = 0.5km/s

15. (D)
Part of theory.

16. (A)
For no deflection 0.8

1
=
R

⇒ R = 2.4mΩ

Temperature fall in 10s = 20 C


ΔR = RαΔt
ΔR −0.6 −2 −1
α = = = −10 C
RΔt 3×20

17. (B)
For first minima a sin θ = λ
λ 1
sin θ = =
a 2

θ = 30
18. (A)
E S IS
Efficiency = E P IP
240IS
0.8 =
4000
3200
IS = = 13.33A
240

19. (B)
−34 8
hc 6.6×10 ×3×10
Eg = = J
λ −9
660×10
−34 8
66×10 ×3×10
= eV
−9 −19
660×10 ×1.6×10
15
= eV
8

So, x = 15

20. (A)
2Q
ϕsmaller cube
=
∈0

5Q
ϕbigger cube =
∈0

ϕ
smaller cube 2
=
ϕ 5
bigger cube

21. (3)

FL
ΔL =
AY
ΔL F
=
L AY
ΔL
1

L F1 30
1
= = = 3
ΔL F2 10
2

L
2
22. (4)

Impulse J = 0.2N − S

J = ∫ F dt = 0.2N − s

Angular impulse (M⃗ )



M c = ∫ τ dt
L
= ∫ F dt
2
L L
= ∫ F dt = × J
2 2
0.3
= × 0.2 = 0.03
2
2
2 2×(0.3)
ML 0.09
Icm = = =
12 12 6

M = Icm (ωf − ωi )
(0.09)
0.03 = (ωf )
6

ωf = 2rad/s

θ = ωt
θ π π
t = = = sec
ω 2×2 4

X = 4
23. (3)

10 1
sin θ = =
20 2

θ = 30
qE
tan θ =
mg
4
q×2×10

tan 30 =
−3
1×10 ×10
1 6
= q × 10
√3
1 −6
q = × 10 C
√3

x = 3

24. (125)
Let intensity of light on screen due to each slit is I0
So internity at centre of screen is 4I0
−−− −
Intensity at distance y from centre I = I0 + I0 + 2√I0 I0 cos ϕ

Imax = 4I0
Imax
= 2I0 = 2I0 + 2I0 cos ϕ
2

cos ϕ = 0
π
ϕ =
2
π
K Δx =
2
2π π
d sin θ =
λ 2
2 y 1
d × =
λ D 2
−7
λD 5×10 ×1
y = = −3
4d 4×10
−6
= 125 × 10 = 125
25. (32)
2 1 1
E = −13.6z ( 2
− 2
)
n n
i f

1 1
E = C ( 2
− 2
)
n n
f i

1 1
hv = C [ 2
− 2
]
n n
f i

1 1
[ − ]
2 2
n n
v1 f i
2−1
=
v2
1 1
[ − ]
2 2
n n
f i
3−1

1 1 3
[ − ]
1 4 4
= =
1 1 8
[ − ]
1 9 9

3 9
= ×
4 8
v1 27
=
v2 32
32 32 15 32 15
v2 = v1 = × 3 × 10 Hz = × 10 Hz
27 27 9

26. (5)
ϕ = N AB cos(ωt)

ε = − = N ABω sin(ωt)
dt

ε max = N ABω

= 200 × 0.2 × 0.01 × π


4π 2π
= = volt
10 5

27. (3)
−−
1 k
f1 = √
2π m
−−

1 k
f2 = √
2π 9m

f


1 9 3
= √ =
f 1 1
2

28. (8)
−−
V = 4√ x
dv
a = V
dx
1
−− 1 − 2
= 4√ x × 4 × x 2 = 8m/s
2

29. (4)
τ = BI N A sin ϕ

C θ = BI N A sin 90
−3 −4
BI N A 0.01×10×10 ×100×2×10
C = =
θ 0.05
−5
= 4 × 10 N − m/rad.

x = 4
30. (60)

In steady state there will be no current in branch of capacitor, so no voltage drop across R 2 = 5Ω

I2 = 0
10
I1 = I3 = = 1A
4+6

V R3 = V c + V R2

V R2 = 0

I3 R 3 = V c

V c = 1 × 6 = 6volt

qc = C V c = 10 × 6 = 60μC

31. (D)
S iO and GeO are acidic while SnO and PbO are amphoteric
2 2

Carbon does not have d-orbitals so can not form pπ − dπ Bond with itself. Due to properties of catenation and
pπ − pπbond formation. carbon is able to show allotropic forms.

32. (A)
3rd Ionisation energy: [NCERT Data]
V : 2833 KJ/mol
Cr : 2990 KJ/mol
Mn : 3260 KJ/mol
Fe : 2962 KJ/mol
Alternative
Min: 3d 4s 5 2

Fe: 3d 4s
6 2

Cr: 3d 4s
5 1

V: 3d 4s3 2

33. (D)

shows -R effect, while rest 3 groups shows +R effect via lone pair.

34. (C)
Strongest reducing agent: BiH explained by its low bond dissociation energy.
3
35. (D)
W ×10
S =
M
2+ 3−
C a 3 (P O 4 ) (s) ⇌ 3C a (aq.) + 2P O (aq.)
2 4
3s
2s
W ×1000 W ×10
S = =
M ×100 M
3 2
Ksp = (3s) (2s)
5
= 108s
5
5 W
= 108 × 10 × ( )
M

5
7 W
= 1.08 × 10 ( )
M

36. (C)
C C l used in fire extinguisher. C H
4 2
C l2 used as paint remover. Freons used in refrigerator and AC. DDT used
as non-Biodegradable insecticide.

37. (B)
I. In p-Block both metals and non metals are present but in d-Block only metals are present.
II. EN and IE of non metals are greater than that of metals
I - False, II-True

38. (D)
Lassaigne's test is used for detection of all element N, S, P, X

39. (A)
2+
Ni + N H 4 OH + dmg →

2 Five member ring


III II

F e 4 [F e(C N ) ]
6 3

Prussian Blue

40. (B)

All are −M , Hence meta directing groups.

41. (D)
C uS O 4 .5H 2 O
2+ 9 0
Cu : 3d 4s

unpaired electron present so it show colour due to d-d transition


42. (D)
For 3p : n = 3, ℓ = 1
Number of radial node = n − ℓ − 1

= 3 − 1 − 1 = 1

43. (B)
A π bonding molecular orbital has higher electron density above and below inter nuclear axis.

44. (A)
Due to H-bonding boiling point of alcohol is High.

45. (C)
H 2 O, N H 3 , C 2 H 5 OH ⇒ Intermolecular H-Bonding

46. (C)
47. (A)
Cr2+ is reducing as it configuration changes from d4 to d3 due to formation of Cr3+, which has half filled t2g
level, on other hand, the change Mn3+ to Mn2+ result half filled d5 configuration which has extra stability.

48. (B)

49. (B)
(Strong field ligand) ⇒
3+ 3+ 6 6 0
[C o(N H 3 ) ] Co 3d (t , eg )
6 2g

Hybridisation: d sp 2 3

Inner orbital complex (spin paired complex) Pairing will take place.
3−
[C oF 6 ]

Co
3+
(Weak field ligand)⇒ 3d
6
(t
4
2g
2
, eg )

Hybridisation: sp d 3 2

Outer orbital complex (spin free complex)


No pairing will take place.

50. (C)

51. (2)
+ −
2H 2 O → O 2 + 4H + 4e
W Q
=
E 96500
Q
mole × n − factor =
96500

Q = 2 × 96500C
5
= 1.93 × 10 C

52. (6)
Let 3 different amino acids are A, B, C then following combination of tripeptides can be formed –
ABC, ACB, BAC, BCA, CAB, CBA
53. (14)
C xH y + O 2 → C O 2 + H 2 O
10ml
y y
C xH y + (x + ) O 2 → xC O 2 + H2 O
4 2
10x 5y

10x = 40

x = 4

5y = 50

y = 10

C 4 H 10

54. (15)
Δ T f = iK f × molality
W
24 = (1) × 1.86 ×
62×18.6

W = 14880gm = 14.880kg

55. (56)
H 2 S O 4 + 2N H 3 → (N H 4 ) S O 4
2

Millimole of H S O → 10 × 2
2 4

So Millimole of N H = 20 × 2 3 = 40

Organic → N H 3

Compound 40 Millimole
∴ Mole of N =
40

1000

Wt. of N =
40

1000
× 14

% Composition of N in organic compound = 40×14

1000×1
× 100 = 56%

56. (2)
A (g) → 2B (g) + C (g)

t = 0 0.1

t = 115 sec. 0.1 − x 2x x

0.1 + 2x = 0.28

2x = 0.18

x = 0.09
1 0.1
K = ℓn =
115 0.1−0.09
−1
= 0.0200sec
−2 −1
= 2 × 10 sec

57. (338)
Cell Rx ; M nO + H C
n −

4
2 2 O4
→ Mn
2+
+ C O2

E
o
= E
cell
of anode +E
o
op
o

RP
of cathode
= 0.49 + 1.51 = 2.00V

At equilibrium
Ecell = 0,
o 0.059
E = log K
cell n

(As per N C ERT =


RT

F
= 0.059 But RT

F
= 0.0591 can also be taken).
0.059
2 = log K
10

log K = 338.98
58. (6)

59. (57)

ΔG = −RT ℓnK

= −8.314 × 300ℓn (10)

= 5744.14 J /mole
−1
= 57.44 × 10 kJ /mole

60. (5)

61. (A)

¯ 2
P (A ) = = p
7
1
P (A ∩ V ) = = q
5
5
P (A) =
7

Ans. P ¯
(A ∩ V ) =
18

35
62. (A)
x − 1; x = even
f (x) = {
2x; x = odd

f (f (f (a))) = 21

Case-1: If a = even
f (a) = a − 1 = odd
f (f (a)) = 2 (a − 1) = even
f (f (f (a))) = 2a − 3 = 21 ⇒ a = 12

Case-2: If a = odd
f (a) = 2a = even
f (f (a)) = 2a − 1 = odd
f (f (f (a))) = 4a − 2 = 21 (Not possible)
Hence a = 12

Now
3
|x|
x
lim ( − [ ])
− 2 12
x→12
3
|x|
x
= lim − lim [ ]
− 12 − 12
x→12 x→12

= 144 − 0 = 144

63. (B)
2 3
4sin x − 4cos x + 9 − 4 cos x = 0; x ∈ [−2π, 2π]
2 3
4 − 4cos x − 4cos x + 9 − 4 cos x = 0
3 2
4cos x + 4cos x + 4 cos x − 13 = 0
3 2
4cos x + 4cos x + 4 cos x = 13

L.H.S. ⩽ 12 cant be equal to 13.

64. (B)
f (0) = 2, lim f (x) = 1
x→−∞

f (x) − x. f (x) = 3

I.F. = e
−αx

−αx −αx
y (e ) = ∫ 3. e dx
−αx
−αx 3e
f (x) . (e ) = + c
−α
−3 3
x = 0 ⇒ 2 = + c ⇒ = c − 2 . . . . (1)
α α
−3 αx
f (x) = + c. e
α
−3
x → −∞ ⇒ 1 = + c (0)
α

α = −3 ∴ c = 1
−3 αx
f (− ln 2) = + c. e
α
3 ln 2
= 1 + e = 9

(But α should be greater than 0 for finite value of c)


65. (B)

P (8λ − 3, 2λ + 4, 2λ − 1)

PR = 6
2 2 2
(8λ − 4) + (2λ + 2) + (2λ − 4) = 36

λ = 0, 1

Hence P (−3, 4, −1) & Q (5, 6, 1)


Centroid of ΔP QR = (1, 4, 1) ≡ (α, β, γ)
2 2 2
(α + β + γ ) = 18

66. (C)
2
f (x) = ∣
∣ 2x + 5 |x| − 3∣

Graph of y = ∣
2
∣ 2x + 5x − 3∣

Graph of f (x)

Number of points of discontinuity = 0 = m


Number of points of non-differentiability = 3 = n
67. (D)

2
p = A, q = AR, r = AR
2 2
Ax + ARx − AR = 0

1 1 3
∴ + =
α β 4

α+β 3 −R 3 4
∴ = ⇒ = ⇒ R =
2 3
αβ 4 −R 4

2 2 2 2 16
(α − β) = (α + β) − 4αβ = R − 4 (−R ) = 5 ( )
9
80
=
9

68. (C)

h = 3 cos θ;
18
k = sin θ
7
2

locus =
2 49y
x
∴ + = 1
9 324
−−−−−−−
324 √ 117 √ 13
e = √1 − = =
49×9 2 7
69. (B)

−→


P N . b = 0?

3 (3λ − 2) + 2 (2λ − 5) + (λ − 7) = 0
23
14λ = 23 ⇒ λ =
14
83 32 51
N ( , , )
14 14 14
α+3 83 62
∴ = ⇒ α =
2 14 7
β+4 32 4
= ⇒ β =
2 14 7
γ+9 −12
51
= ⇒ γ =
2 14 7

Ans. 14 (α + β + γ) = 108

70. (D)

A (1, 3, 2) ; B (−2, 8, 0) ; C (3, 6, 7) ;



−→
^ ^ ^
AC = 2 i + 3 j + 5k
− −−−− −−−− − −
AB = √ 9 + 25 + 4 = √ 38
−−−−− −−−− − −
AC = √ 4 + 9 − 25 = √ 38

−→
1 ^ ^ 3 ^ 1 ^ ^ ^
AD = i − 4j − k = ( i − 8 j − 3k )
2 2 2


−→ −
−→
Length of projection of AD on AC
∣ ∣
−→
− −→

∣ AD . AC ∣ 37
= =
−→

∣ ∣ ∣ ∣ 2√ 38
∣AC ∣
∣ ∣ ∣ ∣
71. (A)
√ x 2 −25
2
f (x) = + log10 (x + 2x − 15)
2
4−x

Domain: x 2
− 25 ⩾ 0 ⇒ x ∈ (−∞, −5] ∪ [5, ∞)
2
4 − x ≠ 0 ⇒ x ≠ {−2, 2}
2
x + 2x − 15 > 0 ⇒ (x + 5) (x − 3) > 0

⇒ x ∈ (−∞, −5) ∪ (3, ∞)

∴ x ∈ (−∞, −5) ∪ [5, ∞)

α = −5, β = 5
2 3
∴ α + β = 150

72. (C)

mOM . mC M = −1
k k−1
. = −1
h h

∴ locus is x 2
+ y (y − 1) = 0
2 2
x + y − y = 0

1
∣ ∣
12
p = ∣ ∣ ⇒ p =
√2
2√ 2
∣ ∣
−− −−−−
2 2
P Q = 2√r − p
−−−−−
1 1 1
= 2√ − =
4 8 √2

73. (A)
S10 = 390
10
[2a + (10 − 1) d] = 390
2

⇒ 2a + 9d = 78 . . . (1)
t10 15 a+9d 15
= ⇒ = ⇒ 8a = 3d . . . . (2)
t5 7 a+4d 7

From (1) & (2),


a = 3 & d = 8
15 5
S15 − S5 = (6 + 14 × 8) − (6 + 4 × 8)
2 2
15×118−5×38
= = 790
2
74. (B)
x1 , x2 . . . . x10
10 10

∑ (xi − α) = 2 ⇒ ∑ xi − 10α = 2
i=1 i=1
∑ xi
Mean μ =
6

5
=
10

∴ ∑ xi = 12

10α + 2 = 12

∴ α = 1
10

Now
2
∑ (xi − β) = 40
i=1

Let y i = xi − β
2 1 2 2
∴ σy = ∑ yi − (ȳ )
10
2
10

⎛ ∑ (x i−β) ⎞
2 1 2 i=1

σx = ∑ (xi − β) ⎜ ⎟
10 10
⎝ ⎠

2
84 12−10β
= 4 − ( )
25 10

2
6−5β 84 16
∴ ( ) = 4 − =
5 25 25

6 − 5β = ±4 ⇒ β =
2

5
(not possible) or β = 2

Hence
β
= 2
α

75. (A)
1 1

3 2 3
I = ∫ (2x − 3x − x + 1) dx
0

2a

Using ∫ f (x) dx where f (2a − x) = −f (x)


0

Here f (1 − x) = f (x)

∴ I = 0

76. (D)
|z| ⩾ 1

Min. value of ∣∣z + 3

2
+ 2i∣
∣ is actually zero.
77. (B)
2 2
aR 1 b ⇔ a + b = 1; a, b ∈ R

(a, b) R 2 (c, d) ⇔ a + f = b + c; (a, b) , (c, d) ∈ N

For R : Not reflexive symmetric not transitive


1

For R : R is reflexive, symmetric and transitive


2 2

Hence only R is equivalence relation.


2

78. (B)
π

4
∫ cos x dx
0
π

3 2
1+cos 2x
= ∫ ( ) dx
2
0
π

3
1 2
= ∫ (1 + 2 cos 2x + cos 2x) dx
4
0
π π π

⎡ 3 3 3 ⎤
1 1+cos 4x
= ∫ dx + 2 ∫ cos 2x dx + ∫ dx
4 2
⎣0 0 0

π π

1 π 3 1 π 1 3
= [ + (sin 2x) + ( ) + (sin 4x) ]
4 3 0 2 3 8 0

π π

1 π 3 1 π 1 3
= [ + (sin 2x) + ( ) + (sin 4x) ]
4 3 0 2 3 8 0

1 π √3 1 √3
= [ + + × (− )]
4 2 2 8 2

π 7√ 3
= +
2 64
1 7
∴ a = ; b =
8 64
9 7
∴ 9a + 8b = + = 2
8 8
79. (A)
x + 2y + 3z = 5

2x + 3y + z = 9

4x + 3y + λz = μ

for infinite following


Δ = Δ1 = Δ2 = Δ3 = 0

∣ 1 2 3 ∣
∣ ∣
Δ = 2 3 1 = 0 ⇒ λ = −13
∣ ∣
∣ 4 3 λ ∣

∣ 5 2 3 ∣
∣ ∣
Δ1 = 9 3 1 = 0 ⇒ μ = 15
∣ ∣
∣ μ 3 −13 ∣

∣ 1 5 3 ∣
∣ ∣
Δ2 = 2 9 1 = 0
∣ ∣
∣ 4 15 −13 ∣

∣ 1 2 5 ∣
∣ ∣
Δ3 = 2 3 9 = 0
∣ ∣
∣ 4 3 15 ∣

For λ = −13, μ = 15 system of equation has infinite solution


Hence λ + 2μ = 17

80. (A)
1 −2 18

x 3 x 3
( + )
3 2

1 12 −2 6

18 x 3 x 3 18 1 1
t7 = C6 ( ) ( ) = C6 12
⋅ 6
3 2
(3) 2

1 6 −2 12

18 x 3 x 3 18 1 1 −6
t13 = C 12 ( ) ( ) = C2 6
⋅ ⋅ x
3 2 12
(3) 2

18 −12 −6 18 −12 −6
m = C 6 .3 .2 : n = C 12 .2 .3
1
1
−12 −6 2
n 2 .3 3 3 9
3
( ) = ( ) = ( ) =
m −12 −6
3 .2 2 4
81. (1.00)
a, ar, ar G.P. 2

Sum of any two sides > third side


2 2 2
a + ar > ar , a + ar > ar, ar + ar > a
2
r − r − 1 < 0
1−√ 5 1+√ 5
r ∈ ( , ) . . . . (1)
2 2

2
r − r + 1 > 0

Always true
2
r + r − 1 > 0
−1−√ 5 −1+√ 5
r ∈ (−∞, ) ∪ ( , ∞) . . . (2)
2 2

Taking intersection of (1) , (2)


−1+√ 5 1+√ 5
r ∈ ( , )
2 2

As r > 1
1+√ 5
r ∈ (1, )
2

[r] = 1, [−r] = −2

3 [r] + [−r] = 1

82. (8.00)
2
ky = 2 (y − x)
2
2y = kx
2
2y
Point of intersection → ky 2
= (y −
k
)

2y
y = 0, ky = 2 (1 − )
k

4y
ky + = 2
k
2 2k
y = 4
= 2
k+ k +4
k

2k

2
k +4
2 2
ky 2y
A = ∫ ((y − ) − ( )). dy
2 k
0
2k
2 3
y k 2 y ∣ k2 +4
= − ( + ) ⋅
2 2 k 3 ∣0
2 2
2k 1 k +4 1 2k
= ( ) [ − × × 2
]
2 2 2k 3
k +4 k +4

1 1
= × 4 × ( )
6 4
k+
k
4
(k+ )
k
A. M . ⩾ G. M ⩾ 2
2
4
k + ⩾ 4
k

Area is maximum when k =


4

k = 2, −2
83. (38.00)
^ ^ ^
a⃗ = i + j + k

⃗ ^ ^ ^
b = i + 8 j + 2k

^ ^ ^
c ⃗ = 4 i + c2 j + c3 k


b × a⃗ = c ⃗ × a⃗

⃗ ⃗
(b − c ) × a⃗ = 0


b − c ⃗ = λα⃗


b = c ⃗ + λα⃗

^ ^ ^ ^ ^ ^ ^ ^ ^
− i − 8 j + 2k = (4 i + c2 j + c3 k ) + λ ( i + j + k )

λ + 4 = −1 ⇒ λ = −5

λ + c2 = −8 ⇒ c2 = −3

λ + c3 = 2 ⇒ c3 = 7

^ ^ ^
c ⃗ = 4 i − 3 j + 7k
12−12+7 7 7
cos θ = = =
√ 26⋅√ 74 √ 26⋅√ 74 2√ 481

2 625×3
tan θ =
49
2
[tan θ] = 38

84. (105.00)
2
(√ x+1)(x −√ x)
1 2 3
y = + (3cos x − 5) cos x
x√ x+x+√ x 15

3
(√ x+1)(√ x)((√ x) −1)
1 5 1 3
y = + cos x − cos x
2 5 3
(√ x)((√ x) +(√ x)+1)

−− −− 1 5 1 3
y = (√ x + 1) (√ x − 1) + cos x − cos x
5 3
′ 4 2
y = 1 − cos x ⋅ (sin x) + cos x (sin x)
′ π 9 1 3 1
y ( ) = 1 − × + ×
6 16 2 4 2
32−9+12 35
= =
32 32

′ π
= 96y ( ) = 105
6
85. (36.00)

4√ 3

sin 30
c
∘ =
sin 120
∘ [By sine rule]
2c = 8 ⇒ c = 4

AB = |(b + 1)| = 4

b = 3, mAB = 0
−1
mBC =
√3

−1
BC : y = (x − 3)
√3

√ 3y + x = 3

Point of intersection: y = x + 3, √ 3y + x = 3

(√ 3 + 1) y = 6
6
y =
√ 3 +1

6
x = − 3
√ 3 +1

6−3√ 3 −3
=
√ 3 +1

(1−√ 3 ) −6
= 3 = 2
(1+√ 3 ) (1+√ 3 )
4
β
= 36
2
α
86. (7.00)

∣ 0 0 1∣
1 ∣ 2
∣ 1 2 2
S2 = a a 1 = (ab + a b)
2 ∣ ∣ 2
2
∣ −b −b 1∣
2 2
2 a −b
PQ : y − a = (x − a)
a+b
2
y − a = (a − b) x − (a − b) a

y = (a − b) x + ab

2 3 a
x x ∣
= (a − b) + (ab) x − ∣
2 3
−b
2 3 3
(a−b) (a+b) (a + b )
= + ab (a + b) −
2 3
2 2
2 (a +b −ab)
(a−b)
+ab−
S1 2 3
=
ab
S2
2
2 2 2
3(a−b) +6ab−2(a +b −ab)
=
3ab

a b
⎡ ⎤
1 + + 2
= b a
3
⎣ ⎦
min = 2
4 m
= =
3 n

m + n = 7

87. (2.00)
T
A = I2 − 2M M
2 T T
A = (I2 − 2M M ) (I2 − 2M M )
T T T T
= I2 − 2M M − 2M M + 4M M MM
T T
= I2 − 4M M + 4M M

= I2

AX = λX
2
A X = λAX

X = λ (λX)
2
X = λ X
2
X (λ − 1) = 0
2
λ = 1

λ = ±1

Sum of square of all possible values = 2


88. (5.00)
2
dx x 1−y
− =
dy y y
1

Integrating factor = e
∫ −
y
dy
=
1

y
2
1 1−y
x ⋅ = ∫ dy
y 2
y

x −1
= − y + c
y y

2
x = −1 − y + cy

x (1) = 1

1 = −1 − 1 + c ⇒ c = 3
2
x = −1 − y + 3y

5x (2) = 5 (−1 − 4 + 6)

= 5

89. (101.00)
are parallel
L1 , L3 , L5 , . . . L19

are Concurrent
L2 , L4 , L6 . . . L20

Total points of intersection = C 20


2

10
C2 −
10
C2 + 1

= 101

90. (219.0)
x

F (x) = ∫ t ⋅ f (t) dt
0

Given F ′
(x) = xf (x)
2 4 5
F (x ) = x + x ,

Let x 2
= t
5
2
F (t) = t + t 2

3
′ 5
F (t) = 2t + t 2

2
3
5
t ⋅ f (t) = 2t + t 2

2
1
5
f (t) = 2 + r 2

2
12 12
2 5
∑ f (r ) = ∑ 2 + r
2
r=1 r=1

12(13)
5
= 24 + [ ]
2 2

= 219

You might also like